Oswaal NTA CUET (UG)| Question Bank Chapterwise & Topicwise Biology For 2024 Exam 9789359580470, 9359580473

Description of the product: • Strictly as per the Latest Exam Pattern issued by NTA • 100% Updated with 2023 Exam Paper

119 3 67MB

English Pages 203 Year 2023

Report DMCA / Copyright

DOWNLOAD PDF FILE

Table of contents :
Cover page
Copyright
Contents
Know your CUET(UG) Exam
Latest CUET (UG) Syllabus
Examination Paper CUET 2023
UNIT I: Reproduction
Chapter-1 – Reproduction in Organisms
Chapter-2 – Sexual Reproduction in
Flowering Plants
Chapter-3 – Human Reproduction
Chapter-4 – Reproductive Health
UNIT II: Genetics and Evolution
Chapter-5 – Heredity and Variation
Chapter-6 – Molecular Basis of Inheritance
Chapter-7 – Evolution
UNIT III: Biology and Human Welfare
Chapter-8 – Human Health and Diseases
Chapter-9 – Improvement in Food Production
Chapter-10 – Microbes in Human Welfare
UNIT IV: Biotechnology and its
Applications
Chapter-11 – Biotechnology : Principles
and Processes
Chapter-12 – Biotechnology and Its
Applications
UNIT V: Organisims & Environment
Chapter-13 – Organisms & Environment
Chapter-14 – Ecosystems
Chapter-15 – Biodiversity and Its
Conservation
Chapter-16 – Environmental Issues
Recommend Papers

Oswaal NTA CUET (UG)| Question Bank Chapterwise & Topicwise Biology For 2024 Exam
 9789359580470, 9359580473

  • 0 0 0
  • Like this paper and download? You can publish your own PDF file online for free in a few minutes! Sign Up
File loading please wait...
Citation preview

For 2024 Exam

Highly Recommended

CHAPTER-WISE

CHAPTER-WISE

QUESTION BANK

QUESTION BANK Includes SOLVED PAPERS (2021 - 2023)

BIOLOGY

BIOLOGY Section-II (Domain Speciifc Subject) Strictly as per the Latest Exam Pattern issued by NTA

The ONLY book you need to #AceCUET(UG)

SECTION II

1

2

3

4

5

100% Updated

Previous Years’ Questions

Revision Notes

Concept Videos

800+ Questions

With 2023 CUET Exam Paper

(2021-2023) for Better Exam Insights

for Crisp Revision with Smart Mind Maps

for Complex Concepts Clarity

for Extensive Practice

(1)

1st EDITION

I S BN

YEAR 2024 "9789359580470"

CUET (UG)

SYLLABUS COVERED

PUBLISHED BY OSWAAL BOOKS & LEARNING PVT. LTD.

COP Y RIGHT

RESERVED

1/11, Sahitya Kunj, M.G. Road, Agra - 282002, (UP) India

BY THE PUBLISHERS

All rights reserved. No part of this book may be reproduced, stored in a retrieval system, or transmitted, in any form or by any means, without written permission from the publishers. The author and publisher will gladly receive information enabling them to rectify any error or omission in subsequent editions.

1010, Cambourne Business Centre Cambridge, Cambridgeshire CB 236DP, United kingdom

0562-2857671

[email protected]

www.OswaalBooks.com

DI SCL A IMER

This book is published by Oswaal Books and Learning Pvt Ltd (“Publisher”) and is intended solely for educational use, to enable students to practice for examinations/tests and reference. The contents of this book primarily comprise a collection of questions that have been sourced from previous examination papers. Any practice questions and/or notes included by the Publisher are formulated by placing reliance on previous question papers and are in keeping with the format/pattern/ guidelines applicable to such papers. The Publisher expressly disclaims any liability for the use of, or references to, any terms or terminology in the book, which may not be considered appropriate or may be considered offensive, in light of societal changes. Further, the contents of this book, including references to any persons, corporations, brands, political parties, incidents, historical events and/or terminology within the book, if any, are not intended to be offensive, and/or to hurt, insult or defame any person (whether living or dead), entity, gender, caste, religion, race, etc. and any interpretation to this effect is unintended and purely incidental. While we try to keep our publications as updated and accurate as possible, human error may creep in. We expressly disclaim liability for errors and/or omissions in the content, if any, and further disclaim any liability for any loss or damages in connection with the use of the book and reference to its contents”.

Kindle Edition (2)

Preface Welcome to the ultimate resource for your Common University Entrance Test (CUET) preparation! The Common University Entrance Test (CUET) marks a significant shift in the admission process for UG programs in Central Universities across India. The introduction of CUET aims to create a level playing field for students nationwide, regardless of their geographical location, and revolutionize the way students connect with these prestigious institutions. CUET (UG), administered by the esteemed National Testing Agency (NTA), is a prestigious all-India test that serves as a single-window opportunity for admissions. The NTA consistently provides timely notifications regarding the exam schedule and any subsequent updates. The curriculum for CUET is based on the National Council of Educational Research and Training (NCERT) syllabus for class 12 only. CUET (UG) scores are mandatory required while admitting students to undergraduate courses in 44 central universities. A merit list will be prepared by participating Universities/organizations. Universities may conduct their individual counselling on the basis of the scorecard of CUET (UG) provided by NTA. Oswaal CUET (UG) Question Bank is your strategic companion designed to elevate your performance and simplify your CUET journey for success in this computer-based test.

Here’s how this book benefits you: • 100% Updated with 2023 CUET Exam Paper • Previous years Questions (2021-2023)for Better Exam insights • Revision Notes for Crisp Revision with Smart Mind Maps • Concept Videos for complex concepts clarity • 800+ questions for Extensive Practice Almost 1.92 million candidates registered for CUET (UG) in 2023. Candidates have been quite anxious about appearing for CUET (UG), however, with the right preparation strategy and resources, you can secure a good rank in CUET (UG). We believe that with dedication, hard work, and the right resources, you can conquer CUET and secure your place in the Central Universities of your choice. Good luck with your preparations, with this trusted companion on your journey to academic success! All the best! Team Oswaal Books

(3)

Order Form JEE (MAIN) 15 Mock Test Papers (For 2024 Exam)

National Talent Search Exam (NTSE) Year-wise Solved Papers (2011 -2021)

JEE (MAIN) 15 Mock Test Papers (Mathematics, Physics, Chemistry)

NTSE 10 Yearwise Solved Papers 2011-2021

JEE (MAIN) Year-wise 22 Solved Papers(All Shifts) (For 2023 Exam)

NRA Question Bank 10th Pass Quantitative Aptitude, Logical Reasoning, General Awareness, General English

Mathematics, Physics, Chemistry JEE (MAIN) Chapter-wise & Topic-wise Solved papers (2019-2023) Question Bank (For 2024 Exam)

NRA Question Bank 12th Pass Quantitative Aptitude, Logical Reasoning, General Awareness, General English

Mathematics, Physics, Chemistry JEE (MAIN) 24 Online Previous Years’ Solved Papers (All Shifts) 2023 (For 2024 Exam)

NRA Question Bank Graduation Pass Quantitative Aptitude, Logical Reasoning, General Awareness, General English

Mathematics, Physics, Chemistry JEE(Main) RMT Cards

Objective General English

Physics Part - I, Physics Part - II, Chemistry Part - I Chemistry Part - II, Mathematics Part -I, Mathematics Part - II

Objective General English Objective Quantitative Aptitude

JEE Advanced (For 2024 Exam) 22 Year-wise Solved Papers (2002-2023)

Objective Quantitative Aptitude CAT

Mathematics, Physics, Chemistry

CAT 15 Mock Test Papers, CAT 25 Years’ Chapterwise & Topicwise Solved Papers (1990-2008) & (2017-2022), CAT 10 Years’ Chapterwise & Topicwise Solved Papers (2005-2008) & (20172022), CAT 5 Years’ Solved Papers Shift-wise (2018-2022)

JEE Advanced 10 Mock Test Papers Paper 1 & Paper 2 (For 2024 Exam) JEE Advanced 10 Mock Test Papers (Paper-1 & 2) GATE 14 Year-wise (For 2024 Exam) Solved Papers (2010 to 2023)

CLAT & AILET 10 Solved Papers (For 2024 Exam) CLAT & AILET 10 Year-wise Solved Papers CLAT (2020-2023) & AILET (2018-2023)

GATE 14 Yearwise Solved Paper (General Aptitude), GATE 14 Yearwise Solved Paper (Engineering Mathematics)

UGC-NET

GATE 14 Years’ (For 2024 Exam) Chapterwise & Topicwise Solved Papers (2010 to 2023)

UGC NET Paper-1 Year-Wise-13 Solved Papers (2015-2023) (For 2024 Exam) General Aptitude, UGC NET Paper-1-15 Years’ Question Bank Chapterwise & Topicwise Solved Papers (20092023) General Aptitude & UGC NET Paper-1 - 15 Mock Test Papers (For 2024 Exam) General Aptitude

GATE 14 Years’ Chapterwise & Topicwise Solved Papers (General Aptitude), GATE 14 Years’ Chapterwise & Topicwise Solved Papers (Engineering Mathematics) NEET (UG) 36 Years’ Solved Papers (For 2024 Exam) Chapterwise & Topicwise 1988-2023

RRB NTPC 2023 1st Stage Exam RRB NTPC 12 Previous Years’ Papers 2016 (All Shifts) & 2019 (Two Shifts) +10 Mock Test Papers

Biology, Physics, Chemistry NEET (UG) 18 Year-wise Solved Papers (2006 to 2023)

UPSC CSE Prelims 10 Years’ Solved Papers (2014 -2023) English Medium (For 2024 Exam)

Biology, Physics, Chemistry

UPSC CSE Prelims General Studies (Paper-1), UPSC CSE Prelims General Studies (Paper-2) (CSAT)

NEET (UG) 15 Mock Test Papers (For 2024 Exam) Physics, Chemistry, Biology

UPSC CSE NCERT One for All (For 2024 Exam)

NEET (UG) RMT Cards

Polity, Economy, History, Geography, General Science

Physics Part-1, Physics Part-2, Chemistry Part-1, Chemistry Part2, Biology Part-1, Biology Part-2

UPSC Power Bank For UPSC CSE & State PSCs Exam (For 2024 Exam)

Handbook Class 11 & 12, Entrance Exams (Engineering & Medical)

Ancient & Medieval History, Art & Culture, Geography, Indian Polity, Indian Economy, Science & Tech., Environment & Ecology, Modern History

Physics, Chemistry, Mathematics & Biology Vedic Maths

UPSC CSE Prelims Mock Test Papers (For 2024 Exam)

Vedic Maths (Ashima Sekhri)

15 Mock Test Papers General Studies Paper-1, 15 Mock Test Papers (CSAT) General Studies Paper-2

CUET (UG) Mock Test Papers (For 2024 Exam)

UPSC CSE (Pre) 29 Years’ Solved Papers (1995-2023) (For 2024 Exam)

English, Accountancy, Biology, Business Studies, Chemistry, Mathematics/Applied Mathematics, Entrepreneurship, Computer Science/Informatics Practices, Sociology, Economics, Geography, History, Physics, Political Science, Psychology, Physical Education, General Test

UPSC 29 Years’ Topicwise Solved Question Paper Prelims GS Paper -1 & 2 UPPSC Prelims 15 Year-Wise Solved Papers

CDS Chapter-Wise & Topic-Wise Solved Papers (2014-2023)

UPPSC Prelims 15 Year-Wise Solved Papers GS Paper-1 & Paper-2

Elementary Mathematics, English, General Knowledge NDA-NA Chapter-Wise & Topic-Wise Solved Papers (2014-2023)

CDS Year wise 11 Solved Papers (2018-2023) Elementary Mathematics, English, General Knowledge

Mathematics, General Ability Test: English, General Ability Test: General Studies

NDA-NA Year-wise 12 Solved Papers (2017-2023) Mathematics, General Ability Test: English, General Ability Test: General Studies

CUET (UG) General Test Question Bank (For 2024 Exam) Quantitative Aptitude, Logical Reasoning, General Awareness

Exam Preparation Books for Class 1 t o 12 : CBSE, CISCE Boar ds & Karnataka Boar d | OLYMPIADS | JEE | NEET | CUET | NDA | CDS | CAT | GATE | UPSC | UGC NET | NRA CET & mo re

We are available in all leading e-comme rce platforms and all good boo kstores.

(4)

Contents l Know your CUET(UG) Exam

6 - 6

l Latest CUET (UG) Syllabus

7 - 8

l Examination Paper CUET 2023



Chapter-10 – Microbes in Human Welfare 110 - 119

UNIT I: Reproduction 1 - 10

Chapter-1 – Reproduction in Organisms Chapter-2 – Sexual Reproduction in Flowering Plants

11 - 22

Chapter-3 – Human Reproduction

23 - 35

Chapter-4 – Reproductive Health

36 - 44

UNIT II: Genetics and Evolution Chapter-6 – Molecular Basis of Inheritance 58 - 71 72 - 84

Chapter-7 – Evolution

UNIT III: Biology and Human Welfare 85 - 98

Chapter-9 – Improvement in Food Production 

UNIT IV: Biotechnology and its Applications Chapter-11 – Biotechnology : Principles and Processes

99 - 109

120 - 131

Chapter-12 – Biotechnology and Its Applications

45 - 57

Chapter-5 – Heredity and Variation

Chapter-8 – Human Health and Diseases

11 - 16

132 - 141

UNIT V: Organisims & Environment Chapter-13 – Organisms & Environment

142 - 150

Chapter-14 – Ecosystems

151 - 161

Chapter-15 – Biodiversity and Its Conservation

162 - 171

Chapter-16 – Environmental Issues

172 - 183

qqq

(5)

2 Languages + 6 Domain Specific Subject + General Test

OR

3 Languages + 5 Domain Specific Subjects + General Test

Subject/Language Choice

Objective Type with MCQs

CBT

Mode of Test

Test Pattern

Know Your CUET (UG) Exam SECTIONS

SECTION I (A) 13 Languages

Tested through reading Comprehension (i) Factual (ii) Literary (iii) Narrative

SECTION III SECTION I (B)

SECTION II

20 Languages

Domain Specific Subjects ( 27 Subjects)

General Test (Compulsory)

INCLUDES : NCERT Model syllabus (only of 12th Standard) is available on all the Subjects

(6)

• • • • • •

General Knowledge Current Affairs General Mental Ability Numerical Ability Quantitative Reasoning Logical & Analytical Reasoning

Syllabus BIOLOGY - 304 Note:

There will be one Question Paper which will have 50 questions out of which 40 questions need to be attempted. BIOLOGY/BIOLOGICAL STUDIES/BIOTECNOLOGY/BIOCHEMISTRY

syndrome, Turner’s and Klinefelter’s syndromes.

Unit I: Reproduction Reproduction in organisms: Reproduction, a characteristic feature of all organisms for continuation of species; Modes of reproduction – Asexual and sexual; Asexual reproduction; Modes- Binary fission, sporulation, budding, gemmule, fragmentation; vegetative propagation in plants.

Molecular Basis of Inheritance: Search for genetic material and DNA as genetic material; Structure of DNA and RNA; DNA packaging; DNA replication; Central dogma; Transcription, genetic code, translation; Gene expression and regulation– Lac Operon; Genome and human genome project; DNA finger printing.

Sexual reproduction in flowering plants: Flower structure; Development of male and female gametophytes; Pollination–types, agencies and examples; Outbreedings devices; Pollen-Pistil interaction; Double fertilization; Post fertilization events– Development of endosperm and embryo, Development of seed and formation of fruit; Special modes– apomixis, parthenocarpy, polyembryony; Significance of seed and fruit formation.

Evolution: Origin of life; Biological evolution and evidences for biological evolution (Paleontological, comparative anatomy, embryology and molecular evidence); Darwin’s contribution, Modern Synthetic theory of Evolution; Mechanism of evolution– Variation (Mutation and Recombination) and Natural Selection with examples, types of natural selection; Gene flow and genetic dirft; Hardy- Weinberg’s principle;Adaptive Radiation; Human evolution.



Unit III: Biology and Human Welfare

Human Reproduction: Male and female reproductive systems; Microscopic anatomy of testis and ovary; Gametogenesis- spermatogenesis & oogenesis; Menstrual cycle; Fertilisation, embryo development upto blastocyst formation, implantation; Pregnancy and placenta formation (Elementary idea); Parturition (Elementary idea); Lactation (Elementary idea).

Reproductive health: Need for reproductive health and prevention of sexually transmitted diseases (STD); Birth control- Need and Methods, Contraception and Medical Termination of Pregnancy (MTP); Amniocentesis; Infertility and assisted reproductive technologies – IVF, ZIFT, GIFT (Elementary idea for general awareness).

Unit II: Genetics and Evolution Heredity and variation: Mendelian Inheritance; Deviations from Mendelism– Incomplete dominance, Co-dominance, Multiple alleles and Inheritance of blood groups, Pleiotropy; Elementary idea of polygenic inheritance; Chromosome theory of inheritance; Chromosomes and genes; Sex determination– In humans, birds, honey bee; Linkage and crossing over; Sex linked inheritance- Haemophilia, Colour blindness; Mendelian disorders in humans– Thalassemia; Chromosomal disorders in humans; Down’s

Health and Disease: Pathogens; parasites causing human diseases (Malaria, Filariasis, Ascariasis, Typhoid, Pneumonia, common cold, amoebiasis, ring worm); Basic concepts of immunology–vaccines; Cancer, HIV and AIDs; Adolescence, drug and alcohol abuse. Improvement in food production: Plant breeding, tissue culture, single cell protein, Biofortification; Apiculture and Animal husbandry. Microbes in human welfare: In household food processing, industrial production, sewage treatment, energy generation and as biocontrol agents and biofertilizers.

Unit IV: Biotechnology and Its Applications Principles and process of Biotechnology: Genetic engineering (Recombinant DNA technology).

Application of Biotechnology in health and agriculture: Human insulin and vaccine production, gene therapy; Genetically modified organisms- Bt crops; Transgenic Animals; Biosafety issues– Biopiracy and patents.

Unit V: Ecology and environment Organisms and environment: Habitat and niche; Population and ecological adaptations; Population interactions–mutualism, competition, predation,

(7)

Cont... conservation; Hotspots, endangered organisms, extinction, Red Data Book, biosphere reserves, National parks and sanctuaries.

parasitism; Population attributes–growth, birth rate and death rate, age distribution. Ecosystems: Patterns, components; productivity and decomposition; Energy flow; Pyramids of number, biomass, energy; Nutrient cycling (carbon and phosphorous); Ecological succession; Ecological Services– Carbon fixation, pollination, oxygen release. Biodiversity and its conservation: Concept of Biodiversity; Patterns of Biodiversity; Importance of Biodiversity; Loss of Biodiversity; Biodiversity

Environmental issues: Air pollution and its control; Water pollution and its control; Agrochemicals and their effects; Solid waste management; Radioactive waste management; Greenhouse effect and global warming; Ozone depletion; Deforestation; Any three case studies as success stories addressing environmental issues. 

(8)

Exclusive School Books Suppliers VIJAYAWADA WEST KAMENG BANGLORE RAJKOT

MAHARASHTRA

ANDHRA PRADESH

Sri Vikas Book Centre, 9848571114, 9440715700,

PUNE

ASSAM

JALNA

Dutta Book Stall, 8729948473

KARNATAKA

CHENNAI

Satish Agencies, 8861630123

GUJRAT

HYDERABAD

Royal Stationers, 9824207514

KOLKATA

ANDHRA PRADESH

INDORE

Akshaya Books Corner, 9666155555

GUWAHATI

PATNA

Bookmark-IT, 7305151653

TELANGANA

Sri Balaji Book Depot , 9676996199, (040) 27613300

WEST BENGAL

United Book House, 9831344622

Bhaiya Industries, 9893326853, Sushil Prakashan,(0731) 2503333, 2535892, 9425322330, Bhaiya Store, 9425318103, Arun Prakashan, 9424890785, Bhaiya Book Centre, 9424081874, Seva Suppliers, 9826451052

ANDAMAN & NICOBAR ISLAND PORTBLAIR

Anil Paper Mart, 9422722522, (02482) 230733

TAMIL NADU

Our Distributors

VISAKHAPATHAM JBD Educational, 9246632691, 9246633693, Sri Rajeshwari Book Link, 9848036014 VIJAYAWADA

Madhusheela Books & Stationery, 7875899892

Krishna Book Centre, 9474205570, Kumar Book Depot, 9932082455, Kumar Book Depot, 9932082455, Sree aditya Book Centre, 8332972720, 7013300914

ASSAM

Book Emporium, 9675972993, 6000763186, Ashok Publication, 7896141127, Kayaan Enterprises, (0361) 2630443, Orchid Book house, 9864624209, Newco, 9864178188

JABALPUR

Vinay Pustak Sadan, 8962362667, Anand Books and Stationers, 9425323508

SAGAR

Princi Book Depot, Sagar, 9977277011

KATNI

Shri Mahavir Agency, 9425363412

UJJAIN BHOPAL

BIHAR

Nova Publisher & Distributors, (0612) 2666404, Shri Durga Pustak Mandir, 9334477386, Sharda Pustak Bhandar, 9334259293, Vikas Book Depot, 9504780402, Alka Book Agency, 9835655005, Metro Book(E&C), Ishu Pustak Bhandar, 8294576789, Gyan Ganga Limited, 6203900312, Ishu Pustak Bhandar, ( E & C ), 9334186300/8294576789

PUNE

Shreenath Book Depot, 9827544045 Gupta Brother, 9644482444

MAHARASHTRA

Natraj Book Depot, (020) 24485054, 9890054092, Vikas Book House, 9921331187, Pravin Sales, 9890683475, New Saraswati Granth Bhandar, 9422323859, Akshar Books & Stationary, 7385089789, Vardhaman Educational, 9860574354, Yash Book Centre, 9890156763, Pragati Book Centre, (ISC), 9850039311, Praveen Sales, Pragati Book Centre, Pune ( E & C ), 9850039311 Shree Sainath Agencies, 7350294089, Maya Book Centre, (ISC), 9372360150 Vidyarthi Sales Agencies, 9819776110, New Student Agencies, 7045065799, Shivam Books & Stationery, 8619805332

Pustak Bhandar, 7870834225

CHATTISGARH

AURANGABAD MUMBAI

AMBIKAPUR

Saini Brothers, 9425582561, M.P Department Stores, 9425254264

JALGAON

BOKARO BHILAI

Bokaro Student Friends Pvt. Ltd, Bokaro, 7277931285 Anil Book Depot, 9425234260

LATUR KOLHAPUR

Yash Book House, 9637936999, Shri Ganesh Pustakalay, 9730172188 Granth the Book World, 9922295522

KORBA

Kitab Ghar, Korba ( E & C ), 9425226528

NANDED

MUZAFFARPUR

DURG

RAIPUR RAIGARH DELHI

Bhagwati Bhawani Book Depot, 0788-2327620, 9827473100

NAGPUR

Sharma Book Depot & Stat. (ISC), 9421393040

Laxmi Pustakalay and Stationers, (0712) 2727354, Vijay Book Depot, 9860122094

Shri Ramdev Traders, 9981761797, Gupta Pustak Mandir, 7974220323, Anil Publication, 9691618258/7999078802

NASHIK

Renuka Book distributor, 9765406133, Novelty Book Depot, 9657690220, Karamveer Book Depot, 9923966466, Arun Book & Stationers, 9423110953 Abhang Pustakalaya, 9823470756/9175940756 Rahul Book Centre, 9970849681, New India Book House, 9623123458

DELHI

YAVATMAL

Shri Ganesh Pustkalaya, 9423131275

Sindhu Book Deopt, 9981935763

DHULE

Mittal Books, (011) 23288887, 9899037390, Singhania Book & Stationer, 9212028238, AoneBooks, New Delhi, 8800497047, Radhey Book Depot, 9818314141, Batheja Super Store, 9871833924, Lov Dev & Sons, Delhi ( E & C ), 9811182352, Zombozone, 9871274082, LDS Marketing, 9811182352/9999353491

VASAI

Navjeevan Book Stall, 7020525561

Prime Book Centre, Vasai, 9890293662

ODISHA A. K. Mishra Agencies, 9437025991, 9437081319

GUJARAT

CUTTACK

BHAVNAGAR DAHOD VAPI

Patel Book, 9898184248, 9824386112, 9825900335, Zaveri Agency, 9979897312, 9979890330, Hardik Book Agency, (ISC) 079-24110043, 9904659821 Samir Book Stall, Bhavnagar (ISC) 9586305305 Collegian Book Corner, 9925501981 Goutam Book Sellers, 9081790813

BHUBANESHWAR M/s Pragnya, 8847888616, 9437943777, Padmalaya, 9437026922, Bidyashree, 9937017070, Books Godown, 7894281110 BARIPADA Trimurti Book World, 9437034735 KEONJHAR Students corner, 7008435418

VALSAD NAVSARI

Mahavir Stationers, 9429474177 College Store, (ISC) NO CALL 02637-258642, 9825099121

AMBALA PATIALA

VADODARA

Umakant Book Sellers & Stationer, 9624920709

HARYANA

FEROZPUR LUDHIANA

ROHTAK

Manish Traders, 9812556687, Swami Kitab Ghar, 9355611088,

CHANDIGARH

REWARI

Sanjay book depot, 9255447231

Kashi Ram Kishan lal, 9289504004, 8920567245 Natraj Book Distributors, 7988917452

AJMER KOTA

BHUNA

Khurana Book Store, 9896572520

BHILWARA

JAMMU

JAIPUR

Sahitya Sangam, 9419190177

UDAIPUR

Nakoda Book Depot, (01482) 243653, 9214983594, Alankar Book Depot, 9414707462 Ravi Enterprises, 9829060694, Saraswati Book House, (0141) 2610823, 9829811155, Goyal Book Distt., 9460983939, 9414782130 Sunil Book Store, 9828682260

Crown Book Distributor & Publishers, (0651) 2213735, 9431173904, Pustak Mandir, 9431115138, Vidyarthi Pustak Bhandar, 9431310228

AGARTALA

Book Corner, 8794894165, 8984657146, Book Emporium, 9089230412

KARNATAKA

COIMBATORE

SURAT

BALLABGARH HISAR

BOKARO RANCHI DUMKA

PUNJAB

Shopping Point, 9824108663

JALANDHAR

Babu Ram Pradeep Kumar, 9813214692

JHARKHAND

JODHPUR

Bokaro Student Friends, (0654) 2233094, 7360021503, Bharati Bhawan Agencies, 9431740797

Renuka Book Distributor, (0836) 2244124, Vidyamandir Book Distributors, 9980773976 CHENNAI

BANGLORE

Krishna book house, 9739847334, Hema Book Stores, 9986767000,

BELLERI

Chatinya book centre, 9886064731

PUDUCHERRY

ERNAKULAM

Academic Book House, (0484) 2376613, H & C Store, 9864196344, Surya Book House, 9847124217, 9847238314 Book Centre, (0481) 2566992 Academic Book House, (0471) 2333349, 9447063349, Ponni Book Stall, 9037591721

TRICHY

KOTTAYAM TRIVANDRUM CALICUT

Sapna Book House Pvt. Ltd., 9980513242, Hema Book World, (Chamrajpet) (ISC) 080-40905110, 9945731121

Aman Book Stall, (0495) 2721282,

MADHYA PRADESH

CHHINDWARA

Pustak Bhawan, ( E & C ), 8982150100

GWALIOR

Agarwal Book Depot, 9425116210

Cheap Book Store, 9872223458, 9878258592, City Book Shop, 9417440753, Subhash Book Depot, 9876453625, Paramvir Enterprises, 9878626248 Sita Ram book Depot, 9463039199, 7696141911 Amit Book, 9815807871, Gupta Brothers, 9888200206, Bhatia Book Centre, 9815277131 Mohindra Book Depot, 9814920226

RAJASTHAN

Laxmi General Store, Ajmer, 0145- 2428942 9460652197 Vardhman Book Depot, 9571365020, 8003221190 Raj Traders, 9309232829

Second Hand Book Stall, 9460004745

TRIPURA

TAMIL NADU

HUBLI

KERALA

Bharat Book Depot, 7988455354 Goel Sons, 9463619978, Adarsh Enterprises, 9814347613

SALEM

THENI MADURAI VELLORE

HYDERABAD

Majestic Book House, (0422) 2384333, CBSC Book Shop, 9585979752

Arraba Book Traders, (044) 25387868, 9841459105, M.R. Book Store (044) 25364596, Kalaimagal Store, (044) 5544072, 9940619404, Vijaya Stores, 9381037417, Bookmark It-Books & Stat. Store, 7305151653, M.K. Store, 9840030099, Tiger Books Pvt. Ltd., 9710447000, New Mylai Stationers, 9841313062, Prince Book House, Chennai, 0444-2053926, 9952068491, S K Publishers & Distributors, 9789865544, Dharma Book Shop, 8667227171 Sri Lakshmi Book Seller, 7871555145 Pattu book centre, 9894816280

P.R.Sons Book Seller, 9443370597, Rasi Publication, 9894816280 Maya Book Centre, 9443929274 Selvi Book Shoppe, 9843057435, Jayam Book Centre, 9894658036 G.K book centre and collections, 9894517994

TELANGANA

Sri Balaji Book Depot, (040) 27613300, 9866355473, Shah Book House, 9849564564 Vishal Book Distributors, 9246333166, Himalaya Book World, 7032578527

(9)

0808

AHMEDABAD

Cont... UTTARAKHAND

GORAKHPUR

Central Book House, 9935454590, Friends & Co., 9450277154, Dinesh book depot, 9125818274, Friends & Co., 9450277154

DEHRADUN

Inder Book Agencies, 9634045280, Amar Book Depot , 8130491477, Goyal Book Store, 9897318047, New National Book House, 9897830283/9720590054

JHANSI

Bhanu Book Depot, 9415031340

MUSSORIE

Ram Saran Dass Chanda kiran, 0135-2632785, 9761344588

KANPUR

Radha News Agency, 8957247427, Raj Book Dist., 9235616506, H K Book Distributors, 9935146730, H K Book Distributors, 9506033137/9935146730

UTTAR PRADESH

LUCKNOW

AGRA

Sparsh Book Agency, 9412257817, Om Pustak Mandir, (0562) 2464014, 9319117771,

MEERUT

Ideal Book Depot, (0121) 4059252, 9837066307

ALLAHABAD

Mehrotra Book Agency, (0532) 2266865, 9415636890

NOIDA

Prozo (Global Edu4 Share Pvt. Ltd), 9318395520, Goyal Books Overseas Pvt.Ltd., 1204655555 9873387003

AZAMGARH

Sasta Sahitya Bhandar, 9450029674

PRAYAGRAJ

Kanhaiya Pustak Bhawan, 9415317109

ALIGARH

K.B.C.L. Agarwal, 9897124960, Shaligram Agencies, 9412317800, New Vimal Books, 9997398868, T.I.C Book centre, 9808039570

MAWANA

Subhash Book Depot, 9760262264

BULANDSHAHAR

Rastogi Book Depot, 9837053462/9368978202

BALRAMPUR

Universal Book Center, 8933826726

KOLKATA

BAREILLY

Siksha Prakashan, 9837829284

RENUKOOT

HARDOI

Mittal Pustak Kendra, 9838201466

Sanjay Publication, 8126699922 Arti book centre, 8630128856, Panchsheel Books, 9412257962, Bhagwati Book Store, (E & C), 9149081912

Vyapar Sadan, 7607102462, Om Book Depot, 7705871398, Azad Book Depot Pvt. Ltd.,

7317000250, Book Sadan, 9839487327, Rama Book Depot(Retail), 7355078254, Ashirwad Book Depot, 9235501197, Book.com, 7458922755, Universal Books,

9450302161, Sheetla Book Agency, 9235832418, Vidyarthi Kendra Publisher & Distributor Pvt Ltd, (Gold), 9554967415, Tripathi Book House, 9415425943

WEST BENGAL Oriental Publishers & Distributor (033) 40628367, Katha 'O' Kahini, (033) 22196313, 22419071, Saha Book House, (033), 22193671, 9333416484, United Book House, 9831344622, Bijay Pustak Bhandar, 8961260603, Shawan Books Distributors, 8336820363, Krishna Book House, 9123083874

Om Stationers, 7007326732

DEORIA

Kanodia Book Depot, 9415277835

COOCH BEHAR

S.B. Book Distributor, Cooch behar, 9002670771

VARANASI

Gupta Books, 8707225564, Bookman & Company, 9935194495/7668899901

KHARAGPUR

Subhani Book Store, 9046891334

MATHURA

Sapra Traders, 9410076716, Vijay Book House , 9897254292

SILIGURI

Agarwal Book House, 9832038727, Modern Book Agency, 8145578772

FARRUKHABAD

Anurag Book Agencies, 8844007575

DINAJPUR

Krishna Book House, 7031748945

NAJIBABAD

Gupta News Agency, 8868932500, Gupta News Agency, ( E & C ), 8868932500

MURSHIDABAD

New Book House, 8944876176

DHAMPUR

Ramkumar Mahaveer Prasad, 9411942550

Entrance & Competition Distributors PATNA

BIHAR

CUTTAK

A.K.Mishra Agencies, 9437025991

Metro Books Corner, 9431647013, Alka Book Agency, 9835655005, Vikas Book Depot, 9504780402

BHUBANESHWAR

M/s Pragnya, 9437943777

CHATTISGARH KORBA

Kitab Ghar, 9425226528, Shri Ramdev Traders, 9981761797

PUNJAB JALANDHAR

Cheap Book Store, 9872223458, 9878258592

DELHI

RAJASTHAN

DELHI

Singhania Book & Stationer, 9212028238, Radhey Book depot, 9818314141, The KOTA Book Shop, 9310262701, Mittal Books, 9899037390, Lov Dev & Sons, 9999353491

Vardhman Book Depot, 9571365020, Raj Traders, 9309232829

NEW DELHI

Anupam Sales, 9560504617, A ONE BOOKS, 8800497047

JAIPUR

HARYANA AMBALA

BOKARO

Goyal Book Distributors, 9414782130

UTTAR PRADESH

Bharat Book Depot, 7988455354

AGRA

BHAGWATI BOOK STORE, 9149081912, Sparsh Book Agency, 9412257817, Sanjay Publication, 8126699922

JHARKHAND

ALIGARH

New Vimal Books, 9997398868

Bokaro Student Friends Pvt. Ltd, 7360021503

ALLAHABAD

Mehrotra Book Agency, (532) 2266865, 9415636890

MADHYA PRADESH

GORAKHPUR

Central Book House, 9935454590

INDORE

Bhaiya Industries, 9109120101

KANPUR

Raj Book Dist, 9235616506

CHHINDWARA

Pustak Bhawan, 9827255997

LUCKNOW

Azad Book Depot PVT LTD, 7317000250, Rama Book Depot(Retail), 7355078254 Ashirwad Book Depot , 9235501197, Book Sadan, 8318643277, Book.com , 7458922755, Sheetla Book Agency, 9235832418

MAHARASHTRA

PRAYAGRAJ

Format Center, 9335115561, Garg Brothers Trading & Services Pvt. Ltd., 7388100499

NAGPUR

Laxmi Pustakalay and Stationers, (0712) 2727354

PUNE

Pragati Book Centre, 9850039311

MUMBAI

New Student Agencies LLP, 7045065799

ODISHA

Inder Book Agancies, 9634045280

WEST BENGAL KOLKATA

Bijay Pustak Bhandar Pvt. Ltd., 8961260603, Saha Book House, 9674827254 United Book House, 9831344622, Techno World, 9830168159

Trimurti Book World, 9437034735

0808

BARIPADA

UTTAR PRADESH DEHRADUN

( 10 )

CUET QUESTION PAPER - 2023 National Testing Agency Held on 25th May 2023

BIOLOGY

(This includes Questions pertaining to Domain Specific Subject only) Max. Marks: 200

Time allowed: 45 Minutes

General Instructions: 1. The test is of 45 Minutes duration. 2. The test contains 50 questions out of which 40 questions needs to be attempted. 3. Marking Scheme of the test: a. Correct answer or the most appropriate answer: Five marks (+5). b. Any incorrect option marked will be given minus one mark (-1). c. Unanswered/Marked for Review will be given 100 mark (0).

1. Select

the organism which does not undergoes parthenogenesis.

4. Identify the organism which is a source of single cell protein and is grown on commercial scale.

(1) Honeybee

(2) Rotifer

(1) Azotobacter

(3) Turkey

(4) Fruit fly

(3) Azospirillum (4) Spirulina 5 Match List I with List II

2. The

method of producing thousands of plants through tissue culture is called as:

LIST I A

Wheat

(2) Micro-propagation

B

(3) Somatic hybridisation

C

(4) Vegetative propagation

D

(1) Somaclones

3. Which

of the following statements are correct regarding decomposition?

(2) Rhizobium

LIST II I

Perisperm

Black pepper

II

Polyembryony

Cashew

II.

Albuminous seed

Citrus

IV False fruits

Choose the correct answer from the options given below:

A. Decomposition is largely oxygen-requiring process.

(1) A-III, B-II, C-I, D-IV

B. The rate of decomposition is controlled by chemical composition of detritus and climatic factors.

(3) A-III, B-I, C-IV, D-II

C. Dry and cold decomposition.

environment

favour

the

D. If detritus is rich in lignin and chitin, the decomposition rate is faster.

Choose the correct answer from the options given below:

.

(2) A-II, B-I, C-III, D-IV (4) A-I, B-II, C-IV, D-III

6. Arrange the following events in correct sequence. A. Formation of zygote B. Formation of blastocyst C. Implantation D. Formation of morula

Choose the correct answer from the options given

(1) A and B only

below:

(2) C and D only

(1) A, D, B, E

(3) B and C only

(2) A, B, D, C

(4) A and D only

(3) A, B, C, D (4) C, D, B, A

12

OSWAAL CUET (UG) Chapterwise Question Bank BIOLOGY

7. Which of the following is used to make the bacterial 13. Match List I with List II cell as 'Competent cell’ to take up DNA? (1) Carbonate ion

(2) Calcium ion

(3) Sodium ion

(4) Sulphate ion

LIST I

8. Identify the equation representing logistic growth of a population. (1)

dN dN KN = rN (2)  rN   dt dt  K 

LIST II

A.

Down's syndrome

I.

44 + XXY

B.

Thalassemia

II.

Autosomal recessive trait

C.

Klinefelter's syndrome

III. 44 + XO

D.

Turner's Syndrome

IV.

45 +XY/ XX

Choose the correct answer from the options given below:

(3)

dN N K  N  dt  K 

(4)

dt = rN dN

(1) A-IV, B-II, C-III, D-I (2) A-IV, B-II, C-I, D-III (3) A-I, B-III, C-IV, D-II

9. Early

Greek thinkers thought that units of life called spores were transferred to different planets including earth. Identify the term associated with the above.

(1) Abiogenesis

(4) A-II, B-IV, C-III, D-I

14. Match List I with List II LIST I (Drugs)

(2) Panspermia (3) Spontaneous generation (4) Biogenesis

10. In

gel electrophoresis the fragments can be visualised :

separated

DNA

(1) In visible light (2) In visible light with staining (3) In UV radiation without staining (4) In UV radiation after staining with ethidium bromide

11. Match List I with List II LIST 1

LIST II (Obtained from)

A. Heroin

I.

Tobacco plant

B.

II.

Erythroxvium coca

Cannabinoids

C. Cocaine

III. Cannabis sativa

D. Nicotine

IV.

Papaver somniferum

Choose the correct answer from the options given below: (1) A-I, B-II, C-III, D-IV (2) A-II, B-I, C-III, D-IV (3) A-I, B-II, C-IV, D-III

LIST II

(4) A-IV, B-III, C-II, D-I

15. Methanogens do not produce :

A.

Tubectomy

I.

Barrier method

B.

Copper ions

II.

Surgical method of sterilisation in human male.

(1) Carbon dioxide and methane

C.

Cervical cap

III.

Surgical method of sterilisation in human female.

(3) Hydrogen and Carbon dioxide

D.

Vasectomy

IV.

Suppress motility of sperms

Choose the correct answer from the options given below:

(2) Methane and Hydrogen (4) Nitrogen and Oxygen

16. The

correct sequence of steps involved in polymerase chain reaction (PCR) are:

(1) A-III, B-IV, C-I, D-II

A. DNA polymerase is used to extends the primers using oligonucleotides.

(2) A-III, B-I, C-IV, D-II

B. Desired DNA fragments are denatured.

(3) A-II, B-I, C-IV, D-III

C. Amplified fragments are ligated with the vector for cloning.

(4) A-IV, B-II, C-III, D-I

12. Mammals are able to survive in Antarctica or in the Sahara desert as they are (1) Conformers

(2) Partial regulators

(3) Regulators

(4) Migrants

D. Oligonucleotides are added

Choose the correct answer from the options given below: (1) C, A, B, D

(2) A, C, B, D

(3) A, D, B, C

(4) B, D, A, C

13

CUET (UG) Exam Paper 2023

17. Select the correct statement/s from the following: A. Spermatogonia always undergo meiotic cell division. B. Primary spermatocytes divide by mitotic cell division. C. Secondary spermatocytes have 23 chromosomes and undergo second meiotic division. D. Spermatozoa are transformed into spermatids.

Choose the correct answer from the options given below:

(1) Oxygen gas

(2) CO2 gas

(3) Ethyl alcohol

(4) Pyruvic acid

24. Select the statements which do not hold true for Cancer : A. Cancerous cells show a property of contact inhibition. B. Metastasis is the most feared property of malignant tumors. C. Malignant tumor cells invade and damage the surrounding tissue. D. Malignant tumor cells grows slowly.

(1) A and C only

(2) C only

(3) D only

(4) B and C only

Choose the correct answer from the options given below:

18. Which of the following metal can not be recovered

(1) A and B only

(2) B and C only

from manual recycling process of e-wastes ?

(3) A and D only

(4) B and D only

(1) Gold

(2) Mercury

(3) Copper

(4) Nickel

19. Bt

25. (p+q)2

= p2 + 2pq+q2 =1 represents Hardy Weinberg equation. It is used in:

toxin produced by Bacillus thuringiensis is resistant to :

(1) Population genetics

(1) Drought

(2) Insects

(3) Biometrics

(3) Nematodes

(4) Viruses

(4) Molecular genetics

20. In some organisms, male has ZZ chromosomes and female has ZW chromosomes. This type of sexdetermination is found in : (1) Drosophila

(2) Hen

(3) Cockroach

(4) Snail

21. The organisation sct up by the Indian Government

(2) Mendelian genetics

26. Mature mRNA is a fully processed (1) hnRNA

(2) snRNA

(3) 28S RNA

(4) 5srRNA

27. Which forest also known as the 'lungs of the planet earth’ ?

to take decisions regarding the validity of GM research and the safety of introducing” GMorganism is:

(1) Amazon rain forest

(1) Genetic Engineering Approval Committee (GEAC)

(4) Tundra forest

(2) Genetic Modification Approval Committee (GMAC)

(2) Rain forest of north-east India (3) Tiaga forest

28. Inbreeding

is carried out in animal husbandry

because it:

(3) Indian Council of Agricultural Research (ICAR)

(1) Increase hybrid vigour

(4) All India Institute of Medical Sciences (AIIMS)

(2) Improves the breed

22. Which

of the following is a hormone releasing

IUD? (1) Multiload 375

(3) Increase heterozygosity (4) Increase homozygosity

29. Which is known as 'Terror of Bengal'.?

(2) LNG-20

(1) Water Lilly

(2) Hydrilla

(3) Cervical Cap

(3) Water hyacinth

(4) Lantana

(4) Vault

23. The puffed up appearance of dough is due to the production of:

30. Threatened animals and plants are taken out from their natural habitat and placed in special settings, protected and given special care is.

14

OSWAAL CUET (UG) Chapterwise Question Bank BIOLOGY

(1) In situ conservation

the control of emission of ozone depleting substances.

(2) Ex situ conservation

C. Use of incinerators is not crucial to dispose off hospital wastes.

(3) Conservation in national park (4) Conservation in biospheres

D. Dobson units is used to measure water quality.

31. Match List I with List II LIST I A. B.

C

D

LIST II

Dominant trait of pod colour in garden pea.

I.

The physical association of two genes on a chromosome.

II.

Polygenic traits Pleiotropy

below: (1) A and B

(2) C and D

(3) A and C

(4) B and D

35. Biofertilisers

The traits generally III. Yellow controlled by three or more genes. When a single gene exhibit multiple phenotypic expression.

IV.

Linkage

are the organisms that enrich the nutrient quality of the soil. Which of the following is not a source of biofertiliser?

(1) Bacteria

(2) Fungi

(3) Baculoviruses

(4) Cyanobacteria

36. Match List I with List II V.

LIST I

Green

Choose the correct answer from the options given below (1) A-III, B-II, C-I, D-V (2) A-II, B-III, C-V, D-I (3) A-III, B-II, C-I, D-IV (4) A-V, B-IV, C-I, D-II

32. Sequence

Choose the correct answer from the options given

the following steps in formation of

female gametophyte of flowering plants A. Cell walls are laid down. B. Formation of seven cells with eight nuclei. C. Meiosis in megaspore mother cell and formation of megaspore tetrad. D. Functional megaspore undergoes three mitotic divisions, results in formation of eight nuclei.

Choose the correct answer from the options given below: (1) A, D, C, B

(2) A, C, B, D

(3) C, D, A. B

(4) C, A, B, D

33. In

which process unusual nucleotide (methyl guanosine triphosphate) is added to the 5' end of hnRNA.

(1) Splicing

(2) Capping

(3) Tailing

(4) Transcription factor

34. Select the incorrect statements given below. A. Methane and Carbon dioxide are green house gases. B. The Montreal Protocol is associated with

LIST II

A. The primates with brain capacity of around 900 cc.

I

Neanderthal man

B. The primates with brain capacity of 1400 cc and lived in east and central Asia

II

Homo erectus

C. The primates which arose in Africa and moved across continents and developed into distinct races

III Homo sapiens

D. The primates with the brain capacities between 650-800 cc and probably did not eat meat.

IV Australopithecines

V

Homo habilis

Choose the correct answer from the options given below: (1) A-III, B-I, C-V, D-II (2) A-II, B-I, C-III, D-V (3) A-I, B-IV, C-III, D-V (4) A-I, B-III, C-IV, D-II

37. Identify

the gene which is effective against corn

bore (1) cryLAc

(2) crylAb

(3) cryILAb

(4) z gene

38. Succession occurring after flood is: (1) Hydrarch succession (2) Primary succession (3) Secondary succession (4) Mesarch succession

15

CUET (UG) Exam Paper 2023

39. Identify the statement which do not hold true for 42. Which of the following is passive immunisation? Deoxyribose Nucleic Acid.

(1) injecting inactivated pathogens

A. Only purine bases are present in DNA.

(2) injecting antigenic preparation

B. Deoxyribose sugar is present

(3) injecting preformed antibodies

C. A nitrogenous base is linked to the 1'C pentose sugar through N-glycosidic linkage.

(4) injected weakened pathogens

43.

D. Phosphate group is linked to OH of 4'C of a nucleotide.

(1) Active immunity

Choose the correct answer from the options given

(2) Passive immunity

below: (1) A and D only

(2) A and B only

(3) C and D only

(4) B and C only

40. Match List I with List II

Immunity provided to the foetus from the mother through placenta during pregnancy is :

(3) Non-specific immunity (4) Innate immunity

44. Production

of antibodies against antigens in the

body is :

LIST I

LIST II

A.

Salmonella phi

I.

Common Cold

B.

Rhmo viruse

II

Typhoid

C.

Sreptecoccus preumoniae

III Malaria

D.

Plasmodium vivax

IV Pneumonia

Choose the correct answer from the options given

(1) Innate immunity (2) Passive immunity (3) Active immunity (4) Non-Specific immunity

45. Match List I with List II LIST I

below:

LIST II

(1) A-II, B-IV, C-I, D-III

A.

Physical Barrier

I.

Saliva

(2) A-II, B-IV, C-III, D-I

B.

Cellular Barrier

II.

Interferons

(3) A-II, B-I, C-IV, D-III

C.

Physiological Barner III. Skin

(4) A-I, B-II, C-III, D-IV

D.

Cytokine Barrier

Read the passage and answer the question given below.

below: (1) A-III, B-I, C-II, D-IV (2) A-III, B-IV, C- l, D-II (3) A-I, B-IV, C-III, D-II (4) A-IV, B-I, C-III, D-II

Observe

of lactation provides passive immunity as it is rich in. (2) Prolactin

(3) Antigen

(4) IgA antibodies

the diagram and answer the question

given below.

41. Colostrum secreted by mother during initial stage (1) Placental lactogen

Polymorpho-nuclear leukocytes

Choose the correct answer from the options given

Malaria is caused by the Plasmodium parasite. The parasite is spread to humans through the bites of infected mosquitoes. When a host is exposed to antigens, which may be in the form of living or dead microbes or other proteins,antibodies are produced in the host body. This type of immunity is called active immunity. Active immunity is slow and takes time to give its full effective response. Injecting the microbes deliberately during immunisation or infectious organisms gaining access into body during natural infection induce active immunity, When antibodies are directly given to protect the body against foreign agents, it is called passive immunity.

IV.



16

OSWAAL CUET (UG) Chapterwise Question Bank BIOLOGY

46. Which of the following constitutes the first trophic level? (1) Plants

(2) Herbivores

(3) Carnivores

(4) Omnivores

47. Sparrow

material at a particular time called as : (1) Standing crop

(2) Living crop

(3) Standing life

(4) Time crop

49. Which

among the following is not a primary consumer?

comes under which of the following categories :

(1) Snake

(2) Grasshopper Cl

(1) Producers

(3) Zooplankton

(4) Caterpillar

(2) Primary consumers (3) Secondary consumers (4) Tertiary consumers

48. Each

trophic level has a certain mass of living

50. Which of the following is a part of detritus food chain? (1) Earthworm

(2) Birds

(3) Man

(4) Snake



UNIT I: Reproduction Study Time

CHAPTER

1

Max. Time: 1:5 Hours Max. Questions: 50

REPRODUCTION IN ORGANISMS can be divided into two types–Asexual and Sexual Production.

 Revision Notes

 Reproduction is biological process of formation of

new offsprings from pre-existing organisms.   Reproduction ensures continuity of species generation after generation.  Life Span is the period from birth to natural death of an organism.  Mitosis is a type of cell division, in which the nucleus divides into two daughter nuclei, each containing the same number of chromosomes as the parent nucleus.  Sex cells or gametes are mature haploid male or female germ cell, produced by gametogenesis.  These gamete cells fuse during fertilisation and lead to formation of a zygote that develops into a new individual.  Juvenility represents the stage when organism develops the capacity to reproduce. Progressive deterioration in body of a living being is called ageing.  The terminal irreversible stage of ageing is known as senescence. Mayfly lives for only one day while giant tortoise lives for about 100–150 years and considered as the longest living animal on the Earth.   Based on whether there is one or two organisms taking part in the process of reproduction the process

Scan to know

more about Asexual Reproduction and Sexual this topic Reproduction  When the offspring is produced by single parents with or without the involvement of gamete formation, Sexual and the reproduction is called asexual Asexual Reproduction reproduction.   When two parents (opposite sex) participates in reproduction process and also involves the fusion of male and female gametes, it is called sexual reproduction  Clones: Offsprings produced are genetically and morphologically similar to each other and to the parent, i.e. they are clones.  Binary fission in this method of asexual reproduction, a cell divides into two halves and rapidly grows into an adult. Ex. amoeba, paramecium.   Budding small buds are produced that remain attached initially with parents and get separated on maturation. e.g., Yeast.  Fungi and simple plants like algae reproduce through special reproductive structures like zoospores (motile structure), conidia (penicillium), buds (hydra) and gemmules (sponges).

Fig 1

2 Oswaal CUET (UG) Chapterwise Question Bank

BIOLOGY

3

REPRODUCTION IN ORGANISMS

Fig 2  Fragmentation is also a kind of asexual reproduction

in which an multicellular organism splits into fragments. Ex. Spirogyra   In plants, vegetative reproduction occurs by vegetative propagules like runner, rhizome, sucker, tuber, offset and bulb.  Asexual reproduction is the most common method of reproduction in organisms having simpler body like in algae and fungi but during unfavourable condition they shift to sexual reproduction.  Vegetative propagation is a type of process in which new plants are obtained without the production of sexual structures i.e., seeds or spores. Different vegetative parts of plants are mentioned below. { { { { { { { {

Runner- oxalis Sucker- Mint and chrysanthemum Tuber- potato Offset- water hyacinth, pistia Bulb- onion, garlic Rhizome- ginger Bulbil- agave Leaf buds- bryophyllum

 Artificial Methods: In this type of method, only a

small part of the plant organ is utilized for obtaining a new complete plant. Amongst them, the most common methods which are used are cutting, layering, and grafting.  Cutting: In cutting, a small piece of root is cut and when planted in moist soil, it will lead to the artificial inducement and development of adventitious roots. For example—lemon.  Layering: This method is used for growing rose, lemon, grape, hibiscus, and jasmine. The lower branches of these plants are bent a little bit and covered with soil in such a way that the tip of the branch protrudes from the ground and the middle part of the plant is inside the soil.

 Grafting: Grafting is carried out

Scan to know

on plants that are having difficulty more about this topic in forming roots or that generally have a weak root system.  Gametes are specialised cells in sexual reproduction. They contain What is half of the maximum number of Grafting? chromosomes of the species and fuse with another gamete to give birth to an organism with double number of chromosomes of the gametic cells.  A meiocyte is a type of cell that differentiates into a gamete through the process of meiosis. Through meiosis, the diploid meiocyte divides into four genetically different haploid gametes. The time of the meiosis through the meiotic cell cycle varies between different groups of organisms.  Sexual dimorphism is the differences in appearance between males and females of the same species. Such as colour, shape, size and structure that are caused by inheritance of one or the other sexual pattern in the genetic material.  Animal dioecy is called gonochory where the separation of sexes in different individuals can be observed. Hydra is a dioecious animal in which an individual produces either sperms or eggs.   Juvenile phase is also called vegetative phase in plant. In angiosperms, end of vegetative stage finishes when they begin to flower.  In flowering plants, both male and female gametes are non-motile. In this case, the non-motile male gametes are transferred to the non-motile female gametes with the help of pollen tube in a process called siphonogamy.  Self-pollination occurs when pollen grains are transferred from anther to stigma of the same flower.  Cross-pollination allows the transfer of pollen grains from anther of the flower into stigma of another flower.

Fig 3

4 Oswaal CUET (UG) Chapterwise Question Bank  The transfer of pollen grains from the anther to the

stigma is called pollination.

BIOLOGY

 Cross-pollination results in increase in genetic

diversity as different plants combine their genetic information to produce new offspring.

OBJECTIVE TYPE QUESTIONS [A] MULTIPLE CHOICE QUESTIONS: 1. A few statements describing certain features of reproduction are given below. i) Gametic fusion takes place. ii) Transfer of genetic material takes place. iii) Reduction division takes place. iv) Progeny have some resemblance with parents. [NCERT Exemplar, Q. 1, Pg. 1] Select the options that are true for both asexual and sexual reproduction from the options given below. (a) i and ii (b) ii and iii (c) ii and iv (d) i and iii 2. There are various types of reproduction. The types of reproduction adopted by an organism depend on: [NCERT Exemplar, Q. 15, Pg. 4] (a) the habitat and morphology of the organism. (b) morphology of the organism. (c) morphology and physiology of the organism. (d) the organism’s habitat, physiology and genetic make-up. 3. The term ‘clone’ cannot be applied to offspring formed by sexual reproduction because [NCERT Exemplar, Q. 2, Pg. 1] (a) offsprings do not possess exact copies of parental DNA. (b) DNA of only one parent is copied and passed on to the offspring. (c) offsprings are formed at different times. (d) DNA of parent and offspring is completely different. 4. Which of the following statements support the view that elaborates sexual reproductive process appeared much later in the organic evolution. [NCERT Exemplar, Q. 13, Pg. 3] i) Lower groups of organisms have simpler body design. ii) Asexual reproduction is common in lower groups. iii) Asexual reproduction is common in higher groups of organisms. iv) The high incidence of sexual reproduction in angiosperms and vertebrates. Choose the correct answer from the options given below. (a) i, ii and iii (b) i, iii and iv (c) i, ii and iv (d) ii, iii and iv 5. Offsprings formed by sexual reproduction exhibit more variations than those formed by asexual reproduction because [NCERT Exemplar, Q. 12, Pg. 3] (a) sexual reproduction is a lengthy process. (b) gametes of parents have qualitatively different genetic compositions. (c) genetic material comes from parents of two different species. (d) greater amount of DNA is involved in sexual reproduction. 6. A few statements with regard to sexual reproduction are given below. [NCERT Exemplar, Q. 4, Pg. 1] i) Sexual reproduction does not always require two individuals. ii) Sexual reproduction generally involves gametic fusion. iii) Meiosis never occurs during sexual reproduction.

iv) External fertilisation is a rule during sexual reproduction. Choose the correct statements from the options below. (a) (i) and (iv) (b) (i) and (ii) (c) (ii) and (iii) (d) (i) and (iv) 7. Given below are a few statements related to external fertilisation. [NCERT Exemplar, Q. 7, Pg. 2] i) The male and female gametes are formed and released simultaneously. ii) Only a few gametes are released into the medium. iii) Water is the medium in a majority of organisms exhibiting external fertilisation. iv) Offsprings formed as a result of external fertilisation have better chances of survival than those formed inside an organism. Choose the correct statements from the given options. (a) (iii) and (iv) (b) (i) and (iii) (c) (ii) and (iv) (d) (i) and (iv) 8. Which of the following is a post-fertilisation event in flowering plants? [NCERT Exemplar, Q. 17, Pg. 4] (a) Transfer of pollen grains (b) Embryo development (c) Formation of flower (c) Formation of pollen grains 9. A multicellular, filamentous alga exhibits a type of sexual life cycle in which the meiotic division occurs after the formation of zygote. The adult filament of this alga has [NCERT Exemplar, Q. 5, Pg. 2] (a) haploid vegetative cells and diploid gametangia. (b) diploid vegetative cells and diploid gametangia. (c) diploid vegetative cells and haploid gametangia. (d) haploid vegetative cells and haploid gametangia. 10. Choose the correct statement from amongst the following. [NCERT Exemplar, Q. 13, Pg. 3] (a) Dioecious (hermaphrodite) organisms are seen only in animals. (b) Dioecious organisms are seen only in plants. (c) Dioecious organisms are seen in both plants and animals. (d) Dioecious organisms are seen only in vertebrates. 11. The male gametes of rice plant have 12 chromosomes in their nucleus. The chromosome number in the female gamete, zygote and the cells of the seedling will be respectively [NCERT Exemplar, Q. 6, Pg. 2] (a) 12, 24, 12. (b) 24, 12, 12. (c) 12, 24, 24. (d) 24, 12, 24. 12. The statements given below describe certain features that are observed in the pistil of flowers. i) Pistil may have many carpels. ii) Each carpel may have more than one ovule. iii) Each carpel has only one ovule. iv) Pistils have only one carpel. Choose the statements that are true from the options below. (a) (i) and (ii) (b) (i) and (iii) (c) (ii) and (iv) (d) (iii) and (iv)

5

REPRODUCTION IN ORGANISMS

13. Which of the following situations correctly describe the similarity between an angiosperm egg and a human egg? i) Eggs of both are formed only once in a lifetime. ii) Both the angiosperm egg and human egg are stationary. iii) Both the angiosperm egg and human egg are motile transported. iv) Syngamy in both results in the formation of zygote. Choose the correct answer from the options given below. (a) (ii) and (iv) (b) (iv) only (c) (iii) and (iv) (d) (i) and (iv) 14. The number of chromosomes in the shoot tip cells of a maize plant is 20. The number of chromosomes in the microspore mother cells of the same plant shall be (a) 20. (b) 10. (c) 40. (d) 15. 15. There is no natural death in single-celled organisms, like Amoeba and bacteria, because (a) they cannot reproduce sexually. (b) they reproduce by binary fission. (c) parental body is distributed amongst the offsprings. (d) they are microscopic. 16. Amoeba and Yeast reproduce asexually by fission and budding respectively, because they are (a) microscopic organisms. (b) heterotrophic organisms. (c) unicellular organisms. (d) uninucleate organisms. 17. Appearance of vegetative propagules from the nodes of plants, such as sugarcane and ginger, are mainly because (a) nodes are shorter than internodes. (b) nodes have meristematic cells. (c) nodes are located near the soil. (d) nodes have non-photosynthetic cells. 18. Identify the incorrect statement. (a) In asexual reproduction, the offsprings produced are morphologically and genetically identical to the parent. (b) Zoospores are sexual reproductive structures. (c) In asexual reproduction, a single parent produces offspring with or without the formation of gametes. (d) Conidia are asexual structures in Penicillium. 19. Match the organisms given in Column A with the vegetative propagules given in Column B. Column A (Organisms)

Column B (Vegetative propagules)

A. Bryophyllum

i) Offset

B. Agave

ii) Eyes

C. Potato

iii) Leaf buds

D. Water hyacinth

iv) Bulbils

(a) A. (ii) B. (iii) C. (i) D. (iv) (b) A. (iii) B. (iv) C. (ii) D. (i) (c) A. (iv) B. (i) C. (iii) D. (ii) (d) A. (iii) B. (ii) C. (iv) D. (i) 20. Which one of the following is correctly matched? (a) Chlamydomonas-Conidia (b) Yeast-Zoospores (c) Onion-Bulb (d) Ginger-Sucker

21. A protozoan reproduces by binary fission. What will be the number of protozoans in its population after six generations? (a) 128 (b) 24 (c) 64 (d) 32 22. Which of the following is not true- (a) Water Hyacinth growing in standing water drains oxygen from water that leads to death of fishes. (b) Offsprings produced by asexual reproduction are called clone. (c) Microscopic, motile asexual reproductive structure called zoospore. (d) In potato , Banana and ginger the plants arise from internode present in the modified stem. 23. Planaria possesses high capacity of – (a) Regeneration (b) Alternation of generation (c) Bioluminescence (d) Metamorphosis 24. Which of the following is not a vegetation propaganda? (a) Runner of grass (b) Offset of water hyacinth (c) Rhizome of Ginger (d) Bud of Hydra 25. Flagellated, motile asexual reproductive structure are called : (a) Megaspores (b) Aplanospores (c) Zoospores (d) Microspores. 26. Which of the following flowers only once in the life time(a) Mango (b) Bamboo species (c) Papaya (d) Jackfruit 27. Which of the following statement is true of date palm? (a) It is monoecious producing both staminate and pistillate flowers in the same plant. (b) It is monoecious producing staminate flower in one and pistillate flower on the another tree (c) It is dioecious producing staminate flower in one and pistillate flower on the another tree (d) None of these 28. The first organisms were (a) autotrophs. (b) eukaryotes. (c) chemoautotrophs. (d) chemo-heterotrophs 29. Which one of the following part of the plant when put into the soil is likely to produce new offspring? (a) part of an internode (b) a stem cutting with a node (c) part of a primary root (d) a flower 30. In sporulation Amoeba undergoes: (a) Binary fission (b) Multiple fission (c) Encystation (d) None of these [B] ASSERTION & REASON Directions: In the following questions a statement of assertion (A) is followed by a statement of reason (R). Mark the correct choice as:

6 Oswaal CUET (UG) Chapterwise Question Bank (1) Both assertion (A) and reason (R) are true and reason (R) is the correct explanation of assertion (A). (2) Both assertion (A) and reason (R) are true but reason (R) is not the correct explanation of assertion (A). (3) Assertion (A) is true but reason (R) is false. (4) Assertion (A) is false but reason (R) is true. 1. Assertion: Small plantlet emerge from the buds (eye) of potato tubers, rhizome of ginger etc,: the site of origin of these plantlets , is invariably the node. Reason: The nodes contain metameric cells, which help in their vegetative propagation. 2. Assertion (A): Hybrids result from a cross between two genetically unlike parents. Reason (R): Hybrid vigour is the superiority of hybrid over either of the parents. 3. Assertion: Several seed bearing plants propagate vegetatively. Reason: Sweet potatoes undergo vegetative propagation by means of root tubers. 4. Assertion: Zygote is the only cell that gives a vital link between two generation of an organisms. Reason: The two gamete fuse to form a single zygote. 5. Assertion: Papaya is a dioecious plant. Reason: Male and female sex organ are born on same plant. 6. Assertion : The life span plant is long then the animals.. Reason: Plant have meristematic tissue to divide continuously through their life. 7. Assertion: Asexual reproduction involves formation of clones of an organism. Reason: Clones are morphologically and genetically identical. 8. Assertion: Plant structure such as tubers, bulbs, corns etc. have asexually reproducing method. Reason: They are common means of propagation in most of the monocot families. 9. Assertion: A plant can be retained and multiplied indefinitely without any change variation through asexual reproduction. Reason: Asexual reproduction does not involve meiosis and syngamy. 10. Assertion: Sponges are simple , cellular level and less specialize organisms. Reason: Sponges shows high power of regeneration. [C] COMPETENCY/CASE-BASED QUESTIONS: I. Read the passage carefully and answer the question given after the passage from 1–5. Organisms like fungi, plants and animals shows diversity. They differ greatly in external as well as internal structures,organisations, their anatomy and physiology. Inspite of that they lived together and have almost similar pattern of reproduction. They have different life forms, life span and individuality. There are different types of reproduction to produce new organisms, like asexual reproduction (in which only one parent is involved to produce offspring), and sexual reproduction (in which two different parents with different genetic compositions are involved). Due to living in large biological diversities, each organism undergoes different mechanisms to reproduce

BIOLOGY

1. Arrange the following in animals with their life spanAnimal name Life Span 1. Banyan tree A. 20-25 years 2. Cow B. 140 years 3. Parrot C. 60 years 4. Crocodile D. 25 years (a) 1-D, 2-A, 3-B, 4- C (b) 1-B 2-A, 3-D, 4- C (c) 1-D, 2-A, 3-C, 4- B (d) 1-D, 2-C, 3-A, 4- B 2. Which of the following is an example of perennial plant? (a) Pea (b) Watermelon (c) Banana (d) none of these 3. Vegetative phase of plant is also called(a) juvenile phase (b) reproductive phase (c) Senescent phase (d) none of these 4. Which of the following statement is false(a) Hydra reproduces through regeneration (b) tuber and corm formation are the method of asexual reproduction (c) Spirogyra reproduce through budding (d) spores in unfavorable condition form cyst around them 5. The asexually reproduced organisms are(a) genetically identical (b) morphologically identical (c) a single parent involved (d) all of these II. Study the following diagram and answer the question given below 6–10.

6. Which type of reproductive bodies seen in the given diagram? (a) Cyst (b) conidia formation (c) budding (d) regeneration 7. Match the sexual reproductive structure in Column I with the organism given in Column II and select the correct optionColumn I (reproductive structure) Column II (Organism) A. Conidia (1) Chlamydomonas B. Gemmules (2) Hydra C. Zoospores (3) Penicillium D. Buds (4) Sponges (5) Banana (a) A-3,  B-4,  C-1,  D-2 (b) A-4,  B-3,  C-2,  D-1 (c) A-2,  B-4,  C-4,  D-2 (d) A-3,  B-4,  C-5,  D-1 8. One of the following is not a method of asexual reproduction : (a) Cutting (b) Grafting (c) Budding (d) Conjugation.

7

REPRODUCTION IN ORGANISMS

10. Asexual reproductive structures of sponge is : (a) Gemmules (b) Conidia (c) Bulb (d) None of these

9. Reproduction occurs through conidia in(a) Paramecium (b) Plasmodium (c) Amoeba (d) Penicillium

ANSWER KEY [A] MULTIPLE CHOICE QUESTIONS 1. (c)

2. (d)

3. (a)

4. (c)

5. (b)

6. (b)

7. (b)

8. (b)

9. (d)

10. (c)

11. (c)

12. (a)

13. (b)

14. (a)

15. (c)

16. (c)

17. (b)

18. (b)

19. (b)

20. (c)

21. (c)

22 (d)

23. (a)

24. (d)

25. (c)

26. (b)

27. (c)

28. (d)

29. (b)

30. (c)

8. (a)

9. (a)

10. (b)

9. (d)

10. (a)

[B] ASSERTION & REASON 1. (a)

2. (b)

3. (b)

4. (a)

5. (c)

6. (a)

7. (b)

[C] COMPETENCY/CASE-BASED QUESTIONS 1. (b)

2. (c)

3. (a)

4. (c)

5. (d)

6. (b)

7. (A)

8. (d)

ANSWERS WITH EXPLANATION [A] MULTIPLE CHOICE QUESTION: 1. Option (c) is correct Explanation: In both types of reproduction,transfer of genetic materials takes place from one generation to another and also progeny have some resemblance with their parents. Gametic fusion (fertilisation of sperm and ovum to form zygote) and reduction division (also called meiosis) are present only in sexual mode of reproduction. 2. Option (d) is correct Explanation: There are different types of reproduction to produce new organisms, like asexual reproduction (in which only one parent is involved to produce offspring), and sexual reproduction (in which two different parents with different genetic compositions are involved). Due to living in large biological diversities, each organism undergoes different mechanisms to reproduce. They all depend on their habitat, physiological properties, genetic constitution and other factors to reproduce and produce new organism. 3. Option (a) is correct Explanation: Clone cannot be applied to offsprings produced from sexual reproduction because sexual reproduction involves the fusion of male and female gametes from two parents. Therefore, the offsprings produced from them are not identical to the parents. This genetic recombination leads to variations which play an important role in evolution. Whereas offsprings produced as a result of asexual reproduction are identical to each other, both physically as well as genetically. They are the exact copies of their parents. Hence, they are called clones. 4. Option (c) is correct Explanation: Statements (i), (ii) and (iv) support the view that elaborates sexual reproductive process, appeared much later in the organic evolution. Asexual reproduction is not common in higher group of organisms. It is common amongst single-celled organisms, and also in plants and animals with relatively simple organisations. 5. Option (b) is correct Explanation: More complex offsprings are formed by sexual reproduction as compared to asexual reproduction. Offsprings produced by sexual reproduction exhibit more variation than those formed by asexual reproduction because gametes of parents have qualitatively different genetic compositions. Due to the genetic variations, individuals within a population have slight differences. In case of asexual reproduction, due to involvement of only one parent there is no chance of genetic variations.

6. Option (b) is correct Explanation: Sexual reproduction involves both male and female gametes (either from same or different parents) and its fusion results in the formation of new organisms. Meiosis (reduction division) takes place during sexual reproduction to produce haploid gametes. It is important to keep the number of chromosomes in species constant. In sexual reproduction, fertilisation occurs externally as well as internally. 7. Option (b) is correct Explanation: External fertilisation occurs outside the female body, usually in water. It takes place in most aquatic organisms, such as a majority of algae and fishes as well as amphibians. During external fertilisation, organisms release large number of gametes in the surrounding medium to increase the chances of syngamy. A major disadvantage is that the offsprings are extremely vulnerable to predators threatening their survival up to adulthood. 8. Option (b) is correct Explanation: During the sexual reproduction, the events which take place after the formation of zygote are called postfertilisation events. The process of embryo development from the zygote (called embryogenesis) takes place after the fertilisation. In all flowering plants, the zygote is formed inside the ovule. In ovule, the zygote divides several times to form an embryo. 9. Option (d) is correct Explanation: Adult filament of multicellular alga have haplontic life cycle in which the meiotic division occurs after the formation of zygote. So, the filament of this alga has haploid vegetative cells and haploid gametangia. A multicellular gametophyte or gametangia which is haploid (n) alternates with a multicellular sporophyte which is diploid (2n). A mature sporophyte produces spores (haploid cells) by meiosis, a process which reduces the number of chromosomes to half, from 2n to n. 10. Option (c) is correct Explanation: Dioecious organisms are those organisms which have male and female reproductive organs in different individuals. These organisms are seen in both plants and animals. In dioecious angiosperms, male and female parts are found in separate flowers e.g. Mulberry. Hence, dioecious plants bear sexual dimorphism. Most animals are dioecious. Insects, spiders, fish, amphibians, reptilians, birds and mammals exhibit sexual dimorphism.

8 Oswaal CUET (UG) Chapterwise Question Bank 11. Option (c) is correct Explanation: In female gamete, the chromosome number will be same as that of the male gamete (12). A zygote is a fertilised egg/seed which means gametes from the parents have been combined (diploid) and thus, the chromosome number will be 24 (2n). A seedling is a young plant. Sporophyte develops out from a plant embryo from a seed. So, the chromosome number in the cells of the seedlings will be 24 (2n), which will further give rise to new diploid individual. Female gamete (n) = 12 Zygote (2n) = 24 Cells of seedling (2n) = 24 12. Option (a) is correct Explanation: (a) Pistil is the female reproductive part (gynoecium) of a flower. It is centrally located and consists of ovary (a swollen base), which contains the potential seeds or ovules; style (a stalk), arising from the ovary; and a pollen receptive tip, the stigma. Inside ovary it is attached to a parenchymatous cushion called placenta, either singly or in cluster. 13. Option (b) is correct Explanation: Both angiosperms and humans exhibit internal fertilisation and remain reproductively active throughout their reproductive phase. It means that the formation of egg takes place not only once, but also many times in a life time. In humans, egg released from the ovary reaches the Fallopian tube (for fertilisation) and from there to uterus for implantation. So, the egg is considered as motile not stationary. Similarly, in flowering plants (angiosperms), the gametes are non-motile cells within gametophytes, but for the fusion to take place the non-motile male gametes are carried to female gamete by pollen tubes. 14. Option (a) is correct Explanation: The whole plant body of maize plant including shoot tip cells remains in diploid (2n) condition. As the microspore mother cell is a part of reproductive organ, the chromosome number in these cells will remain same as the individual, that is, 2n = 20. These microspore mother cells are further responsible for producing male gametes, that is, haploid (n) by reduction division. 15. Option (c) is correct Explanation: There is no natural death in single-celled organisms, like Amoeba and bacteria, because parental body is distributed amongst the offsprings. In Amoeba and bacteria, the parental body itself divides into two parts and becomes daughter cells. Then, each of these daughter cells rapidly grow into adult organisms. Therefore, the parent organism cannot be said to have died, it continues living as daughter cells. 16. Option (c) is correct Explanation: Asexual reproduction is common in unicellular organisms due to their simple cell organisation. These organisms carry out all of their life processes as one single cell. All prokaryotes, fungi and protists (e.g., Amoeba, etc.) are unicellular organisms. Fission and budding are types of asexual reproduction in which a single parent will reproduce new organisms. Amoeba and Yeast reproduce by fission (binary fission) and budding respectively. In binary fission, parent cell divides into two equal halves (called daughter cells). Daughter cells are identical to each other and to their parent cell, for example, Amoeba, Euglena and bacteria. In budding, a new organism is

BIOLOGY

formed from a small part (called as bud) of the parent’s body. Then this bud detaches from the parent body and develops into a new organism, for example, Hydra and Yeast. 17. Option (b) is correct Explanation: Appearance of vegetative propagules from the nodes of plants, such as sugarcane and ginger, are mainly because of the presence of meristematic cell in nodes. Meristematic cells are responsible to control the growth and development of tissues and organs in plants. Nodes (present in the modified stems) when come in contact with damp soil or water, they produce roots and gives rise to new plants. 18. Option (b) is correct Explanation: Zoospores are microscopic motile asexual reproductive structures. They are produced by certain algae and some fungi. They move with the help flagella. Other common asexual reproductive structures are conidia (penicillium), buds (Hydra) and gemmules (sponge). 19. Option (b) is correct The correct matches of the vegetative propagules are given below: S. No. Column A(Organisms) Column B (Vegetative propagules) A. Bryophyllum iii) Leaf buds B. Agave iv) Bulbils C. Potato ii) Eyes D. Water hyacinth i) Offset Column A (Organisms)

Column B (Vegetative propagules)

A. Bryophyllum

iii) Leaf buds

B. Agave

iv) Bulbils

C. Potato

ii) Eyes

D. Water hyacinth

i) Offset

20. Option (c) is correct Explanation: The bulbs are underground condensed shoots which have one or more buds that give rise to new plants. Onion is vegetatively propagated by bulbs. 21. Option (c) is correct Explanation : Binary fission is a mode of asexual reproduction in protozoan through which it produces two offsprings from parent individual. So, In 1st generation : there are 2 protozoans. In 2nd generation : there are 2 × 2 = 4 protozoans In 3rd generation : there are 4 × 2 = 8 protozoans In 4th generation : there are total 16 protozoans. In 5th generation : there are total 32 protozoans In 6th generation : there are 64 protozoans. Thus, the population of protozoan will be 64, after six generations. 22. Option (d) is correct Explanation: In potato, Banana and ginger new plantlets arise from nodes of the modified stem not from internode. 23. Option (a) is correct Explanation: Planaria is an organism which shows high capacity of regeneration in which the cutted part or damaged part form again. 24. Option (d) is correct Explanation: Considering the misprinted words vegetation propaganda as vegetative propagules. Vegetative propagation is a form of asexual reproduction. It is a process in which plants reproduce from stem, roots and leaves.

9

REPRODUCTION IN ORGANISMS

In plants, the units of vegetative propagation are runner, rhizome, sucker, tuber, offset, bulb etc. as they are capable of giving rise to new offspring. Buds in Hydra are asexual reproductive structures.

6. Option (a) is correct Explanation: Animal body have limited body growth while plant have meristematic tissue at the tip which has division capacity throughout life.

25. Option (c) is correct.

7. Option (b) is correct.

Explanation: Zoospore is a motile, a sexual spore, which uses its flagella for locomotion. Example : Euglena.

Explanation: (b) The reproduction is known as asexual reproduction, when an offspring is produced by a single parent without the involvement of gamete formation. As a result, the offspring that are produced are not only similar to one another but are also exact copies of their parent. Such a group of morphologically and genetically similar individuals are called clones.

26. Option (b) is correct Explanation: Bamboo species have a life cycle of about 80100 years and generally flowers only once in its lifetime. So, the correct answer is option b. 27. Option (c) is correct Explanation: Date palm is dioecious producing staminate (male) flowers in one tree and pistillate (female) flowers in another tree. 28. Option (d) is correct Explanation: The first living beings were single-celled prokaryotes, like bacteria. These living beings were present in the environment of soupy sea having abundant organic molecules. Nutritionally, they were chemo-heterotrophs. 29. Option (b) is correct Explanation: The plant when put into the soil is likely to produce new offspring by a stem cutting with a node, this type of propagation is called layering. 30. Option (c) is correct Explanation: In sporulation Amoeba undergoes encystation due to unfavorable condition around them, whenever it get favorable condition it comes out from this condition and germinate.

8. Option (a) is correct Explanation: (a) Asexual reproduction produces individuals that are genetically identical to the parent plant. Roots such as corms, stem tubers, rhizomes, and stolon undergo vegetative reproduction. They are usually found in monocotyledonous families. 9. Option (a) is correct Explanation: (a) In asexual mode of reproduction, the genetic constituent remains the same. So the offspring and parents are morphologically and genetically same. 10. Option (b) is correct Explanation: Sponges are non-motile animals attached to some solid support. The body design involves very minimal differentiation and division into tissues. [C] COMPETENCY/CASE-BASED QUESTIONS: 1. Option (b) is correct Explanation: Animal name

Life Span

1. Banyan tree

A. 140 years

[B] ASSERTION & REASON:

2. Cow

B. 20-25 years

1. Option (a) is correct

3. Parrot

C. 25 years

Explanation: Small plantlet emerge from the buds (eye) of potato tubers, rhizome of ginger etc,: the site of origin of these plantlets, is invariably the node which contain metameric (dividing cells) cells, which help in their vegetative propagation

4. Crocodile

D. 60 years

2. Option (b) is correct. Explanation: Hybrid vigour is superior in characteristic such as size, growth rate, fertility, etc., of a hybrid organism over it's parents. Hybrid, on the other hand is a result of cross between parents of two different types. 3. Option (b) is correct Explanation: Formation of new plants by means of vegetative units as tubers, buds, rhizomes is called vegetative propagation. It is useful for producing large number of offsprings within a short time and for preserving qualities such as disease resistance. In sweet potato, root tubers take part in vegetative propagation. 4. Option (a) is correct Explanation: Zygote forms after fusion of two gamete (male and female). It ensure the characteristics of both the parents. 5. Option (c) is correct Explanation: Papaya is dioecious plant as it has male and female reproductive part in different plants,

2. Option (c) is correct Explanation: Perennial plants grow again and again , have shorter blooming period. They can live for two or more years. So, Banana is an example of perennial plant rest two are annual plants. 3. Option (a) is correct Explanation: Juvenile phase is the phase of plant from birth to reproductive maturity, this phase shows full development of the plant body, hence also called vegetative phase. 4. Option (c) is correct Explanation: Spirogyra is type of algae which reproduces through fragmentation. 5. Option (d) is correct Explanation: The asexually reproduced organisms are genetically identical, morphologically identical and only single parent involved, hence all are correct. 6. Option (b) is correct. Explanation: Conidium or conidia is an asexual reproductive bodies found in fungi which is situated at the extreme side of the top of the hyphae.

10 Oswaal CUET (UG) Chapterwise Question Bank 7. Option A is correct Explanation: Column I (reproductive structure) Column II (Organism) A. Conidia

(1) Penicillium

B. Gemmules

(2) Sponges

C. Zoospores

(3) Chlamydomonas

D. Buds

(4) Hydra

8. Option (d) is correct Explanation: Cutting, grafting, and budding all are asexual

24

BIOLOGY

method of reproduction while conjugation is a sexual method in which genetic material transfer between bacterial cells. 9. Option (d) is correct Explanation: Paramecium , Amoeba and plasmodium reproduce through fission. While penicillium reproduce through spores like conidia. 10. Option (a) is correct Explanation: Gemmules are the internal buds whenever they attain maturity ,they detach from parental body and survive as new invidual.

Study Time

CHAPTER

2

Max. Time: 1:5 Hours Max. Questions: 50

SEXUAL REPRODUCTION IN FLOWERING PLANTS

 Revision Notes Flower Morphology

 Flowers are fascinating to biologists due to their intricate morphological and embryological features. The structure of a typical flower can be described in several parts, each with its own specific function in reproduction. Here’s an overview of the main parts of a flower: 1. Receptacle: The receptacle is the tip of the stem to which all the flower parts are attached. It is the base or foundation of the flower. 2. Sepals: Sepals are the outermost part of the flower and are usually green, although they can be colored in some species. Sepals protect the flower during its bud stage. Collectively, the sepals are known as the calyx. 3. Petals: Petals are often brightly colored and are located just inside the sepals. They serve to attract pollinators like bees and butterflies. Collectively, the petals are known as the corolla. 4. Stamens (Androecium): Stamens are the male reproductive parts of the flower. Each stamen typically consists of two main parts: { Filament: This is a thin, elongated stalk that supports the anther. { Anther: The anther is the top part of the stamen, and it contains pollen sacs. Pollen grains are produced in these sacs. 5. Pistil (Gynoecium): The pistil is the female reproductive part of the flower, and it usually consists of three parts: { Stigma: The stigma is the sticky, often knob-like structure at the top of the pistil. It serves as the landing platform for pollen. { Style: The style is a slender tube-like structure that connects the stigma to the ovary. { Ovary: The ovary is the swollen base of the pistil, and it contains the ovules. Once fertilization occurs, the ovary develops into a fruit. 6. Ovules: Ovules are tiny structures within the ovary that contain the female gametes (eggs). These are the structures that will eventually become seeds when fertilized. 7. Nectaries: Some flowers have nectaries, which are specialized structures that produce nectar. Nectar attracts pollinators by providing them with a food source. 8. Pollen Grains: Pollen grains are produced in the anthers of the stamen. These tiny grains contain the male gametes (sperm cells) needed for fertilization.

9. Floral Axis: The floral axis is the central axis of the flower, and it is where all the floral parts are attached. It extends from the receptacle to the tip of the floral bud. 10.  Pedicel: The pedicel is a small stalk that connects the flower to the main stem of the plant. It’s especially important when flowers are borne on stalks rather than directly on the main stem. These are the basic structural components of a flower. The arrangement and characteristics of these parts can vary greatly between different species of plants, leading to the wide diversity of flowers seen in nature. Each part plays a specific role in the flower’s reproductive process, ensuring the plant’s ability to produce seeds and propagate itself.

Pre-fertilization: Structures and Events { Prior

to the visible appearance of a flower, the plant undergoes hormonal and structural changes, leading to the development of the floral primordium. { Inflorescences are formed, bearing floral buds that eventually become flowers. { Within the flower, the male and female reproductive structures, the androecium (stamens) and the gynoecium (female reproductive parts), differentiate and develop. {  The androecium comprises stamens representing the male reproductive organ, while the gynoecium represents the female reproductive organ.

Anther { A

typical stamen consists of two parts: the slender filament and the terminal bilobed structure known as the anther. { The anther’s proximal end attaches to the thalamus or petal of the flower. { The number and length of stamens vary among different flower species.

Fig 1: Transverse section of a young anther

12 Oswaal CUET (UG) Chapterwise Question Bank Oswaal CBSE Question Bank Chapterwise & Topicwise, BIOLOGY, Class-XII 2

BIOLOGY

13

SEXUAL REPRODUCTION IN FLOWERING PLANTS { Stamens

from various species exhibit significant variations in size, shape, and anther attachment.

Microsporangium { A typical

angiosperm anther is bilobed, with each lobe containing two microsporangia (dithecous). { Microsporangia develop into pollen sacs, which are filled with pollen grains. { A transverse section of a micro­sporangium reveals four wall layers: epidermis, endothecium, middle layers, and tapetum.

{ The

tapetum nourishes developing pollen grains and has dense cytoplasm with multiple nuclei. { Sporogenous tissue in the microsporangium undergoes meiotic divisions to form microspore tetrads, with each tetrad containing potential pollen or microspore mother cells. { This process of microspore formation from a pollen mother cell through meiosis is known as microsporogenesis. {  Microspores within each microsporangium cluster as tetrads and mature into individual pollen grains. Scan to know more about this topic

Formation of Male Gametes

Fig 2: Enlarged view of an microsporangium

Fig 3: Structure of two-celled male gametophyte (pollen grain)

Meiosis I

Microspore mother cell

Meiosis II

Microspore dyad

Microspore tetrad

Fig 4: Microsporogenesis

Vacuoles

Vegetative Cell

Nucleus Generative cell Asymmetric Spindle

Fig 5: Stages of a microspore maturing into a pollen grain

14 Oswaal CUET (UG) Chapterwise Question Bank Pollen Grain { Pollen

Scan to know more about this topic

grains are the male game­ tophytes. { They typically have a spherical shape with sizes ranging from 2550 micrometers in diameter. Formation of The structure of a pollen grain consists of Pollen several layers, each with distinct functions. These layers include: 1. Exine: The outermost layer of the pollen grain is called the exine. It is a tough and resistant layer primarily made up of a substance called sporopollenin. The exine protects the delicate contents of the pollen grain from environmental factors, such as UV radiation and desiccation. It also provides structural support. 2. Germ Pores: The exine often features small openings called germ pores. These pores are essential for the germination of the pollen grain when it lands on the stigma of a flower. The sporopollenin layer is absent or thinner around these pores, allowing for the emergence of the pollen tube. 3. Intine: Beneath the exine, there is a layer called the intine. The intine is the innermost layer of the pollen grain wall. It is much thinner than the exine and is composed mainly of cellulose and pectin. The intine is responsible for facilitating the growth of the pollen tube after pollination. It provides a path for the pollen tube to penetrate through to reach the ovule within the flower’s ovary. 4. Cytoplasm: Inside the intine, there is a small amount of cytoplasm. This cytoplasm contains the two cells found in most mature pollen grains: { Vegetative Cell: This cell is larger and contains abundant food reserves. It also has a large, irregularly shaped nucleus. { Generative Cell: The generative cell is smaller and floats within the cytoplasm of the vegetative cell. It contains dense cytoplasm and a nucleus. In many flowering plants, this cell undergoes mitosis to form two sperm cells before fertilization. The unique structure of pollen grains, with their tough exine, germ pores, and inner layers, allows them to withstand various environmental challenges during pollination and fertilization. The pollen tube, which grows from the germinating pollen grain, plays a critical role in delivering sperm cells to the female reproductive structures of the flower for fertilization.

Pollen Viability and Preservation

BIOLOGY

(syncarpous) or free (apocarpous). pistil comprises three parts: stigma, style, and ovary. { The stigma serves as the landing platform for pollen grains, the style is an elongated structure beneath the stigma, and the ovary has the ovarian cavity (locule). { The placenta inside the ovary gives rise to megasporangia, also known as ovules. { Each

The Megasporangium (Ovule) { The

ovule is attached to the placenta via a stalk called the funicle. { The body of the ovule fuses with the funicle at the hilum. { Integuments protect the ovule and encircle the nucellus, leaving a micropyle at the tip. { The chalaza represents the ovule’s basal part. { Inside the ovule, the nucellus houses the embryo sac or female gametophyte, generally formed from a single megaspore.

Megasporogenesis {  A

megaspore mother cell (MMC) undergoes meiotic division to produce four megaspores. { One of these megaspores becomes functional, while the others degenerate. { The functional megaspore develops into the female gametophyte (embryo sac) through a process known as monosporic development.

Female Gametophyte { The

functional megaspore divides mitotically to form the 2-nucleate embryo sac. { Subsequent mitotic divisions lead to the 4-nucleate and then the 8-nucleate stages. { Cell walls form after the 8-nucleate stage, organizing the typical 7-celled female gametophyte or embryo sac. { The embryo sac consists of cells at the micropylar end (egg apparatus), cells at the chalazal end (antipodals), and a central cell with polar nuclei. { The egg apparatus includes synergids and an egg cell. { The central cell contains polar nuclei. { The micropyle serves as the site for pollen tube entry for fertilization.

{ The

period of pollen grain viability varies based on temperature and humidity. { Some pollen grains lose viability within minutes of release, while others remain viable for months. { Pollen grains can be stored for extended periods in liquid nitrogen (-196°C) for use in breeding programs.

The Pistil { The gynoecium represents the female reproductive part

of the flower. can consist of a single pistil (monocarpellary) or multiple pistils (multicarpellary), either fused

{ It

Fig 6: A diagrammatic view of a typical anatropous ovule

15

SEXUAL REPRODUCTION IN FLOWERING PLANTS Pollen

Chalazal end Antipodals

Polar nuclei Central cell Egg cell Synergids

Micropylar end

]

Egg apparatus

Filiform apparatus

Fig 7: A diagrammatic view of the mature enmbryo

Pollination { Definition:

Pollination is the process of transferring pollen grains from the anther of a flower to the stigma of the same or a different flower, facilitating fertilization. { Types of Pollination: 1. Autogamy: Pollination occurs within the same flower, where pollen from the anther fertilizes the stigma of the same flower. 2. Geitonogamy: Pollination involves transferring pollen from the anther to the stigma of another flower on the same plant. 3. Xenogamy: Pollination involves transferring pollen from the anther to the stigma of a flower on a different plant, promoting genetic diversity. {  Agents of Pollination: Plants use various agents to achieve pollination, including: Biotic Agents: Animals such as bees, butterflies, flies, beetles, wasps, ants, moths, birds (sunbirds and hummingbirds), and bats. Abiotic Agents: Wind and water (less common). { Examples of Pollination by Biotic Agents: Bees: Dominant pollinators among insects; attracted to colorful, fragrant flowers rich in nectar.  Flies and Beetles: Attracted to flowers with foul odors.  Birds and Bats: Visit specific types of flowers, often brightly colored and rich in nectar. { Outbreeding Devices: Mechanisms to prevent selfpollination and encourage cross-pollination. Synchrony: Timing mismatch between pollen release and stigma receptivity. Spatial Separation: Anther and stigma placed at different positions within the flower. Self-Incompatibility: Genetic mechanism preventing self-pollen from fertilizing ovules. Unisexual Flowers: Some plants have separate male and female flowers, preventing self-pollination. { Pollen-Pistil Interaction: The process by which the pistil accepts or rejects pollen based on compatibility. Pollen Recognition: Pistil recognizes compatible pollen, allowing it to germinate on the stigma.

Germination: Pollen grains form a pollen tube, which grows through the stigma and style. Pollen Tube Entry: The tube enters the ovary through the micropyle and reaches the ovule. { Artificial Hybridization: Breeding technique used to create desired hybrids by manually controlling pollination. Emasculation: Removal of anthers from bisexual flowers to prevent self-pollination. Bagging: Covering emasculated or female flowers with bags to protect them from unwanted pollen. Pollination: Controlled pollination is performed using desired pollen, and the flowers are rebagged to prevent contamination. Observing pollen germination can be done by placing pollen grains on a glass slide with sugar solution and observing the pollen tubes under a microscope.

Double Fertilization { Double

Fertilization: A unique Scan to know reproductive process in flowering more about this topic plants where two fertilization events occur within the embryo sac. { Syngamy: The first fertilization event where one male gamete fuses with the egg cell to form a Double Fertilization in diploid zygote. Angiosperms { Triple Fusion: The second fertilization event where the other male gamete fuses with two polar nuclei to form a triploid primary endosperm nucleus (PEN). { Double Fertilization Significance: It ensures the formation of both the embryo and endosperm in the same embryo sac, a crucial adaptation in flowering plants.

Post-Fertilization: Structures and Events Endosperm { Endosperm

Development: Occurs before embryo development and provides nourishment to the growing embryo. { Types of Endosperm Development: Free-Nuclear Endosperm: Formation of free nuclei followed by cellularization; found in coconut. Cellular Endosperm: Endosperm cells form walls; varies in the number of nuclei. { Endosperm Fate: Can be completely consumed by the embryo (e.g., peas) or persist in the mature seed (e.g., castor, coconut).

Embryo { Embryo

Development: Occurs at the micropylar end of the embryo sac, after endosperm formation. { Embryonic Stages: Common stages in both monocots and dicots: proembryo, globular, heart-shaped, and mature embryo. { Dicotyledonous Embryo: Contains an embryonal axis with two cotyledons, epicotyl, hypocotyl, and radicle.

16 Oswaal CUET (UG) Chapterwise Question Bank { Monocotyledonous

Embryo: Contains one cotyledon (scutellum), an embryonal axis with an epicotyl, coleoptile, and radical enclosed in coleorrhiza.

Seed { Seed

Composition: Typically consists of seed coat(s), cotyledon(s), and an embryo axis. { Mature Seeds: May be non-albuminous (e.g., pea) or albuminous (e.g., wheat); some have a persistent nucellus (perisperm). { Seed Coat: Derived from ovule integuments; has a micropyle for gas and water exchange during germination. { Seed Dehydration: As seeds mature, they lose water and become relatively dry, allowing for storage. { Seed Dormancy: Some seeds enter a state of dormancy, while others germinate under favorable conditions. { Seed Viability: Varies among species; some seeds remain viable for years, with records of exceptionally old seeds germinating (e.g., date palm, lupine).

Apomixis and Polyembryony { Apomixis: Asexual reproduction mechanism in flowering

plants, where seeds develop without fertilization.

BIOLOGY

{ Polyembryony:

Occurs in some Scan to know apomictic species where multiple more about embryos form in a single seed. this topic {  Apomictic Seed Development: Involves diploid egg cell formation without reduction division or the development of nucellar cells into Apomixis and embryos. Polyembryony {  Examples of Apomictic Plants: Some Citrus and Mango varieties exhibit apomixis. { Polyembryony Observation: Seeds of orange can show polyembryony with numerous embryos of varying sizes in each seed. {  Genetic Nature of Apomictic Embryos: Apomictic embryos are typically genetic clones of the parent plant. { Significance for Agriculture: Apomixis has implications in agriculture, especially for hybrid seed production and maintenance, reducing costs for farmers. { Research on Apomixis: Active research aims to understand the genetics of apomixis and transfer apomictic genes into hybrid varieties to make hybrid seed production more economical.

OBJECTIVE TYPE QUESTIONS [A] MULTIPLE CHOICE QUESTIONS: 1. Fruit which develop only from the ovary are called— [CUET 2022] (a) False fruits (b) Parthenocarpic fruits (c) True fruits (d) Apomictic fruits 2. Which layer of microsporangium is nutritive in function? [CUET 2022] (a) Epidermis (b) Endothecium (c) Middle Layers (d) Tapetum 3. In an embryo sac, the cells that degenerate after fertilisation are: (a) Synergids and primary endosperm cell (b) Synergids and antipodals (c) Antipodals and primary endosperm cell (d) Egg and antipodals 4. Which one of the cell in an embryo-sac produce endosperm after double fertilisation? (a) Synergids cell (b) Antipodal cell (c) Central Cell (d) Egg 5. The structure of bilobed anther consists of: (a) 2 thecae, 2 sporangia (b) 4 thecae, 4 sporangia (c) 4 thecae, 2 sporangia (d) 2 thecae, 4 sporangia 6. From among the situations given below, choose the one that prevents both autogamy and geitonogamy. (a) Monoecious plant bearing unisexual flowers. (b) Dioecious plant bearing only male or female flowers.

(c) Monoecious plant with bisexual flowers. (d) Dioecious plant with bisexual flowers 7. Which among the following cell is binucleate in an embryo sac? (a) Antipodal cell (b) Central cell (c) Synergid (d) Female gamete 8. Flowers with both androecium and gynoecium are called: (a) Bisexual flowers (b) Anther (c) Unisexual flowers (d) Androgynous 9. Among the terms listed below, those that of are not technically correct names for a floral whorl are: (i) Androecium (ii) Carpel (iii) Corolla (iv) Sepal (a) (i) and (iv) (b) (iii) and (iv) (c) (ii) and (iv) (d) (i) and (ii) 10. Enclosed within the integument of a typical anatropus ovule is a diploid mass of cellular tissue known as: (a) Nucellus (b) Embryo sac (c) Megaspore mother cell. (d) Synergids 11. During microsporogenesis, meiosis occurs in: (a) Endothecium (b) Microspore mother cells (c) Microspore tetrads (d) Pollen grains 12. The outermost and innermost wall layers of micro­ sporangium in an anther are respectively. (a) endothecium and tapetum. (b) epidermis and endodermis. (c) epidermis and middle layer. (d) epidermis and tapetum.

SEXUAL REPRODUCTION IN FLOWERING PLANTS

13. The phenomenon observed in some plants wherein parts of the sexual apparatus are used for forming embryos without fertilisation is called: (a) parthenocarpy. (b) apomixis. (c) vegetative propagation. (d) sexual reproduction 14. The phenomenon wherein, the ovary develops into a fruit without fertilisation is called (a) parthenocarpy. (b) apomixis. (c) asexual reproduction. (d) sexual reproduction. 15. In a flower, if the megaspore mother cell forms megaspores without undergoing meiosis and if one of the megaspores develops into an embryo sac, its nuclei would be: (a) haploid. (b) diploid. (c) a few haploids and a few diploids. (d) with varying ploidy 16. A dicotyledonous plant bears flowers but never produces fruits and seeds. The most probable cause for the above situation is: (a) Plant is dioecious and bears only pistillate flowers. (b) Plant is dioecious and bears both pistillate and staminate flowers. (c) Plant is monoecious. (d) Plant is dioecious and bears only staminate flowers. 17. While planning for an artificial hybridisation programme involving dioecious plants, which of the following steps would not be relevant? (a) Bagging of female flower (b) Dusting of pollen on stigma (c) Emasculation (d) Collection of pollen 18. From the statements given below, choose the options that are true for a typical female gametophyte of a flowering plant. (i) It is 8-nucleate and 7-celled at maturity. (ii) It is free-nuclear during the development. (iii) It is situated inside the integument but outside the nucellus. (iv) It has an egg apparatus situated at the chalazal end. (a) (i) and (iv) (b) (ii) and (iii) (c) (i) and (ii) (d) (ii) and (iv) 19. Which of these are the carriers of male gametes in some seed plants? (a) Microspore mother cells (b) Pollen Grains (c) Anthers (d) Megaspore mother cells Ans. 20. Select the appropriate options with reference to post fertilisation changes in angiosperms. A. Inner integument of ovule develops into tegmen. B. Ovary wall forms pericarp. C. Ovule forms fruit. D. Zygote forms endosperm. E. Outer integument of ovule develops into Testa. Choose the correct answer from the options given below: (a) B, E only (b) A, B, E only (c) C, D, B only (d) A, C, B only Ans.

17 21. Which of the following is a post-fertilisation event in flowering plants? [NCERT Exemplar, Q. 17, Pg. 4] (a) Transfer of pollen grains (b) Embryo development (c) Formation of flower (d) Formation of pollen grains 22. Choose the correct statement from the following. [NCERT Exemplar, Q. 17, Pg. 14] (a) Cleistogamous flowers always exhibit autogamy. (b) Chasmogamous flowers always exhibit geitonogamy. (c) Cleistogamous flowers exhibit both autogamy and geitono­ gamy. (d) Chasmogamous flowers never exhibit autogamy. 23. A Particular species of plant produces light, nonsticky pollen in large numbers and its stigmas are long and feathery. These modifications facilitate pollination by [NCERT Exemplar, Q. 12, Pg. 11] (a) insects. (b) water. (c) wind. (d) animals. 24. The male gametes of rice plant have 12 chromosomes in their nucleus. The chromosome number in the female gamete, zygote and the cells of the seedling will be respectively. [NCERT Exemplar, Q. 6, Pg. 2] (a) 12, 24, 12. (b) 24, 12, 12. (c) 12, 24, 24. (d) 24, 12, 24. 25. The statements given below describe certain features that are observed in the pistil of flowers. [NCERT Exemplar, Q. 8, Pg. 2] (i) Pistil may have many carpels. (ii) Each carpel may have more than one ovule. (iii) Each carpel has only one ovule. (iv) Pistils have only one carpel. Choose the statements that are true from the options below. (a) (i) and (ii) (b) (i) and (iii) (c) (ii) and (iv) (d) (iii) and (iv) 26. In a fertilised embryo sac, the haploid, diploid and triploid structures are [NCERT Exemplar, Q. 14, Pg. 11] (a) synergids, zygote and primary endosperm nucleus. (b) synergids, antipodal and polar nuclei. (c) antipodal, synergids and primary endosperm nucleus. (d) synergids, polar nuclei and zygote. 27. The number of chromosomes in the shoot tip cells of a maize plant is 20. The number of chromosomes in the microspore mother cells of the same plant shall be [NCERT Exemplar, Q. 18, Pg. 4] (a) 20. (b) 10. (c) 40. (d) 15. 28. Autogamy can occur in a chasmogamous flower if [NCERT Exemplar, Q. 10, Pg. 10] (a) pollen matures before maturity of ovule. (b) ovules mature before maturity of pollen. (c) both pollen and ovules mature simultaneously. (d) both anther and stigma are of equal lengths. 29. Embryo sac is to ovule as _______ is to an anther. (a) stamen (b) filament (c) pollen grain (d) androecium [NCERT Exemplar, Q. 2, Pg. 9]

18 Oswaal CUET (UG) Chapterwise Question Bank 30. In a typical complete, bisexual and hypogynous flower the arrangement of floral whorls on the thalamus from the outermost to the innermost is. [NCERT Exemplar, Q. 3, Pg. 9] (a) calyx, corolla, androecium and gynoecium. (b) calyx, corolla, gynoecium and androecium. (c) gynoecium, androecium, corolla and calyx. (d) androecium, gynoecium, corolla and calyx. [B] ASSERTION & REASON: Directions for Questions Number 1-10 Each question consists of two statements, namely, Assertion (A) and Reason (R).For selecting the correct answer, use the following code: (a) Both Assertion (A) and Reason (R) are the true and Reason (R) is a correct explanation of Assertion (A). (b) Both Assertion (A) and Reason (R) are the true but Reason (R) is not a correct explanation of Assertion (A). (c) Assertion (A) is true and Reason (R) is false. (d) Assertion (A) is false and Reason (R) is true. 1. Assertion (A): The microsporangium is generally surrounded by three wall layers. Reason (R): The innermost wall layer, the tapetum, nourishes developing pollen grains. 2. Assertion (A): The exine of a pollen grain is primarily composed of cellulose and pectin. Reason (R): The exine is highly resistant to environmental factors due to its sporopollenin content. 3. Assertion (A): The pollen grain’s inner wall is called the exine. Reason (R): The inner wall, called the intine, is made up of sporopollenin. 4. Assertion (A): Pollen grains are released with the dehiscence of the anther. Reason (R): The anther remains closed to protect the pollen grains. 5: Assertion (A): The generative cell within a pollen grain is larger than the vegetative cell. Reason (R): In some angiosperms, pollen grains are shed in a 3-celled stage. 6. Assertion (A): In many angiosperms, pollen grains remain viable for months. Reason (R): The viability of pollen grains is influenced by temperature and humidity. 7. Assertion (A): The micropyle serves as the site for pollen tube entry into the ovule. Reason (R): The egg apparatus is located at the chalazal end of the embryo sac. 8. Assertion (A): The style is responsible for providing a landing platform for pollen grains. Reason (R): The stigma is the elongated slender part of the pistil. 9. Assertion (A): The endosperm in seeds can be completely consumed by the embryo. Reason (R): Some seeds have a persistent nucellus known as perisperm. 10. Assertion (A): Apomixis is a reproductive mechanism where seeds develop without fertilization. Reason (R): Polyembryony can occur in apomictic species, resulting in multiple embryos in a single seed.

BIOLOGY

[C] COMPETENCY/CASE BASED QUESTIONS: I. Read the following paragraph and answer the given questions. Flowering plants, or angiosperms, exhibit a complex and fascinating reproductive process. Before the visible emergence of a flower, the plant undergoes hormonal and structural changes, initiating the development of floral primordium. This process involves the formation of inflorescences that bear floral buds, leading to the differentiation and development of the male (androecium) and female (gynoecium) reproductive structures within the flower. The androecium consists of stamens, while the gynoecium represents the female reproductive organ. Within the stamen, a microsporangium houses the pollen grains, which are the male gametophytes. These pollen grains exhibit remarkable diversity in size, shape, and ornamentation. 1. What are the two primary reproductive structures found within a flower? (a) Stamens and petals (b) Anther and ovule (c) Androecium and gynoecium (d) Filament and style 2. What is the role of the microsporangium within the stamen? (a) It houses the female gametophyte. (b) It is the site of pollen deposition. (c) It stores nectar for pollinators. (d) It protects the ovule. 3. What is the outermost layer of the pollen grain responsible for its high resistance to environmental factors? (a) Intine (b) Cytoplasm (c) Exine (d) Tapetum 4. At what stage of development are pollen grains typically shed from the anther? (a) 1-celled stage (b) 2-celled stage (c) 3-celled stage (d) 4-celled stage 5. What is the term for the structure that serves as the site for pollen tube entry into the ovule during fertilization? (a) Chalaza (b) Micropyle (c) Synergids (d) Integuments II. Read the paragrapgh and answer given questions 5-10. Pollen development in angiosperms is a crucial process in the reproduction of flowering plants. It involves a series of intricate steps that result in the formation of pollen grains, which carry the male gametes necessary for fertilization. Understanding the stages of pollen development and the unique features of pollen grains is essential for comprehending the reproductive biology of angiosperms. Table: Pollen Development in Angiosperms Stage

Description

Key Characteristics

1. M  icrosporogenesis

Formation of Four wall microspore layers in the tetrads within the microsporangium microsporangium

2. M  icrospore Formation

Microspores develop from pollen mother cells

Microspore tetrads, each containing potential pollen cells

3. Pollen Grain Structure

Mature pollen grain composition

Exine (sporopollenin) and intine (cellulose and pectin) layers

19

SEXUAL REPRODUCTION IN FLOWERING PLANTS

4. V  iability and Factors affecting Storage pollen grain viability

Temperature and humidity influence viability

5. P  ollen Germination

Pollen tube enters the ovule through the micropyle

Process of pollen tube formation and growth

This table summarizes the key stages of pollen development in angiosperms, including microsporogenesis, microspore formation, the structure of a pollen grain, factors influencing viability, and the process of pollen germination. Each stage has unique characteristics and contributes to the successful repro­ duction of flowering plants. 6. During which stage of pollen development do microspore tetrads form? (a) Microsporogenesis (b) Pollen Viability (c) Pre-fertilization (d) The Generative Stage 7. What is the primary function of the tapetum in the microsporangium?

(a) (b) (c) (d) 8. (a) (c) 9.

Nourishing the developing pollen grains Protection against environmental factors Formation of the exine layer Maturation of the vegetative cell Which part of the pollen grain contains the male gametes? Exine (b) Intine Vegetative cell (d) Generative cell In which part of the flower does the pollen tube enter during fertilization? (a) Stigma (b) Style (c) Ovary (d) Micropyle 10. What is the primary factor that influences the duration of pollen grain viability? (a) The number of microsporangia (b) The thickness of the exine (c) Temperature and humidity (d) The size of the vegetative cell

ANSWER KEY [A] MULTIPLE CHOICE QUESTIONS 1. (c)

2. (d)

3. (b)

4. (c)

5. (d)

6. (b)

7. (b)

8. (a)

9. (c)

10. (a)

11. (b)

12. (d)

13. (b)

14. (a)

15. (b)

16. (d)

17. (c)

18. (c)

19. (b)

20. (b)

21. (b)

22 (a)

23. (c)

24. (c)

25. (a)

26. (a)

27. (a)

28. (c)

29. (c)

30. (a)

1. (a)

2. (d)

3. (b)

4. (c)

8. (c)

9. (a)

10. (a)

1. (c)

2. (b)

3. (c)

9. (d)

10. (c)

[B] ASSERTION & REASON 5. (d)

6. (a)

7. (a)

[C] COMPETENCY/CASE-BASED QUESTIONS 4. (b)

5. (b)

6. (a)

7. (a)

8. (d)

ANSWERS WITH EXPLANATION [A] MULTIPLE CHOICE QUESTION: 1. Option (c) is correct Explanation: Fruit which develops only from the ovary are called true fruits, while in false fruits, parts other than ovary also contributes in its formation 2. Option (d) is correct Explanation: Tapetum provides nourishment to developing pollen grains and it is the innermost wall layer of microsporangium. While layers like epidermis, endothecium and middle layers perform the function of protection and help in dehiscence of anther to release pollen 3. Option (b) is correct. Explanation: In an unfertilised embryo sac, the antipodals and synergids are distinctly present at chalazal end and micropylar end respectively while, in fertilised embryo sac, antipodals and synergids gradually degenerate after the formation of zygote. 4. Option (c) is correct Explanation: In female gametophyte,central cell is involved in the double fertilisation that help in the endosperm development. While antipodal cells provides nourishment to the egg cell, and synergid cell help in pollen tube growth.

5. Option (d) is correct. Explanation: Anther is a sac-like structure that produces pollen grains. It is bilobed and each lobe has two theca i.e., dithecous 6. Option (b) is correct. Explanation: Dioecious plants (bearing only male or female flowers) prevent both autogamy and geitonogamy. Autogamy is a method of self-pollination in which the transfer of pollen grains from anther to stigma of the same flower. takes place. Geitonogamy is the transfer of pollen grains from anther to stigma of another flower of the same plant. It is ecologically cross - pollination which is supposed to be equivalent to selfpollination because all flowers on a plant are genetically identical. 7. Option (b) is correct. Explanation: Central cell form binucleate endosperm mother cell upon fertilisation with one of the two sperm cells, forms triploid endosperm to nourish embryo development. 8. Option (a) is correct. Explanation: Androecium is the male part and gynoecium is the female part, and in those flower have both of these they are called bisexual flower.

20 Oswaal CUET (UG) Chapterwise Question Bank 9. Option (c) is correct. Explanation: All the four whorls of the plant with their relative position in flower can be indicated through following diagram. Sepals collectively form a whorl, called as calyx while technically the carpel is known as gynoecium. The floral whorls formed by petals and stamens are called as corolla and androecium, respectively. 10. Option (a) is correct. Explanation: In a typical anatropous ovule, several structures can be found, and one of them is the nucellus. Let’s break down the components: 1. Anatropous ovule: This refers to the orientation of the ovule within the ovary. Anatropous ovules are bent at an angle, with the micropyle (the opening through which the pollen tube enters during fertilization) and funiculus (the stalk-like structure that attaches the ovule to the ovary) at opposite ends. 2.  Diploid mass of cellular tissue: This is the structure enclosed within the integument (the protective layers surrounding the ovule) and is indeed diploid. 3. Nucellus: The diploid mass of cellular tissue enclosed within the integument of a typical anatropous ovule is called the nucellus. The nucellus plays a crucial role in the development of the embryo sac, which is part of the female reproductive structure of the flower. So, in summary, the correct answer is (A) Nucellus because it is the diploid mass of cellular tissue enclosed within the integument of the anatropous ovule. 11. Option (b) is correct. Explanation: During microsporogenesis, meiosis occurs in microspore mother cells. As the anther develops, the microspore mother cells of the sporogenous tissue undergo meiotic divisions to form microspore tetrads. The microspore tetrad after dehydration is developed into pollen grains. 12. Option (d) is correct. Explanation: The outermost and innermost wall layers of microsporangium in an anther are respectively, epidermis and tapetum. A typical microsporangium is generally surrounded by four-wall layers, that is, the epidermis, (outermost protective layer), endothecia, (middle fibrous layers) and the tapetum (innermost nutritive layer). 13. Option (b) is correct. Explanation: Apomixis refers to the formation of seeds without fertilization. The embryos are genetically identical to the parental plant. 14. Option (a) is correct Explanation: Parthenocarpy is the formation of seedless fruits without fertilization. The fruits developed from unfertilised ovary are called parthenocarpic fruits. 15. Option (b) is correct. Explanation: In some species, the diploid egg cell is formed without reduction division and develops into an embryo without fertilisation. It is an asexual reproduction which occurs in the absence of pollinators or in extreme environments. 16. Option (d) is correct. Explanation: In dioecious plants, the unisexual male flower is staminate, that is, bearing stamens only, while the female is pistillate or bearing pistil only. For the production of fruits and

BIOLOGY

seeds fertilisation must take place, which is possible only in the presence of both male and female flowers. When the plant is dioecious, it will give rise to the following situations: (i) If the plant is dioecious and bears only pistillate flowers, fertilisation can take place with the help of pollinators. (ii) If the plant is dioecious and bears only staminate flowers, fertilisation cannot take place, because female gamete is non-motile which can’t reach the male gamete in order to fuse with it. When the plant is monoecious, that is, carrying both stamen and pistil together, it may lead to self-fertilisation and production of seed. 17. Option (c) is correct. Explanation: Artificial hybridisation is one of the major methods of crop improvement programme. This cross will make sure that only the desired pollen grains are used for pollination and the stigma is protected from contamination (from unwanted pollen). This is achieved by emasculation and bagging techniques. 18. Option (c) is correct. Explanation: Statement (i) and (ii) are correct regarding female gametophyte of flowering plant. The female gametophyte or embryo sac is located inside the nucellus, enclosed within the integuments. In a majority of flowering plants, one of the megaspore is functional while the other three degenerates. Three repeated mitotic divisions of the functional megaspore results in the formation of seven-celled or eight-nucleate embryo sac. Six of the eight nuclei are organised at the two poles. Three cells grouped at micropylar end forms egg-apparatus and 3 at the chalazal end form antipodal cells. The large central cell at the centre has two polar nuclei. The meiotic divisions in the formation of embryo sac are strictly free nuclear, that is nuclear divisions are not followed immediately by cell-wall formation. Gametophyte is situated at micropylar end not at chalazal end. 19. Option (b) is correct Explanation: Pollen grain carries the male gametes which can be transferred to the female gamete via pollination. All the events - from pollen deposition on the stigma until pollen tubes enter the ovule - are together referred as pollen-pistil interaction. 20. Option (b) is correct Explanation: Post fertilization events includes a. Development of endosperm from primary endosperm cell. b. Development of embryo from zygote c. Formation of seed from ovule d. At the time of seed formation, an integument develops into seed coats. Outer integument forms testa (outer seed coat) while inner integument forms tegmen (inner seed coat). e. Formation of pericarp from ovary wall 21. Option (b) is correct Explanation: During the sexual reproduction, the events which take place after the formation of zygote are called post-fertilisation events. The process of embryo development from the zygote (called embryogenesis) takes place after the fertilisation. In all flowering plants, the zygote is formed inside the ovule. In ovule, the zygote divides several times to form an embryo. 22. Option (a) is correct Explanation: The pollination that occurs in opened flowers is called chasmogamy. It is of two types, that is, self-pollination (autogamy) and cross-pollination. Cross-pollination is of two

SEXUAL REPRODUCTION IN FLOWERING PLANTS

types, that is, geitonogamy and xenogamy. So, we can say that chasmogamous flowers exhibit both autogamy (self-pollination) and allogamy (crosspollination). While, in cleistogamous flower the anthers and stigma lies close to each other within the closed flowers. When anthers dehisce in the flower buds, pollen grains come in contact with the stigma for effective pollination. Thus, these flowers are invariably autogamous as there is no chance of cross-pollen landing on the stigma 23. Option (c) is correct Explanation: Plants use two abiotic (wind and water) and one biotic (animals) agent to achieve pollination. Majority of plants use biotic agents for pollination. Pollination by wind is more common amongst abiotic pollination. It requires the light and non-sticky pollen grains so that, they can be transported in wind currents. They often possess well-exposed stamens (so that the pollens are easily dispersed into wind currents) and large often feathery stigma to easily trap air-borne pollen grains. Wind pollination is common in grasses. Pollination by water is called hydrophily which is quite rare in flowering plants but occurs in aquatic plants. Zoophily is pollination through the agency. Entomophily (pollination by insects) is the most common type of zoophily which occurs through the agency of animals. 24. Option (c) is correct Explanation: In female gamete, the chromosome number will be same as that of the male gamete (12). A zygote is a fertilised egg/seed which means gametes from the parents have been combined (diploid) and thus, the chromosome number will be 24 (2n). A seedling is a young plant. Sporophyte develops out from a plant embryo from a seed. So, the chromosome number in the cells of the seedlings will be 24 (2n), which will further give rise to new diploid individual. Female gamete (n) = 12 Zygote (2n) = 24 Cells of seedling (2n) = 24 Q. 25. Option (a) is correct Explanation: (a) Pistil is the female reproductive part (gynoecium) of a flower. It is centrally located and consists of ovary (a swollen base), which contains the potential seeds or ovules; style (a stalk), arising from the ovary; and a pollen receptive tip, the stigma. Inside ovary it is attached to a parenchymatous cushion called placenta, either singly or in cluster. 26. Option (a) is correct Explanation: In a fertilised embryo sac, the haploid, diploid and triploid structures are synergids, zygote and primary endosperm nucleus respectively. 27. Option (a) is correct Explanation: The whole plant body of maize plant including shoot tip cells remains in diploid (2n) condition. As the microspore mother cell is a part of reproductive organ, the chromosome number in these cells will remain same as the individual, that is, 2n = 20. These microspore mother cells are further responsible for producing male gametes, that is, haploid (n) by reduction division 28. Option (c) is correct Explanation: Autogamy is a method of self-pollination. It is a process in which the stigma of a flower receives pollens from the anther of same flower. For autogamy both the sex organs of a chasmogamous flower should mature at the same time. As chasmogamous flowers open at maturity, pollen release and stigma receptivity should be synchronised for the process of autogamy. In such flowers, the length of anther and stigma plays secondary role in autogamy, e.g., in case of protandry (in which pollens mature early) and protogyny (in which stigma matures early) leads to cross-pollination.

21 29. Option (c) is correct Explanation: The pollen grains represent the male gametophytes. As the anthers mature and dehydrate, the microspores dissociate from each other and develop into pollen grains. So, embryo sac is to ovule as pollen grain is to an anther. 30. Option (a) is correct Explanation: In a typical complete, bisexual and hypogynous flower the arrangement of floral whorls on the thalamus from the outermost to the innermost is as follows: (i) Calyx: It is the outermost whorl of sepals. (ii) Corolla: It is a whorl of petals inside the calyx. (iii) Androecium: It is a whorl of stamens inside the corolla. (iv) Gynoecium: It is a whorl of pistils (in the centre of the flower forming inner most whorls). [B]. ASSERTION AND REASON QUESTIONS: 1. Option (a) is correct Explanation: Assertion (A) correctly mentions that the microsporangium is generally surrounded by three wall layers. Reason (R) provides the correct explanation by specifying that the innermost wall layer, the tapetum, nourishes developing pollen grains. 2. Option (d) is correct Explanation: Assertion (A) is incorrect because the exine is primarily composed of sporopollenin, not cellulose and pectin. Reason (R) correctly explains the resistance of the exine to environmental factors. 3. Option (b) is correct Explanation: Assertion (A) is correct in identifying the inner wall as the intine, not the exine. Reason (R) is not the correct explanation because the intine is not made up of sporopollenin. 4. Option (c) is correct Explanation: Assertion (A) is correct as pollen grains are indeed released with the dehiscence (opening) of the anther. Reason (R) is not correct because the anther opens to facilitate pollen release, not to protect the pollen grains. 5. Option (d) is correct Explanation: Assertion (A) is incorrect because the generative cell is typically smaller than the vegetative cell in a mature pollen grain. Reason (R) is correct as some angiosperms shed pollen grains when they are in a 3-celled stage. 6. Option (a) is correct Explanation: Assertion (A) is correct, as many angiosperms indeed maintain pollen viability for months. Reason (R) provides the correct explanation by stating that the viability of pollen grains is influenced by factors like temperature and humidity. 7. Option (a) is correct Explanation: Assertion (A) is correct because the micropyle is indeed the site for pollen tube entry into the ovule. Reason (R) provides the correct explanation by specifying the location of the egg apparatus. 8. Option (c) is correct Explanation: Assertion (A) is correct in describing the function of the style. Reason (R) is not correct because the stigma is typically the receptive surface for pollen grains, and it is not the elongated slender part of the pistil. 9. Option (a) is correct Explanation: Assertion (A) is correct because the endosperm in some seeds can be fully consumed by the developing embryo. Reason (R) provides the correct explanation by mentioning the presence of a persistent nucellus known as perisperm in some seeds.

22 Oswaal CUET (UG) Chapterwise Question Bank 10. Option (a) is correct Explanation: Assertion (A) is correct in describing apomixis as a reproductive mechanism without fertilization. Reason (R) provides the correct explanation by mentioning polyembryony, which can occur in apomictic species, leading to the development of multiple embryos within a single seed. [C] COMPETENCY/CASE-BASED QUESTIONS : 1. Option (c) is correct Explanation: The androecium represents the male reproductive structures of a flower, and it includes the stamens. The gynoecium represents the female reproductive structures and includes the pistil or carpel. Together, the androecium and gynoecium are the two main reproductive parts within a flower. 2. Option (b) is correct Explanation: The microsporangium is the part of the stamen where pollen grains are produced and stored. It acts as the site where pollen grains are deposited, making it available for pollination when the flower is ready to reproduce. 3. Option (c) is correct Explanation: The outermost layer of a pollen grain is called the exine. It primarily consists of sporopollenin, a substance known for its high resistance to environmental factors such as heat, chemicals, and physical damage. This robust exine layer protects the pollen grain during its journey to the stigma. 4. Option (b) is correct Explanation: Pollen grains are typically shed from the anther in the 2-celled stage. At this stage, each pollen grain contains

24

BIOLOGY

two cells: the vegetative cell and the generative cell. This stage is well-suited for pollination, as it allows the generative cell to participate in fertilization. 5. Option (b) is correct Explanation: The micropyle is the small opening at the tip of the ovule’s integuments. It serves as the site for pollen tube entry into the ovule during fertilization. The pollen tube grows through the micropyle to reach the ovule, facilitating the fertilization process. 6. Option (a) is correct Explanation: Microsporogenesis is the stage where microspore tetrads form within the microsporangium. 7. Option (a) is correct Explanation: The tapetum nourishes developing pollen grains within the microsporangium. 8. Option (d) is correct Explanation: The generative cell within the pollen grain contains the male gametes responsible for fertilization. 9. Option (d) is correct Explanation: The micropyle is the site where the pollen tube enters the ovule for fertilization. 10. Option (c) is correct Explanation: The duration of pollen grain viability is influenced by external factors such as temperature and humidity. Optimal conditions can extend pollen viability. These questions should provide a more challenging assessment of your knowledge regarding pollen development in angi­ osperms.

Study Time

CHAPTER

Max. Time: 1:5 Hours Max. Questions: 50

3

 Revision Notes {  Humans

HUMAN REPRODUCTION

are sexually reproducing and viviparous, meaning they give birth to live offspring. { The reproductive events in humans involve a series of intricate pro­ cesses that lead to the formation of a new individual.

Scan to know more about this topic

Human reproduction

Male Reproductive System { Primary

Sex Organs:

Testes, a pair of oval-shaped organs, are the primary

male sex organs. are located outside the abdominal cavity in a sac called the scrotum, which helps maintain a lower temperature essential for spermatogenesis, the process of sperm production.  Within each testis, there are approximately 250 testicular lobules, each containing highly coiled seminiferous tubules where sperms are produced. The seminiferous tubules are lined with two types of cells: spermatogonia (male germ cells) and Sertoli cells, which play a crucial role in nurturing and supporting developing spermatogenic cells.  Surrounding the seminiferous tubules are Leydig cells, also known as interstitial cells, responsible for synthesizing and secreting androgens, including testosterone, which are essential for male sexual characteristics and function. { Secondary Sex Organs:  The male duct system includes rete testis, vasa efferentia, epididymis, vas deferens, ejaculatory duct, and associated glands.  They

Ureter

Urinary bladder

Vas deferens

Seminal vesicle Prostate Bulbourethral gland

Epididymis Urethra

Vasa efferentia Rete testis Testicular lobules

Testis Foreskin Glans penis

Fig 1: Human Male Reproductive System

Fig 2: Sectional view of human seminiferous tubule The

epididymis is a coiled tube where sperm from the testes mature and gain motility. The vas deferens, which ascends into the abdominal cavity and loops over the urinary bladder, carries sperm from the epididymis. The ejaculatory duct receives secretions from the seminal vesicles and opens into the urethra. The urethra, originating from the urinary bladder, extends through the penis to its external opening called the urethral meatus.  Together, these ducts store and transport sperm from the testes to the outside through the urethra. { External Genitalia: The penis, the primary external male genital organ, plays a crucial role in the process of insemination.  It contains specialized tissues that facilitate erection, allowing for the introduction of sperm into the female reproductive tract. The enlarged end of the penis is known as the glans, and it is typically covered by a loose fold of skin called the foreskin. { Accessory Male Genital Glands:  Paired seminal vesicles, the prostate gland, and paired bulbourethral glands (Cowper’s glands) make up the accessory male genital glands.  These glands secrete various components that together constitute seminal plasma, a fluid rich in fructose, calcium, and certain enzymes. Seminal vesicles are responsible for producing the majority of the semen, typically contributing to about 60-70% of its volume.

HUMAN REPRODUCTION

24 Oswaal CUET (UG) Chapterwise Question Bank

31

BIOLOGY

25

HUMAN REPRODUCTION The secretion of the bulbourethral glands is alkaline

and mucous-rich, aiding in lubricating the penis, supplying nutrients to the sperm, and counteracting the acidic environment of the female reproductive tract.

Female Reproductive System { Primary

Sex Organs:

 The

primary female sex organs are a pair of ovaries, which serve as the site for the production of ova (eggs) and the secretion of various steroid ovarian hormones, including estrogen and progesterone.

These ovaries are located on either side of the lower

abdomen and are connected to the pelvic wall and uterus by ligaments. Each

ovary is enveloped by a thin epithelial layer that encloses the ovarian stroma, consisting of an outer cortex and an inner medulla.

Within

the ovaries, groups of cells form structures known as Ovarian or Graafian follicles, each housing a centrally located ovum.

{ Secondary

Sex Organs:

The female duct system comprises a pair of fallopian

tubes (oviducts), a uterus, cervix, and vagina. Each fallopian tube extends from the periphery of an

ovary to the uterus. The section closer to the ovary is a funnel-shaped structure called the infundibulum, which has finger-like projections called fimbriae. The

fallopian tube further consists of the ampulla, isthmus, and uterine part.

The uterus, also known as the womb, has an inverted

pear shape and is supported by ligaments attached to the pelvic wall. It

connects to the vagina through a narrow passage called the cervix, which forms the cervical canal, serving as part of the birth canal.

The

uterine wall is composed of three layers: the external perimetrium, the middle myometrium (consisting of smooth muscle), and the inner endometrium, which undergoes cyclic changes during the menstrual cycle.

Fig 3: Human Female Reproductive system { External

Genitalia:

The

external female genitalia, collectively referred to as the vulva, includes the mons pubis, labia majora, labia minora, hymen, and clitoris.

The

mons pubis is a cushion of fatty tissue covered by skin and pubic hair. Labia majora are a pair of thick, fleshy folds of tissue that surround the vaginal opening. Labia minora are narrower folds of tissue located beneath the labia majora. The vaginal opening is often partially covered by a membrane known as the hymen. The clitoris is a small, sensitive structure situated at the upper junction of the labia minora, above the urethral opening.

{ Mammary

Glands: pair of mammary glands, containing glandular tissue and adipose (fat) tissue, is present in the chest region of females.  The glandular tissue is organized into 15-20 mammary lobes, each containing clusters of milksecreting cells called alveoli. Milk produced by the alveoli is stored within the cavities or lumens of these structures. Mammary tubules transport milk from the alveoli, and several mammary ducts converge to form a wider mammary ampulla.  The lactiferous ducts connect to the lactiferous sinus, through which milk is drawn out during breastfeeding.  A

26 Oswaal CUET (UG) Chapterwise Question Bank Gametogenesis { The

Scan to know more about this topic

process of gamete fo­ rmation occurs in the testes and ovaries: Spermatogenesis takes place in the testes, beg­ Gametogenesis | Human inning at puberty. Reproduction | Oogenesis occurs in the ovaries, starting during embryonic development. { Spermatogenesis:  Spermatogonia, located within the seminiferous tubules of the testes, undergo mitotic divisions to increase in number. Spermatogonia then differentiate into spermatocytes and undergo meiotic divisions to produce secondary spermatocytes, each with 23 chromosomes.  Spermatids are formed from secondary spermato­ cytes, and spermiogenesis transforms them into spermatozoa. These mature spermatozoa are eventually released from the seminiferous tubules in a process called spermiation.

Sperm: Structure of sperm: { Head: The

head is the front part of the sperm cell. contains the nucleus, which carries the genetic material (DNA) of the sperm.  The acrosome is a cap-like structure covering the front part of the nucleus. It contains enzymes that help the sperm penetrate the egg during fertilization. { Midpiece: The midpiece is the region just behind the head.  It contains numerous mitochondria, which provide energy (in the form of ATP) for the sperm’s movement.  Mitochondria are arranged in a spiral or helical pattern within the midpiece. { Tail (Flagellum):  The tail is a long, whip-like structure extending from the midpiece. It is responsible for the sperm’s motility, allowing it to swim in search of the egg for fertilization. The flagellum has a complex arrangement of microtubules that form the axoneme, which provides the structural support for movement. { Plasma Membrane: The plasma membrane surrounds the entire sperm cell. It contains receptors that can recognize and bind to specific molecules on the surface of the egg during fertilization. { Acrosomal Vesicle:  This is a membrane-bound structure within the acrosome.  It contains enzymes necessary for breaking down the outer layers of the egg, allowing the sperm to penetrate and fertilize it.  It

BIOLOGY

{ Centrioles:  Sperm

typically have a pair of centrioles located near the nucleus.  Centrioles play a crucial role in organizing the microtubules during cell division, particularly in the formation of the mitotic spindle during fertilization. { Flagellar Apparatus: The flagellum of the sperm contains microtubules arranged in a “9+2” pattern. This arrangement provides the structural framework and allows for coordinated, whip-like movement. { Glycoprotein Coat:  The sperm’s plasma membrane is coated with glycoproteins, which are involved in sperm-egg recognition and binding. { Sperm Tail Sheath: The tail is enclosed by a sheath that helps streamline the sperm for efficient movement through the female reproductive tract. { Oogenesis:  Oogenesis begins during embryonic development when millions of oogonia (gamete mother cells) are formed in the fetal ovaries. Primary oocytes, derived from oogonia, are arrested in prophase-I of meiotic division.  Upon puberty, one primary oocyte typically completes meiosis-I, resulting in the release of a secondary oocyte during each menstrual cycle. This secondary oocyte is arrested in metaphase-II and only completes meiosis-II upon fertilization by a sperm. { Ovum: The ovum is a specialized female reproductive cell with a unique structure designed to support fertilization and early embryonic development.  It contains a nucleus with genetic material, a nutrient-rich cytoplasm, protective layers (zona pellucida and corona radiata), and various organelles essential for cellular processes.

Structure of Ovum { Nucleus:  The

nucleus of the ovum contains a haploid set of chromosomes (half the normal number of chromosomes in most human cells).  It carries the genetic material necessary for fertilization and the development of a new organism. { Cytoplasm: The cytoplasm of the ovum is rich in nutrients and organelles. It provides the necessary energy and resources for the initial stages of embryonic development before implantation in the uterus. { Plasma Membrane (Cell Membrane): The plasma membrane surrounds the entire ovum. It acts as a selectively permeable barrier that controls the entry of substances into and out of the cell.

27

HUMAN REPRODUCTION { Zona

Pellucida: zona pellucida is an extracellular matrix or glycoprotein layer surrounding the plasma membrane of the ovum. It plays a critical role in sperm-egg recognition and binding during fertilization. { Corona Radiata: The corona radiata consists of layers of granulosa cells that surround the ovum. These cells are connected to the ovum by long, hairlike structures called cumulus oophorus.  The corona radiata helps protect and nourish the ovum as it moves through the female reproductive tract. { Vesicles and Organelles:  The cytoplasm of the ovum contains various vesicles and organelles, including Golgi apparatus, endoplasmic reticulum, mitochondria, and lysosomes. These organelles are involved in processes such as protein synthesis, energy production, and cellular metabolism. { Polar Body (sometimes): In some cases, a polar body may be present.  A polar body is a smaller cell that is produced during oogenesis and contains a minimal amount of cytoplasm.  The polar body is a byproduct of the unequal division of cytoplasm during meiosis and typically does not play a role in fertilization.  The

Menstrual Cycle {  The

menstrual cycle is a complex Scan to know hormonal and physiological more about process that occurs in females and this topic is typically around 28 days long. { It consists of four phases: menstrual, follicular, ovulatory, and luteal. { Hormonal regulation involving FSH The Menstrual (Follicle-Stimulating Hormone) Cycle and LH (Luteinizing Hormone) orchestrates the events of the menstrual cycle. { Each phase is characterized by specific changes in the female reproductive tract and hormone levels. { Menstrual Phase: Lasts from approximately cycle days 3-5. LH and FSH production is reduced. Decreased LH leads to corpus luteum degeneration, reducing progesterone production. The endometrial lining of the uterus breaks down, leading to menstruation. { Follicular Phase: Typically occurs from cycle days 6-13 or 14.  FSH stimulates ovarian follicles to produce estrogen.  Estrogen promotes the proliferation of the endometrial lining, increasing the number of uterine glands and blood vessels.

{ Ovulatory Takes

Phase: place around the 14th day of the menstrual

cycle. and FSH levels peak. A surge in LH induces ovulation, the release of a mature egg (secondary oocyte) from the ovarian follicle. { Luteal Phase: Occurs from cycle days 15 to 28.  The corpus luteum, formed from the ruptured ovarian follicle, secretes progesterone. The endometrial lining thickens, and uterine glands become secretory. { Hormonal fluctuations and the subsequent changes in the endometrial lining are essential for preparing the uterus for potential embryo implantation. LH

Fertilisation and Implantation { Fertilisation

is the process of the fusion of a sperm cell with an ovum (egg) to form a zygote. { During sexual intercourse (coitus), semen is ejaculated into the vagina. {  Spermatozoa swim rapidly through the female reproductive tract to reach the ampullary-isthmic junction of the fallopian tube, where fertilization typically occurs. {  The acrosome of the sperm undergoes an acrosomal reaction, releasing enzymes that dissolve the egg’s protective layers, allowing the sperm to penetrate the egg. { Cortical Reaction:  After sperm entry, the egg undergoes a cortical reaction to prevent polyspermy (fertilization by multiple sperm).  Cortical granules beneath the egg’s plasma membrane release chemicals into the space between the ooplasm and the plasma membrane.  This elevates the vitelline membrane, creating a fertilization membrane that hinders the entry of additional sperm. Ensuring only one sperm fertilizes the egg prevents genetic abnormalities. { The haploid gametes, the sperm, and the ovum fuse to create a diploid zygote. As the zygote begins its journey toward the uterus, it undergoes mitotic divisions, forming a multicellular structure known as a blastocyst. { Implantation:  The blastocyst eventually attaches itself to the uterine wall in a process called implantation. Implantation is a crucial step in the establishment of pregnancy, as it initiates the interaction between the developing embryo and the maternal body.

Pregnancy and Embryonic Development

Embryonic development in humans can be explained in terms of cellular stages, each representing a crucial step in the formation of a new individual: { Fertilisation Stage:  Fertilisation occurs when a sperm cell penetrates and fuses with an egg cell (ovum).

28 Oswaal CUET (UG) Chapterwise Question Bank  This

fusion results in the Scan to know formation of a diploid zygote, more about this topic the first cell of the new individual. { Cleavage Stage:  The zygote undergoes a series Embryology | of rapid cell divisions known as Fertilization, cleavage. Cleavage, Blastulation  Cleavage divisions do not | First week increase the overall size but of embryonic create a multicellular structure development | Zygote called a morula. { Blastocyst Formation:  Further cleavage divisions transform the morula into a blastocyst. The blastocyst comprises two main parts: the outer trophoblast and the inner cell mass. { Implantation Stage:  The blastocyst attaches to the uterine lining (endometrium) in a process called implantation, typically occurring 6-10 days after fertilization.  This attachment allows the embryo to receive nourishment from the mother’s body. { Gastrulation Stage:  Gastrulation is a key process during which the embryo undergoes reorganization, forming three primary germ layers: ectoderm, mesoderm, and endoderm. These germ layers give rise to different tissues and organs in the body. { Neurulation Stage: Within the ectoderm, a process called neurulation occurs. It leads to the formation of the neural tube, which eventually develops into the central nervous system, including the brain and spinal cord. { Organogenesis Stage: {  Organogenesis is the phase during which cells within the germ layers differentiate and migrate to form specific organs and organ systems. { Major organs like the heart, lungs, liver, and kidneys begin to take shape during this stage. { Fosetal Development Stage: The embryonic period ends around the eighth week after fertilization, and the developing organism is now referred to as a fetus. During the fetal stage, the focus is on growth and maturation of the organs and tissues formed during organogenesis. { Placental Development Stage: Concurrently with embryonic and fetal development, the placenta, a temporary organ, develops.  The placenta plays a crucial role in nutrient and oxygen exchange between the mother and the fetus.

BIOLOGY

{ Birth

(Parturition) Stage: marks the end of embryonic and fetal development. It involves uterine contractions, cervical changes, and hormonal signals that lead to the birth of the baby. { Postnatal Development Stage:  After birth, the newborn continues to grow and develop outside the womb. This stage includes further maturation of organs and systems, adaptation to the external environment, and the development of various skills and abilities. { During human embryonic development: At one month, the embryo’s heart begins to form. By the end of the second month, limbs and digits are recognizable. By the end of the third month, major organs and external genitalia are well developed.  Fetal movement becomes noticeable around the fifth month. By the end of the sixth month, the body is covered in fine hair, eyelids form, and the eyes open.  At the end of nine months, the fetus is fully developed and ready for birth.  Parturition

Parturition and Lactation { Parturition:

Parturition refers to the process of delivering a fully

developed fetus and typically occurs near the end of pregnancy.  Signals for parturition originate from the fully developed fetus and placenta.  These signals induce mild uterine contractions known as the fetal ejection reflex. The release of oxytocin from the maternal pituitary gland intensifies uterine contractions, leading to the expulsion of the baby from the uterus. { Lactation: Lactation is the production of milk by the mammary glands. The mammary glands begin producing milk towards the end of pregnancy.  Initially, the milk produced is called colostrum, which is rich in antibodies and essential nutrients.  Colostrum provides crucial immune protection to the newborn.  As lactation continues, the milk transitions to mature breast milk, providing complete nutrition for the infant.

Hormonal Control of Reproductive Processes {  Various

hormones, including gonadotropin-releasing hormone (GnRH), luteinizing hormone (LH), folliclestimulating hormone (FSH), estrogen, and progesterone, play pivotal roles in regulating the male and female reproductive systems, menstrual cycle, and pregnancy. { The balance and timing of these hormones are crucial for the successful functioning of the reproductive processes outlined above.

29

HUMAN REPRODUCTION

OBJECTIVE TYPE QUESTIONS [A] MULTIPLE CHOICE QUESTIONS: 1. Select the correct statements regarding menstrual cycle in human female: [CUET 2023] (A) The first menstruation begins at puberty and is called menopause. (B) The ovulation takes place in the middle of cycle (about 14th day), when the level of progesterone is at maximum level. (C) In absence of fertilisation, the corpus luteum degenerates which causes the disintegration of endometrium leading to menstruation. (D) In human beings, menstrual cycle ceases around 50 years of age. Choose the correct answer from the options given below: (a) (A) and (B) only (b) (B) and (D) only (c) (A) and (C) only (d) (C) and (D) only 2. Select the hormone which is not secreted by human placenta ? [CUET 2023] (a) Estrogen (b) Progestogen (c) Human chorionic gonadotropin (d) Luteinising hormone 3. Select the hormones produced in women only during pregnancy. [CUET 2022] A. Estrogen B. Human chronic gonadotroph C. Progesterone D. Human placental lactogen E. Relaxin Choose the correct answer from the options given below: (a) B and D only (b) B and E only (c) A, B and C only (d) B, D and E only 4. Which of the following gets embedded in the endometrium during implantation? [CUET 2022] (a) Zygote (b) Morula (c) Blastocyst (d) Embryo 5. Arrange the following events in the female reproductive cycle in their natural sequence. [CUET 2022] A. Ovulation B. Growth of corpus luteum C. Sudden increase in level of LH D. Secretion of FSH E. Growth of ovarian follicle and oogenesis Choose the correct answer from the options given below: (a) E - D - A - B – C (b) D - E - C - A - B (c) E - C - A - D – B (d) D - A - C - E – B 6. Which of the following is effect of steroid in males? [CUET 2022]

(a) (b) (c) (d) 7.

Premature baldness Deepening of voice Excessive hair growth on face and body Enlargement of clitoris Match List-I with List-II List 1

(a) (c) 8.

List 2

A. Ampulla

I. Present at the upper part of labia minora

B. Fimbriae

II. Along with vagina is birth canal

C. Clitoris

III. Site of fertilisation

D. Cervical Canal

IV. Helps in collection of ovum after ovulation

Choose the correct answer from the options given below: [CUET 2022] A-IV, B-I, C-II, D-III (b) A-III, B-IV, C-I, D-II A-I, B-II, C-IV, D-III (d) A-II, B-III, C-I, D-IV The Graafian follicle ruptures to release the ovum from the ovary by the process called: [CUET 2022] (a) Ovulation (b) Menstruation (c) Implantation (d) Copulation 9. The embryo with 8 to 16 blastomeres is called _____. [CUET 2022] (a) Morula (b) Blastula (c) Zygote (d) Foetus 10. The part of the human sperm that helps in the fertilisation of the ovum is a cap-like structure called: [CUET 2022] (a) Head (b) Acrosome (c) Middle piece (d) Tail 11. Layers surrounding the ovum from outside to inside are: [CUET 2022] A. Zona pellucida B. Vitelline membrane C. Corona radiata Choose the correct answer from the options given below: [CUET 2022] (a) A, C, B (b) С, В, А (c) C, A, B (d) A, B, C 12. In humans females at the time of birth, the stage of cell cycle of oocyte is: (a) Prophase-I (b) Prophase-II (c) Meiosis-II (d) Mitosis 13. Identify the correct statements. [CUET 2022] A. Oogenesis is initiated at puberty. B. Second meiotic division in secondary oocyte occurs in the ovary. C. Middle piece of sperm provides energy for movement. D. Fertilisation of gametes occurs in the ampulla isthmic region of oviduct. E. Myometrium helps in muscular contractions of the uterus. Choose the correct answer from the options given below: (a) A, D, E only (b) A, B, D only (c) B, C, D only (d) C, D, E only

30 Oswaal CUET (UG) Chapterwise Question Bank 14. Hormones responsible for menstrual cycle are produced from: [CUET 2022] (a) uterus only (b) ovaries only (c) ovaries and anterior pituitary (d) uterus and anterior pituitary 15. Which one of the following is not a male accessory gland? (a) Seminal vesicle (b) Ampulla (c) Prostate (d) Bulbo-urethral gland 16. Prostate glands are located below (a) gubernaculum (b) seminal vesicles (c) epididymis (d) bulbourethral glands 17. The membranous cover of the ovum at ovulation is (a) corona radiata. (b) zona radiata. (c) zona pellucida. (d) chorion. 18. In human adult females oxytocin (a) stimulates pituitary to secrete vasopressin (b) causes strong uterine contractions during parturition (c) is secreted by anterior pituitary (d) stimulates growth of mammary glands 19. Acrosomal reaction of the sperm occurs due to (a) its contact with zona pellucida of the ova. (b) reactions within the uterine environment of the female. (c) reactions within the epididymal environment of the male. (d) androgens produced in the uterus. 20. Seminal plasma, the fluid part of semen, is contributed by (i) Seminal vesicle (ii) Prostate (iii) Urethra (iv) Bulbo-urethral gland (a) i and ii (b) i, ii and iv (c) ii, iii and iv (d) i and iv 21. Spot the odd one out from the following structures with reference to the male reproductive system.  [NCERT Exemplar, Q. 3, Pg. 1] (a) Rete testis (b) Epididymis (c) Vasa efferentia (d) Isthmus 22. The vas deferens receives duct from the seminal vesicle and opens into urethra as: [NCERT Exemplar, Q. 14, Pg. 3] (a) epididymis. (b) ejaculatory duct. (c) efferent ductule. (d) ureter. 23. Urethral meatus refers to the: [NCERT Exemplar, Q. 15, Pg. 4] (a) urinogenital duct. (b) opening of vas deferens into urethra. (c) external opening of the urinogenital duct. (d) muscles surrounding the urinogenital duct.

BIOLOGY

24. Identify the odd one from the following.  [NCERT Exemplar, Q. 12, Pg. 4] (a) Labia minora (b) Fimbriae (c) Infundibulum (d) Isthmus 25. Identify the wrong statement from the following.  [NCERT Exemplar, Q. 2, Pg. 1] (a) High levels of oestrogen triggers the ovulatory surge. (b) Oogonial cells start to proliferate and give rise to functional ova in regular cycles from puberty onwards. (c) Sperms released from seminiferous tubules are poorly motile/non-motile. (d) Progesterone level is high during the post ovulatory phase of menstrual cycle. 26. The immature male germ cells undergo division to produce sperms by the process of spermatogenesis. Choose the correct one with reference to above. [NCERT Exemplar, Q. 9, Pg. 2] (a) Spermatogonia have 46 chromosomes and always undergo meiotic cell division. (b) Primary spermatocytes divide by mitotic cell division. (c) Secondary spermatocytes have 23 chromosomes and undergo second meiotic division. (d) Spermatozoa are transformed into spermatids. 27. Spermiation is the process of the release of sperms from [NCERT Exemplar, Q. 5, Pg. 2] (a) seminiferous tubules. (b) vas deferens. (c) epididymis. (d) prostate gland. 28. Mature Graafian follicle is generally present in the ovary of a healthy human female around stet. (a) 5–8 days of menstrual cycle. (b) 11–17 days of menstrual cycle. (c) 18–23 days of menstrual cycle. (d) 24–28 days of menstrual cycle. 29. Which among the following has 23 chromosomes? [NCERT Exemplar, Q. 11, Pg. 3] (a) Spermatogonia (b) Zygote (c) Secondary oocyte (d) Oogonia 30. Choose the incorrect statement from the following. (a) In birds and mammals internal fertilisation takes place. (b) Colostrum contains antibodies and nutrients. (c) Polyspermy is prevented by the chemical changes in the egg surface. (d) In the human female implantation occurs almost seven days after fertilisation. [B] ASSERTION & REASON: Directions for Questions Number 1-10 Each question consists of two statements, namely, Assertion (A) and Reason (R).For selecting the correct answer, use the following code: (a) Both Assertion (A) and Reason (R) are the true and Reason (R) is a correct explanation of Assertion (A). (b) Both Assertion (A) and Reason (R) are the true but Reason (R) is not a correct explanation of Assertion (A). (c) Assertion (A) is true and Reason (R) is false. (d) Assertion (A) is false and Reason (R) is true.

31

HUMAN REPRODUCTION

1. Assertion (A): Leydig cells synthesise and secrete male testicular hormones called androgens. Reason (R): Androgens, stimulate the process of spermatogenesis. [CUET 2022] 2. Assertion (A): Spermatogenesis occurs in the seminiferous tubules of the testes. Reason (R): Sertoli cells play a crucial role in nurturing and supporting developing spermatogenic cells. 3. Assertion (A): The menstrual phase is characterized by the breakdown of the endometrial lining of the uterus. Reason (R): Reduced LH levels during the menstrual phase lead to corpus luteum degeneration. 4. Assertion (A): The zona pellucida is a glycoprotein layer surrounding the sperm cell’s plasma membrane. Reason (R): The zona pellucida plays a crucial role in sperm-egg recognition and binding during fertilization. 5. Assertion (A): Parturition refers to the process of fertilisation in humans. Reason (R): Parturition is initiated by the release of oxytocin from the maternal pituitary gland. 6. Assertion (A): The primary male sex organs are the seminal vesicles. Reason (R): Seminal vesicles produce and release androgens, including testosterone. 7. Assertion (A): Ovulation marks the end of the menstrual cycle. Reason (R): Ovulation is triggered by a surge in LH levels, causing the release of a mature egg from the ovarian follicle. 8. Assertion (A): The zona pellucida is essential for protecting the sperm cell during its journey through the female reproductive tract. Reason (R): The zona pellucida is a thick, gel-like substance that surrounds the sperm’s plasma membrane. 9. Assertion (A): The corpus luteum secretes progesterone during the luteal phase of the menstrual cycle. Reason (R): Progesterone is essential for maintaining the thickened endometrial lining of the uterus in preparation for potential embryo implantation. 10. Assertion (A): Lactation refers to the process of delivering a fully developed fetus. Reason (R): Lactation begins during pregnancy when the mammary glands produce colostrum. [C] COMPETENCY/CASE-BASED QUESTIONS: Directions for question 1-5. I. Read the following passage and answer the given questions.  The human reproductive system is a complex and intricately organised system responsible for the formation of new individuals. It comprises primary and secondary sex organs in both males and females. These organs play pivotal roles in the processes of gametogenesis and reproductive function. Reproductive Process

Male

Female

Primary Sex Organs

Testes

Ovaries

Secondary Sex Organs

Epididymis, vas deferens, ejaculatory duct, accessory glands, penis

Fallopian tubes, uterus, cervix, vagina, mammary glands

Gametogenesis Site

Testes

Ovaries

Gametogenesis Process

Spermatogenesis

Oogenesis

Hormones Regulating Process

Testosterone, FSH, LH

Estrogen, Progesterone, FSH, LH

1. Which organ is responsible for the production of ova (eggs) in females? (a) Testes (b) Ovaries (c) Epididymis (d) Uterus 2. What is the primary function of the epididymis in males? (a) Sperm production (b) Hormone secretion (c) Sperm maturation and storage (d) Uterine lining development 3. Which of the following hormones is responsible for initiating ovulation in females? (a) Testosterone (b) Estrogen (c) Progesterone (d) Luteinizing hormone (LH) 4. In the process of spermatogenesis, what is the end product that is eventually released from the seminiferous tubules? (a) Spermatogonia (b) Spermatocytes (c) Spermatids (d) Spermatozoa 5. What is the primary role of the zona pellucida in the structure of the ovum? (a) It contains the genetic material (DNA) of the ovum. (b) It provides energy for the initial stages of embryonic development. (c) It acts as a selectively permeable barrier. (d) It plays a critical role in sperm-egg recognition and binding during fertilization. II. Read the following passage and answer the question 6-10. Read the following passage and answer the given questions. Fertilisation is a complex process that culminates in the fusion of a sperm cell with an ovum (egg) to form a zygote. During sexual intercourse, semen is ejaculated into the vagina, and spermatozoa embark on a rapid journey through the female reproductive tract, with the destination typically being the ampullary-isthmic junction of the fallopian tube, where fertilisation occurs. The sperm head, which houses the nucleus carrying genetic material, is equipped with an acrosome containing enzymes that assist in penetrating the egg’s protective layers. Once a sperm successfully penetrates the egg, a cortical reaction is triggered, preventing polyspermy by blocking the entry of additional sperm. This ensures that only one sperm fertilises the egg, reducing the risk of genetic abnormalities. The fusion of the haploid sperm and ovum results in the formation of a diploid zygote, which undergoes mitotic divisions to develop into a blastocyst. Subsequently, the blastocyst attaches itself to the uterine wall in a process called implantation, marking a critical step in establishing pregnancy and initiating the interaction between the developing embryo and the maternal body.

32 Oswaal CUET (UG) Chapterwise Question Bank 6. What is the primary role of the acrosome in a sperm cell during fertilisation? (a) Energy production (b) Penetrating the egg’s protective layers (c) Housing genetic material (d) Initiating mitotic divisions 7. What is the purpose of the cortical reaction that occurs after a sperm enters the egg? (a) To increase the egg’s size (b) To initiate fertilisation (c) To prevent polyspermy (d) To form a zygote

BIOLOGY

8. Which structure of the sperm contains enzymes for breaking down the egg’s protective layers? (a) Nucleus (b) Midpiece (c) Tail (d) Acrosome 9. What is the outcome of the fusion between a sperm and an ovum? (a) Formation of a blastocyst (b) Initiation of cortical reaction (c) Release of enzymes (d) Penetration of the uterine wall 10. During which phase of the menstrual cycle does the ovum typically undergo fertilisation? (a) Menstrual phase (b) Follicular phase (c) Ovulatory phase (d) Luteal phase

ANSWER KEY [A] MULTIPLE CHOICE QUESTIONS 1. (b)

2. (d)

3. (d)

4. (c)

5. (b)

6. (a)

7. (b)

8. (a)

9. (a)

10. (b)

11. (c)

12. (a)

13. (c)

14. (c)

15. (b)

16. (b)

17. (a)

18. (b)

19. (a)

20. (b)

21. (d)

22 (b)

23. (c)

24. (a)

25. (b)

26. (c)

27. (a)

28. (b)

29. (c)

30. (c)

8. (c)

9. (a)

10. (d)

9. (a)

10. (c)

[B] ASSERTION & REASON 1. (a)

2. (a)

3. (a)

4. (c)

5. (b)

6. (c)

7. (b)

[C] COMPETENCY/CASE-BASED QUESTIONS 1. (b)

2. (c)

3. (d)

4. (d)

5. (d)

6. (b)

7. (c)

8. (d)

ANSWERS WITH EXPLANATION [A] MULTIPLE CHOICE QUESTIONS: 1. Option (b) is correct Explanation: (A) The first menstruation begins at puberty and is not called menopause. Puberty marks the beginning of the menstrual cycle, but menopause is the cessation of menstruation, which typically occurs around the age of 50. (B) This statement is correct. Ovulation, the release of an egg from the ovary, usually occurs in the middle of the menstrual cycle, approximately on the 14th day in a 28day cycle. Progesterone levels are indeed at their highest during this phase. (C) This statement is incorrect. If fertilisation does not occur, the corpus luteum, which is formed from the follicle after ovulation, will degenerate. The decline in progesterone from the corpus luteum causes the endometrial lining to break down and result in menstruation. (D) This statement is correct. Menopause is the natural cessation of menstruation and reproductive capacity in women, and it typically occurs around the age of 50 in human beings. 2. Option (d) is correct Explanation: Luteinizing hormone is not secreted by the placenta. It is primarily produced and secreted by the anterior pituitary gland in the brain. Estrogen, progestogen, and human chorionic gonadotropin (hCG) are hormones that are indeed secreted by the human placenta during pregnancy.

3. Option (d) is correct. Explanation: hCG (human chorionic gonadotropin), hpL (human placental lactogen) and relaxin are produced in humans only during pregnancy while Estrogen and progesterone are secreted in a non-pregnant woman also. 4. Option (c) is correct Explanation: The blastocyst gets embedded in the endometrium of the uterus. After attachment, the uterine cells divide rapidly and causes the blastocyst to become embedded in the endometrium of the uterus. This leads to pregnancy 5. Option (b) is correct Explanation: The correct sequence of the events in the female reproductive cycle is: D - Secretion of FSH E - FSH leads to the growth of ovarian follicles into a fully mature Graafian follicle. C - During the mid of menstrual cycle, there is sudden increase in the level of LH (LH surge). A - LH surge leads to ovulation. B - It is followed by luteal phase in which the remaining parts of the Graafian follicle transform as the corpus luteum. 6. Option (a) is correct Explanation: The side effects of the use of anabolic steroids in males include premature baldness, increased aggressiveness, breast enlargement, etc 7. Option (b) is correct Explanation: The fertilisation of the ovum with sperm takes place in the ampulla region of the fallopian tube. As the egg is released (a process called ovulation) it is captured by fingerlike projections of the end of the fallopian tubes (fimbriae). The fimbriae sweep the egg into the tube. Clitoris, is a small, sensitive protrusion, where two labia minora meet. The

HUMAN REPRODUCTION

cervical canal passes through the cervix [the lower end of the womb (uterus)]. It helps the baby to pass from the womb into the vagina. Thus, along with vagina it is a birth canal 8. Option (a) is correct Explanation: Ovulation is defined as the process in which a mature Graafian follicle ruptures and releases an ovum (oocyte). 9. Option (a) is correct Explanation: Morula is a solid mass of blastomeres, produced by a series of cleavage divisions of the early embryo. The embryo with 8 to 16 blastomeres is called morula. 10. Option (b) is correct Explanation: Acrosome is a special kind of organelle with a cap-like structure that covers the anterior portion of the head of the spermatozoon and contains digestive enzymes It contains lytic enzymes that help in fertilization. Head is oval shaped and consists of a haploid nucleus and a cap like acrosome. Middle piece is composed of axial filament surrounded by mitochondria and cytoplasm. Tail consists of a central axial filament. The sperm moves in fluid medium of female genital tract by the undulating movement of the tail. 11. Option (c) is correct Explanation: Ovum is spherical and is surrounded by four membranes: a. Plasma membrane (Oolemma), which is the innermost layer. b. Vitelline membrane, which is attached to the plasma membrane. c. Zona pellucida. It is the transparent noncellular layer found outer to the Vitelline membrane. d. Corona radiata: It is the outer layer formed of follicle cells. These cells are bound together by hyaluronic acid. 12. Option (a) is correct Explanation: Oogenesis is initiated in embryonic stage when millions of egg mother cells (oogonia) are formed within each foetal ovary. No more oogonia are formed and added after birth. These cells (Oogonia) undergo meiosis but get temporarily arrested at the prophase-I stage and are called primary oocytes. 13. Option (c) is correct Explanation: Certainly, here’s an explanation for each of the correct statements: B. Second meiotic division in the secondary oocyte occurs in the ovary: This statement is correct. In the process of oogenesis, the second meiotic division in the secondary oocyte is arrested and does not complete until fertilisation occurs. The secondary oocyte is released from the ovary during ovulation in an arrested state and will only complete the second meiotic division if it is fertilised by a sperm cell. D. Fertilisation of gametes occurs in the ampulla isthmic region of the oviduct: This statement is correct. Fertilisation typically occurs in the ampulla of the oviduct (fallopian tube). After ovulation, if a sperm cell successfully meets and penetrates the secondary oocyte, fertilisation occurs in the ampulla region of the oviduct before the newly formed zygote moves toward the uterus for implantation. E. Myometrium helps in muscular contractions of the uterus: This statement is correct. The myometrium is the middle layer of the uterine wall, and its main function is to contract during labor and menstruation. These contractions are responsible for pushing the fetus out during childbirth and for expelling the menstrual blood during menstruation.

33 14. Option (c) is correct. Explanation: The menstrual cycle is regulated by the coordinated functions of the hypothalamus, pituitary, ovaries, and endometrium. Hypothalamus causes the nearby pituitary gland to produce certain chemicals, which prompt the ovaries to produce the sex hormones oestrogen and progesterone. 15. Option (b) is correct. Explanation: Ampulla is a part of Fallopian tube where fertilisation occurs. Fallopian tube is part of female reproductive system. 16. Option (b) is correct. Explanation: Prostate gland is the part of male reproductive system that is located just be low the bladder. Small and walnut shaped, the prostate surrounds the beginning of the urethra. 17. Option (a) is correct. Explanation: Corona radiata is the innermost layer of the cells of the cumulus oophorus and is directly adjacent to the zona pellucida, the outer protective glycoprotein layer of the ovum. Zona radiata is a striated membrane situated next the yolk of an ovum. Chorion is an extra embryonic foetal membrane which is responsible for the formation of placenta. 18. Option (b) is correct. Explanation: Oxytocin is produced in the hypothalamus and is secreted by the posterior pituitary gland. It causes contraction during parturition and help to brings baby out from the mother womb. 19. Option (a) is correct. Explanation: Acrosomal reaction of the sperm occurs due to its contact with zona pellucida of the ova. The reaction that occurs in acrosome of sperm is triggered by the release of fertilizin. The main purpose of the acrosomal reaction is to start the fusion of the oocyte membrane with the sperm cell membrane allowing the combination of genetic material contained in both gametes, leading to the fertilisation of the oocyte. 20. Option (b) is correct Explanation: Secretion of seminal vesicle (paired), prostate gland (unpaired) and bulbo-urethral glands or Cowper’s glands (paired) constitute the seminal plasma. It contains various proteins and fructose as energy suppliers for sperm motility and is also responsible for making the largest proportion of the alkaline buffer. 21. Option (d) is correct Explanation: Rete testis, vasa efferentia and epididymis are parts of male reproductive system. Whereas isthmus is part of Fallopian tube which carries an egg from the ovary to the uterus. 22. Option (b) is correct Explanation: The vas deferens receives duct from the seminal vesicle and opens into urethra as ejaculatory duct. These ejaculatory ducts which open into the urethra about half-way through the prostate gland function to mix the sperm stored in the ampulla with fluids secreted by the seminal vesicles and to transport these substances to the prostate. 23. Option (c) is correct Explanation: Urethral meatus, also known as the external urethral orifice, is the external opening or meatus of the urethra. The urethra originates from the urinary bladder and extends through the penis to its external opening called urethral meatus.

34 Oswaal CUET (UG) Chapterwise Question Bank 24. Option (a) is correct Explanation: Fimbriae, infundibulum’ isthmus and ampulla are the parts of Fallopian duct while labia minora is female external genitalia. 25. Option (b) is correct Explanation: (a) Rapid secretion of high level of oestrogen and LH triggers the ovulatory surge. (b) Oogenesis is initiated during the embryonic developmental stage when a couple of million Unlike sperm formation that starts at puberty egg formation begins before birth. No more oogonia are formed and added after birth. (c) Sperms released from seminiferous tubules are poorly motile/non-motile. They undergo physical maturation in the head of epididymis and acquire increased motility and fertilising capacity. (d) Progesterone level is high during the postovulatory phase of menstrual cycle because after ovulation the remaining part of Graafian follicle becomes a temporary endocrine gland, corpus luteum. Corpus luteum secretes progesterone hormone to maintain the pregnancy. 26. Option (c) is correct Explanation: (a) Spermatogonia have 46 chromosomes and always undergo mitotic cell division. (b) Primary spermatocytes divide by meiotic cell division. (d) Spermatids are transformed into spermatozoa. 27. Option (a) is correct Explanation: Spermiation is the process by which mature spermatids or spermatozoa are released from sertoli cells into the seminiferous tubule lumen prior to their passage to the epididymis. 28. Option (b) is correct Explanation: Mature Graafian follicle is the follicular stage present in the ovary. It is formed after the completion of first mitotic division but before ovulation. It therefore contains a 2N diploid oocyte. Graafian follicle is characterised by a large follicular antrum and releases one or more ova into the Fallopian tube and leaving behind the corpus luteum. It is generally present in the ovary of a healthy human female around 11–17 days of menstrual cycle. 29. Option (c) is correct Explanation: Secondary oocyte (n = 23). Primary oocyte completes first meiotic division to form secondary oocyte (23 chromosomes) and polar body (23 chromosomes), whereas spermatogonia, zygote and oogonia have 46 chromosomes, hence diploid. 30. Option (c) is correct Explanation: Polyspermy may be defined as the fertilisation of an ovum by more than one sperm. During fertilisation, a sperm comes in contact with the zona pellucida layer of the ovum and induces changes in the membrane that block the entry of additional sperms. Thus, it ensures that only one sperm can fertilise an ovum. [B]. ASSERTION AND REASON QUESTIONS: 1. Option (a) is correct. Explanation: LH acts on the Leydig cells and stimulates synthesis and secretion of androgens. This happens at the age of puberty due to significant increase in the secretion of gonadotropin releasing hormone there is an increase in the secretion of gonadotropins i.e., LH and FSH.

BIOLOGY

2. Option (a) is correct. Explanation: Spermatogenesis indeed occurs in the seminiferous tubules of the testes, and Sertoli cells are crucial for nurturing and supporting developing spermatogenic cells, making Reason (R) an appropriate explanation for Assertion (A). 3. Option (a) is correct. Explanation: The menstrual phase is indeed characterized by the breakdown of the endometrial lining, and reduced LH levels during this phase contribute to corpus luteum degeneration, making Reason (R) a valid explanation for Assertion (A). 4. Option (c) is correct. Explanation: The zona pellucida surrounds the ovum (egg), not the sperm cell’s plasma membrane. While Reason (R) is correct regarding its role in sperm-egg recognition, it inaccurately describes the zona pellucida’s location. 5. Option (b) is correct. Explanation: Assertion (A) is incorrect because parturition refers to the process of delivering a fully developed fetus, not fertilization. Reason (R) is true as oxytocin does play a role in initiating uterine contractions during parturition, but it does not explain Assertion (A) accurately. 6. Option (c) is correct. Explanation: The primary male sex organs are the testes, not the seminal vesicles. Seminal vesicles do not produce androgens; instead, they contribute to seminal plasma. 7. Option (b) is correct. Explanation: Assertion (A) is not entirely accurate, as the menstrual cycle consists of multiple phases, and ovulation occurs within the cycle. However, Reason (R) correctly describes the process of ovulation but does not explain why it marks the end of the menstrual cycle. 8. Option (c) is correct. Explanation: While Assertion (A) is correct, Reason (R) inaccurately describes the zona pellucida as surrounding the sperm’s plasma membrane. The zona pellucida surrounds the ovum (egg) and plays a role in protecting it during fertilization. 9. Option (a) is correct. Explanation: Assertion (A) and Reason (R) are both accurate. The corpus luteum does secrete progesterone during the luteal phase, and progesterone is crucial for maintaining the endometrial lining for potential embryo implantation. 10. Option (d) is correct. Explanation: Assertion (A) is incorrect, as lactation does not refer to the process of delivering a fetus but rather to the production and secretion of milk by the mammary glands. Reason (R) is true, as lactation indeed begins during pregnancy when colostrum is produced. [C] COMPETENCY/CASE-BASED QUESTIONS: 1. Option (b) is correct. Explanation: The ovaries are responsible for the production of ova (eggs) in females. 2. Option (c) is correct. Explanation: The primary function of the epididymis in males is to store and mature sperm. 3. Option (d) is correct. Explanation: LH is responsible for initiating ovulation in females, specifically the release of a mature egg from the ovarian follicle.

35

HUMAN REPRODUCTION

4. Option (d) is correct. Explanation: Spermatozoa, also known as sperm cells, are the end product of spermatogenesis and are eventually released from the seminiferous tubules. 5. Option (d) is correct. Explanation: The zona pellucida is an extracellular matrix surrounding the ovum’s plasma membrane, and it is essential for sperm-egg recognition and binding during fertilization. 6. Option (b) is correct. Explanation: The acrosome of a sperm contains enzymes that assist in penetrating the egg’s protective layers during fertilization. 7. Option (c) is correct. Explanation: The cortical reaction’s primary purpose is to prevent polyspermy, the entry of multiple sperm into the egg, ensuring only one sperm fertilizes it.

24

8. Option (d) is correct. Explanation: The acrosome is the structure in the sperm that contains enzymes necessary for breaking down the outer layers of the egg, allowing the sperm to penetrate it. 9. Option (a) is correct. Explanation: The fusion of a sperm and an ovum results in the formation of a diploid zygote, which then undergoes mitotic divisions to develop into a blastocyst. 10. Option (c) is correct. Explanation: Fertilization typically occurs during the ovulatory phase of the menstrual cycle when a mature egg (secondary oocyte) is released from the ovarian follicle and is ready for fertilization in the fallopian tube.

Study Time

CHAPTER

4

Max. Time: 1:5 Hours Max. Questions: 50

REPRODUCTIVE HEALTH

 Revision Notes

  The term ‘reproductive health’ simply refers to

healthy reproductive organs with normal functions. According to WHO (World Health Organisation), the word ‘reproductive health’ means a total wellbeing in all aspects of reproduction i.e., physical, emotional, behavioural and social. The world population was estimated to have reached. 6 billion as of December 2017.  Strategies of Reproductive Health Programmes: To ensure total reproductive health, several programmes like reproductive health programmes and family planning were started in 1951.  Consequences of Population Explosion: Poverty, unemployment, shortage of food, unhygienic conditions, education problems, residential problems, pollution, crime, excessive consumption of natural resources etc.  Amniotic fluid is a clear, slightly yellowish liquid that surrounds the unborn baby (foetus) during pregnancy. It is contained in the amniotic sac.  Insemination is a process of introducing semen into the reproductive tract of a female, either through sexual intercourse or with the help of syringe in the process known as artificial insemination.  Ejaculation is the action of ejecting semen from the body.  Dr. Gregory Goodwin Pincus was the first to discover birth control pills.  The first Lippes Loop IUD was introduced in 1962. It was a plastic double “S” loop, a trapezoid-shaped IUD that closely fit around the contours of the uterine cavity, reducing the incidence of expulsion.  IUDs are currently the most effective long- acting, reversible birth control option available. The least effective methods of birth control are spermicides and withdrawal.  Women are less likely to have symptoms of common STDs, such as chlamydia and gonorrhoea, compared to men.  Human papillomavirus (HPV) is the most common STD in women and is the main cause of cervical cancer.

 It is well known that fertility in women decreases with increasing age, as showed by the following statistics: { Infertility in married women ages 16-20 = 4.5% { Infertility in married women ages 35-40 = 31.8% { Infertility in married women over the age of 40 = 70%.  Louise Joy Brown (born 25 July 1978) is the world’s first baby to be conceived via IVF at Oldham and District General Hospital in Manchester, England, to parents Lesley and Peter Brown.  Harsha Chawla born on 6th August 1986 is recorded as the India’s first IVF baby.  Artificial Insemination (AI) technique: { The semen collected from the husband or a healthy donor is artificially introduced into the vagina or the uterus (IUI– intra-uterine insemination) of the female. { This technique is useful for the male partner having an inability to inseminate female or low sperm counts.   Surrogacy- In this technique a woman (surrogate mother) bears a child for a couple unable to produce children, because the wife is infertile or unable to carry. The surrogate is impregnated either through artificial insemination or through the implantation of an embryo produced by in vitro fertilization.   Zygote Intra Fallopian Transfer (ZIFT): Transfer of zygote or early embryos (with upto 8 blastomeres) into the fallopian tube.  Intra Uterine Transfer (IUT): Transfer of embryos with more than 8 blastomeres into the uterus. The embryo formed by in vivo fertilization (fertilization within the female) is also used for such transfer to assist those females who cannot conceive.   Gamete Intra Fallopian Transfer (GIFT): Transfer of an ovum from a donor into the fallopian tube of another female who cannot produce ovum, but can provide a suitable environment for fertilization and development.  Intra Cytoplasmic Sperm Injection (ICSI): A laboratory procedure in which a single sperm (from a male partner) is injected directly into an egg (from a female partner). Then the fertilized egg is implanted into the woman’s uterus.

58

Oswaal CBSE Question Bank Chapterwise & Topicwise, BIOLOGY, Class-XII

REPRODUCTIVE HEALTH

37

38 Oswaal CUET (UG) Chapterwise Question Bank

BIOLOGY

OBJECTIVE TYPE QUESTIONS [A] MULTIPLE CHOICE QUESTIONS 1. Increased IMR and decreased MMR in a population will [NCERT Exemplar, Q. 2, Pg. 24] (a) cause rapid increase in growth rate. (b) result in decline in growth rate. (c) not cause significant change in growth rate. (d) result in an explosive population/exp. 2. A national level approach to build up a reproductively healthy society was taken up in our country in [NCERT Exemplar, Q. 5, Pg. 25] (a) 1950s. (b) 1960s. (c) 1980s. (d) 1990s. 3. Intensely lactating mothers do not generally conceive due to the [NCERT Exemplar, Q. 3, Pg. 24] (a) suppression of gonadotropins. (b) hyper-secretion of gonadotrophins. (c) suppression of gametic transport. (d) suppression of fertilisation. 4. Emergency contraceptives are effective if used within [NCERT Exemplar, Q. 6, Pg. 25] (a) 72 hrs of coitus. (b) 72 hrs of ovulation. (c) 72 hrs of menstruation. (d) 72 hrs of implantation. 5. Diaphragms are contraceptive devices used by the females. Choose the correct option from the statements given below. [NCERT Exemplar, Q. 13, Pg. 26] (i) They are introduced into the uterus. (ii) They are placed to cover the cervical region. (iii) They act as physical barriers for sperm entry. (iv) They act as spermicidal agents. (a) i and ii (b) i and iii (c) ii and iii (d) iii and iv 6. Condoms are one of the most popular contraceptives because of the following reasons. [NCERT Exemplar, Q. 10, Pg. 25] (a) These are effective barriers for insemination. (b) They do not interfere with coital act. (c) These help in reducing the risk of STDs. (d) All of the above 7. Choose the right one amongst the statements given below. [NCERT Exemplar, Q. 7, Pg. 25] (a) IUDs are generally inserted by the user herself. (b) IUDs increase phagocytosis reaction in the uterus. (c) IUDs suppress gametogenesis. (d) IUDs once inserted need not to be replaced. 8. Following statements are given regarding MTP. Choose the correct options given below. [NCERT Exemplar, Q. 2, Pg. 25] (i) MTPs are generally advised during first trimester. (ii) MTPs are used as a contraceptive method. (iii) MTPs are always surgical. (iv) MTPs require the assistance of qualified medical personnel. (a) ii and iii (b) i and iii (c) i and iv (d) i and ii

9. From the sexually transmitted diseases mentioned below, identify the one which does not specifically affect the sex organs. [NCERT Exemplar, Q. 9, Pg. 25] (a) Syphilis (b) AIDS (c) Gonorrhoea (d) Genital warts 10.  Choose the correct statement regarding the ZIFT procedure. [NCERT Exemplar, Q. 11, Pg. 26] (a) Ova collected from a female donor are transferred to the Fallopian tube to facilitate zygote formation. (b) Zygote is collected from a female donor and transferred to the Fallopian tube. (c) Zygote is collected from a female donor and transferred to the uterus. (d) Ova collected from a female donor and transferred to the uterus. 11. Match List-I with List-II. [CUET 2023] List-I List-II (a) Lippes loop (I) Barrier (b) Vaults (II) Hormone releasing device (c) Periodic abstinence (III) Non medicated IUDs (d) Progestasert (IV) Natural method Choose the correct answer from the options given below: (a) (a)-(I), (b)-(III), (c)-(IV), (d)-(II) (b) (a)-(III), (b)-(II), (c)-(IV), (d)-(I) (c) (a)-(III), (b)-(I), (c)-(IV), (d)-(II) (d) (a)-(III), (b)-(I), (c)-(II), (d)-(IV) 12. Identify the terminal method used to prevent pregnancy is: [CUET 2023] (a) Lactational amenorrhea (b) Sterilisation (c) Intra Uterine Device (d) Periodic abstinence 13. Transfer of an ovum collected from a donor into fallopian tube is called _____ method. (a) ZIFT (b) ICST (c) GIFT (d) IVF 14. Which of the following is INCORRECT statement?. [CUET 2022] A. In IUD’s, released Cu ions increases sperm motility and fertilising capacity of sperms. B. Multiload 375 is medicated IUD. C. Lippes loop is non-medicated IUD. D. LNG-20 is hormones releasing IUD. E. Vault is an IUD. Choose the correct answer from the options given below: (a) A & E only (b) A & B only (c) B & D only (d) B & C only 15. Which of the following is a natural method of birth control? [CUET 2022] (a) Condoms (b) Lactational amenorrhea (c) IUD (d) Diaphragms 16. A couple is unable to produce a child as the male partner has low sperm count. Infertility caused by this condition can be treated using- [CUET 2022] (a) ZIFT (b) AI (c) GIFT (d) IUT

39

REPRODUCTIVE HEALTH

17. What is correct for test tube baby? [CUET 2022] (a) Fertilisation of ova and embryonic development takes place in test tube (b) Fertilisation of ova occurs in uterus while the development takes place in test tube (c) Fertilisation of ova occurs in test tube whereas development of embryo occurs in uterus (d) Unfertilised ova develops in test tube 18. The function of copper ions in copper releasing IUD’s: (a) They suppress sperm motility and fertilising capacity of sperms. (b) They inhibit gametogenesis (c) They make uterus unsuitable for implantation (d) They inhibit ovulation 19. In case of a couple where the male is having a very low sperm count which technique will be suitable for fertilisation ? (a) Intrauterine transfer (b) Gamete intra-cytoplasmic fallopian transfer (c) Artificial Insemination (d) Intra-cytoplasmic sperm injection (4) Intra-cytoplasmic sperm injection 20. Select the option including all sexually transmitted diseases. (a) Gonorrhoea, Malaria, Genital herpes (b) AIDS, Malaria, Filaria (c) Cancer, AIDS, Syphilis (d) Gonorrhoea, Syphilis, Genital herpes 21. In vitro fertilisation is a technique that involves transfer of which one of the following into the fallopian tube? (a) Embryo only up to 8 celled stage (b) Either zygote or early embryo up to 8 celled stage (c) Embryo of 32 celled stage (d) Zygote only 22. Artificial insemination means (a) Transfer of sperms of a healthy donor to a test tube containing ova (b) Transfer of sperms of husband to a test tube containing ova (c) Artificial introduction of sperms of a healthy donor into the vagina (d) Introduction of sperms of healthy donor direct-ly into the ovary 23. Choose the correct statement regarding the ZIFT procedure. (a) Ova collected from a female donor are transferred to the Fallopian tube to facilitate zygote formation. (b) Zygote is collected from a female donor and transferred to the Fallopian tube. (c) Zygote is collected from a female donor and transferred to the uterus. (d) Ova collected from a female donor and transferred to the uterus. 24. The correct surgical procedure as a contraceptive method in male is: (a) ovariectomy. (b) hysterectomy. (c) vasectomy. (d) castration.

25. In human adult females oxytocin (a) stimulates pituitary to secrete vasopressin (b) causes strong uterine contractions during par­turition (c) is secreted by anterior pituitary (d) stimulates growth of mammary glands. 26. An IUD recommended to promote the cervix hostility to the sperms is: [CUET 2021] (a) Copper-T (b) Multiload-375 (c) LNG-20 (d) Cu7 27. An infertile couple was advised to undergo In-vitro fertilisation by the doctor. Out of the options given below, select the correct stage for transfer to the fallopian tube for successful results? (a) Zygote only (b) Zygote or early embryo upto 8 blastomeres (c) Embryos with more than 8 blastomeres (d) Blastocyst Stage 28. Given below are four contraceptive methods and their modes of action. Select the correct match: Method Mode of action (a) Condom (i) Ovum not able to reach Fallopian tube (b) Vasectomy (ii) Prevents ovulation (c) Pill (iii) Prevents sperm reaching the cervix (d) Tubectomy (iv) Semen contains no sperms (a) (a)–(i); (b)–(ii); (c)– (iii); (d)–(iv) (b) (a)–(ii); (b)–(iii); (c)–(iii); (d) – (i) (c) (a)–(iii); (b)–(iv); (c)–(ii); (d)–(i) (d) (a)–(iv); (b)–(i); (c)– (iii); (d)–(ii) 29. Condoms are one of the most popular contraceptives because of the following reasons. (a) These are effective barriers for insemination. (b) They do not interfere with coital act. (c) These help in reducing the risk of STDs. (d) All of the above 30. The term ‘Health’ is defined in many ways. The most accurate definition of the health would be: (a) health is the state of body and mind in a balanced condition. (b) health is the reflection of a smiling face. (c) health is a state of complete physical, mental, and social well-being. (d) health is the symbol of economic prosperity. [B] ASSERTION & REASON:

Directions: In the following questions, A statement of Assertion (A) is followed by a statement of Reason (R). Mark the correct choice as.

(a) Both assertion (A) and reason (R) are true and reason (R) is the correct explanation of assertion. (b) Both assertion (A) and reason (R) are true but reason (R) is NOT the correct explanation of assertion. (c) Assertion (A) is true but reason (R) is false. (d) Assertion (A) is false and reason (R) is true. 1. Assertion (A): Lactational amenorrhoea (absence of menstruation) is a temporary contraceptive method.

40 Oswaal CUET (UG) Chapterwise Question Bank

Reason (R): It is based on the fact that ovulation and therefore the cycle do not occur during the period of intense lactation following parturition. 2. Assertion (A): Saheli is considered as an improved form of contraceptive for human females. Reason (R): It is a non-steroidal preparation and is once a week pill. 3. Assertion (A): Intra cytoplasmic sperm injection (ICSI) technique is used to develop embryo in vitro. Reason (R): In ICSI technique, the sperm is directly injected into the ovum. 4. Assertion (A): Abortions could happen spontaneously too. Reason (R): Oral pills are very popular contraceptives among the educated urban women. 5. Assertion (A): Diaphragms are contraceptive devices used by the male. Reason (R): These devices are made of rubber that is used to cover the cervical region. 6. Assertion (A): Very often persons suffering from Sexually Transmitted Diseases (STD) do not go for timely detection and proper treatment. Reason (R): Absence or less significant symptoms in the early stages of STDs and the social stigma attached to the disease. 7. Assertion (A): Vasectomy is a sterilisation procedure advised for females as a terminal method. Reason (R): In vasectomy, a small part of the vas deferens is removed or tied by blocking gamete transport therefore preventing conception. 8. Assertion (A): Interstitial spaces outside the seminiferous tubule have blood vessels and sertoli cells. Reason (R): Sertoli cells provide nutrition to the germ cells. 9. Assertion (A): Determining the sex of an unborn child followed by MTP is an illegal process. Reason (R): Amniocentesis is a practice to test the presence of genetic disorder also. 10. Assertion (A): ICSI is an assisted reproductive technique. Reason (R): In ICSI sperm is directly injected into ovum to form the embryo in vitro. [C] COMPETENCY/CASE-BASED QUESTIONS: I. Read the following passage and answer the 1-5 given below: The term sexually transmitted disease (STD) is applied to the group of diseases that spread by sexual contact. Some infections like hepatitis-B and HIV can spread by sharing injection needles, surgical instruments etc. Except for hepatitis-B, HIV infections, genital herpes, most of the STDs are completely curable if detected early and treated properly. Though all persons are vulnerable to these infections, their incidences are high in the age group of 15-24 years. 1. Which among the following is a venerable disease, caused by a virus, which also gets transmitted by blood contact ? (a) Gonorrhoea (b) Syphilis (c) Trichomoniasis (d) Hepatitis-B.

BIOLOGY

2. Site of infection in a female suffering from chlamydiasis? (a) Urethra (b) Ureters (c) Cervix (d) Peritoneum. 3. AIDS is caused by HIV. HIV is transmitted through: (a) Handshake (b) Blood contact (c) Sexual contact (d) Both (b) and (c). 4. Which among the following STDs is caused by a protozoan ? (a) Syphilis (b) Gonorrhoea (c) Chlamydiasis (d) Trichomoniasis. 5. Syphilis is an infectious disease caused by Treponema pallidum with 3 stages: a) Infections painless ulcers on the genitals    i) First b) Blindness, heart trouble, aortic impairment   ii) Second c) Skin lesions, hair loss, swollen joints   iii) Third (a) a-I, b-II, c-III (b) a-I, b-III, c-II (c) a-III, b-I, c-II (d) a-II, B-III, c-I II. Read the following passage and answer the questions 6-10 given below:

Lactational amenorrhea, is also called as postpartum infertility. It occurs when a women is not menstruating and fully breast feeding. It is a temporary family planning method. It is a contraceptive method where the mother is informed and supported in how to use breast feeding for contraception. As breast feeding delays the return of a mother’s ovulation (when the woman’s ovary releases an egg) and may make her infertile for six months or more.

6. (a) (b) (c) (d)

A procedure misused for female foeticide is: Amniocentesis Parturition Lactational amenorrhea Artificial insemination.

7. (a) (b) (c) (d)

Lactational amenorrhea is due to action of which hormone: Prolactin induced inhibition of GnRH Prolactin induced inhibition of FSH Oxytocin induced inhibition of GnRH Oxytocin induced inhibition of FSH

8. (a) (b) (c)

What is the meaning of lactational amenorrhea? Absence of menstruation Delay in ovulation during lactational period. Chances of contraception are almost nil upto six months following parturition. (d) All of these. 9. (a) (b) (c) (d)

The only birth control that is 100% effective: Abstinence Natural family planing Birth Control pills Emergency contraceptive

10. Which among the following chemicals is used under chemical methods for contraception ? (a) Lactic acid (b) Citric acid (c) Boric acid (d) All of these.

41

REPRODUCTIVE HEALTH

ANSWER KEY [A] MULTIPLE CHOICE QUESTIONS 1. (b)

2. (a)

3. (a)

4. (a)

5. (c)

6. (d)

7. (b)

8. (c)

9. (b)

10. (b)

11. (b)

12. (b)

13. (c)

14. (a)

15. (b)

16. (b)

17. (c)

18. (a)

19. (a)

20. (c)

21. (b)

22 (c)

23. (b)

24. (c)

25. (a)

26. (c)

27. (b)

28. (c)

29. (d)

30. (a)

8. (d)

9. (b)

10. (b)

9. (a)

10. (d)

[B] ASSERTION & REASON 1. (a)

2. (a)

3. (a)

4. (b)

5. (d)

6. (a)

7. (d)

[C] COMPETENCY/CASE-BASED QUESTIONS 1. (d)

2. (c)

3. (d)

4. (d)

5. (b)

6. (a)

7. (a)

8. (d)

ANSWERS WITH EXPLANATION [A] MULTIPLE CHOICE QUESTIONS 1. Option (b) is correct Explanation: Both IMR (infant mortality rate) and MMR (maternal mortality rate) are responsible for affecting the growth rate inversely. It means decline in IMR as well as MMR will result in high population growth and vice-versa. Hence, if there is an increased IMR and decreased MMR in a population mothers are there to give births to infants whereas the survival rate of infants becomes low. Therefore, in a particular population, increased IMR and decreased MMR will hinder their growth rate. 2. Option (a) is correct Explanation: A national level approach to build up a reproductively healthy society was taken up in India in 1950s. 3. Option (a) is correct Explanation: Breast-feeding is one of the temporary and natural contraceptive methods which reduce the secretion of oestrogen hormone and thereby suppress ovulation (release of egg) and cause amenorrhoea. The contraceptive effect of the lactational amenorrhoea method is a result of increased levels of prolactin. In this condition, production and secretion of gonadotrophin releasing hormones are inhibited and resulted in low secretion of oestrogen. As ovulation cannot occur without a surge in oestrogen levels, and if a woman does not ovulate, pregnancy is prevented. 4. Option (a) is correct Explanation: Emergency contraceptives are effective if they are used within 72 hours of coitus. They can prevent unwanted pregnancies after unprotected sex, failed used of contraceptive devices or due to rape, etc. Two types of emergency contraceptives include emergencies pills or IUDs. 5. Option (c) is correct Explanation: Diaphragms, cervical caps and vaults are contraceptive devices used by females to avoid pregnancies. These devices are made of rubber that is introduced into the female reproductive tract to cover the cervix during coitus. Spermicidal creams, jellies and foams are usually used along with these barriers to increase their contraceptive efficiency. They block the entry of sperm through cervix and thereby prevent conception. They are reusable. 6. Option (d) is correct Explanation: Condoms can both prevent pregnancy by stopping sperm from meeting an egg. They also protect against sexually-transmitted infections (STIs). Condoms act as a

barrier method of contraception. They are made up of very thin latex (rubber) and are designed to prevent pregnancy by stopping sperm from meeting an egg. They do not interfere with coital act. 7. Option (b) is correct Explanation: IUDs are of two types: copper IUD and hormonal IUD. Both types of IUDs work primarily by preventing sperm from reaching and fertilising an egg. The copper IUD releases copper into the uterus, which works as a spermicide and suppresses the motility and fertilising ability of sperm whereas the other IUDs release progestin hormone into the uterus. The progestin thickens the cervical mucous so that sperm can’t reach the egg. 8. Option (c) is correct Explanation: Medical termination of pregnancy is also called induced abortion. It cannot be used as a contraceptive method. MTP is done to get rid from unwanted pregnancies arises due to rape, genetic abnormalities in child, or when mother has some risks by the developing child. MTP is safe up to first trimester of pregnancy, that is, 12 weeks. MTP done in the supervision of medical practitioner under some law 9. Option (b) is correct Explanation: (b) Sexually transmitted diseases are transmitted through sexual intercourse. They are also called as venereal diseases (VDs) or reproductive tract infections (RTI). Examples of STDs are syphilis, gonorrhoea, genital warts, trichomonas, AIDs, etc. In the above-mentioned diseases, AIDS does not affect any sex organ. It is a set of symptoms or illness that arises as a result of advanced HIV infection which has destroyed the immune system. 10. Option (b) is correct Explanation: In zygote intra-Fallopian transfer, the zygote is collected from a female donor and placed in Fallopian tubes rather than directly in the uterus.ZIFT is the result of combining IVF and GIFT. 11. Option (b) is correct Explanation: List-I  List-II (A) Lippes loop   (i) Non-medicated IUDs (B) Vaults   (ii) Barrier (C) Periodic abstinence   (iii) Natural method (D) Progestasert   (iv) Hormone releasing device

42 Oswaal CUET (UG) Chapterwise Question Bank

BIOLOGY

12. Option (b) is correct

19. Option (c) is correct.

Explanation: The terminal method of contraception means surgical methods. In males it is called vasectomy and in females is called tubectomy , due to this transport of gametes blocked. 13. Option (c) is correct

Explanation: Artificial insemination (AI) is a technique in which the semen collected from the husband or a healthy donor is artificially introduced either into the vagina or the uterus. 20. Option (c) is correct.

Explanation: GIFT is the method in which transfer of ovum collected from a donor into the fallopian tube of another female who cannot produce one, but can provide suitable environment for fertilisation takes place. Whereas, ZIFT stands for Zygote intra fallopian transfer which involves transfer of zygote (upto 8 blastomeres) into the fallopian tube. ICSI stands for Intracytoplasmic sperm injection and is the method of injecting the sperms directly into the ovum in-vitro. IVF stands for invitro fertilisation. 14. Option (a) is correct Explanation:. There are three different IUD types: (i) Non-medicated IUDs—These devices, such as Lippes loops, promote the phagocytosis of sperm within the uterus. (ii) Copper-releasing IUDs—In addition to phagocytosis of the sperms, the released copper ions also inhibit sperm motility and sperm fertility. For example: Multiload-375, Cu-T, and Cu-7. (iii) Hormone-releasing IUDs, such as progestasert and LNG-20, render the uterus unsuitable for implantation and the cervix hostile to sperm. Vaults are a type of barrier method which blocks the entry of sperm and prevents conception. 15. Option (b) is correct Explanation: Lactational amenorrhea is also known as postpartum infertility Lactation means your body is making breastmilk and amenorrhea refers to absence of monthly period in females. It is a method of natural contraception, which is based on the fact that the process of ovulation does not occur during the period of intense lactation. 16. Option (b) is correct Explanation: Artificial insemination (AI) technique is useful for the male partner having inability to inseminate female or has low sperm counts. It is the method of transferring semen (sperm) collected from the husband or a healthy donor into the vagina or the uterus (IUI-intra-uterine insemination) of the recipient Female Zygote Intra Fallopian Transfer (ZIFT) involves transfer of zygote or early embryos (with up to 8 blastomeres) into fallopian tube. Gamete Intra Fallopian Transfer (GIFT) is the method of transfer of gamete (ovum) from a donor into the fallopian tube of another female who is unable to produce ovum, but can provide right conditions for fertilization and development of an embryo. Intra Uterine Transfer (IUT) involves transfer of embryos with more than 8 blastomeres into the uterus. The babies thus produced from this method are known as test tube babies. 17. Option (c) is correct. Explanation: Test tube baby is a term used to refer to a child that is conceived outside the women’s body by a scientific process. The entire process is done in a laboratory. In this method, ova from the wife/donor and sperms from the husband/donor are collected and are fertilised to form zygote under simulated conditions in the laboratory. The fertilised ovum is then placed inside the uterus of a female. 18. Option (a) is correct. Explanation: Cu ions in Cu releasing IUDs suppresses sperm motility and fertilising capacity of sperm.

Explanation: Gonorrhoea caused by Neisseria gonorrhoeae, Syphilis caused by Treponema pallidum, Genital herpes caused by HSV-2. 21. Option (b) is correct. Explanation: In vitro fertilisation, the zygote or early embryos develop up to 8 blastomeres is transferred into the fallopian tube. 22. Option (c) is correct. Explanation: In artificial insemination [AI] technique, the semen collected either from the husband or a healthy donor is artificially introduced either into the vagina or into the uterus (IUI—Intra Uterine Insemination) of the female. 23. Option (b) is correct. Explanation: In zygote intra-Fallopian transfer, the zygote is collected from a female donor and placed in Fallopian tubes rather than directly in the uterus. ZIFT is the result of combining IVF and GIFT. 24. Option (c) is correct. Explanation: Vasectomy is the correct surgical procedure as a contraceptive method in male. During this procedure, vas deferens is cut down to prevent sperm from entering into the urethra and thereby preventing fertilisation. In female, this procedure is called tubectomy in which Fallopian tubes are held and blocked or severed and sealed, either of which prevents eggs from reaching the uterus for implantation. Removal of ovary, uterus and testes are called as ovariectomy, hysterectomy and castration respectively. 25. Option (b) is correct. Explanation: Oxytocin is produced in the hy­pothalamus and is secreted by the posterior pituitary gland. It causes contraction during parturition and help to brings baby out from the mother womb. 26. Option (c) is correct. Explanation: Intra Uterine Devices (IUDs) presently available are: (A) The non-medicated IUDs e.g., Lippes loop. (B) Copper releasing IUDs e.g., CuT, Cu7, Multiload 375. (C) The hormone releasing IUDs e.g., LNG- 20 and Protesters. These IUDs increase phagocytosis of sperms within the uterus and the Cu ions released suppress sperm motility and the fertilising capacity of sperms. The hormone releasing IUDs, in addition,make the uterus unsuitable for implantation & the cervix hostile to the sperms. 27. Option (b) is correct. Explanation: In-vitro fertilisation is the technique in which the fertilisation takes place outside the body but the conditions are similar as that present in the body. Fertilisation process is followed by embryo transfer (ET). In this process the zygote or early embryos (with up to 8 blastomeres) are transferred into the fallopian tube.

43

REPRODUCTIVE HEALTH

28. Option (c) is correct.

6. Option (a) is correct.

Explanation: Condoms are barriers, which are made up of thin rubber or latex sheath. They are used to cover the penis in male or vagina and cervix in the female. It is a type of barrier method which prevent the physical meeting of ovum and sperms. Vasectomy is surgical intervention, which blocks gamete transport thus prevents conception. This sterilisation procedure in the male is called vasectomy. In this a small part of the vas deferens is removed or tied up, where as in tubectomy, a small part of the fallopian tube is removed or tied up. Pills are small doses of either progestogens or progestogen estrogen combination, used orally by females in the form of tablets

Explanation: Absence or less significant symptoms in the early stages of infection and the social stigma attached to the STDs deter the infected persons from going for timely detection and proper treatment leading to pelvic inflammatory diseases (PID), abortions, still births, ectopic pregnancies, infertility or even cancer of the reproductive tract. 7. Option (d) is correct.

29. Option (d) is correct. Explanation: Condoms can both prevent pregnancy by stopping sperm from meeting an egg. They also protect against sexually-transmitted infections (STIs). Condoms act as a barrier method of contraception. They are made up of very thin latex (rubber) and are designed to prevent pregnancy by stopping sperm from meeting an egg. They do not interfere with coital act. 30. Option (c) is correct. Explanation: Health may be defined as a person’s mental and physical condition. It is a state of complete physical, mental and social well-being and not merely the absence of disease or infirmity. When people are healthy, they are more efficient in their work. It also increases longevity of people and reduces infant and maternal mortality. [B] ASSERTION & REASON QUESTIONS. 1. Option (a) is correct. Explanation: ‘Lactational amenorrhoea’ means lack of menses resulting from breast-feeding. During this period high level of prolactin and a reduction of gonadotropin-releasing hormone from the hypothalamus suppress ovulation. 2. Option (a) is correct. Explanation: ‘Saheli’ is considered as an improved form of contractive for a human female because: (i) it is non-steroidal preparation. (ii) it has lesser or no side effects. (iii) it has a high contraceptive value. (iv) It is once a week pill. 3. Option (a) is correct. Explanation: In infertility cases artificial insemination (AI) techniques are used to cure the patient. 4. Option (b) is correct. Explanation: When the fertilisation become successful than the woman become pregnant but it become terminated is some case intentionally if she is unmarried or not didn’t planned for babies in some case, abortion happen accidentally or negligence in case of pregnant mother. 5. Option (d) is correct. Explanation: Diaphragms are contraceptive devices used by the females to avoid pregnancies. These devices are made of rubber that is introduced into the female reproductive tract to cover the cervix during coitus. They act as physical barriers for sperm entry.

Explanation: Vasectomy is a sterilisation procedure in the male while that in the female is called tubectomy. In vasectomy, a small part of the vas deferens is removed or tied up through a small incision on the scrotum while in tubectomy a small part of the fallopian tube is removed or tied up through a small incision in the abdomen or through vagina. 8. Option (d) is correct. Explanation: The interstitial spaces are the regions outside the seminiferous tubules, contains small blood vessels and interstitial cells or Leydig cells. Each seminiferous tubule is lined on its inside by two types of cells called male germ cells (spermatogonia) and Sertoli cells. Sertoli cells provide nutrition to the germ cells during spermatogenesis. 9. Option (b) is correct Explanation: Determining the sex of an unborn child followed by MTP is an illegal process in India , amniocentesis is a technique to test the presence of genetic disorder also. 10. Option (b) is correct Explanation: ICSI is an assisted reproductive technique in this process sperm is directly injected into ovum to form the embryo in vitro [C] COMPETENCY/CASE-BASED QUESTIONS: I. 1.  Option (d) is correct. Explanation: Hepatitis B is a serious infection of the liver caused by a virus. Symptoms may include tiredness, loss of appetite, stomach discomfort and yellow skin. The virus is found in blood, semen, vaginal fluids and saliva. Hepatitis B is the only sexually transmitted disease that has a safe and effective vaccine to protect against infection. 2. Option (c) is correct. Explanation: The most common manifestation of chlamydia infection is infection of the cervix with inflammation (cervicitis) in women. 3. Option (d) is correct. Explanation: HIV is transmitted through the exchange of certain types of bodily fluids including: blood, semen, breast milk, and vaginal fluids 4. Option (d) is correct. Explanation: Trichomonas is a very common sexually transmitted disease (STD). It is caused by infection with a protozoan parasite called Trichomonas vaginalis. 5. Option (b) is correct. Explanation: A person with first stage of syphilis generally has a sore or sores at the original site of infection. These sores usually occur on or around the genitals. These sores are usually (but not always) firm, round, and painless. Symptoms of secondary syphilis include skin rash, swollen lymph nodes, and fever. Tertiary syphilis is associated with severe medical problems. It can affect the heart, brain, and other organs of the body.

44 Oswaal CUET (UG) Chapterwise Question Bank II. 6.

Option (a) is correct.

BIOLOGY

8. Option (d) is correct.

Explanation: Amniocentesis is the technique of sex determination of embryo in pregnancy using amniotic fluid from mothers womb.

Explanation: After giving birth to the child, women lactates milk for feeding baby and there is no chances of fertilisation till 6 months.

7. Option (a) is correct.

9. Option (a) is correct.

Explanation: The lactational amenorrhea is a result of increased level of prolactin . In this condition, production and secretion of Gonadotrophin releasing hormone (GnRH) from the hypothalamus is inhibited which results in low secretion of luteinising hormone (LH). An ovulation cannot occur without a surge of LH.

Explanation: Abstinence is the prevention of birth control by eliminating all the chances of fertilisation of egg with sperm.

24

10. Option (d) is correct Explanation: Lactic acid, boric acid and citric acid all are used as chemical contraceptive methods.

UNIT II: Genetics and Evolution Study Time

CHAPTER

Max. Time: 1:5 Hours Max. Questions: 50

5

HEREDITY AND VARIATION { Collect pollen grains from the other plant (male parent)

 Revision Notes Heredity: {  Heredity,

derived from the Latin word “hereditas,” refers to the transfer of specific characteristics from parents to offspring. { It encompasses all biological processes responsible for passing on traits, whether through asexual or sexual reproduction.

Variation: { Variation pertains to the natural tendency for differences

to exist among the traits of individuals within a progeny. differences can manifest as distinctions between offspring and their parents, highlighting the diversity within a population.

{ These

Mendel’s Laws of Inheritance { Gregor

Mendel conducted groundbreaking hybridi­sation experiments on garden peas from 1856 to 1863, leading to the formulation of the laws of inheritance.

Trait Studied Seed Colour Seed Texture Flower Colour Flower Position Pod Colour Pod Shape Stem Length

Scan to know more about this topic

Mendel´s Laws - Inheritance

Contrasting Traits Yellow vs. Green Smooth vs. Wrinkled Purple vs. White Axial vs. Terminal Green vs. Yellow Inflated vs. Constricted Long vs. Short

These traits were instrumental in Mendel’s development of the principles of heredity and the laws of inheritance.

Monohybrid Cross: { A

monohybrid cross involves two plants that differ in one pair of contrasting characters, typically controlled by a single gene. { For example, Mendel crossed tall and dwarf pea plants to study the inheritance of a single gene governing plant height. Steps in Making a cross of Pea: { Select two pea plants with contrasting characters, such as tall and dwarf. {  Remove the anthers (emasculation) of one plant to prevent self-pollination; this plant becomes the female parent.

and transfer them to the flower of female parent for pollination. {  Collect seeds produced by the female parent, which represent the offspring. { Mendel made similar observations for other pairs of traits and proposed that these traits are due to inherited “factors” which were later called genes. { The F1 generation (Tt) produces gametes T (allele for tall stem) and t (allele for short stem) in equal proportion when self-pollinated. { Mendel self-pollinated the F2 generation plants and found that dwarf F2 plants continued to generate dwarf plants in the F3 and F4 generations. { He concluded that the genotype of the dwarf plants was homozygous (tt). Monohybrid Phenotypic Ratio: 3 Tall : 1 Dwarf = 3 : 1 Monohybrid Genotypic Ratio: 1 Homozygous tall (TT) : 2 Heterozygous tall (Tt) : 1 Homozygous dwarf (tt) Back Cross and Test Cross: { A back cross involves the crossing of an F1 hybrid with either of its parent plants. { A test cross is the crossing of an F1 hybrid with its recessive parent. It is used to determine the unknown genotype of the F2 generation.

Mendel’s Principles or Laws of Inheritance: Principle of Dominance: { Characters

are governed by discrete units known as factors (later called genes). { These factors occur in pairs on homologous chromosomes. { In a dissimilar pair of factors (contrasting alleles), only one member of the pair (dominant allele) expresses its effect in the hybrid. { The manifestation of the other allele (recessive allele) is masked in the presence of the dominant allele. { Dominant alleles determine the observable traits (phenotype) in heterozygous individuals. { Recessive alleles are only expressed when an individual is homozygous for the recessive allele. { The dominance relationship is specific to each gene and its alleles. { Mendel’s experiments with pea plants demonstrated the principle of dominance and led to the formulation of this fundamental genetic concept.

T

Tt

Tt

Tt

t

Selfing T T

Tt

T

Flower position

Pod shape

Pod colour

Seed shape

Seed colour

2.

3.

4.

5.

6.

7.

Yellow/green

Round/ wrinkled

Green/yellow

Inflated/constricted

Axial/terminal

Violet/white

Tall/dwarf

t

Tall Tt Gametes

t

Dwarf tt Gametes

F2 generation

ry

Gametes

F1 generation

Gametes

P generation

RrYy

Ry

ry

rrYy

RRYy

rY

Rryy

RrYy

rryy

Rryy

RrYy

rrYY

RRyy

RY

RrYY

RRYY

RrYY

RY

rrYy

RRYy

Yy

rY

Round yellow RrYy Selfing

RY

RrYy

Ry

Gametes ry

Round yellow Wrinkled green yy rr YY

When two pairs of traits are combined in a hybrid, segregation of one pair of characteristics is independent of the other pair of characters.

r Rr

R

rr

RR Rr

R

All pink (Rr)

r

r

Gametes

Phenotypic ratio : red : pink : white 1 : 2 : 1 Genotypic ratio : RR : Rr : rr 1 : 2 : 1

F1 generation

Gametes

R

Red (RR) White (rr)

F1 generation

Gametes

P generation

The heterozygous offspring shows intermediate character between two parental characteristics. Phenotypic ratio and Genotypic ratio are same. e.g., Flower colour in Antirrhinum sp. and Mirabilis jalapa.

Gregor

d Mendel pose Pro ws a l ee thr

Genetic Disorders

n ity a d V

Two alleles of a gene are equally dominant and express themselves even when they are together. e.g., ABO blood grouping in human.

A gene exists in more than two allelic forms e.g. , ABO blood grouping

Non-Mendelian Inheritance

Hered

Contrasting Traits

During gamete formation, the factors (alleles) of a character pair present in parents segregate from each other such that a gamete receives only one of the two factors.

tt Phenotypic ratio : tall : dwarf : 3 1 Genotypic ratio : TT : Tt : tt 1 : 2 :1

t TT F2 generation all Tt Tt

T Tt Gametes

F1 generation

Tall TT Gametes

Flower colour

1.

In heterozygous condition, only one member of a pair expresses itse l f and the hybrid is called as dominant while the manifestation of the other is masked and is known as recessive

Characters

Stem height

S.No.

These are caused due to absence or excess or abnormal arrangement of one or more chromosomes. Example, Down's syndrome (trisomy of 21). Klinefelter's Syndrome (XXY in male). Turner's syndrome (XO in female).

First Level

Second Level

Trace the Mind Map Third Level

Ability of a gene to have multiple phenotypic effects as it influences a number of characters simultaneously.

Chromosomes are mutable.

Chromosomes are immortal. Two identical chromosomes form a homologous pair. They segregate at the time of gamete formation.

Proposed by Walter Sutton and Theodore Boveri in 1902. Thomas Hunt Morgan formulated chromosomal theory of inheritance using fruit flies (Drosophila melanogaster). Morgan coined the term Linkage.

Chromosomal Theory of inheritance

ZZ—ZW mechanism - Example: Birds XX—XO mechanism - Example: grasshopper XX–XY mechanism -Example: Human being

Sex Determination



tion 

ia ar 

These are caused by alteration or mutation in the single gene. Autosomal dominant: e.g., Muscular dystrophy. Autosomal recessive: e.g., Sickle cell anaemia, Albinism Sex linked: e.g., Haemophilia.

46 Oswaal CUET (UG) Chapterwise Question Bank BIOLOGY

47

HEREDITY AND VARIATION

Example of the Principle of Dominance: us consider the gene responsible for flower colour in a particular plant species. { There are two alleles for this gene: “R” for red flowers (dominant) and “r” for white flowers (recessive). { When a plant has two dominant alleles (RR), it will produce red flowers. { When a plant has one dominant allele and one recessive allele (Rr), it will also produce red flowers because the dominant allele (R) masks the effect of the recessive allele (r). { Only when a plant is homozygous for the recessive allele (rr) it will produce white flowers. { In this example, the principle of dominance is illustrated by the fact that the dominant allele (R) determines the flower colour (red) in both homozygous dominant (RR) and heterozygous (Rr) individuals, while the recessive allele (r) is only expressed in homozygous recessive (rr) individuals, resulting in white flowers.



{ Let

Law of Segregation: { Allelic

pairs (gene variants) segregate during gamete formation. { Each individual has two alleles for each gene, one inherited from each parent. { These alleles segregate into separate gametes during the formation of eggs and sperm. { The segregation is random, meaning that each gamete receives one allele, and it is a matter of chance which allele is passed on. { Homozygous individuals have two identical alleles for a gene (e.g., RR or rr), and they produce gametes containing only one type of allele (R or r). { Heterozygous individuals have two different alleles for a gene (e.g., Rr), and they produce gametes with an equal chance of containing either allele (R or r). { When fertilisation occurs, the alleles from the male and female gametes unite, restoring the diploid number of alleles in the offspring. { The Law of Segregation ensures genetic diversity in offspring and is a fundamental principle of Mendelian genetics.

Example of the Law of Segregation:

1. Parental Generation: { In the parental generation, there are two pea plants: one

with purple flowers (genotype PP) and one with white flowers (genotype pp). { The purple-flowered plant has two identical dominant alleles (PP), and the white-flowered plant has two identical recessive alleles (pp).

2. Gamete Formation (Segregation): { During

gamete formation in the purple-flowered plant (PP), it produces gametes with the allele for purple flowers (P). { Similarly, the white-flowered plant (pp) produces gametes with the allele for white flowers (p).

3. Fertilisation (Random Union): { When these gametes unite through fertilisation, random

pairing occurs. The offspring inherit one allele from each parent. { Possible combinations include PP (homozygous dominant), Pp (heterozygous), and pp (homozygous recessive).

4. Offspring Phenotypes: { The

resulting offspring may have the following phenotypes: Homozygous Dominant (PP): Purple flowers. Heterozygous (Pp): Purple flowers (due to the dominance of the P allele). Homozygous Recessive (pp): White flowers. In this example, the Law of Segregation is evident as alleles (P and p) segregate into separate gametes during gamete formation. The random union of gametes during fertilisation leads to the inheritance of different combinations of alleles, resulting in varying flower colours in the offspring (purple or white).

Law of Independent Assortment: { It

states that when more than one pair of contrasting characters (genes) is involved in a cross, the segregation of one pair of contrasting characters is independent of the segregation of another pair of contrasting characters. {  In other words, the inheritance of one trait does not influence the inheritance of another trait during gamete formation. Key Points of the Law of Independent Assortment:

1. Multiple Gene Pairs: The law applies when considering the inheritance of traits governed by different gene pairs (genes located on different chromosomes).



2. Random Assortment: The segregation of alleles for one trait into gametes is independent of the segregation of alleles for another trait. It occurs randomly.



3. Dihybrid Cross: The law is most evident in dihybrid crosses, which involve the study of two pairs of alleles for two different traits simultaneously.

Example of Law of Independent Assortment: Let us consider a dihybrid cross involving pea plants with two pairs of contrasting traits: seed colour and seed shape. {  Trait 1 (Seed Colour): There are two alleles, “Y” for yellow seeds (dominant) and “y” for green seeds (recessive). { Trait 2 (Seed Shape): There are two alleles, “R” for round seeds (dominant) and “r” for wrinkled seeds (recessive). Cross Involving Seed Colour (Y/y) and Seed Shape (R/r): Parental Generation (P): { Plant 1: Yellow seeds and round seeds (genotype: YYRR) { Plant 2: Green seeds and wrinkled seeds (genotype: yyrr) Gamete Formation: { Each parent produces gametes (sperm and egg cells) with alleles for both traits. { Plant 1 (YYRR) produces gametes with alleles YR, YR, YR, and YR.

48 Oswaal CUET (UG) Chapterwise Question Bank { Plant

2 (yyrr) produces gametes with alleles yr, yr, yr, and yr. { Resultant progeny has YyRr genotype and yellow, round seeds. This is F1 generation. Fertilisation in F1 generation (Punnett’s Square): { Plant 1 (YYRR) produces gametes with alleles YR, Yr, yR, and yr.

Offspring Phenotypes: { The genotypes lead to different phenotypes for seed colour and seed shape. { The phenotypic ratios among the offspring include various combinations of yellow/green seed colour and round/wrinkled seed shape. Phenotypic Ratio: { The phenotypic ratio among the offspring is typically 9:3:3:1, which means that the offspring exhibit four different combinations of phenotypes: 1. Yellow and round (YyRr): 9 individuals 2. Yellow and wrinkled (Yyrr or YyRr): 3 individuals 3. Green and round (yyRr or YyRr): 3 individuals 4. Green and wrinkled (yyrr): 1 individual This dihybrid cross illustrates the Law of Independent Assortment, as the segregation of alleles for seed colour (Y/y) occurs independently of the segregation of alleles for seed shape (R/r). The offspring exhibit various combinations of genotypes and phenotypes, showcasing the independent assortment of these two pairs of contrasting traits.

Non-Mendelian Inheritance:

(a)  Incomplete Dominance: { Incomplete

dominance occurs when the heterozygous offspring exhibit an intermediate phenotype that is a blend or mixture of the two parental characteristics.

BIOLOGY

{ Plant

2 (yyrr) produces gametes with alleles YR, Yr, yR, and yr Let’s use a Punnett’s square to show the possible combinations of alleles when these gametes combine: Offspring Genotypes: { As shown in the Punnett’s square, there are 16 possible combinations of alleles among the offspring.

{ An

example of incomplete dominance is seen in flower colour in Snapdragon (Antirrhinum sp.) and Mirabilis jalapa (4 o’clock plant). Example 1: Snapdragon Flower Colour: { In Snapdragon flowers, there are two alleles for colour: “R” for red and “W” for white. { Homozygous RR plants have red flowers, homozygous WW plants have white flowers, but heterozygous RW plants have pink flowers. { The phenotype of the heterozygous offspring (RW) is intermediate between the two homozygous parental phenotypes. Example 2: 4 o’clock Plant Flower Colour: { In the 4 o’clock plant (Mirabilis jalapa), there are two alleles for flower colour: “R” for red and “W” for white. { Homozygous RR plants have red flowers, homozygous WW plants have white flowers, and heterozygous RW plants have flowers of an intermediate colour , often described as pink or variegated. (b) Co-dominance: { Co-dominance

occurs when both alleles of a gene are expressed equally and independently in a hybrid, without blending. Example: ABO Blood Grouping in Humans: { The ABO blood group is determined by a gene with three alleles: IA, IB, and i.

49

HEREDITY AND VARIATION { Allele

IA codes for the A antigen on the surface of red blood cells. { Allele IB codes for the B antigen on the surface of red blood cells. { Allele i does not produce any antigen and is recessive. {  Individuals with two dominant alleles (IAIA or IBIB) express either A or B blood types, respectively. A B { Heterozygous individuals (I I ) express both A and B antigens equally, resulting in the AB blood type. {  The co-dominance of IA and IB alleles leads to the presence of both A and B antigens on the surface of red blood cells in AB blood type individuals. (c)  Multiple Allelism: { Multiple

allelism occurs when more than two alleles govern the same trait or character, but each individual has only two alleles. Example: ABO Blood Grouping { In the ABO blood group system, there are three alleles: IA, IB, and i. A B { I and I are co-dominant alleles, while i is recessive. { In a population, individuals can have combinations of these three alleles, leading to different blood types (A, B, AB, and O) based on the specific allele combinations present in an individual. (d) Pleiotropy: { Pleiotropy occurs when a single gene controls multiple,

seemingly unrelated traits or characteristics. Example: Pea Plant Seed Texture and Starch Grain Size: { In Mendel’s pea plants, there is a gene called “R” that controls both seed texture and starch grain size. { Homozygous RR plants have round seeds and large starch grains. { Homozygous rr plants have wrinkled seeds and small starch grains. { The same gene (R) affects both the texture of the seeds (round or wrinkled) and the size of starch grains (large or small). Pleiotropy in this case demonstrates that a single gene can have a broad influence on multiple traits or characteristics, even when those traits may seem unrelated at first glance. { It often involves the impact of a gene on multiple metabolic pathways.

Chromosomal Theory of Inheritance: { Gregor

Mendel published his groundbreaking work on inheritance in 1865, introducing the principles of heredity. { Unfortunately, Mendel’s work remained largely unrecognised until 1900 due to limited communication, resistance to his gene concept, and the mathematical nature of his research. { In 1900, three scientists independently rediscovered Mendel’s work, bringing his concepts to the forefront of genetics. { Advances in microscopy allowed scientists to observe cell division and discover chromosomes, providing a critical link between Mendel’s laws and physical structures.

{ Walter

Sutton and Theodore Boveri further connected chromosome behaviour to Mendel’s laws, proposing the chromosomal theory of inheritance. { The chromosomal theory revealed that chromosomes and genes occur in pairs, with alleles located on homologous sites. { During meiosis, chromosomes segregate independently, similar to how genes assort independently.

Linkage and Recombination

Scan to know Hunt Morgan’s research more about on Drosophila (fruit flies) involved this topic dihybrid crosses. { Morgan observed that genes on the same chromosome could be linked, deviating from the expected Linkage, 9:3:3:1 phenotypic ratio predicted Recombination by Mendel. { The terms “linkage” and “recombination” were introduced to describe the physical association of genes on a chromosome and the generation of non-parental gene combinations. { Some genes on the same chromosome are tightly linked (low recombination), while others are loosely linked (high recombination). {  Genetic maps, based on recombination frequencies, were developed to help determine the relative positions of genes on chromosomes. { Thomas

Polygenic Inheritance: { While

Mendel studied traits with distinct alternate forms controlled by single genes, many traits exhibit continuous variation and are controlled by multiple genes. These are called polygenic traits. { Polygenic traits involve additive effects of alleles from multiple genes. { Human traits like height and skin colour are classic examples of polygenic inheritance.

Sex Determination: { In

many organisms, sex determination is based on differences in sex chromosomes. 1. Sex Determination in Humans: { Human sex determination is based on the presence of X or Y chromosomes. { Males have one X and one Y chromosome (XY), while females have two X chromosomes (XX). 2. Sex Determination in birds: { Birds, sex determination is based on the presence of Z and W chromosomes where females have ZW chromosomes, and males have ZZ chromosomes. 3. Sex Determination in Honey Bee: { Honey bees have haplodiploid sex determination, where females have 32 chromosomes (diploid) and males have 16 chromosomes (haploid). { Males develop from unfertilised eggs.

Sex-Linked Inheritance: { Sex-linked inheritance refers to the inheritance of genes

located on the sex chromosomes, X and Y.

50 Oswaal CUET (UG) Chapterwise Question Bank { In

humans, males have one X and one Y chromosome (XY), while females have two X chromosomes (XX). { Genes located on the X chromosome are called X-linked genes and play a significant role in sex-linked inheritance.



Examples of X-linked Inheritance:

1. Hemophilia: { Hemophilia

is an X-linked recessive disorder. affects the blood’s ability to clot properly. { Males who inherit one X-linked hemophilia allele from their carrier mother are more likely to develop the disorder because they lack a second X chromosome with a healthy allele. { Example: Queen Victoria of England passed the hemophilia allele to several of her descendants, leading to its historical association as the “Royal Disease.” { It



2. Colour Blindness: { Red-green

colour blindness is another X-linked recessive trait. { Affected individuals have difficulty of distinguishing between red and green colours. { Since the gene for colour vision is located on the X chromosome, it is more common in males, who have only one X chromosome. { Example: A son may inherit the colour blindness allele from his mother, who is a carrier but does not exhibit the trait herself.

exhibit the disorder because they lack a second healthy X chromosome. { Example: DMD primarily affects males, while females are usually carriers, passing the mutated allele to their sons.

Scan to know more about this topic

Sex linked inheritance

3. Duchenne’s Muscular Dystrophy (DMD): { Duchenne’s

muscular dystrophy is an X-linked recessive disorder. { It leads to progressive muscle weakness. { Males with a single mutated X-linked DMD allele often

BIOLOGY

4. X-Linked Dominant Inheritance: { Some

X-linked disorders are dominant, meaning that one mutated allele is sufficient to cause the condition. { Affected individuals can inherit the condition from an affected parent of either sex. { Example: Rett syndrome is an X-linked dominant disorder, and females who inherit a mutated allele often exhibit the condition.

Genetic Disorders: Pedigree Analysis { Pedigree analysis is the study of the inheritance patterns of traits in families over generations. { It helps trace the inheritance of genetic disorders in humans. Mendelian Disorders { Mendelian disorders are caused by mutations in single genes. { Examples include haemophilia, cystic fibrosis, sicklecell anaemia, colour blindness, phenylketonuria, and thalassemia. { Mendelian disorders can be dominant or recessive. Chromosomal Disorders { Chromosomal disorders result from alterations in chromosome number or structure. { Examples include Down’s syndrome (trisomy 21), Klinefelter’s syndrome (XXY), and Turner’s syndrome (X0). { Down’s syndrome results from an extra chromosome 21, while Klinefelter’s and Turner’s syndromes involve sex chromosome abnormalities.

OBJECTIVE TYPE QUESTIONS [A] MULTIPLE CHOICE QUESTIONS: 1. Match list I with list II List I

List II

(A) Metabolic Disorder

I. Family tree over generation

(B) Pedigree Analysis

II. Blood disease

(C) Thalassemisa

III. Phenylketonuria st

(D) Trisomy of 21 chromosome (a) (b) (c) (d)

IV. Down’s Syndrome

Choose the correct answer from the options given below: [CUET 2023] A-II, B-IV, C-III, D-I A-1, B-III, C-II, D-IV A-III, B-1, C-II, D-IV A-III, B-II, C-IV, D-1

2. Which of the following is not a Mendelian Disorder? [CUET 2023] (a) Haemophilia (b) Sickle-cell anaemia (c) Down’s Syndrome (d) Phenylketonuria 3. One of the following is NOT a characteristic/criteria of genetic materials, identify it: [CUET 2023] (a) Genetic material should be able to generate its replica. (b) Genetic material should be stable chemically and structurally. (c) It should not provide the scope for mutations. (d) It should be able to express itself in the form of Mendelian characters. 4. A pure breeding garden pea plant was crossed with a pure dwarf plant. The plant produced 400 seeds. The seeds were sown to produce plants. The phenotype of the plants in next generation will be: [CUET 2023] (a) All tall (b) All dwarf

51

HEREDITY AND VARIATION

(c) 300 tall and 100 dwarf plants (d) All plants of intermediate height 5. Father’s blood group is A and mother’s blood group is B. One of their offspring is with group AB. What is the percentage of probability of AB blood group offspring? [CUET 2022] (a) Nil (c) 50%



(b) 25% (d) 75%

6. Match List I with List II

[CUET 2022]

List I

List II

A.

Phenylketonuria

I. Sex linked recessive disease

B.

Down’s syndrome

II. Autosomal Recessive disease

C.

Turner’s syndrome

III. Trisomy of chromosome number 21

D.

Hemophilia

IV. Monosomy of X chromosome

Choose the correct answer from the options given below: (a) A-I, B-III, C-IV, D-II (b) A-II, B-III, C-IV, D-I (c) A-II, B-IV, C-III, D-I (d) A-III, B-II, C-IV, D-I 7. The haplodiploid sex-determination system where all the males of the population are haploid and females are diploid, is seen in:  [CUET 2022] (a) Drosophila (b) Honey bees (c) Birds (d) Grasshoppers 8. How many gametes will be formed in the pea plant having genotype RrTT? [CUET 2022] (a) One (b) Two (c) Four (d) Eight 9. (a) (b) (c) (d)

The percent crossing over is high when genes are unlinked. genes are located on different chromosomes. genes are linked and are very close to each other. genes are linked and far away from each other.

10. Sex in honeybee is determined by (a) X : A ratio. (b) the presence of Y chromosome. (c) environmental temperature. (d) haplodiploidy.

[CUET 2022]

11. ZZ / ZW type of sex determination is seen in: (a) Platypus (b) snails. (c) cockroach (d) peacock 12. A human female with Turner’s syndrome: (a) has 45 chromosome with XO (b) has one additional X chromosome (c) exhibits male characters (d) is able to produce children with normal husband

13. A woman has an X-linked condition on one of her X-chromosomes. This chromosome can be inherited by  (a) only daughters (b) both sons and daughters (c) only grandchildren (d) only sons 14. A person with sickle cell anaemia is: (a) more prone to malaria. (b) more prone to typhoid. (c) less prone to malaria. (d) less prone to typhoid. 15. If a genetic disease is transferred from a phenotypically normal but carrier female to only some of the male progeny, the disease is (a) autosomal dominant. (b) autosomal recessive. (c) sex-linked dominant. (d) sex-linked recessive 16. The genotype of a plant showing the dominant phenotype can be determined by:  (a) test cross (b) dihybrid cross (c) pedigree analysis (d) back cross. 17. What is the genetic disorder in which an individual has an overall masculine development, gynaecomastia, and is sterile ?  (a) Turner’s syndrome (b) Klinefelter’s syndrome (c) Down’s syndrome (d) Edward syndrome 18. If both parents have sickle cell trait, then there is _______________ of the child having sickle cell anaemia. (a) 25 % risk (b) 50 % risk (c) 75% risk (d) No risk 19. F2 generation in a Mendelian cross showed that both genotypic and phenotypic ratios are same as 1 : 2 : 1. It represent a case of: (a) Co-dominance. (b) Dihybrid cross. (c) Monohybrid cross with incomplete dominance. (d) Monohybrid cross with complete dominance. 20. Which of the following pairs is wrongly matched? (a) Starch synthesis in pea: Multiple alleles (b) ABO blood grouping: Co-dominance (c) Flower colour in Snapdragon: Incomplete dominance (d) T.H. Morgan: Linkage 21. A cross between two tall plants resulted in offspring having few dwarf plants. What would be the genotypes of both the parents? (a) TT and Tt (b) Tt and Tt (c) TT and TT (d) Tt and tt 22. In a dihybrid cross, if you get 9 : 3 : 3 : 1 ratio it denotes that:  [NCERT Exemplar, Q. 9, Pg. 30] (a) the alleles of two genes are interacting with each other. (b) it is a multigenic inheritance. (c) it is a case of multiple allelism. (d) the alleles of two genes are segregating independently

52 Oswaal CUET (UG) Chapterwise Question Bank 23. Person having genotype IA IB would show the blood group as AB. This is because of [NCERT Exemplar, Q. 6, Pg. 30] (a) pleiotropy. (b) co-dominance. (c) segregation. (d) incomplete dominance. 24. Mendel’s Law of independent assortment holds good for genes situated on the:  [NCERT Exemplar, Q. 11, Pg. 31] (a) non-homologous chromosomes. (b) homologous chromosomes. (c) extra nuclear genetic element. (d) same chromosome. 25. Occasionally, a single gene may express more than one effect. The phenomenon is called:  [NCERT Exemplar, Q. 12, Pg. 31] (a) multiple allelism. (b) mosaicism. (c) pleiotropy. (d) polygeny. 26. It is said that Mendel proposed that the factor controlling any character is discrete and independent. This proposition was based on the: [NCERT Exemplar, Q. 15, Pg. 31] (a) results of F3 generation of a cross. (b) observations that the offspring of a cross made between the plants having two contrasting characters shows only one character without any blending. (c) self-pollination of F1 off springs. (d) cross pollination of parental generations. 27. Two genes ‘A’ and ‘B’ are linked. In a dihybrid cross involving these two genes, the F1 heterozygote is crossed with homozygous recessive parental type (aa bb). What would be the ratio of offspring in the next generation?  [NCERT Exemplar, Q. 16, Pg. 32] (a) 1 : 1 : 1 : 1 (b) 9 : 3 : 3 : 1 (c) 3 : 1 (d) 1 : 1 28. Which of the following will not result in variations among siblings? [NCERT Exemplar, Q. 10, Pg. 30] (a) Independent assortment of genes (b) Crossing over (c) Linkage (d) Mutation 29. All genes located on the same chromosome:  [NCERT Exemplar, Q. 1, Pg. 29] (a) form different groups depending upon their relative distance. (b) form one linkage group. (c) will not from any linkage groups. (d) form interactive groups that affect the phenotype. 30. Distance between the genes and percentage of recombina­ tion shows: [NCERT Exemplar, Q. 3, Pg. 29] (a) a direct relationship. (b) an inverse relationship. (c) a parallel relationship. (d) no relationship.

BIOLOGY

[B] ASSERTION & REASON: Directions: Each question consists of two statements, namely, Assertion (A) and Reason (R).For selecting the correct answer, use the following code: (a) Both Assertion (A) and Reason (R) are the true and Reason (R) is a correct explanation of Assertion (A). (b) Both Assertion (A) and Reason (R) are the true but Reason (R) is not a correct explanation of Assertion (A). (c) Assertion (A) is true and Reason (R) is false. (d) Assertion (A) is false and Reason (R) is true. 1. Assertion (A): In F2 generation of a Mendelian cross, the traits seen in the progeny are identical to their parents. Reason (R): The progeny of the F2 generation in a Mendelian cross shows no blending of traits. [CUET 2022] 2. Assertion (A): Hybrids result from a cross between two genetically unlike parents. Reason (R): Hybrid vigour is the superiority of hybrid over either of the parents 3. Assertion (A): Haemophilia is an autosomal disorder. Reason (R): A haemophilic father can never pass the gene for haemophilia to his son. 4. Assertion (A): Incomplete dominance is a genetic phenomenon where heterozygous individuals exhibit an intermediate phenotype. Reason (R): Incomplete dominance results from the blending of alleles in heterozygous individuals. 5. Assertion (A): The Law of Independent Assortment states that genes located on different chromosomes segregate independently during gamete formation. Reason (R): Mendel’s dihybrid crosses involving two pairs of genes support the Law of Independent Assortment. 6. Assertion (A): Pleiotropy is a genetic phenomenon where a single gene affects multiple phenotypic traits. Reason (R): Pleiotropy is commonly observed in polygenic inheritance. 7. Assertion (A): X-linked dominant inheritance means that a single mutated allele on the X chromosome can cause a genetic disorder. Reason (R): X-linked dominant disorders affect both males and females equally. 8. Assertion (A): Chromosomal disorders can result from alterations in the number or structure of chromosomes. Reason (R): Down’s syndrome, characterized by trisomy 21, is an example of a chromosomal disorder.  9. Assertion (A): Pedigree analysis is a valuable tool for studying the inheritance patterns of genetic traits in families over generations. Reason (R): Pedigree analysis can determine the exact genetic mutations responsible for genetic disorders.  10. Assertion (A): Mendelian disorders are caused by mutations in multiple genes. Reason (R): Mendelian disorders often involve complex inheritance patterns and are not governed by Mendel’s principles. 

53

HEREDITY AND VARIATION

[C] COMPETENCY/CASE-BASED QUESTIONS: I. Read the following text and answer the following questions on the basis of the same: Down syndrome (sometimes called Down’s syndrome) is a condition in which a child is born with an extra copy of their 21st chromosome hence its other name, trisomy 21. The affected individual mental retarded, short statured with small round, head, furrowed tongue and partially open mouth, Physical, psychomotor and mental development is retarded. 1. The number of chromosomes a child with Down’s syndrome has is (a) 45 (b) 46 (c) 47 (d) 48 2. Down’s syndrome is (a) Sex-linked (b) Chromosomal (c) Dominant (d) Recessive

3. One of this trait is seen in a person with Down’s syndrome (a) Upward slant eye (b) Baldness (c) Short stature (d) Long neck 4. (a) (b) (c) (d)

Down’s Syndrome is an extra copy of which chromosome 22nd chromosome 21st chromosome 45th chromosome 47th chromosome

5. Down’s syndrome is caused due to: (a) bacterial infection. (b) lack of oxygen supply to the brain during birth. (3) Viral infection. (d) a chromosomal abnormality.

II. Read the following text and answer the following questions on the basis of the same:  Genetic inheritance encompasses a set of principles and laws that govern how traits are passed from one generation to the next. Concept

Description

Mendel’s Principles

Dominance, Segregation, Independent Assortment

Chromosomal Theory of Inheritance

Genes on paired chromosomes, independent segregation

Genetic Disorders

Result from gene mutations or chromosome alterations

Polygenic Inheritance

Traits influenced by multiple genes with additive effects

Sex Determination

Mechanisms determining an individual’s sex

Pleiotropy

A single gene affecting multiple traits

Non-Mendelian Inheritance

Inheritance patterns deviating from Mendel’s principles

Linkage and Recombination

Genes located close on a chromosome may not segregate independently

6. Which genetic concept involves traits influenced by multiple genes with additive effects? (a) Pleiotropy (b) Polygenic Inheritance (c) Non-Mendelian Inheritance (d) Linkage and Recombination 7. In the chromosomal theory of inheritance, what does “independent segregation” refer to? (a) Genes located on different chromosomes. (b) Genes located on the same chromosome. (c) The random assortment of alleles. (d) The formation of gametes. 8. What phenomenon occurs when a single gene affects multiple seemingly unrelated traits? (a) Non-Mendelian Inheritance

(b) (c) (d) 9.

Pleiotropy Polygenic Inheritance Linkage and Recombination Which concept explains the inheritance patterns that deviate from Mendel’s principles, including incomplete dominance and co-dominance? (a) Genetic Disorders (b) Mendel’s Principles (c) Non-Mendelian Inheritance (d) Chromosomal Theory 10. How is an individual’s sex determined in many organisms? (a) By Mendel’s Principles (b) Through polygenic inheritance (c) Via sex determination mechanisms (d) By linkage and recombination

ANSWER KEY [A] MULTIPLE CHOICE QUESTIONS 1. (d)

2. (c)

3. (c)

4. (a)

5. (b, c)

6. (b)

7. (b)

8. (c)

9. (d)

10. (d)

11. (d)

12. (a)

13. (b)

14. (c)

15. (d)

16. (a)

17. (b)

18. (a)

19. (c)

20. (a)

21. (b)

22 (d)

23. (b)

24. (a)

25. (c)

26. (a)

27. (a)

28. (c)

29. (b)

30. (a)

8. (a)

9. (a)

10. (c)

9. (c)

10. (c)

[B] ASSERTION & REASON 1. (a)

2. (b)

3. (d)

4. (a)

5. (a)

6. (a)

7. (b)

[C] COMPETENCY/CASE-BASED QUESTIONS 1. (c)

2. (b)

3. (c)

4. (b)

5. (d)

6. (b)

7. (a)

8. (b)

54 Oswaal CUET (UG) Chapterwise Question Bank

BIOLOGY

ANSWERS WITH EXPLANATION [A] MULTIPLE CHOICE QUESTIONS: 1. Option (d) is correct Explanation: Phenylketonuria (PKU) is a metabolic disorder. Pedigree Analysis involves creating family trees over generations. Thalassemia is a type of blood disease. Trisomy of the 21st chromosome results in Down’s Syndrome. 2. Option (c) is correct Explanation: 1. Haemophilia and Sickle-cell anemia are Mendelian disorders caused by mutations in single genes and follow Mendelian inheritance patterns.

2. Phenylketonuria (PKU) is also a Mendelian disorder caused by a mutation in a single gene and follows Mendelian inheritance.

However, Down’s Syndrome is not a Mendelian disorder. It is caused by the presence of an extra copy of chromosome 21 (trisomy 21) and is a chromosomal disorder. It does not follow the principles of Mendelian inheritance because it involves an abnormality in the number of chromosomes, rather than a single gene mutation. 3. Option (c) is correct Explanation: The following requirements must be met by a molecule before it may serve as genetic material. It needs to be stable both chemically and structurally. It should be able to create a copy of itself. It ought to allow for the kind of slow mutation that evolution requires. It ought to be able to manifest itself as Mendelian characteristics. 4. Option (a) is correct Explanation: Tallness and dwarfness in pea plants are dominant and recessive characters respectively. The cross can be represented as follows:

6. Option (b) is correct Explanation: Phenylketonuria is an autosomal recessive disorder. The individuals suffering from this disorder lack an enzyme that converts the amino acid phenylamine into tyrosine. Down’s syndrome is caused due to an additional copy of the chromosome number 21 (trisomy of 21) Turner’s syndrome is caused due to the absence of one of the X-chromosomes, i.e., 45 with X0 (monosomy of X-chromosome) Hemophilia is sexlinked recessive disease. 7. Option (b) is correct Explanation: The haplo-diploid sex determination system is seen in honey bees. In honeybee, the male have half the number of chromosome than that of female. The female are diploid having 32 chromosomes and males are haploid i.e., having 16 numbers of chromosomes. In Drosophila and grasshopper, sexdetermination is XX-XO type. Here, female is homogametic, i.e., XX (all gametes are with X chromosomes) while male is heterogametic, i.e., XO (Gametes with X and gametes without X) In birds, female heterogamety and male homogamety is seen. Female birds have two different sex chromosomes designated as Z and W whereas male birds have two similar sex chromosomes and called ZZ. 8. Option (c) is correct Explanation: Since genotype RrTT is heterozygous for two genes i.e. it has both dominant and recessive alleles of two genes, it will form total four types of gametes. The gametes are RT, Rt, rt and tr. 9. Option (d) is correct. Explanation: Frequency of crossing over will increase if the distance between linked genes is longer. 10. Option (d) is correct. Explanation: Haplodiploidy or arrhenotoky type of sex determination is used by bees. It is a sex-determination system in which males develop from unfertilized eggs and are haploid, and females develop from fertilised eggs and are diploid. 11. Option (d) is correct.

5. Option (b, c) are correct Explanation: The ABO blood group system in humans is determined by the presence of three alleles: A, B, and O. Each person inherits two alleles, one from each parent. Alleles A and B are codominant, while allele O is recessive.

Given that the father’s blood group is A (genotype AA or AO) and the mother’s blood group is B (genotype BB or BO), let’s consider the possible combinations for their offspring’s blood group:



Father is AO (A allele from the father) and mother is BO (B allele from the mother): { {



 ffspring can inherit AO from the father and BO O from the mother. This results in the AB blood group because the A and B alleles are codominant.

So, there is a 100% probability that their offspring can have an AB blood group, not just a percentage chance.

Explanation: ZZ / ZW type of sex determination is seen in birds, reptiles and fish. Thus, peacock shows ZZ/ZW type sex determination. In this type, female has heteromorphic (ZW) sex chromosomes and the male has homomorphic (ZZ) sex chromosomes. In Platypus the sex determination is of XX-XY type in which both male and females has ten sex chromosome each. The male has XY, XY, XY, XY, XY and female has XX, XX, XX, XX, XX. In snails, the sex determination is environmentally induced, while in cockroaches it is of XX-XO types. In this type Y-chromosome is absent. In this the presence of unpaired X-chromosomes determines the masculine sex. 12. Option (a) is correct. Explanation: It is a disorder caused due to the absence of one of the X-chromosomes, i.e., 45 with XO. 13. Option (b) is correct. Explanation: Here is this case, the women is a carrier. Both the son and daughter will inherit the X-chromosome but only the son will be diseased.

55

HEREDITY AND VARIATION

14. Option (c) is correct. Explanation: A person with sickle cell anaemia is less prone to malaria because in sickle cell anaemia red blood cells becomes abnormally shaped, if they inherit two faulty copies of the gene for the oxygen-carrying protein haemoglobin. The faulty gene persists because even carrying one copy of it confers some resistance to malaria. Therefore, people with normal haemoglobin A are susceptible to death from malaria but people with sickle cell trait who have one gene for haemoglobin A and other for haemoglobin S have a greater chance of surviving malaria and do not suffer adverse consequences from the haemoglobin S gene. 15. Option (d) is correct. Explanation: If a genetic disease is transferred from a phenotypically normal but carrier female to only some of the male progeny, the disease is sex linked recessive disease. Most sex-linked (X-linked) conditions are recessive. Because, in this condition a person with two X-chromosomes (females) must have a change or mutation whereas in a person with one X-chromosomes (males), only one copy of a gene must have a mutation. A female with a mutation in one copy of a gene on the X-chromosome is said to be a ‘carrier’ for an X-linked condition. For X-linked recessive disorders, an unaffected carrier mother who has a mutation in a gene on the X-chromosome can transmit either the X-chromosome with this mutation or a normal X-chromosome to her children (as shown below in cross). Autosomal dominant inheritance refers to the pattern of inheritance of a condition directly or indirectly due to a dominant faulty gene located on autosome. Autosomal recessive inheritance is caused directly or indirectly due to presence of recessive faulty gene copy on autosome. Sex-linked dominant is a rare trait, caused by a single abnormal gene on the X-chromosome. 16. Option (a) is correct. Explanation: In a test cross an organism showing a dominant phenotype whose phenotype is to be determined (Whether it is homozygous or heterozygous for that trait) is crossed with a recessive parent. 17. Option (b) is correct. Explanation: Klinefelter’s syndrome is the genetic disorder which is caused due to the presence of an additional copy of X-chromosome resulting in karyotype of 47, XXY chromosome but only the son will be affected by the disease.

When true breeding plants were crossed to each other, this is called a parental cross and offspring comprise the first filial or F1 - generation. When the members of the F1 - generation were crossed, this produced the F2-generation or second filial generation. A cross between true breeding tall and dwarf plants of the parent generation yield phenotypically tall plants. The cross between TT and Tt is called back cross, which results into two homozygous and two heterozygous dominant gametes. The cross between Tt and tt is called test cross which results into 1 : 1 ratio of gametes. 22. Option (d) is correct. Explanation: In a dihybrid cross, if you get 9 : 3 : 3 : 1 ratio, it denotes that alleles of two genes are segregating independently. Law of independent assortment states that “When two pairs of traits are combined in a hybrid, segregation of one pair of characters is independent of the other pair of characters”. Suppose crosses are made between a pea plant with round and yellow seeds and one with wrinkled and green ones. All F1 hybrids give yellow and round seeds. Since. yellow colour is dominant over the green and the round shape is dominant over the wrinkled. When the F1 hybrid plants are crossed to each other or allowed to self-fertilize, F2 generation form as represented in the following cross.

18. Option (a) is correct. Explanation: If both parents have sickle cell trait, there is a 25% chance with each pregnancy that the baby will have sickle cell anaemia. A child with sickle cell anaemia appears normal at birth. 19. Option (c) is correct. Explanation: Monohybrid cross with incomplete dominance shows both genotypic and phenotypic ratio as same (1 : 2 : 1). 20. Option (a) is correct. Explanation: Starch synthesis in pea seed is controlled by one gene which has two alleles (B and b). Pleiotropy is shown by B gene. 21. Option (b) is correct. Explanation: The genotypes of both the parents are Tt and Tt. Refer the given cross between true breeding tall plants and true breeding dwarf plants.

Phenotypic ratio-9:3:3:1 P-generation Gametes F1-generation Gametes F2-generation Yellow-round seed Green-wrinkled seed YYRR yyrr YR YyRr yr YyRr YyRr YR Yr yR yr 1 4 1 4 1 4 1 4 YR Yr yR yr 1 4 1 4 1 4 1 4 Yellow-round seed

56 Oswaal CUET (UG) Chapterwise Question Bank (dominant character) The F1 hybrid plants are allowed to self fertilise YR Yr yR yr YR Yr yR yr YYRR YYRr YyRR YyRr YYRr YYrr YyRr Yyrr yyRr yyRR yyRr yyrr Yyrr YyRR YyRr YyRr The outcome of the dihybrid cross makes it clear that segregation of the seed colour is independent of the seed shape and both the parental and new combinations of the characters appear in the offspring, that is, assortment of genes of one pair is independent of the other pair. 23. Option (b) is correct. Explanation: ABO blood grouping in humans is an example of co-dominance. ABO blood groups are controlled by gene I. Gene I has three alleles IA, IB, and Ii. When IA, and IB are present together, both express equally and produce the surface antigens A and B, whereas i is the recessive allele and does not produce any antigen. Pleiotropy referred the genetic effect of a single gene on multiple phenotypic traits. Incomplete dominance does not completely dominate another allele. Segregation is the separation of allele during the process of gametogenesis. This is the basis of reappearance of recessive character in F2 generation. 24. Option (a) is correct. Explanation: Mendel’s Law of independent assortment holds good for genes situated on the non-homologous chromosome. According to law of independent assortment when two or more characteristics are inherited, individual hereditary factors assort independently of one another during gamete production, giving different characters an equal opportunity of occurring together. It can be illustrated by the dihybrid cross (a cross between two true-breeding parents) that express different traits for two characteristics. 25. Option (c) is correct. Explanation: Pleiotropy is a phenomenon in which a single gene affects multiple effects. Sometimes, one trait will be very evident and others will be less evident, e.g., a gene for white eye in Drosophila also affect the shape of organs in male responsible for sperm storage as well as other structures. Similarly, sickle-cell anaemic individuals suffer from a number of problems, all of which are pleiotropic effects of the sicklecell alleles. Multiple allelism is the state of having more than two alternative contrasting characters controlled by multiple alleles at a single genetic locus, for e.g. ABO blood group. Mosaicism describes the occurrence of cells that differ in their genetic component from other cells of the body. Polygeny refers to a single characteristic that is controlled by more than two genes. (it is also known as multifactorial inheritance). 26. Option (a) is correct. Explanation: According to law of segregation or law of purity of gametes, factors or alleles of a pair segregate from each other during gamete formation, such that a gamete receives only one of the two factors and do not show any blending. Therefore, the proposition made by Mendel that the factor controlling any character is discrete and independent and based on the observations that the offspring of a cross made between the plants having two contrasting characters shows only one character without any blending. Suppose, when two contrasting characters (for e.g. seed colour and seed shape round yellow and wrinkled green, given below) of a plant are crossed, their dominant phenotypic traits were expressed in F1 generation and their recessive characters appear in the F2 generation. This shows that there is no blending of Mendelian factors in F1 generation but they stay together and only one of the character is expressed. Therefore, at the time of formation of gametes, these two factors separate or segregate otherwise recessive types will not appear in F2 generation.

BIOLOGY

27. Option (a) is correct. Explanation: As per the cross discussed, the ratio of offspring in the next generation will be 1 : 1 : 1 : 1. In monohybrid crosses, to know if a dominant trait is homozygous (RR) or heterozygous (Rr) it is necessary to carry out a test cross. This is done with a homozygous recessive (rr) individual. The same is true for a dihybrid cross where the test cross is made with an individual which is homozygous recessive for both characters (rryy). It can be explained by the following test cross. 28. Option (c) is correct. Explanation: Linkage may be defined as the relationship between genes on the same chromosome that causes them to be inherited together, therefore it will not result in variations among siblings. In linkage there is a tendency to maintain the parental gene recombination except for occasional crossovers. Independent assortment of genes means that allele pair separate during the formation of gametes independently; it means that traits are transmitted to offspring independently of one another. Crossing over is the exchange of genetic material between homologous chromosomes. It is one of the final phases of genetic recombination. Mutation is the sudden inheritable change in genetic material of an organism which transfers to next generation. 29. Option (b) is correct. Explanation: All the genes, present on a particular chromosome form a linkage group. The number of linkage group of a species correspond to the total number of different chromosomes of that species. It is not simply the number of chromosomes in haploid set. For example, in human male there are 22 pairs of autosomes and X and Y sex chromosomes, that is, 24 linkage groups and in female = 22 pairs autosomes + 2X-chromosomes, that is, 23 linkage groups. 30. Option (a) is correct. Explanation: Crossing over separates genes away from each other. So, the physical distance between the two genes and percentage of recombination shows a direct relationship. More the distance between two genes, more is the frequency of crossing over between them and hence more is the percentage of recombination. [B] ASSERTION & REASON: 1. Option (a) is correct Explanation: In monohybrid Mendelian cross, the F1 hybrid always showed one of the parental forms of the trait whereas both the parental forms of the trait (contrasting forms of the trait) appeared without any change in the F2 generation. This occurred because there is no blending of traits. The form of the trail that appeared in the F2 hybrids is called dominant form and it appeared in the F2 generation about three times in frequency as its alternate (recessive) form. 2. Option (b) is correct. Explanation: Hybrid vigour is superior in characteristic such as size, growth rate, fertility, etc., of a hybrid organism over it’s parents. Hybrid, on the other hand is a result of cross between parents of two different types. 3. Option (d) is correct. Explanation: Haemophilia is a sex-linked recessive disorder in which X-chromosome has the haemophilic gene. A haemophilic father can never pass the gene for haemophilia to his son.

57

HEREDITY AND VARIATION

4. Option (a) is correct.

2. Option (b) is correct.

Explanation: Incomplete dominance does result in an intermediate phenotype in heterozygous individuals, and this happens due to the blending of alleles.

Explanation: Down syndrome is an autosomal recessive disorder which can be inherited through normal parents in the child.

5. Option (a) is correct.

3. Option (c) is correct.

Explanation: The Law of Independent Assortment states that genes located on different chromosomes segregate independently, and Mendel’s dihybrid crosses involving two pairs of genes support this law.

Explanation: The person affected with Down syndrome has symptoms like mental retarded, short statured with small round, head, furrowed tongue, etc.

6. Option (a) is correct.

Explanation: Down Syndrome is due to extra copy of 21st chromosome forming trisomy condition.

Explanation: Pleiotropy indeed involves a single gene affecting multiple phenotypic traits and is commonly observed in polygenic inheritance. 7. Option (b) is correct. Explanation: X-linked dominant inheritance does mean that a single mutated allele on the X chromosome can cause a genetic disorder, but it doesn’t necessarily affect both males and females equally. 8. Option (a) is correct. Explanation: Chromosomal disorders can indeed result from alterations in the number or structure of chromosomes, and Down’s syndrome is an example of such a disorder. 9. Option (a) is correct. Explanation: Pedigree analysis is a valuable tool for studying inheritance patterns, especially in tracking genetic traits within families. 10. Option (c) is correct. Explanation: Mendelian disorders are typically caused by mutations in single genes, not multiple genes. They follow Mendel’s principles of inheritance.

4. Option (b) is correct.

5. Option (d) is correct. Explanation: Down syndrome is due to autosomal chromosome abnormality. 6. Option (b) is correct. Explanation: Polygenic Inheritance involves traits influenced by multiple genes with additive effects, leading to continuous variation in phenotypes. 7. Option (a) is correct. Explanation: In the chromosomal theory of inheritance, “independent segregation” refers to the fact that genes located on different chromosomes segregate independently during gamete formation. 8. Option (b) is correct. Explanation: Pleiotropy refers to the phenomenon where a single gene affects multiple seemingly unrelated traits. 9. Option (c) is correct. Explanation: Non-Mendelian Inheritance explains inheritance patterns that deviate from Mendel’s principles, including incomplete dominance and co-dominance.

[C] COMPETENCY/CASE-BASED QUESTIONS:

10. Option (c) is correct.

1. Option (c) is correct.

Explanation: An individual’s sex is determined in many organisms through specific sex determination mechanisms, which can involve various genetic and environ­mental factors.

Explanation: The affected person inherited with one extra copy of 21st chromosome that forms trisomy condition.

24

Study Time

CHAPTER

6

Max. Time: 1:5 Hours Max. Questions: 50

MOLECULAR BASIS OF INHERITANCE

 Revision Notes: DNA

DNA is a long polymer made up of deoxyribonucleotides, and its length is measured in terms of the number of nucleotides or base pairs. { Different organisms have varying DNA lengths, such as bacteriophages, bacteria, and humans. {  A nucleotide consists of three components: a nitrogenous base, a pentose sugar (ribose in RNA, deoxyribose in DNA), and a phosphate group. { There are two categories of nitrogenous bases: Purines (Adenine and Guanine) and Pyrimidines (Cytosine, and Uracil in RNA, Thymine in DNA). {  Nucleosides are formed when a nitrogenous base is linked to the pentose sugar through a N-glycosidic linkage. Those with ribose sugar i.e. in RNA are called ribonucleosides while those with deoxyribose sugar i.e, in DNA are called deoxyribonucleosides. {  Nucleotides, are formed when a phosphate group is linked to the pentose sugar through a phosphoester linkage. Those with ribose sugar i.e. in RNA are called ribonucleotides while those with deoxyribose sugar i.e, in DNA are called deoxyribonucleotides. { Polynucleotide chains are created by linking nucleotides together through 3’-5’ phosphodiester linkages. { The backbone of a polynucleotide chain is formed by sugar and phosphate molecules, while the nitrogenous bases project from this backbone. { In DNA, there are free phosphate moieties at the 5’-end and free OH groups at the 3’-end of the polynucleotide chain. {  DNA has a double-helix structure, where two polynucleotide chains wind around each other. { The chains are anti-parallel, with one having a 5’ to 3’ polarity and the other having a 3’ to 5’ polarity. { Base pairing between the chains is achieved through hydrogen bonds, with Adenine pairing with Thymine and Guanine pairing with Cytosine. {  The DNA double helix is right-handed, with a pitch of 3.4 nm and approximately 10 base pairs per turn, resulting in a constant distance between the two strands. { Stacking of base pairs and hydrogen bonds contribute to the stability of the double-helix structure. { The discovery of the DNA double helix structure had a profound impact on molecular biology, leading to the formulation of the Central Dogma, which describes

the flow of genetic information from DNA to RNA to protein. The central dogma of molecular biology is a theory stating that genetic information flows only in one direction, from DNA, to RNA, to protein, or RNA directly to protein.

Packaging of DNA Helix {  The

Scan to know

more about length of a typical DNA this topic double helix in a mammalian cell is about 2.2 meters, which is much longer than the size of a typical nucleus (approximately 10-6 meters). DNA- Structure and function of { In prokaryotes like E. coli, DNA Deoxyribonucleic is organised in a region called Acid (DNA) the ‘nucleoid,’ where it is held in large loops by proteins. {  In eukaryotes, DNA packaging is more complex. Positively charged, basic proteins known as histones play a crucial role. {  Histones, rich in lysine and arginine amino acid residues with positive charges, form an octamer unit called a histone octamer. { DNA, which is negatively charged, wraps around the positively charged histone octamer to form a nucleosome, with each nucleosome containing 200 base pairs of DNA. {  Nucleosomes are the repeating units of chromatin, thread-like structures visible in the nucleus. {  Chromatin appears as a ‘beads-on-string’ structure when viewed under an electron microscope. {  The number of nucleosomes in a mammalian cell is theoretically very high. { Chromatin fibers, formed by packaging nucleosomes, further coil and condense during cell division to create chromosomes. { The packaging of chromatin at higher levels involves non-histone chromosomal (NHC) proteins. { Chromatin can be categorised into euchromatin, which is loosely packed and transcriptionally active, and heterochromatin, which is densely packed and inactive.

The Search for Genetic Material Transforming Principle (1928) { Frederick Griffith’s experiments with Streptococcus pneumoniae demonstrated a transformation in bacteria. {  Two types of bacterial colonies: S (smooth) with a polysaccharide coat and R (rough) without the coat were used in experiment.

102

Oswaal CBSE Question Bank Chapterwise & Topicwise, BIOLOGY, Class-XII

MOLECULAR BASIS OF INHERITANCE

59

60 Oswaal CUET (UG) Chapterwise Question Bank { Mice

infected with the S strain (virulent) died, while those infected with the R strain did not. { Heat-killed S bacteria did not kill mice, but a mixture of heat-killed S and live R bacteria did. { Griffith concluded that something in the heat-killed S strain transformed the R strain, but the nature of this “transforming principle” was unknown.

Biochemical ­Principle

Characterisation

of

Transforming

{ Oswald

Avery, Colin MacLeod, and Maclyn McCarty (1933-44) aimed to determine the biochemical nature of the transforming principle. { They isolated various biochemicals (proteins, DNA, RNA) from heat-killed S cells and tested their ability to transform R cells into S cells. { DNA from S bacteria was found to cause transformation. {  While protein digesting enzymes and RNA digesting enzymes had no effect, DNase inhibited transformation, indicating that DNA was the genetic material. DNA Confirmed as the Genetic Material (Hershey-Chase Experiment, 1952): {  Alfred Hershey and Martha Chase worked with bacteriophages (viruses that infect bacteria) to confirm DNA as the genetic material. { Phages were grown in the presence of radioactive phosphorus (labeling DNA) and radioactive sulfur (labeling protein). { Phages were allowed to infect E. coli bacteria. {  Bacteria infected with DNA-labeled phages became radioactive, while those infected with protein-labeled phages did not. { This experiment provided conclusive evidence that DNA, not protein, was the genetic material.

Properties of Genetic Material (DNA versus RNA): { A

molecule serving as genetic material must fulfill specific criteria. { DNA and RNA both have the ability to replicate due to base pairing and complementarity. { Genetic material should be chemically and structurally stable, with DNA being more stable than RNA due to its lack of a reactive 2’-OH group and thymine instead of uracil. { Both DNA and RNA can mutate, but RNA mutates at a faster rate. {  RNA can directly code for protein synthesis, while DNA relies on RNA for protein synthesis.

RNA World: RNA as the First Genetic Material { RNA is

considered the first genetic material. { Evidence suggests that essential life processes, such as metabolism, translation, and splicing, evolved around RNA. { RNA served as both genetic material and a catalyst, but its reactivity made it unstable. { DNA evolved from RNA with chemical modifications for stability, such as double-stranded structure and complementary base pairing.

BIOLOGY

Replication Watson and Crick’s Proposal {  Watson

and Crick proposed a scheme for DNA replication when they discovered the double-helical structure of DNA. { Their scheme suggested that the two DNA strands would separate and serve as templates for the synthesis of complementary strands. {  This process was termed “semi­ conservative DNA replication.”

Scan to know more about this topic

DNA Replication | Molecular Basis of Inheritance

Experimental Proof of Semiconservative Replication: {  Matthew

Meselson and Franklin Stahl conducted an experiment in 1958 to prove that DNA replicates semiconservatively. 15 { They used heavy nitrogen ( N) to label DNA and tracked its distribution in subsequent generations of Escherichia coli. 15 { DNA from the first generation after switching from N to 14N showed an intermediate density, and DNA from the second generation had equal amounts of hybrid and “light” DNA. { This experiment confirmed semi-conservative DNA replication.

Replication Machinery and Enzymes { DNA

replication in living cells requires a set of enzymes, including DNA-dependent DNA polymerase. {  These enzymes must work quickly and accurately to replicate a large number of nucleotides in a short time. { DNA polymerases catalyse polymerisation in the 5’ to 3’ direction. {  Replication occurs at a replication fork, where one strand replicates continuously, and the other replicates discontinuously with fragments later joined by DNA ligase.

Origin of Replication { DNA

replication begins at specific regions in DNA called origins of replication. { These regions are essential for initiating the replication process. { Vectors used in recombinant DNA procedures provide origins of replication.

Transcription Transcription Unit { A transcription

unit consists of a promoter, a structural gene, and a terminator. {  The promoter is located upstream Scan to know more about of the structural gene and provides this topic a binding site for RNA polymerase. { The terminator defines the end of transcription.

Transcription Unit and the Gene {  Genes

are functional units of

Transcription in Prokaryotes

61

MOLECULAR BASIS OF INHERITANCE

inheritance, but defining a gene in terms of DNA sequence is complex. { Cistron are the smallest unit of genetic material which coded for a single polypeptide for the transmission of genetic information. Structural genes in transcription units can be monocistronic or polycistronic. { Eukaryotic genes often have interrupted coding sequences (exons and introns).

Types of RNA and Transcription Process { Three

major types of RNA exist in bacteria: mRNA, tRNA, and rRNA. { RNA polymerase catalyses transcription in bacteria. {  In eukaryotes, there are multiple RNA polymerases with distinct functions. {  In eukaryotes, due to presence of exons, primary transcripts undergo splicing, capping, and tailing to form mature mRNA for translation.

Genetic Code Introduction to the Genetic Code { Replication

and transcription involve copying nucleic acids, while translation synthesises amino acids. {  There is no inherent complementarity between nucleotides and amino acids. {  The genetic code concept emerged to explain how changes in nucleic acids affect amino acids in proteins. { George Gamow proposed a triplet code for amino acids, which raised questions about redundancy.

Deciphering the Genetic Code { Scientists

from various fields collaborated to decipher the genetic code. { Har Gobind Khorana’s chemical method and Marshall Nirenberg’s cell-free system played key roles. { Severo Ochoa’s enzyme helped synthesise RNA with defined sequences. { A checkerboard for the genetic code was prepared.

Salient Features of the Genetic Code { The

genetic code is triplet, with 61 codons coding for amino acids and 3 as stop codons. {  Some amino acids are coded by multiple codons, making the code degenerate. { Codons are read in mRNA without punctuations. { The code is nearly universal but has exceptions in mitochondria and some protozoans. { AUG serves as both the initiator codon and codes for methionine (Met). { UAA, UAG, and UGA are stop codons.

Applications of the Genetic Code {  Predicting

the amino acid sequence from an mRNA sequence using the genetic code. {  Predicting the mRNA sequence from an amino acid sequence.

Mutations and Genetic Code { Exploring

the impact of mutations on the genetic code.

{ Frame-shift

insertions or deletions change the reading frame, affecting all codons. {  Insertions or deletions of three or multiples of three bases usually maintain the reading frame.

tRNA - The Adapter Molecule { tRNA acts

as an adapter molecule: It reads the code on one end and binds to specific amino acids on the other. { Each amino acid has its specific tRNA.

Translation { Translation

is the process of synthesising a polypeptide using the mRNA code. { Amino acids are linked by peptide bonds. { Energy is required for peptide bond formation. {  Charging of tRNA with amino acids occurs at the beginning. { Ribosomes, composed of structural RNAs and proteins, are responsible for protein synthesis. { They have large and small subunits. { The ribosome serves as a catalyst for peptide bond formation. { A translational unit in mRNA codes for a polypeptide and is flanked by a start codon (AUG) and a stop codon. { Untranslated regions (UTRs) exist at both the 5’ and 3’ ends of mRNA, aiding efficient translation.

Phases of Translation {  Translation

occurs in initiation, elongation, and termination phases. { Initiation involves the ribosome binding to the start codon (AUG). { In elongation, tRNA molecules bind sequentially to the mRNA codons. {  Termination is signaled by a stop codon, and the polypeptide is released.

Regulation of Gene Expression  efinition: Regulation of gene expression involves controlling D when and to what extent a gene is activated to produce its corresponding protein or RNA product. Levels of Regulation: { Transcriptional Level: Regulation at the stage of gene transcription, determining whether the gene is turned on or off. { Processing Level: Regulation of RNA splicing, ensuring correct processing of pre-mRNA into mature mRNA. { mRNA Transport: Control over the transport of mRNA from the nucleus to the cytoplasm, affecting its availability for translation. { Translational Level: Regulation of protein synthesis from mRNA, controlling the rate of translation. Factors Influencing Gene Expression: {  Metabolic and Physiological Conditions: Gene expression adapts to the metabolic needs of the cell. { Environmental Conditions: External factors, such

62 Oswaal CUET (UG) Chapterwise Question Bank as nutrient availability, temperature, and stress, can influence gene expression. { Development and Differentiation: During develop­ ment, genes are selectively expressed to shape an organism’s form and function. Prokaryotic Regulation: In prokaryotes, Scan to know gene expression is primarily regulated at more about this topic the transcriptional level. Regulatory proteins, activators, and repressors influence RNA polymerase activity. Promoter regions have adjacent Prokaryotic Gene operator sequences where Regulation: repressors bind. Lac Operon Lac Operon: Components: The lac operon includes one regulatory gene (i gene) and three structural genes (z, y, and a). Function: These genes are involved in the metabolism of lactose. Repressor: The i gene codes for a repressor protein that binds to the operator, preventing transcription of the operon. Inducer: Lactose acts as an inducer by inactivating the repressor, allowing RNA polymerase access to the promoter. Negative Regulation: The repression of the operon by the repressor is an example of negative regulation. { Positive Regulation: Some operons are also regulated positively by activator proteins, which enhance transcription when bound to specific sites. { Scope of Expression: Lac operon is expressed as long as lactose is present and actively transported into the cell. { Inducer Specificity: Glucose or galactose cannot act as inducers for the lac operon. { Complex Regulation: Regulation of gene expression can involve multiple levels and factors, ensuring precise control over cellular processes.

Human Genome Project Introduction to HGP { The

Human Genome Project (HGP) aimed to sequence the entire human genome. { It began in 1990, driven by the desire to understand genetic makeup and variations.

Scale and Cost of the Project

human genome consists of approximately 3 x 109 base pairs. { At an estimated cost of $3 per base pair, the project was estimated to cost around $9 billion. {  Storing the DNA sequence information required thousands of books due to the enormous data volume. { The

Bioinformatics Development { The HGP led to the rapid development of bioinformatics,

BIOLOGY

a field dealing with data storage, retrieval, and analysis.

Goals of HGP { Identify

20,000-25,000 genes in human DNA. { Determine the sequence of the 3 billion base pairs in human DNA. { Store this information in databases. { Improve tools for data analysis. { Transfer related technologies to other sectors. { Address ethical, legal, and social issues (ELSI) related to the project.

Collaborative Effort {  The

HGP was a 13-year project coordinated by the U.S. Department of Energy and the National Institute of Health. {  The Welcome Trust (U.K.) and contributions from other countries played significant roles.

Applications of HGP { Knowledge

about DNA variations among individuals can revolutionise disease diagnosis, treatment, and prevention. { It aids in understanding human biology and studying DNA sequences in non-human organisms. { Non-human model organisms like bacteria, yeast, and plants have also been sequenced.

Methodologies Used {  Two

major approaches: Expressed Sequence Tags (ESTs) for gene identification and sequencing the entire genome for annotation. { DNA fragments from a cell were cloned into vectors like BAC and YAC for sequencing. {  Automated DNA sequencers were used, based on Frederick Sanger’s method. { Specialised computer programs were developed for sequence alignment.

Salient Features of Human Genome { The

human genome has 3164.7 million base pairs. vary in size, with an average gene consisting of 3000 bases. { Over 50% of discovered genes have unknown functions. { Less than 2% of the genome codes for proteins. { Repetitive sequences, which are non-coding, make up a substantial portion. {  Chromosome 1 has the most genes, while the Y chromosome has the fewest. { 1.4 million single-base DNA differences (SNPs) have been identified in humans. { Genes

Applications and Challenges { DNA

sequences provide valuable insights into biological systems. {  High-throughput technologies allow the study of thousands of genes and proteins simultaneously. { Challenges include interpreting the vast amount of genomic data.

63

MOLECULAR BASIS OF INHERITANCE

DNA Fingerprinting { Definition: DNA fingerprinting is a technique used to identify individuals based on variations in specific regions of their DNA sequences. { Human Genome Project Connection: DNA fingerprinting relies on polymorphisms, which are naturally occurring variations in DNA sequences. These polymorphisms form the basis for differences among individuals and are a part of the human genome. { Repetitive DNA: DNA fingerprinting primarily targets repetitive DNA sequences, particularly satellite DNA regions. These sequences consist of short DNA segments that are repeated many times. { Polymorphism: Polymorphism refers to genetic variations within a population. In the context of DNA fingerprinting, it involves variations in the number of repeats in satellite DNA sequences. Polymorphisms can occur in both coding and non-coding DNA. { Alec Jeffreys: Alec Jeffreys is credited with developing the technique of DNA fingerprinting. He used Variable Number of Tandem Repeats (VNTRs), a type of satellite DNA, as probes for this purpose. { VNTRs: VNTRs are mini-satellite DNA sequences consisting of short repeats arranged in tandem. The number of repeats can vary significantly between individuals, leading to a high degree of polymorphism. { DNA Fingerprinting Process: 1.  Isolation of DNA: DNA is extracted from the biological sample, such as blood or hair. 2.  Digestion: DNA is digested using restriction endonucleases, resulting in fragments of varying lengths.

5'

H | C | H

P

P

H | C | H

3.  Electrophoresis: DNA fragments are separated by size through electrophoresis, forming a pattern of bands. 4.  Blotting: Separated DNA fragments are transferred (blotted) onto synthetic membranes. 5.  Hybridisation: The membranes are exposed to radiolabeled VNTR probes, allowing for binding to complementary sequences. 6.  Autoradiography: The hybridised DNA fragments are detected using autoradiography, creating a characteristic pattern of bands. { Characteristic Pattern: The resulting pattern of bands on the autoradiogram is unique to each individual, except for monozygotic (identical) twins. It serves as a DNA fingerprint. { PCR Advancements: Polymerase Chain Reaction (PCR) technology has improved the sensitivity of DNA fingerprinting. PCR allows DNA analysis even with minute samples and has expanded the applications of DNA fingerprinting. { Applications: 1. DNA fingerprinting is widely used in forensic science for identifying suspects and victims. It is also employed in paternity testing, tracking genetic diseases, and studying genetic diversity in populations. 2. DNA fingerprinting is crucial for solving crimes, establishing biological relationships, and understanding genetic variations within populations. It plays a significant role in both forensic science and genetics research.

P

H | C | H

P

T

A

H | C | H

OH 3'

C

G

Hydrogen bond A

T H HO 3'

|

C

G

C H

P

|

|

C

H

H

|

P

H | C | H

H | C | H

P

P 5'

Fig 1: Double Stranded polynucleotide chain

Replication DNA

Transcription

mRNA

Translation

Protein

Fig 2: Central Dogma Fig 3: Nucleosome

64 Oswaal CUET (UG) Chapterwise Question Bank

BIOLOGY

Bacteriophage Radioactive ( 35 S) labelled protein capsule

Radioactive ( 32P) labelled DNA

1.Infection

2.Blending

3.Centrifugation

Radioactive ( 32 P) detected in cells + No radioactivity detected in supernatant

No Radioactive ( 35 S) detected in cells + Radioactive [ 35 S] detected in supernatant

Fig 4: The Hershey and Chase Experiment Generation I 15 15

N-DNA

14

Fig 5: Watson Crick model of Semiconservative DNA replication

Generation II

N-DNA

N-DNA 40 min

20 min Gravitational force

15

N15N Heavy

14

N14N 14N15N Light Hybrid

N15N Hybrid

14

14

N-DNA

15

N-DNA

14

N-DNA

14

N-DNA

Separation of DNA by Centrifugation

Fig 6: Meselson and Stahl's Experiment

Fig 7: Replicating Fork

Fig 8: Schematic structure of a transcription unit 5'

3'

Initiation

5' Promoter

RNA polymerase



DNA helix Sigma factor 5'

3' Elongation

3'

5' RNA



Te rminator

3' Termination

3'

5'

5'  RNA

Fig 9: Process of Transcription in Bacteria

3' RNA Polymerase Rho factor

65

MOLECULAR BASIS OF INHERITANCE

OBJECTIVE TYPE QUESTIONS [A] MULTIPLE CHOICE QUESTIONS: 1. Match List - I with List - II. [CUET 2023] List – I List - II (A) Ribosome (I) Replication (B)  Histone (II)  Transcription (C)  DNA polymerase (III)  Translation (D) RNA polymerase (IV) Nucleosome Choose the correct answer from the options given below: (a) (A)-(1), (B)-(II), (C)-(III), (D)-(IV) (b) (A)-(II), (B)-(I), (C)-(III), (D)-(IV) (c) (A)-(1), (B)-(II), (C)-(IV), (D)-(III) (d) (A)-(III), (B)-(IV), (C)-(I), (D)-(II) 2. Arrange the following steps of experiments performed by Griffith in correct series. [CUET 2023] (A) ‘S’ strain injected into mice → Mice died (B) ‘S’ strain (Heat killed) → injected into mice → Mice lived (C) ‘R’ strain injected into mice → Mice lived (D) ‘S’ strain (Heat killed) +’R’ Strain (Live) → injected into mice → Mice died Choose the correct answer from the options given below: (a) (A), (B), (C), (D) (b) (B), (A), (C), (D) (c) (B), (C), (D), (A) (d) (A), (C), (B), (D) 3. In DNA N-glycosidic linkage is present between [CUET 2023] (a) Pentose sugar and Phosphate group. (b) A nitrogenous base and a Pentose sugar. (c) Two nitrogenous bases. (d) Two pentose sugars. 4. Identify the statements true for RNA. [CUET 2023] (A) RNA acts as a genetic material for some viruses. (B) RNA also functions as an adapter molecule. (C) RNA has hexose sugar as its backbone. (D) RNA also acts as catalyst in some cases. Choose the correct answer from the options given below: (a) (A), (B) and (C) only (b) (A), (B) and (D) only (c) (B), (C) and (D) only (d) (A), (C) and (D) only 5. The protein formed by the encoded gene expression in a heterologous host is called: [CUET 2022] (a) Structural protein (b) Recombinant protein (c) Transposons (d) Prohormone 6. With reference to processing of hn RNA, which of the following statements is/are INCORRECT? [CUET 2022] A. Introns are removed and exons are joined directly splicing. B. Capping and Tailing occurs at 5’ end and 3’ end respectively. C. Addition of 200-300 adenylated residues means capping.

D. Addition of guanosine triphosphate takes place at 5’ end. E. Processing take place in the nucleus and converts hn RNA into functional RNA. Choose the correct answer from the options given below: (a) B only (b) C only (c) B and C only (d) B and D only 7. Match List I with List II [CUET 2022] List-I List-II A. Alec Jeffreys I. Lac Operon B. Francois Jacob and II. D  eciphering of genetic code Jacque Monod C. Marshall Nirenberg III. S  emi-conservative replication of DNA D. Meselson Stahl IV. D  NA Fingerprinting Choose the correct answer form the options given below: (a) A-IV, B-I, C-II, D-III (b) A-I, B-III, C-IV, D-II (c) A-II, B-III, C-I, D-IV (d) A-IV, B-II, C-III, D-I 8. One of the following is NOT a characteristic/criteria of genetic materials, identify it: [CUET 2022] (a) Genetic material should be able to generate its replica. (b) Genetic material should be stable chemically and structurally. (c) It should not provide the scope for mutations. (d) It should be able to express itself in the form of Mendelian characters. 9. Adenine pairs with Thymine through how many hydrogens bonds? [CUET 2022] (a) 2 (b) 3 (c) 4 (d) 5 10. Select the correct statements from the following [CUET 2022] (A) The presence of Thymine at the place of Uracil gives more stability to DNA. (B) Both nucleic acids, i.e., DNA and RNA mutate. (C) DNA is dependent on RNA for synthesis of proteins. (D) RNA mutate at slower rate than DNA. (E) DNA is better genetic material. Choose the correct answer from the options given below: (a) A and D only (b) A and B only (c) A, B, C and D only (d) A, B, C and E only 11. Arrange the following steps in order for the technique of DNA finger printing. [CUET 2022] A. Isolation of DNA. B. Separation of DNA fragments by gel, electrophoresis. C. Digestion of DNA by restriction endonucleases. D. Blotting of separated DNA fragments to nylon membrane. E. Hybridisation using VNTR probe followed by autoradiography. Choose the correct answer from the options given below: (a) A,C,B,D,E (b) A,B,C,D,E (c) A,B,D,C,E (d) A,E,C,B,D

66 Oswaal CUET (UG) Chapterwise Question Bank 12. The term ribosome is applicable to the following molecule in bacteria [CUET 2022] (a) mRNA (b) 23S rRNA (c) 5S rRNA (d) hn RNA 13. What happened when heat-killed S cells along with live R cells were injected into mice in Griffith’s experiment? [CUET 2022] (a) Mice died and showed live R cells in the blood. (b) Mice survived and showed live S cells in the blood. (c) Mice died and showed live S cells in the blood. (d) Mice survived and showed live R cells in the blood. 14. Match List I with List II [CUET 2022] List-I (Genes in lac operon) List-II (Products coded for) A. i-gene I. b-galactosidase B. z-gene II. Transacetylase C. y-gene III. Repressor protein D. a-gene IV. Permease Choose the correct answer from the options given below: (a) A-I, B-II, C-IV, D-III (b) A-III, B-I, C-IV, D-II (c) A-III, B-IV, C-II, D-I (d) A-I, B-II, C-III, D-IV 15. The promoter site and the terminator site for transcription are located at [CUET 2022] (a) 3’ (downstream) end and 5’ (upstream) end, respectively of the transcription unit. (b) 5’ (upstream) end and 3’ (downstream) end, respectively of the transcription unit. (c) the 5’ (upstream) end. (d) the 3’ (downstream) end 16. The net electric charge on DNA and histone is (a) both positive. (b) both negative. (c) negative and positive, respectively. (d) zero. 17. Which of the following steps in transcription is catalysed by RNA polymerase? (a) Initiation (b) Elongation (c) Termination (d) All of the above 18. If the sequence of nitrogeneous bases of the coding strand of DNA in a transcription unit is: 5’-ATGAATG-3’, the sequence of bases in its RNA transcript would be: (a) 5’-AUGAAUG-3’ (b) 5’-UACUUAC-3’ (c) 5’-CAUUCAU-3’ (d) 5’-GUAAGUA-3’ 19. With regard to mature mRNA in eukaryotes: (a) exons and introns do not appear in the mature RNA. (b) exons appear, but introns do not appear in the mature RNA. (c) introns appear but exons do not appear in the mature RNA. (d) both exons and introns appear in the mature RNA. 20. What provides binding site to RNA polymerase? (a) Exons (b) Promoter (c) Inducer (d) Repressor 21. The fact that a purine base always paired through hydrogen bonds with a pyrimidine base leads to, in the DNA double helix [NCERT Exemplar., Q.4, Pg-36] (a) the antiparallel nature. (b) the semi-conservative nature. (c) uniform width throughout DNA. (d) uniform length in all DNA.

BIOLOGY

22. A nucleoside differs from a nucleotide. It lacks the [NCERT Exemplar., Q.5, Pg-36] (a) base (b) sugar (c) phosphate group (d) hydroxyl group 23. The first genetic material could be [NCERT Exemplar., Q.9, Pg-37] (a) protein. (b) carbohydrates. (c) DNA. (d) RNA. 24. Which of the following are the functions of RNA? [NCERT Exemplar., Q.19, Pg-39] (a) It is a carrier of genetic information from DNA to ribosomes synthesising polypeptides. (b) It carries amino acids to ribosomes. (c) It is a constituent component of ribosomes. (d) All of the above. 25. While analysing the DNA of an organism a total number of 5,386 nucleotides were found out of which the proportion of different bases were: Adenine = 29%, Guanine = 17%, Cytosine = 32% and Thymine = 17%. Considering the Chargaff’s rule, it can be concluded that [NCERT Exemplar., Q.20, Pg-39] (a) it is a double-stranded circular DNA. (b) it is single-stranded DNA. (c) it is a double-stranded linear DNA. (d) no conclusion can be drawn. 26. DNA is a polymer of nucleotides which are linked to each other by 3′–5′ phosphodiester bond. To prevent polymerisation of nucleotides, which of the following modifications would you choose? [NCERT Exemplar., Q.13, Pg-38] (a) Replace purine with pyrimidines (b) Remove/replace 3′-OH group in deoxyribose (c) Remove/replace 2′-OH group with some other group in deoxyribose (d) Both (b) and (c) 27. Discontinuous synthesis of DNA occurs in one strand, because: [NCERT Exemplar., Q.14, Pg-38] (a) DNA molecule being synthesised is very long. (b) DNA-dependent DNA polymerase catalyses polymerisation only in one direction (5′→3′). (c) it is a more efficient process. (d) DNA ligase has to have a role. 28. In some viruses, DNA is synthesised by using RNA as template. Such a DNA is called: [NCERT Exemplar., Q.21, Pg-39] (a) A-DNA. (b) B-DNA. (c) cDNA. (d) rDNA. 29. The amino acid attaches to the tRNA at its: [NCERT Exemplar., Q.20, Pg-40] (a) 5’-end. (b) 3’-end. (c) anti-codon site. (d) DHU loop. 30. Regulatory proteins are the accessory proteins that interact with RNA polymerase and affect its role in transcription.

67

MOLECULAR BASIS OF INHERITANCE

(a) (b) (c) (d)

Which of the following statements is correct about regulatory protein? [NCERT Exemplar., Q.17, Pg-38] They only increase expression. They only decrease expression. They interact with RNA polymerase, but do not affect the expression. They can act both as activators and as repressors.

[B] ASSERTION AND REASON QUESTIONS: Directions In the following questions a statement of assertion (A) is followed by a statement of reason (R) Mark the correct choice as: (a) Both assertion (A) and reason (R) are true and reason (R) is the correct explanation of assertion (A). (b) Both assertion (A) and reason (R) are true but reason (R) is not the correct explanation of assertion (A). (c) Assertion (A) is true but reason (R) is false. (d) Assertion (A) is false but reason (R) is true. 1. Assertion (A): Termination codons or stop codons are UAA, UAG and UGA. Reason (R): Stop codons represent termination of translation. 2. Assertion (A): In Griffith’s experiment, the dead R strain bacteria was capable of causing the transformation of the live S-strain bacteria. Reason (R): The S-strain is virulent strain. 3. Assertion (A): Aminoacylation is an essential step for the synthesis of protein. Reason (R): It is the process of adding an activated amino acid to the acceptor arm of a transfer RNA. 4. Assertion (A): DNA fingerprinting is applied in paternity testing in case of disputes. Reason (R): It employs the principle of polymorphism in DNA sequences as polymorphisms are inheritable from parent to children. 5. Assertion (A): Chargaff’s rule is applicable to RNA. Reason (R): RNA contains ribose sugar in them. 6. Assertion (A): Primary transcripts in eukaryotes are nonfunctional. Reason (R): Methyl guanosine triphosphate is attached to 5’ – end of hnRNA. 7. Assertion (A): The newly formed mRNA has same sequence as the coding strand of transcriptional unit with uracil present at place of thymine. Reason (R): The rule of complementarity guides the formation of DNA and RNA. 8. Assertion (A): The nucleosome is a repeating unit of a structure in nucleus called chromatin. Reason (R): The negatively charged DNA is wrapped around the positively charged histone octamer to form a structure called nucleosome. 9. Assertion (A): In RNA, uracil is present at the place of thymine. Reason (R): 5-methyl uracil is the chemical name of thymine. 10. Assertion (A): The enzyme involved in the continuous replication of DNA strand is DNA polymerase.



Reason (R): The polarity of the template strand is 3’ → 5’.

[C] COMPETENCY/CASE-BASED QUESTIONS: I. Read the passage carefully and answer the question given after the passage from 1–5. The process of replication requires a set of catalyst (enzymes). The main enzyme is referred to as DNA dependent DNA Polymerase. E.coli that has only 4.6×106 bp (compare it with human whose diploid content is 6.6×106 bp), completes the process of replication within 18 minutes, that means the average rate of polymerisation has to be approximately 2000 bp per second these polymerase also have catalyse the reaction with high degree of accuracy. Deoxyribonucleoside triphosphates serve dual purposes. In addition to DNA dependent DNA polymerases, many additional enzymes are required to complete the process of replication with high degree of accuracy. For long DNA molecules, since the two strands of DNA cannot be separated in its entire length (due to very high energy requirement), the replication occur with in a small opening of the DNA helix, referred to as replication fork. The DNA dependent DNA polymerase catalyse polymerisation only in one direction, that 5’-3’. This creates some additional complications at the replicating fork. Consequently, on one strand (the template with polarity 3’ → 5’) the replication is continuous, which on the others (the template with polarity 5’ → 3’), it is the discontinuous. The non-continuously synthesized fragments are later joined by the enzyme DNA ligase. 1. The DNA dependent-DNA polymerase catalyse polymerisation in one direction only that is: (a) 5’-3’ of DNA (b) 3’-5’ of DNA (c) 5’-3’ of RNA (d) 3’-5’ of RNA 2. In human beings DNA polymerase complete the process of replication in: (a) 25 Minutes (b) 16 Minutes (c) 12 Minutes (d) 18 Minutes 3. The number of nucleotides in E. coli is: (a) 1.2 × 106 bp (b) 2.3 × 106 bp (c) 4.6 × 106 bp (d) 6.6 × 106 bp 4. In E. coli the main enzyme required to catalyse the polymerisation of deoxynucleotide is: (a) DNA-Dependent DNA Polymerase (b) DNA-Dependent RNA Polymerase (c) RNA-Dependent DNA Polymerase (d) RNA-Dependent DNA Polymerase 5. Which of the following statements are correct? A. DNA dependent DNA polymerase is a slow catalyst B. Deoxyribonucleoside triphosphate provide energy for polymerisation C. Replication fork is a small opening of the DNA helix D. The replication is discontinuous on DNA template with polarity 3’-5’ E. The replication is continuous on DNA template with polarity 3’-5’. Choose the correct answer form the options given below: (a) A and B only (b) B and C only (c) A and D only (d) C and E only II. Study the following diagram and answer the question given below 6–10. The lac operon consists of a regulation gene and three

68 Oswaal CUET (UG) Chapterwise Question Bank structural gene. The lactose acts as inducer. In the presence of an Inducer such as lactose, the repressor is inactivated during the interaction. This allows RNA polymerase access to the promoter and transcription proceeds. The repressor is synthesized which in turn binds with the operator region of the operon and prevents RNA polymerase from transcribing the operon. 6. When the process of Lac operon is blocked by a repressor it represents: (a) Positive regulation. (b) Negative regulation. (c) sometimes positive sometimes negative. (d) both positive and negative regulation. 7. Identify the correct sequence of the structural genes in the lac operon. (a) lacA-lacZ-lacY (b) lacZ-lacA-lacY (c) lacZ-lacY-lacA (d) lacA-lacY-lacZ 8. Which of the following statement is true in reference to the lac operon process in E.coli?

BIOLOGY



(i) Galactosidase is the only enzyme produced in large quantities when lac operon is turned on (ii) The messenger RNA in lac operon is a polycistronic mRNA . (a) Only i is correct (b) Only ii is correct (c) Both (i) and (ii) are correct (d) None of them are correct 9. What provides binding site to RNA polymerase? (a) Exons (b) Promoter (c) Inducer (d) Repressor 10. The lac operon of E. coli contains genes involved in lactose metabolism. It’s expressed only when lactose is____________(1) and glucose is _________(2). (a) 1: Present, 2: Absent (b) 1: Absent, 2: Present (c) 1: More, 2: less (d) 1: repressed, 2: promoted

ANSWER KEY [A] MULTIPLE CHOICE QUESTIONS 1. (d)

2. (c)

3. (b)

4. (d)

5. (b)

6. (b)

7. (a)

8. (c)

9. (a)

10. (d)

11. (a)

12. (b)

13. (c)

14. (b)

15. (b)

16. (c)

17. (b)

18. (a)

19. (b)

20. (b)

21. (c)

22 (c)

23. (d)

24. (d)

25. (b)

26. (b)

27. (b)

28. (c)

29. (b)

30. (d)

1. (a)

2. (d)

3. (a)

4. (b)

8. (b)

9. (b)

10. (b)

1. (a)

2. (d)

3. (c)

9. (b)

10. (a)

[B] ASSERTION & REASON 5. (d)

6. (b)

7. (a)

[C] COMPETENCY/CASE-BASED QUESTIONS 4. (a)

5. (b)

6. (b)

7. (c)

8. (b)

ANSWERS WITH EXPLANATION [A] MULTIPLE CHOICE QUESTIONS: 1. Option (d) is correct Explanation: { Ribosome is involved in translation, the process of protein synthesis. {  Histone proteins are associated with DNA in the nucleus, forming nucleosomes. {  DNA polymerase is essential for DNA replication, where it synthesises new DNA strands. { RNA polymerase is responsible for transcription, which involves the synthesis of RNA from a DNA template. 2. Option (c) is correct Explanation: 1. Griffith first injected the heat-killed ‘S’ strain into mice, and the mice survived. This indicated that the heat-killed ‘S’ strain alone was not lethal. 2. Then, he injected the ‘R’ strain (non-virulent) into mice, and the mice also survived. This confirmed that the ‘R’ strain was not lethal to mice. 3. Griffith then injected the live ‘S’ strain into mice, and the mice died. This showed that the live ‘S’ strain was virulent.

4.  Finally, he performed the experiment where he mixed the heat-killed ‘S’ strain with the live ‘R’ strain and injected it into mice. The mice died in this case, demonstrating that something in the heat-killed ‘S’ strain had transformed the ‘R’ strain into a virulent form, which is now known as transformation. 3. Option (b) is correct Explanation: N-glycosidic linkage is the bond between the nitrogenous base (e.g., adenine, guanine, cytosine, thymine) and the pentose sugar (deoxyribose) in DNA. 4. Option (d) is correct Explanation: Statements true for RNA are: (A) RNA acts as a genetic material for some viruses. (B) RNA also functions as an adapter molecule. (D) RNA also acts as a catalyst in some cases. 5. Option (b) is correct Explanation: Structural proteins are the most abundant class of proteins in nature and form structural elements. Transposons are repetitive DNA sequences that have the capacity to move from one location to another in genome and prohormones are the precursors of hormones. Therefore, Option (B) is the correct answer as any protein encoding gene is expressed in a heterologous host is called recombinant protein.

69

MOLECULAR BASIS OF INHERITANCE

6. Option (b) is correct Explanation: Splicing involves cutting off introns and linking exons in a specific order takes place during the processing of human RNA. At the 5’ end of hn RNA, methyl guanosine triphosphate is added. It is known as capping. Tailing occurs by the addition of adenylate residues of about 200-300 at 3’ end of hn RNA. The mRNA, or completely processed in RNA, is transferred outside of the nucleus for translation. 7. Option (a) is correct Explanation: Alec Jeffreys pioneered the DNA fingerprinting process. The Operon model of gene control in bacteria was proposed by Jacob and Monod. By converting a synthetic poly U RNA into polyphenylalanine, Nirenberg and Matthaei were able to interpret the genetic code. Meselson and Stahl demonstrated that E. coli DNA replication follows a semiconservative paradigm. 8. Option (c) is correct

{ S-strain

(Heat killed) + R-strain (live) → Inject into mice → Mice die He concluded that some ‘transforming principle’, transferred from heat-killed S-strain to R strain. It enabled R-strain to synthesise smooth polysaccharide coat and become virulent. This must be due to the transfer of some genetic material.

14. Option (b) is correct Explanation: The ‘i’ gene codes for the repressor protein of the lac operon. The ‘z’ gene, b-galactosidase hydrolyses lactose to galactose and glucose. The ‘y’ gene codes for Permease, which increases the permeability of the cell to b-galactosidase. 15. Option (b) is correct. Explanation: The promoter site and the terminator site for transcription are located at 5’ (upstream) end and 3’ (downstream) end, respectively of the transcription unit. The promoter is the binding site for RNA polymerase for initiation of transcription.

Explanation: The following requirements must be met by a molecule before it may serve as genetic material; It needs to be stable both chemically and structurally. It should be able to create a copy of itself. It ought to allow for the kind of slow mutation that evolution requires. It ought to be able to manifest itself as Mendelian characteristics.

16. Option (c) is correct.

9. Option (a) is correct

17. Option (b) is correct.

Explanation: Thymine base pair is held together by 2 hydrogen bonds while the Guanine. Cytosine base pair is held together by 3 hydrogen bonds. 10. Option (d) is correct Explanation: DNA is better genetic material. Structurally and chemically DNA is more stable than RNA. This additional stability to DNA is due to the presence of thymine at the place of uracil. DNA mutate slowly with respect to RNA. 2-OH groups in RNA makes it easily degradable. DNA, is dependent on RNA for synthesis of proteins. 11. Option (a) is correct Explanation: Different steps of DNA fingerprinting are: AIsolation of DNA. (From any cells like blood stains, semen stains or hair roots) C- Digestion of DNA by restriction endonucleases. B- Separation of DNA fragments by gel electrophoresis. D- Transferring (blotting) of separated DNA fragments to synthetic membranes, such as nitrocellulose or nylon and then baked in a vacuum oven at 80°C for 3-5 hours (to fix the DNA fragment on the membrane). E- Hybridization using labeled VNTR probe, followed by autoradiography. 12. Option (b) is correct Explanation: In prokaryotes, the larger subunit 50S of ribosome consists of 23S and 5S rRNA whereas in eukaryotes, the larger subunit 60S of ribosome has 28S, 5.8S and 5S rRNA. mRNA copies DNA’s instructions and carries them to the ribosome where proteins can be made. hnRNA is the precursor of mRNA in eukaryotes. 13. Option (c) is correct Explanation: To test for the trait of pathogenicity, Griffith injected mice with mixes of the two strains { S-strain → Inject into mice → Mice die { R-strain → Inject into mice → Mice live { S-strain (Heat killed) → Inject into mice → Mice live

Explanation: DNA consists of a nitrogenous base, pentose sugar and a phosphate group. DNA has negative charge due to the presence of phosphate group. Histone is rich in the basic amino acids lysine and arginine, which carry positive charges in their side chains. Therefore, histone is positively charged. Explanation: The DNA-dependent RNA polymerase helps in transcription by catalysing the polymerisation in only one direction (i.e., 5’-3’). 18. Option (a) is correct. Explanation: 5’-ATGAATG-3’ (coding strand) 5’-TACTTAC-3’ (complementary strand) 5’-AUGAAUG-3’ (RNA). 19. Option (b) is correct. Explanation: In eukaryotes, the monocistronic structural genes have interrupted coding sequences, that is, the genes in eukaryotes are split. The coding sequences or expressed sequences are defined as exons. These sequences (exons) appear in mature or processed RNA, thus exons are interrupted by introns or intervening sequences which do not appear in mature or processed RNA. 20. Option (b) is correct. Explanation: Promoter helps in starting the process of transcription and provides a binding site to RNA polymerase. 21. Option (c) is correct. Explanation: The diameter of the strand is always constant due to a pairing of purine (adenine and guanine) and pyrimidine (cytosine and thymine). This specific bonding gives uniform width to the DNA. 22. Option (c) is correct. Explanation: A nitrogenous base is attached to the pentose sugar by an N-glycosidic linkage to form a nucleoside, that is, Nucleoside = Nitrogen base + Pentose sugar. When a phosphate group is attached to the 5’-OH of a nucleoside through phosphodiester linkage, a nucleotide is formed, that is, Nucleotide = Nitrogen base + Pentose sugar + Phosphate (PO4). So, a nucleoside differs from a nucleotide as it lacks the phosphate group. 23. Option (d) is correct.

70 Oswaal CUET (UG) Chapterwise Question Bank Explanation: RNA was the first genetic material. There is now enough evidence to suggest that essential life processes (e.g., metabolism, translation and splicing), evolved from RNA. RNA is used to act as a genetic material as well as a catalyst (there are some important biochemical reactions in living systems that are catalysed by RNA catalysts and not by protein enzymes). But, RNA being a catalyst was reactive and hence unstable. Therefore, DNA has evolved from RNA with chemical modifications that make it more stable. DNA being double-stranded and having complementary strand, further resists changes by evolving a process of repair. 24. Option (d) is correct. Explanation: Ribosomal RNA (rRNA), messenger RNA (mRNA) and transfer RNA (tRNA) are major classes of RNAs that are involved in gene expression. rRNAs bind protein molecules and give rise to ribosomes. mRNA carries coded information for translation into polypeptide formation. rRNA is also called soluble or adaptor RNA and carries amino acids to mRNA during protein synthesis. 25. Option (b) is correct. Explanation: According to Chargaff’s rules of base pairing, (i) The amount of adenine is always equal to the amount of thymine and the amount of guanine is always equal to the amount of cytosine. (ii) Adenine is joined to thymine with two hydrogen bonds and guanine is joined to cytosine by three hydrogen bonds. (iii) The ratio of adenine to thymine and that of guanine to cytosine is always equal to one, that is., A/T = G/C = 1. In the given organism, the DNA is not following the Chargaff’s rule, hence it can be concluded that it is a singlestranded DNA, not double-stranded. 26. Option (b) is correct. Explanation: The enzyme called DNA polymerase progressively adds deoxyribonucleotides to the free 3′-end of the growing polynucleotide chain so that replication of the 3′–5′ strand of the DNA molecule is continuous (growth of the new strand in 5′→3′direction). So, to prevent polymerisation of nucleotides 3′-OH group in deoxyribose should be replaced/ removed. 27. Option (b) is correct. Explanation: DNA polymerase adds deoxyribose nucleotides to the free 3′-end of the growing polynucleotide chain so that replication of the 3′→5′ strand of the DNA molecule is continuous (growth of the new strand in 5′→3′ direction). Since, DNA-dependent DNA polymerase catalyses polymerisation only in one direction (5′→3′), discontinuous synthesis of DNA occurs in the other strand. 28. Option (c) is correct. Explanation: In some viruses, like retro-viruses (e.g., HIV), an enzyme called reverse transcriptase is used to generate complementary DNA (cDNA) from an RNA template. This process is termed reverse transcription. 29. Option (b) is correct. Explanation: AA-binding site (amino acid binding site) lies at the 3′-end opposite the anti-codon and has CCA-OH group. It is the site where amino acid attaches to the tRNA. 30. Option (d) is correct. Explanation: Regulatory proteins, the accessory proteins that interact with RNA polymerase and affect its role in

BIOLOGY

transcription. It controls the functions of structural genes and are called regulatory genes. Promoters, terminators, operators and repressor are some important regulatory genes. They can act both as activators and as repressors. [B] ASSERTION & REASON: 1. Option (a) is correct. Explanation: Both assertion and reason are true. Stop codons are also known as non-sense codons or termination codons as they do not code for an amino acid. If they are not present, then protein synthesis will continue and result in defective protein. 2. Option (d) is correct. Explanation: It was concluded by the transforming principle, that bacteria is transformed from heat killed S-strain to live R-strain which is non-virulent. 3. Option (a) is correct. Explanation: This attachment is an essential step in the synthesis of protein. This attachment is brought by aminoacylt-RNA synthetase. 4. Option (b) is correct. Explanation: DNA fingerprinting involves identifying differences in some specific regions in DNA sequence called as respective DNA, because in these sequences, a small stretch of Solutions 63 DNA is repeated many times. These sequences normally do not code for any proteins, but they form a large portion of human genome. These sequence show high degree of polymorphism and form the basis of DNA fingerprinting. As the polymorphisms are inheritable from parents to children, DNA fingerprinting is the basis of paternity testing in case of disputes. 5. Option (d) is correct. Explanation: According to Chargaff’s rule the DNA helix contain equal molar ratio of A & T, G & C.... This does not apply to RNA, as uracil is present in RNA instead of DNA. In place of deoxyribose sugars RNA contains ribose sugar, which make it more unstable and highly sensitive to degradation. 6. Option (b) is correct. Explanation: Primary transcripts contains both introns and exon, in which introns are non- coding parts. At 5’ end of hnRNA, a cap is formed by modification of GTP into 7-methyl guanosine in the process called capping. 7. Option (a) is correct. Explanation: The sequence of mRNA will be identical to the given sequence of coding strand Question given for practice and its solution is given at the end of the chapter 128 except for the presence of uracil in place of thymine in mRNA. 8. Option (b) is correct. Explanation: The length of DNA in a human diploid cell is around 2.2 metres. It is greater than the dimension of a typical nucleus. In order to fit themselves in the nucleus, DNA is wrapped around histone octomer to form nucleosome. The nucleosome in chromatin gives a ‘beads’ on string appearance. 9. Option (b) is correct. Explanation: Uracil is more reactive and unstable than thymine. Since the RNA is comparatively short-lived, uracil would suffice to achieve the function of RNA and any potential errors do not lead to any lasting damage. In DNA if the the base was uracil instead thymine, any damage to DNA can change the nucleotide bases causing mutation. 10. Option (b) is correct. Explanation: DNA polymerase enzyme is responsible for synthesising DNA, they add nucleotides one by one to the growing DNA chain, adding those which are complementary

71

MOLECULAR BASIS OF INHERITANCE

to the template. The template strand has polarity in 3’ → 5’ because the RNA-polymerase can polymerize RNA only in 5’ - 3’ direction. [C] COMPETENCY/CASE-BASED QUESTIONS: 1. Option (a) is correct Explanation: The DNA dependent DNA polymerase catalyse polymerisation only in one direction that is 5’ → 3’ of DNA. 2. Option (d) is correct Explanation: In human beings DNA polymerase complete the process of replication within 18 minutes. 3. Option (c) is correct Explanation: The number of nucleotide in E.Coli is 4.6 × 10s bp. 4. Option (a) is correct Explanation: In E.Coli the main enzyme required to catalyse the polymerisation of deoxynucleotide is DNA dependent DNA polymerase. 5. Option (b) is correct Explanation: The two terminal phosphates in a deoxynucleoside triphosphates are high-energy phosphates, same as ATP. It is also a building block of DNA strand. Since, the two strands of DNA cannot be separated in its entire length (due to high energy requirement), the replication occurs within a small opening of the DNA helix, called replication fork. The replication is continuous on the template strand with polarity 3'→5' while on the other strand replication is discontinuous with polarity 5'→3'. As per NTA option (B) is correct however

24

as per NCERT, statements (B), (C) and (E) are correct but none of the option includes all three statements that are (B), (C) and (E). 6. Option (b) is correct. Explanation: The lac operon regulation can be in both negative and positive ways. It is a negative control system because expression is typically blocked by an active repressor (the lac repressor) that turns off transcription. And when CAP (catabolite gene activating protein) binds upstream of this operator region near the promoter and transcription increases, this is an example of a positive system. 7. Option (c) is correct. Explanation: The lac operon consists of 3 structural genes, and a promoter, a terminator, regulator, and an operator. The three structural genes are: lacZ, lacY, and lacA. 8. Option (b) is correct. Explanation: The messenger RNA produced by transcription carries information for the synthesis of all three proteins found in all three structural genes. Hence, it is a polycistronic messenger RNA. 9. Option (b) is correct. Explanation: Promoter helps in starting the process of transcription and provides a binding site to RNA polymerase. 10. Option (a) is correct. Explanation: The lac operon of E. coli contains genes involved in lactose metabolism. It’s expressed only when lactose is present and glucose is absent.

Study Time

CHAPTER

7

Max. Time: 1:5 Hours Max. Questions: 50

EVOLUTION

Origin of Life: Peering into the Past { When observing stars in the night sky, we are essentially

looking back in time due to the vast distance and the time it takes for their light to reach us. { Stellar distances are measured in light years, meaning the light we see today started its journey millions of years ago from trillions of kilometers away. { In contrast, objects in our immediate surroundings are seen instantly, representing the present time.

The Universe and Earth’s Insignificance { The universe is immense, approximately 20 billion years

old, and contains galaxies, stars, and cosmic clouds. {  Earth is minuscule in comparison, formed about 4.5 billion years ago within the Milky Way galaxy. { The Big Bang theory explains the origin of the universe, describing a colossal explosion and the subsequent formation of galaxies.

Early Earth and the Emergence of Life {  Early

Earth lacked an atmosphere, with gases like water vapour, methane, carbon dioxide, and ammonia covering its surface. {  Sun’s UV rays broke up water into hydrogen and oxygen, forming the basis for the atmosphere and the ozone layer. {  As Earth cooled, water vapour condensed into rain, filling depressions and forming oceans. {  Life emerged approximately 500 million years after Earth’s formation, nearly four billion years ago.

{ However,

the origin of the first life form on Earth remained a mystery.

Chemical Evolution and Oparin-Haldane ­Hypothesis { Oparin

(Russia) and Haldane (England) proposed that life’s first forms could have emerged from non-living organic molecules like RNA and proteins. { They suggested that chemical evolution, the formation of organic molecules from inorganic constituents, preceded life. {  Earth’s conditions at the time included high temperatures, volcanic storms, and a reducing atmosphere with methane and ammonia.

Stanley Miller’s Experiment and Meteorite Analysis {  In

1953, Stanley L. Miller conducted laboratory experi­ ments simulating early Earth conditions, producing amino acids through electric discharge in a closed flask. { Other experiments under similar conditions generated sugars, nitrogen bases, pigments, and fats.

Scan to know more about this topic

What Was The Miller-Urey Experiment?

The Panspermia Hypothesis {  Some

scientists propose the possibility of life originating from outside Earth. {  Ancient Greek thinkers suggested the transfer of life units or spores to different planets, including Earth. { The concept of panspermia remains a favourite among some astronomers.

4

Water inlet

The Demise of Spontaneous Generation { Early

theories suggested that life arose from decaying and rotting matter, such as straw and mud, known as spontaneous generation. { Louis Pasteur’s experiments definitively disproved spontaneous generation, demonstrating that life only arises from pre-existing life.

Fig 1: Miller and Urey’s Experiment

EVOLUTION

EVOLUTION

133

73

74 Oswaal CUET (UG) Chapterwise Question Bank {  Analysis

of meteorite content revealed similar compounds, suggesting these processes may occur in space.

The Mystery of the First Self-Replicating Life Forms { The

origin of the first self-replicating metabolic life form remains unknown. { The first non-cellular life forms, potentially giant molecules like RNA and proteins, could have originated around three billion years ago. { The first cellular life forms, likely single-celled, might not have emerged until approximately two billion years ago, all in a water-based environment.

The Accepted Theory: Slow Evolution from Non-Living Molecules { The prevailing view suggests that life evolved gradually

Common Ancestry and Geological History { According to evolutionary theory, all existing life forms

share common ancestors, although these ancestors existed at different periods in Earth’s history, spanning epochs, periods, and eras. { The geological history of Earth is closely intertwined with its biological history. {  A widely accepted conclusion is that Earth is not merely thousands of years old, as previously believed, but instead, it is billions of years old. The theory of evolution through natural selection has provided a robust framework for understanding the diversity of life on Earth and the vast expanse of geological time over which this diversity has developed.

Evidence for Evolution

from non-living molecules through evolutionary forces. {  This theory implies that complex biodiversity developed from these early forms of life. The journey from the origin of life to the complex diversity we observe today remains a captivating and ongoing scientific exploration.

Challenging the Theory of Special Creation religious literature advocates the theory of special creation, which encompasses three key ideas: 1. All living organisms we see today were created as they are. 2. Biodiversity has remained constant since creation and will remain so in the future. 3. Earth is approximately 4,000 years old.

{ Conventional



Charles Darwin’s Observations and Evolutionary Theory Darwin, during his voyage on the H.M.S. Beagle, observed that existing living forms share similarities, both with each other and with life forms that existed millions of years ago. { Many of these ancient life forms have become extinct, while new forms have emerged over Earth’s history. { Darwin proposed the concept of gradual evolution of life forms, challenging the idea of fixed, unchanging species. { He noted that within any population, there is inherent variation in characteristics. { Characteristics that provide better adaptation to natural conditions, such as climate, food sources, and physical factors, lead to greater reproductive success. { Darwin emphasised that “fitness” in this context refers primarily to reproductive fitness. {  Individuals or populations better suited to their environment produce more offspring, leading to their increased survival. This mechanism he termed as “natural selection.”

{  Charles

Alfred Wallace’s Parallel Contribution {  Alfred

Wallace, a naturalist working in the Malay Archipelago, independently arrived at similar conclusions around the same time as Darwin.

BIOLOGY





1. Fossils and the Geological Record {  Fossils are the preserved Scan to know remains of organisms found more about this topic in rocks. {  Sedimentary rock layers contain fossils of different life forms, indicating the arrangement of sediments Evolution of Man - Evolution over Earth’s history. { Fossils in different sedi­mentary layers provide in­ sights into the geological period in which they existed, showing that life forms have varied over time. { This evidence is known as paleontological evidence and supports the concept of evolution. 2. Embryological Evidence {  Ernst Haeckel proposed embryological evidence (“ontogeny repeats phylogeny)” for evolution, suggesting common features in the embryonic stages of all vertebrates. { For example, vertebrate embryos develop vestigial gill slits, which are functional only in fish. {  However, Karl Ernst von Baer later disapproved this proposal, noting that embryos do not pass through the adult stages of other animals. 3. Comparative Anatomy and Morphology {  Comparative anatomy reveals similarities and differences among modern and ancient organisms. {  Similarities in anatomical structures suggest common ancestry. {  For example, the bone structure of forelimbs in whales, bats, cheetahs, and humans is similar, despite different functions. {  Homologous structures, such as these, indicate common ancestry. { Analogous structures, like butterfly and bird wings, serve similar functions but are not anatomically similar. These are the result of convergent evolution. 4. Biochemical Evidence {  Similarities in proteins and genes performing specific functions among diverse organisms suggest common ancestry.

75

EVOLUTION { These

biochemical similarities support the idea of shared evolutionary history. 5. Artificial Selection {  Humans have selectively bred plants and animals for various purposes like agriculture, horticulture, and sports. {  This selective breeding has led to the creation of different breeds within the same species. { It highlights how selection pressures can result in the rapid development of new varieties. 6. Industrial Melanism as an Example of Natural Selection {  Observations of moths in England before and after industrialisation provide evidence for natural selection. {  Before industrialisation, there were more whitewinged moths, as they could blend in with the lichen-covered trees. { After industrialisation, dark-winged moths became more common as they could better camouflage against soot-covered tree trunks. { This demonstrates how survival and reproduction of moths were influenced by their ability to blend into their environment i.e. ability to escape predators. 7. Anthropogenic Evolution {  Human actions, such as the excessive use of herbicides, pesticides, antibiotics, and drugs, have led to the selection of resistant varieties of plants and microbes in a much shorter time frame. {  This demonstrates the rapidity of evolution in response to selective pressures. 8. Stochastic Nature of Evolution { Evolution is a stochastic process driven by chance events in nature and mutations in organisms. {  It is not a directed or deterministic process but results from natural selection acting on random variations. These various lines of evidence collectively support the theory of evolution by natural selection as a mechanism for the development of life forms on Earth over vast period of time.

Adaptive Radiation: The P ­ henomenon { Definition:

Adaptive radiation is the process of evolution in which multiple species diverge from a common ancestor and adapt to different ecological niches or habitats.

The factors favouring ­adaptive radiation { Geographical

Isolation: Adaptive radiation often occurs in a specific geographical area where a single ancestral species faces diverse environmental conditions and ecological Scan to know opportunities. more about { Diversification of Traits: Over this topic time, the ancestral species gives rise to multiple descendant species, each with unique adaptations and traits that Adaptive enable them to exploit different Radiation resources or habitats.

{ Example

1: Darwin’s Finches: Charles Darwin observed this phenomenon during his visit to the Galapagos Islands, where he found multiple varieties of finches with different beak shapes adapted to various food sources, such as insectivory and herbivory. {  Example 2: Australian Marsupials: In Australia, a diverse range of marsupials evolved from a common ancestral stock, each adapting to different ecological niches within the isolated Australian continent.

Convergent Evolution within Adaptive ­Radiation { In cases where more than one adaptive radiation occurs

within the same geographical area, it can result in convergent evolution. { Convergent Evolution is the condition when different species develop similar traits or characteristics independently due to the selection pressures of similar habitats or ecological niches. {  Example: Placental mammals and marsupials in Australia exhibit convergent evolution, with similar species of animals like placental wolves resembling Tasmanian wolves (marsupials).

Biological Evolution and Natural Selection {  Darwin’s

theory of evolution is primarily based on natural selection, which is the driving force behind biological evolution. { Rate of Evolution: The rate of evolution is linked to an organism’s life cycle or life span. Organisms with shorter life cycles, like microbes, can evolve rapidly in response to changing conditions. { Selective Pressures: Changes in the environment can lead to the survival of certain variants within a population, resulting in new species over time. { Adaptive Ability and Fitness: Adaptive abilities that increase an organism’s fitness are inherited and have a genetic basis. { Branching Descent and Natural Selection: These are the two central concepts of Darwinian Theory of Evolution. { Darwin’s theory contrasts with Lamarck’s earlier idea that evolution occurred through the use and disuse of organs, a notion that is no longer widely accepted.

Natural Selection and Population Dynamics {  Natural

selection is based on factual observations, including limited natural resources, stable population sizes with fluctuations, genetic variation within populations, and the heritability of traits. { The competition for limited resources in a population results in the survival and reproduction of individuals with advantageous traits. {  Over many generations, these traits become more prevalent in the population, leading to evolutionary change and the emergence of new forms. Darwin’s groundbreaking insights into natural selection and adaptive radiation have fundamentally shaped our

76 Oswaal CUET (UG) Chapterwise Question Bank understanding of the diversity of life on Earth and the mechanisms that drive its evolution.

Scan to know more about this topic

Mechanism of Evolution { Mendel’s

Inheritance Principles (19th Century): What is Natural Selection?  Gregor Johann Mendel’s work on pea plants revealed the existence of inheritable “factors” that influence an organism’s phenotype. These factors, now known as genes, are responsible for the transmission of traits from one generation to the next. { Darwin’s Theory of Natural Selection (19th Century):  Charles Darwin proposed the theory of natural selection, which emphasised that variations within a population are essential for evolution.  Darwinian variations are small, heritable changes that accumulate gradually over time. Natural selection acts on these variations, favouring individuals with traits that enhance their survival and reproduction. Over generations, the traits of the fittest individuals become more prevalent in the population, leading to evolutionary change. { Hugo de Vries and the Idea of Mutations (Early 20th Century): In the early 20th century, Hugo de Vries introduced the concept of mutations as a mechanism of evolution. He proposed that mutations, which are sudden and large changes in an organism’s genetic material, could drive evolutionary change. Unlike Darwinian variations, mutations are random and typically not adaptive.  De Vries called this rapid evolutionary change through mutation “saltation.” { Population Genetics (Mid-20th Century):  Studies in population genetics, particularly the work of scientists like J.B.S. Haldane, R.A. Fisher, and Sewall Wright, provided a more comprehensive understanding of how genes and variations function in populations. These researchers explained how the frequency of alleles (different forms of genes) in a population can change over time due to factors like natural selection, genetic drift, mutation, and gene flow.  They showed that both gradual changes and sudden mutations can contribute to the evolution of species. { Hardy-Weinberg Principle:  The Hardy-Weinberg principle is a fundamental concept in population genetics that describes the genetic equilibrium in a population.  It states that allele frequencies in a population remain stable from generation to generation if certain conditions are met.

BIOLOGY

 The

gene pool, which includes all the genes and their alleles in a population, remains constant over time.  The principle is expressed mathematically as: p² (frequency of AA) + 2pq (frequency of Aa) + q² (frequency of aa) = 1, where p and q represent the frequencies of allele A and allele a, respectively.  This equation illustrates that the sum of the frequencies of all possible genotypes for a given gene locus equals 1, representing the entire gene pool of the population.  The Hardy-Weinberg equilibrium provides a baseline against which changes in allele frequencies due to evolutionary forces can be measured. { Factors Affecting Genetic Equilibrium:

Gene Migration or Gene Flow: {  When

individuals from one population migrate to another population and interbreed, they introduce new alleles into the gene pool of the receiving population. { This can lead to changes in allele frequencies in both the source and recipient populations, disrupting the genetic equilibrium.

Genetic Drift: { Genetic

drift refers to the random fluctuations in allele frequencies that can occur in small populations. { Due to chance events, some alleles may become more or less prevalent over time, leading to deviations from the Hardy-Weinberg equilibrium.

Mutation: { Mutations

are random and spontaneous changes in an organism’s DNA sequence. {  New alleles can arise through mutations, and if these mutations are beneficial and increase an individual’s fitness, they can become more common in the population.

Genetic Recombination: {  Genetic

recombination occurs during gametogenesis when chromosomes exchange genetic material through processes like crossing over. {  This can generate new combinations of alleles, contributing to genetic diversity in the population.

Natural Selection: {  Natural

selection favours individuals with traits that enhance their survival and reproductive success in a given environment. {  Alleles associated with advantageous traits become more common in the population over time, while less favourable alleles may decrease in frequency.

Timeline of Evolution { Emergence

of Cellular Life: on Earth traces back approximately 2000 million years ago (2 billion years) when the first cellular life forms emerged.

 Life

77

EVOLUTION  Some

early cells developed the ability to release oxygen (O2) as a result of a process possibly analogous to photosynthesis, splitting water with captured solar energy.  Single-celled organisms gradually evolved into multicellular life forms. { 500 Million Years Ago (mya): Invertebrates emerged and became active in Earth’s oceans. { 350 mya: Jawless fish likely evolved during this period.  Some aquatic plants, such as seaweeds, began to exist around this time. { Plants Invading Land: The first organisms to colonise land were plants. They established themselves on land before animals. Some fish developed strong fins which allowed them to move between land and water around 350 mya. { The Coelacanth Discovery (1938): In 1938, a fish called the Coelacanth was discovered off the coast of South Africa. Previously, it was thought to be extinct. Coelacanths are lobefin fish, believed to be ancestors of modern amphibians, frogs, and salamanders. { Evolution of Amphibians: Amphibians were among the first vertebrates to live both in water and on land. Ancestors of modern frogs and salamanders likely evolved from these early amphibians. { Transition to Reptiles:  Amphibians gave way to reptiles, which had the advantage of laying thick-shelled eggs that did not dry out in the sun.  Modern descendants of these reptiles include turtles, tortoises, and crocodiles. { Era Dominated by Reptiles: Reptiles of various shapes and sizes dominated the Earth for approximately 200 million years. { Giant Ferns and Pteridophytes:  During this era, giant ferns (pteridophytes) were prevalent, eventually contributing to the formation of coal deposits. { Fish-like Reptiles (Ichthyosaurs): Some land reptiles returned to the water, evolving into fish-like reptiles, such as Ichthyosaurs, around 200 mya. { Dinosaur Dominance: Dinosaurs were the most prominent reptiles during this era, with species like Tyrannosaurus rex growing to heights of around 20 feet and featuring fearsome teeth.  Approximately 65 mya, dinosaurs mysteriously disappeared from the Earth, and the exact cause remains a subject of debate.

{ Mammalian

Evolution: initially small and resembling shrews, evolved during this time.  Mammals were viviparous (giving birth to live young) and displayed more advanced sensory and survival capabilities than reptiles. { Continental Drift Impact: The shifting of continents due to continental drift had significant effects on the distribution of animals.  For example, South American mammals faced competition from North American fauna when the continents joined and the Australian continent has some endemic species due to geographical isolation. { Unique Evolutionary Histories: Various animal species, including horses, elephants, and dogs, have unique evolutionary histories that are studied in detail. { Origin and Evolution of Man The origin and evolution of humans (Homo sapiens) is a fascinating journey that spans millions of years.  The evolution of humans is characterised by the development of language skills and selfconsciousness This evolutionary process has led to the development of modern humans with advanced cognitive abilities and complex societies. { Around 15 Million Years Ago (mya): Primates known as Dryopithecus and Ramapithecus existed. These early primates exhibited features similar to gorillas and chimpanzees. R amapithecus displayed more human-like characteristics compared to Dryopithecus. { 3-4 mya:  Evidence suggests that human-like primates lived in eastern Africa during this period. These hominids likely stood upright and were not taller than about 4 feet.  Australopithecines, dating back around 2 mya, inhabited East African grasslands. They were likely omnivorous, using stone tools for hunting but primarily consuming fruits.  The earliest hominids, Homo habilis, with brain capacities ranging from 650-800cc, were part of this evolutionary stage. { 1.5 mya: Fossils discovered in Java in 1891 provided insights into the next stage of human evolution, Homo erectus. Homo erectus had a larger brain, approximately 900cc, and is believed to have incorporated meat into their diet. { Neanderthal Man:  Neanderthal man, with a brain size of around 1400cc, inhabited the Near East and Central Asia between 100,000 to 40,000 years ago.  They used animal hides for protection and had burial practices, suggesting a degree of cultural sophistication.  Mammals,

78 Oswaal CUET (UG) Chapterwise Question Bank { Modern

Homo sapiens: sapiens, modern humans, originated in Africa and eventually migrated to different continents. The emergence of distinct races occurred as Homo sapiens adapted to various environments and regions. { Ice Age and Cave Art: During the ice age, which lasted from approximately 75,000 to 10,000 years ago, modern Homo sapiens developed.  Prehistoric cave art, dating back around 18,000 years ago, provides evidence of early humans’ artistic expression. Homo

BIOLOGY

Notable examples of such art can be found in places

like the Bhimbetka rock shelter in the Raisen district of Madhya Pradesh, India. { Agriculture and Human Settlements:  Agriculture emerged around 10,000 years ago, marking a significant shift in human lifestyles. The ability to cultivate crops led to the development of permanent human settlements and the growth of civilizations. { Human History:  The subsequent history of humanity involves the rise and fall of civilisations, technological advancements, cultural developments, and the shaping of modern societies.

OBJECTIVE TYPE QUESTIONS [A] MULTIPLE CHOICE QUESTIONS: 1. Identify the factor which does not affect the HardyWeinberg equilibrium? [CUET 2023] (a) Genetic drift (b) Natural selection (c) Genetic recombination (d) Genetic equilibrium 2. Single step large mutation leading to speciation is also called: [CUET 2023] (a) Founder effect (b) Saltation (c) Branching descent (d) Natural selection 3. Paleontological evidences for evolution refers to: [CUET 2023] (a) Development of embryo (b) Homologous Organs (c) Fossils (d) Analogous Organs 4. Oparin and Haldane proposed the theory that the first form of life could have come from [CUET 2022] (a) Non-living organic molecules (b) Inorganic molecules (c) UV rays (d) CO2 and water 5. The theory of Chemical evolution was studied and tested in laboratory by— [CUET 2022] (a) Charles Darwin (b) S.L. Miller (c) Louis Pasteur (d) Haldane 6. Development of analogous structure or organs is a result of:...................: [CUET 2022] (a) Divergent evolution (b) Convergent evolution (c) Adaptive radiation (d) Saltation 7. Match List I with List II [CUET 2022] List I A) Homo habilis

List-I I. Used hides to protect their body and buried their dead

B) Dryopithecus

II. Hunted with stone weapons but essentially ate fruits

C) Australopithecines

III. First human-like being

D) Neanderthal man

IV. Ape-like ancestor

Choose the correct answer from the options given below: (a) A - I, B - II, C - III, D - IV (b) A - III, B - IV, C - II, D - I (c) A - IV, B - III, C - I, D - II (d) A - II, B - I, C - IV, D - III 8. Match List I with List II [CUET 2022] List I

List-I

A) Genetic drift

I. Change in allele frequency due to immigration and emigration

B) Stabilising selection

II. Change in gene frequency due to chance factor

C) Gene flow

III. More individuals acquire mean character value

D) Saltation

IV. S  ingle step large mutation

Choose the correct answer form the options given below: (a) A - I, B – II, C - III, D - IV (b) A - III, B - 1, C - II, D - IV (c) A - I, B - III, C - IV, D - II (d) A - II, B - III, C - I, D – IV 9. Which of the following sequences fits into the organic evolution theory proposed by Charles Darwin and Alfred Wallace? [CUET 2021] (a) Variations, constancy of population size, overproduction, natural selection (b) Variations, overproduction, constancy of population size, natural selection (c) Overproduction, variations, constancy of population size, natural selection (d) Overproduction, constancy of population size, variations, natural selection

79

EVOLUTION

10. Evolution occurs as a result of mutations in [CUET 2021] (a) somatic DNA (b) somatic RNA (c) germplasm DNA (d) germplasm RNA 11. Match the scientists listed under column ‘A’ with ideas listed column ‘B’. Column A

Column B

A) Darwin

I. Abiogenesis

B) Oparin

II. Use and disuse of organs

C) Lamarck

III. Continental drift theory

D) Wegener IV. Evolution by natural selection A B C D         A B C D (a) (i) (iv) (ii) (iii) (b) (iv) (i) (ii) (iii) (c) (ii) (iv) (iii) (i) (d) (iv) (iii) (ii) (i) 12. Variations during mutations of meiotic recombinations are: (a) random and directionless (b) random and directional (c) random and small (d) random, small and directional 13. Which one of the following changes involved is not correct regarding the evolution of man? (a) Absence of tail (b) Perfection of hand for making tool (c) Change of diet from hard nuts and hard roots to soft food (d) Increase in the ability to communicate with others and develop community behaviour. 14. Weismann cut off tails of mice generation after generation but tails neither disappeared nor shortened. According to this, which of the following statement is correct? (a) Darwin was correct. (b) Tail is an essential organ. (c) Mutation theory is wrong. (d) Lamarckism was wrong in inheritance of acquired characters. 15. The process by which organisms with different evolutionary history evolve similar phenotypic adaptation in response to a common environmental challenge, is called: (a) Natural selection (b) Convergent evolution (c) Non-random evolution (d) Adaptive radiation 16. Among the following sets of examples for divergent evolution, select the incorrect option: (a) Forelimbs of man, bat and cheetah (b) Eye of octopus, bat and man (c) Brain of bat, man and cheetah (d) Heart of bat, man and cheetah 17. Biogas is composed of majorly: (a) methane, CO2 and O2 (b) CO2, H2S, and H (c) methane, CO2 (d) H2S, H and O2 18. Excreta preserved as a fossil is known as: (a) Coprolite (b) Stromatolite (c) Compressed fossil (d) None of the above

19. The presence of NaCl in body fluid shows that life is originated in: (a) Salt solution (b) Primitive oceans (c) Rivers (d) All of the above. 20. Evolutionary convergence is development of a: (a) common set of functions in groups of different ancestry. (b) dissimilar set of functions in closely related groups. (c) common set of structures in closely related groups. (d) dissimilar set of functions in unrelated groups. 21. The theory of spontaneous generation stated that: [NCERT Exemplar Q.21, Pg.45] (a) life arose from living forms only. (b) life can arise from both living and non-living. (c) life can arise from non-living things only. (d) life arises spontaneously, neither from living nor from the non-living. 22. In 1953, S. L. Miller created primitive earth conditions in the laboratory and gave experimental evidence for origin of first form of life from pre-existing non-living organic molecules. The primitive earth conditions created include [NCERT Exemplar Q.17, Pg.48] (a) low temperature, volcanic storms and atmosphere rich in oxygen. (b) low temperature, volcanic storms and reducing atmosphere. (c) high temperature, volcanic storms and nonreducing atmosphere. (d) high temperature, volcanic storms, reducing atmosphere containing CH4, NH3, etc. 23. Theory of “Continuity of Germplasm” was given by August: (a) G. Mendel (b) Lamarck (c) Haeckel (d) August Weismann. 24. Fossils are generally found in [NCERT Exemplar Q.11, Pg.46] (a) sedimentary rocks. (b) igneous rocks. (c) metamorphic rocks. (d) any type of rock. 25. The bones of forelimbs of whale, bat, cheetah and man are similar in structure, because: [NCERT Exemplar Q.5, Pg.45] (a) one organism has given rise to another. (b) they share a common ancestor. (c) they perform the same function. (d) they have bio-chemical similarities. 26. Analogous organs arise due to: [NCERT Exemplar Q.6, Pg.46] (a) divergent evolution. (b) artificial selection. (c) genetic drift. (d) convergent evolution. 27. Evolution of life shows that life forms had a trend of moving from: [NCERT Exemplar Q.9, Pg.46] (a) land to water. (b) dry land to wet land. (c) fresh water to sea water. (d) water to land. 28. Which of the following is an example for link species? [NCERT Exemplar Q.15, Pg.47]

80 Oswaal CUET (UG) Chapterwise Question Bank (a) Lobe fish (b) Dodo bird (c) Sea weeds (d) Chimpanzee 29. Viviparity is considered to be more evolved because: [NCERT Exemplar Q.10, Pg.46] (a) the young ones are left on their own. (b) the young ones are protected by a thick shell. (c) the young ones are protected inside the mother’s body and are looked after they are born leading to more chances of survival. (d) the embryo takes a long time to develop. 30. The most accepted line of descent in human evolution is: [NCERT Exemplar Q.14, Pg.49] (a) Australopithecus → Ramapithecus → Homo sapiens → Homo habilis. (b) Homo erectus → Homo habilis → Homo sapiens. (c) Ramapithecus → Homo habilis → Homo erectus → Homo sapiens. (d) Australopithecus → Ramapithecus → Homo erectus → Homo habilis → Homo sapiens. [B] ASSERTION & REASON: Directions In the following questions, a statement of assertion (A) is followed by a statement of reason (R). Mark the correct choice as: (a) Both assertion (A) and reason (R) are true and reason (R) is the correct explanation of assertion  (A). (b) Both assertion (A) and reason (R) are true but reason (R) is not the correct explanation of assertion (A). (c) Assertion (A) is true but reason (R) is false. (d) Assertion (A) is false but reason (R) is true. 1. Assertion (A): Fossils are proof of evolution. Reason (R): Fossils are the preserved remains or traces of organisms from the distant past. 2. Assertion (A): Wings of birds and bats are different in origin but similar function. Reason (R): Wings of birds and bats are example of homologous structure. 3. Assertion (A): The scientific theory of evolution by natural selection was given by Charles Darwin and A R Wallace. Reason (R): Evolution refers to the changes in a population or species through time. 4. Assertion (A): Best adapted and less adapted individuals reproduce at the same rate. Reason (R): Adaptations help an organism to survive in a particular environment. 5. Assertion (A): Amphibians have evolved from fishes. Reason (R): Archaeopteryx is a fossil linking reptiles and amphibians. 6. Assertion (A): The most significant trend in the evolution of Modern man (Homo sapiens) is its increased cranial capacity. Reason (R): The volume of human brain has increased from about 600 cc in Homo habilis to 1400 cc in Modern man (Homo sapiens). 7. Assertion (A): Hugo de-Vries, defined the mechanism of evolution as saltation.

BIOLOGY



Reason (R): Saltation is a single step large mutation which brings major change as speciation. 8. Assertion (A): Ramapithecus and Dryopithecus were early primates that existed around 15 million years ago. Reason (R): Ramapithecus displayed more human-like characteristics compared to Dryopithecus. 9. Assertion (A): Homo habilis had brain capacities ranging from 650-800cc. Reason (R): Homo habilis primarily consumed meat as a significant part of their diet. 10. Assertion (A): Agriculture emerged around 10,000 years ago, leading to the development of permanent human settlements. Reason (R): The ability to cultivate crops allowed early humans to shift from a nomadic lifestyle to a more settled one. [C] COMPETECY/CASE-BASED QUESTIONS I. Read the passage carefully and answer the question given after the passage from 1–5. Charles Darwin’s theory of natural selection is a cornerstone of our understanding of evolution. During his voyage on the H.M.S. Beagle, Darwin observed that living organisms, both past and present, share similarities and differences. He proposed that life evolves gradually, challenging the belief in unchanging species. Darwin emphasized the presence of variations within populations, where traits promoting adaptation to the environment lead to greater reproductive success. “Fitness” primarily refers to reproductive fitness in this context. Individuals better suited to their surroundings produce more offspring, increasing their survival. Alfred Wallace independently arrived at similar conclusions, contributing significantly to the theory of natural selection. The idea of common ancestry suggests that all life forms share ancestors from different periods in Earth’s extensive history, highlighting the intertwined relationship between Earth’s geological and biological pasts.  1. What is the central concept of Charles Darwin’s theory of natural selection? (a) Gradual evolution of life forms (b) Rapid appearance of new species (c) Fixed, unchanging species (d) Sudden, random mutations 2. According to Darwin, what primarily defines “fitness” in the context of natural selection? (a) Physical strength (b) Reproductive success (c) Longevity (d) Adaptation to the environment 3. Who independently arrived at conclusions similar to Charles Darwin’s theory of natural selection? (a) Gregor Mendel (b) Alfred Wallace (c) Louis Pasteur (d) Stanley Miller 4. What does the concept of common ancestry propose? (a) All life forms share the same habitat. (b) Living beings have no similarities with ancient species.

81

EVOLUTION

(c) Existing life forms have no common ancestors. (d) All life forms share ancestors from different time periods. 5. How does the passage describe the relationship between Earth’s geological and biological histories? (a) They are unrelate(d) (b) Geological history is more important. (c) They are intertwine(d) (d) Biological history is more important. II. Directions from question no 46-50: Read the table and answer the questions.  Time Period (mya) ~2000 ~500 ~350 ~200

~200 ~200 ~65 ~18,000 ~10,000

Key Evolutionary Events Emergence of cellular life forms. Invertebrates appear in oceans. Jawless fish and early plants exist. Transition to reptiles; dominance of reptiles. Giant ferns contribute to coal deposits. Emergence of fish-like reptiles. Dinosaurs dominate; Tyrannosaurus rex. Early mammals resembling shrews evolve. Coelacanth rediscovered; ancestor of amphibians. Mysterious disappearance of dinosaurs. Development of prehistoric cave art. Emergence of agriculture; settled human societies.

6. During which time period did the emergence of cellular life forms occur? (a) ~500 mya (b) ~200 mya (c) ~2000 mya (d) ~10,000 mya 7. What significant event is associated with the time period of ~350 mya? (a) Emergence of invertebrates (b) Development of giant ferns (c) Transition to reptiles (d) Mysterious disappearance of dinosaurs 8. Which ancient group of organisms is believed to be the ancestor of modern amphibians, frogs, and salamanders? (a) Invertebrates (b) Giant ferns (c) Jawless fish (d) Coelacanth 9. Approximately how many million years ago did the mysterious disappearance of dinosaurs occur? (a) ~200 mya (b) ~500 mya (c) ~65 mya (d) ~18,000 mya 10. What event is linked to the time period of ~10,000 years ago? (a) Emergence of agriculture (b) Development of prehistoric cave art (c) Transition to reptiles (d) Emergence of invertebrates

ANSWER KEY [A] MULTIPLE CHOICE QUESTIONS 1. (d)

2. (b)

3. (c)

4. (a)

5. (b)

6. (b)

7. (b)

8. (d)

9. (c)

10. (c)

11. (b)

12. (a)

13. (c)

14. (d)

15. (b)

16. (d)

17. (c)

18. (a)

19. (b)

20. (a)

21. (c)

22 (d)

23. (c)

24. (a)

25. (b)

26. (d)

27. (d)

28. (a)

29. (c)

30. (c)

8. (b)

9. (b)

10. (a)

9. (c)

10. (a)

[B] ASSERTION & REASON 1. (a)

2. (c)

3. (a)

4. (d)

5. (c)

6. (a)

7. (a)

[C] COMPETENCY/CASE-BASED QUESTIONS 1. (a)

2. (b)

3. (b)

4. (d)

5. (c)

6. (c)

7. (c)

8. (c)

ANSWERS WITH EXPLANATION [A] MULTIPLE CHOICE QUESTIONS: 1. Option (d) is correct Explanation: The Hardy-Weinberg equilibrium describes the theoretical state of a population in which allele frequencies remain constant from generation to generation. In this equilibrium, no evolution is occurring. Factors such as genetic drift, natural selection, and genetic recombination can disrupt the equilibrium and lead to changes in allele frequencies. However, genetic equilibrium, by definition, represents a state where no such disruptions are occurring. 2. Option (b) is correct Explanation: Saltation refers to a single-step, large mutation

that can result in significant changes in an organism’s traits, potentially leading to the formation of a new species. It is a type of abrupt and substantial genetic change that contrasts with the gradual accumulation of small variations in the process of natural selection. 3. Option (c) is correct Explanation: Paleontological evidence for evolution primarily refers to fossils. Fossils are the preserved remains or traces of organisms from the distant past. They provide valuable insights into the history of life on Earth, showing the existence of different life forms in different geological periods. Fossils offer tangible proof of the changes and diversification of species over time, supporting the concept of evolution.

82 Oswaal CUET (UG) Chapterwise Question Bank 4. Option (a) is correct Explanation: Oparin and Haldane hypothesized that the emergence of life was preceded by chemical evolution and that the earliest forms of life may have evolved from pre-existing nonliving molecules (such as RNA, protein, etc.). 5. Option (b) is correct Explanation: S.L. Miller provided experimental evidence of the chemical evolution in 1953 in a laboratory set-up based on the hypothesis proposed by Oparin and Haldane. 6. Option (b) is correct Explanation: Analogous organs, are those organs which have similar function but are different in anatomical structures, e.g., wings of an insect and bird. Convergent evolution is the independent evolution of similar features in species of different periods. Thus, convergent evolution creates analogous structures. 7. Option (b) is correct Explanation: Homo habilis were the first human like being the hominid. They have fully erect posture. Dryopithecus were existing about 15 mya and they were more ape-like. They were hairy and walked like gorillas and chimpanzees. Australopithecus hunted with stone weapons, lived at trees, canines and incisors were small. Neanderthal man was cave dweller, used hides to protect their bodies, and buried their dead. 8. Option (d) is correct Explanation: Genetic drift is defined as sudden change in gene frequency by chance. It is the accidental gene flow causing change in frequency. Stabilizing selection: It is the selection that favours the intermediate phenotype. Hence, both extremes of the variation of a trait are eliminated. This selection results in favouring the average individuals in a population. Gene flow is the change in allele frequency due to immigration and emigration. Saltation is the single step large mutation. 9. Option (c) is correct. Explanation: The sequence proposed by Darwin and Wallace for organic evolution is overproduction, variations, the constancy of population size, natural selection. First, the species produces an enormous number of offsprings which contain variations. Due to overproduction, competition for survival will occur among the organisms of same or different species in a population. This results in a struggle for existence. This struggle for existence and heritable variations from parents to progeny results in the survival of the fittest or natural selection. The survival of the fittest results in a phenotype picked which is best suited for the environment. But with time even the environment changes, causing the species to change accordingly as well. After many years this changing environment along with the species changing to fit in it, results in the origin of a new species. 10. Option (c) is correct. Explanation: A mutation refers to the change in DNA of an organism, whereas evolution refers to change in the population over time. Mutation is one of the driving forces of evolution, which is responsible for the variation. Thus, evolution occurs as a result of mutations in Germplasm DNA. 11. Option (b) is correct Explanation: (iv) Darwin proposed the theory of evolution by Natural Selection. According to this theory, individuals that are better adapted to their environment survive and reproduce more successfully than less well-adapted individuals. (i) Oparin put forth abiogenesis theory. According to abiogenesis, life is originated from the non-living things spontaneously. (ii) Lamarck proposed theory of inheritance of acquired characters. Use and disuse of organs is one of the important principles of

BIOLOGY

Lamarckism. D (iii) Alfred Wegener proposed continental drift theory in 1912. According to this theory, 250 million years ago all the continents were combined into one super-continent (landmass) called Pangaea. Then Pangaea begins to break and has been slowly been moving towards the current position of today’s continent. 12. Option (a) is correct. Explanation: Variations during mutations of meiotic recombinations are random and directionless. Hugo de Vries proposed mutation theory on the basis of his work on evening primrose. He stated that mutations are sudden, heritable and persistent in successive generation. He contradicted Darwinian variations that are small and directional. 13. Option (c) is correct. Explanation: In ancient period, hands were used to collect food and to save themselves. Gradually men learnt to cook food; to make tools for their own purpose, this change in habit brings perfection in their hand. Similarly, there is an increase in the ability to communicate with others and develop community behaviour. Loss of tail takes a great role in course of evolution. But as in ancient period, men still eat hard nuts and hard roots (though they often take soft food also). Thus, change in diet is the most irrelevant change in the evolution of man. 14. Option (d) is correct. Explanation: Lamarck gave first theory of evolution known as Lamarckism in which he explained inheritance of acquired characters. According to which whatever an individual acquires characters in its lifetime due to internal vital force, effect of environment, new needs and use and disuse of organs they are inherited to the next generations. After several generations, the variations are accumulated up to such extent that they give rise to new species. This theory was proved wrong by August Weismann. He put forward the theory of continuity of germplasm. Weismann cut off the tails of rats for as many as 22 generations and allowed them to breed, but tailless rats were never born 15. Option (b) is correct. Explanation: Convergent evolution creates analogous structures that have similar form or function but were not present in the last common ancestor to those groups. 16. Option (d) is correct. Explanation: Divergent evolution refers to the process where two or more species that share a common ancestor evolve different traits or characteristics over time due to different environmental or selective pressures. In divergent evolution, these species become less similar in terms of specific traits. (A), (B), and (C) are examples of divergent evolution because they involve traits that have evolved differently in different species: (a) Forelimbs of man, bat, and cheetah: These species have forelimbs that have evolved differently to serve different purposes (e.g., human arms for manipulation, bat wings for flight, and cheetah forelimbs for running). (b)  Eye of octopus, bat, and man: These species have eyes that have evolved independently with different structures and functions due to their different environments and needs. (c) Brain of bat, man, and cheetah: These species have brains that have evolved differently to accommodate their specific behaviors and lifestyles. However, (d) Heart of bat, man, and cheetah is not an example of divergent evolution because the heart’s basic structure and function are quite conserved among vertebrates, including mammals like bats, humans,

EVOLUTION

and cheetahs. While there may be some variations in heart size and capacity to accommodate the different physiological demands of these species, the fundamental structure and function of the heart remain quite similar across mammals. 17. Option (c) is correct Explanation: Palaeontological evidences for evolution primarily refer to fossils. Fossils are the preserved remains or traces of organisms from the distant past. They provide valuable insights into the history of life on Earth and support the concept of evolution in several ways 18. Option (a) is correct Explanation: Excreta preserved as a fossil is known as “Coprolite.” Coprolites are fossilised feces or dung that have mineralized over geological time. They provide valuable information about the diet and digestive systems of ancient organisms, as well as the environments in which these organisms lived. 19. Option (b) is correct. Explanation: As marine waters are generally rich in salts like sodium chloride and it is generally accepted that life first evolved in water and presence of NaCl in body fluid  proves  that life is originated in primitive oceans. Top of Form 20. Option (a) is correct. Explanation: Development of similar adaptive functional structures in unrelated groups of organisms is called convergent evolution. 21. Option (c) is correct. Explanation: The theory of spontaneous generation (also called abiogenesis) was first suggested by Aristotle in his book “On the Generation of Animals” around 350 B.C. According to this theory, life can arise from non-living matter only. For example, organisms such as rats, flies and maggots arise within rotting meat and other decomposable items. This theory suggests that organisms do not descend from other organisms or from a parent, but only require certain environmental conditions to be fulfilled in order for their creation. This theory was disapproved by Francesco Redi, John Needham, Lazzaro Spallanzani and Louis Pasteur. 22. Option (d) is correct. Explanation: The Miller-Urey experiment created primitive earth condition in the laboratory and demonstrated the origin of first form of life from pre-existing non-living inorganic molecules. These primitive earth conditions include high temperature, volcanic storms and reducing environment containing methane (CH4), ammonia (NH3), hydrogen (H2) and water (H2O). They ultimately found that a large number of simple organic compounds including some amino acids such as alanine, glycine and aspartic acid can be synthesised during chemical origin of life. 23. Option (c) is correct. Explanation: Palaeontological evidences for evolution refer to the evidences from fossils. Fossils are the naturally preserved remains or traces of organisms that lived in the geologic past. These organisms are preserved in the form of skeletal, foot prints, trails, casts and moulds, resin (amber), etc. The study of fossils is called palaeontology. Development of embryo refers to embryological evidences for evolution. Whereas homologous and analogous organs provide evidences for comparative anatomy and morphology. 24. Option (a) is correct. Explanation: Fossils are the remains or impression of a primitive plant or animal which are embedded in rock and

83 preserved in petrified form. These are generally found in sedimentary rocks because of the following reasons: (i) These rocks are formed (on the Earth surface, under the water, etc.) from slits and muds at temperatures and pressures that do not destroy fossil remnants. (ii)  These rocks are not formed due to volcanic eruption. (iii) These rocks remain the same. 25. Option (b) is correct. Explanation: The bones of fore-limbs of whale, bat, cheetah and man are similar in structure, because they share a common ancestor. 26. Option (d) is correct. Explanation: Analogous organs arise due to convergent evolution. Analogous organs are those organs which have different origin but similar function (e.g., wings of butterfly and birds look alike). Anatomically these are not similar structures though they perform similar functions. Hence, analogous structures are a result of convergent evolution (i.e., different structures which are not closely related but evolve for the same function and hence having similarity). 27. Option (d) is correct. Explanation: Evolution of life shows that life forms had a trend of moving from water to land. The early vertebrates were fishes (which lived in only water). Some fishes gradually changed into amphibians which can live both on land and in water. Certain amphibians then transformed into the reptiles (live on land) and some of the latter finally evolved into birds (can fly) and then mammals. Thus, showing life forms moved from water to land. 28. Option (a) is correct. Explanation: Lobe fish is an example for link species. About 350 mya, fish with stout and strong fins could move on land and go back to water. These were called lobe and they evolved into the first amphibians that lived on both land and water (e.g., Coelacanth). Coelacanth is an ancient fish and refers to as a living fossil. Dodo is an extinct flightless bird. Sea weed is multicellular benthic marine algae and chimpanzees are the closest living relatives of humans. 29. Option (c) is correct. Explanation: Viviparity is considered to be more evolved because the developed young ones are protected inside the mother’s body and are looked after they are born leading to more chances of survival. These young ones are capable of independent existence (e.g., mammals). 30. Option (c) is correct. Explanation: The most accepted line of descent in human evolution is - Ramapithecus → Homo habilis → Homo erectus → Homo sapiens. [B] ASSERTION & REASON: 1. Option (a) is correct. Explanation: While it is true that fossils provide valuable evidence for the existence of organisms in the past, the mere existence of fossils doesn’t directly prove the theory of evolution. Fossils are evidence, but the theory of evolution involves more complex concepts, including the processes and mechanisms that lead to the changes observed in fossils. 2. Option (c) is correct. Explanation: Assertion (A) is true, but reason is false, because analogous organs are different in evolutionary origin but perform same function. 3. Option (a) is correct. Explanation: Both the assertion (A) and reason (R) are correct. Charles Darwin and Alfred Russel Wallace independently

84 Oswaal CUET (UG) Chapterwise Question Bank proposed the theory of evolution by natural selection. The reason (R) correctly defines evolution as the changes that occur in populations or species over time. 4. Option (d) is correct. Explanation: Individuals that are better adapted tend to survive and reproduce more offsprings while the less well adapted tends to produce fewer offsprings. 5. Option (c) is correct. Explanation: Archaeopteryx is a fossil linking reptilia and aves. while the less well adapted tends to produce fewer offsprings. 6. Option (a) is correct Explanation: Both the assertion (A) and reason (R) are correct. The evolution of Homo sapiens is characterized by a significant increase in cranial capacity, which is reflected in the larger volume of the human brain. This increase in brain size and complexity is considered a significant trend in human evolution. 7. Option (a) is correct Explanation: Both the assertion (a) and reason (R) are correct. Hugo de Vries was a botanist who introduced the concept of “saltation,” which involves the idea that evolution can occur through sudden and significant mutations, leading to the formation of new species. Saltation is indeed described as a single-step, large mutation that can bring about substantial changes and potentially lead to speciation. De Vries’ work contributed to our understanding of evolutionary mechanisms. 8. Option (b) is correct Explanation: Both Ramapithecus and Dryopithecus were early primates, which is true (Assertion A). However, the reason provided (Assertion R) is not the correct explanation of Assertion A. The assertion doesnot mention that Ramapithecus displayed more human-like characteristics; rather, it simply states that both species existed around 15 million years ago. 9. Option (b) is correct Explanation: The assertion (a) correctly states that Homo habilis had brain capacities ranging from 650-800cc. However, the reason (R) is not the correct explanation of Assertion A. While Homo habilis may have consumed meat as part of their diet, brain capacity is not directly related to their dietary habits, and this reason does not explain the brain capacity range mentioned. 10. Option (a) is correct Explanation: Both the assertion (a) and the reason (R) are true. Agriculture did indeed emerge around 10,000 years ago,

24

BIOLOGY

and it played a crucial role in the development of permanent human settlements. The ability to cultivate crops provided a stable source of food, which allowed early humans to transition from a nomadic lifestyle, where they followed food sources, to a settled one, where they could live in one place and develop communities. Therefore, reason (R) correctly explains why agriculture led to the development of permanent settlements.

[C] COMPETENCY/CASE-BASED QUESTIONS

1. Option (a) is correct Explanation: Charles Darwin’s theory of natural selection emphasizes the gradual evolution of life forms over time. 2. Option (b) is correct Explanation: In Darwin’s theory, “fitness” primarily refers to reproductive fitness, where individuals better adapted to their environment have more offspring. 3. Option (b) is correct Explanation: Alfred Wallace independently reached conclusions similar to Darwin’s theory of natural selection. 4. Option (d) is correct Explanation: The concept of common ancestry suggests that all life forms share ancestors from different periods in Earth’s history. 5. Option (c) is correct Explanation: The passage describes the relationship between Earth’s geological and biological histories as intertwined, with each influencing the other. 6. Option (c) is correct Explanation: The emergence of cellular life forms occurred approximately 2000 million years ago (~2000 mya). 7. Option (c) is correct Explanation: The transition to reptiles is associated with the time period of ~350 mya. 8. Option (c) is correct Explanation: The Coelacanth is believed to be the ancestor of modern amphibians, frogs, and salamanders. 9. Option (c) is correct Explanation: The mysterious disappearance of dinosaurs occurred approximately 65 million years ago (~65 mya). 10. Option (a) is correct Explanation: The emergence of agriculture is linked to the time period of ~10,000 years ago, leading to settled human societies.

UNIT III: Biology and Human Welfare

Study Time

CHAPTER

8

Max. Time: 1:5 Hours Max. Questions: 50

HUMAN HEALTH AND DISEASES

 Revision Notes:

 Health {  Health is the state of complete physical, mental, and social well-being. Health increases productivity and ensures longevity.  Ways to Ensure Good Health { Balanced diet { Personal hygiene { Exercise { Awareness about prevention and control of diseases { Proper waste disposal and control of vectors { Vaccination  Diseases { Causes of Diseases { Genetic reasons − Innate deficiencies and inheritable defects { Infections { Sedentary life style − Junk food, consumption of alcohols/drugs, lack of exercise   Pathogenic Diseases Pathogens; Parasites causing

Human Diseases { Pathogens

are the parasites that enter the human body through various means, then multiply, and interfere with normal vital activities. { Parasites are pathogens as they harm the host by living in or on them. { Pathogens have to adapt to live within the environment of the host.  Bacterial Diseases { Typhoid Pathogen − Salmonella typhi Spreads through − Contaminated food and water Site of infection − Small intestine Symptoms − High fever, stomach pain, headache, loss of appetite, constipation, and intestinal perforations in severe cases Confirmatory test − Widal test { Pneumonia Pathogens − Streptococcus pneumoniae and Haemophilus influenzae Spreads through − Droplets/aerosols released from infected person, sharing of glasses or utensils

Site

of infection − Alveoli (gets filled with fluid, difficulty in breathing) Symptoms − Fever, chills, cough, headache, lips and nails become grey in severe cases  Viral Diseases { Common cold Pathogen − Rhino viruses Site of infection − Nose and respiratory passage Spreads through − Droplets released from coughing or sneezing, or contaminated objects Symptoms − Nasal congestion and discharge, sore throat, cough, headache, tiredness  Protozoan Diseases { Malaria Pathogen − Plasmodium sps. (P.vivax, P. falciparum, P. malaria) Vector − Female Anopheles mosquito Symptoms − High grade fever, chills due to release of a toxic substance called haemozoin from the burst RBCs. { Amoebiasis Pathogen − Entamoeba histolytica Vector − Housefly Site of infection − Large intestine Symptoms − Constipation, abdominal pain, cramps, stools with mucous, and blood clots  Fungal Diseases { Ringworms Pathogens − Genera Microsporum, Trichophyton, and Epidermophyton Spreads through − Towels, clothes, comb (Fungus is acquired from soil) Symptoms − Appearance of dry, scaly lesions on various body parts with intense itching  Diseases Caused by Worms { Ascariasis Pathogen − Round worm, Ascaris Spreads through − Water, vegetables, fruits contaminated by faeces of infected person Symptoms − Internal bleeding, muscular pain, fever, anaemia, blockage of intestinal passage { Elephantiasis (filariasis) Pathogen − Wuchereria (W.malayi and W.bancrofti)

t

Diag

Treatmen

tion &

Detec nosis

Ca

AIDS

First Level

Second Level

Third Level

e

n gu

De

h and Disea a lt

Auto Immunity

e.g.,

e.g.,

ea dy

.

ies

e tiv Ac

ev en

s

tio

n/

l

s ign

Con tro

Causes

om

s nd sa

gs ru

T-Lymphocy t es

Types of barriers

Produces primary & secondary response

B-Lymph ocy te

Innate (Non-Specific)

Immunity

Drugs and Alcohol abuse Allergy

s

ant ibodies) Acquired (Specific)

tibod

ma de a n

Lymphoid organs

Primary

iku ngun ya

Diseases

Health Cancer

Trace the Mind Map

IV by H ed us



Typ es

Entamoeba



2 Types

bancrofti



S ec o n da r y

Ch

He

(O

wn

m Sy

eR D

si v Pas

pt

Aedes mosquito

e.g., Skin e.g., Saliva, stomach acid e.g., WBC e.g., Interferon

-

86 Oswaal CUET (UG) Chapterwise Question Bank

Pr

ses

BIOLOGY

87

HUMAN HEALTH AND DISEASES Spreads

through − Bite of female mosquito vector

{ Parasites

Symptom

− Chronic inflammation of the organs, usually the lymphatic vessels of lower limb

 Life Cycle of Plasmodium { Plasmodium

requires two hosts to complete its life

cycle. { When

female Anopheles mosquito bites a healthy human being, it releases Plasmodium, which lives in its body as sporozoite (infectious form).

{ The

parasites multiply (asexual reproduction) in the liver cells and finally burst the liver cells. Sporozoites are released in blood.

enter RBCs and further multiply (asexual reproduction) here and finally burst RBCs also.

{ Bursting

of RBCs is accompanied by release of a toxic substance called haemozoin (associated with fever and chills).

{ In the RBCs, only sporozoites change into gametocytes

(sexual stage). Gametocytes multiply. { When

the diseased person is bitten by a female Anopheles mosquito, gametocytes are introduced into the mosquito.

{ Gametocytes

fertilise and develop inside the intestine of mosquito to form sporozoites. Scan to know more about this topic

When the mosquito bites another human, sporozoites are injected with bite Mature infective stages (sporozoites) escape from intestine and migrate to the mosquito salivary glands

................... ................... ................... ................... ................... ................... ................... ................... ................... ................... ................... ................... ................... ................... ................... ................... ................... ................... ................... ...................

Life cycle of plasmodium

Parasites (sporozoites) reach the liver through blood

Sporozoites Salivary glands Fertilisation and development take place in the mosquito's intestine

Mosquito Host

Female mosquito takes up gametocytes with blood meal

Human Host

The parasite reproduces asexually in liver cells, bursting the cell and releasing into the blood

Female

Parasites reproduce asexually in red blood cells, bursting the red Male blood cells and causing cycles of fever and other Sexual stages (gametocyte) symptoms. Released parasites develop in red blood cells infect new red blood cells

Fig 1: Stage in the Life Cycle of Plasmodium

Physiological barriers − Acid in stomach, saliva in

Sporozoites { 

are stored in the salivary glands of mosquito and are released into the healthy person who is bitten by this mosquito.

mouth, tears from eyes Cellular

barriers − Blood has leukocytes such as polymorpho-nuclear leukocytes, monocytes, etc. and tissue has macrophages which phagocytose the microbes.

Immunity Basic concepts of immunology What is immunity? { The

ability of body to fight the disease-causing organisms is called immunity.

Cytokine

barriers − Special proteins called interferons are secreted by virus- infected cells that prevent the further spread of viral infection.

Types of immunity { Immunity

is of two types − innate immunity and acquired immunity.

{ Innate

immunity − It is present from the time of birth. It is non-specific. It consists of 4 kinds of barriers. Physical

barriers − Skin and mucus coating of respiratory, gastrointestinal, and urogenital tract prevent entry of microbes into body.

Acquired immunity { It

is acquired, which means that it is produced in response to an encounter with a pathogen based on memory. It is pathogen specific. When

a pathogen for the first time infects a person, low intensity immune response is generated (primary response).

88 Oswaal CUET (UG) Chapterwise Question Bank

 hen the same pathogen attacks again, W intensified immune response in generated, thereby preventing the occurrence of disease (secondary response).

Acquired

immunity involves two types of cells − B-lymphocytes and T- lymphocytes.

B-lymphocytes − Secrete proteins called antibodies

in response to pathogens Antibodies are specialized proteins with 4 peptide chains (2 light and 2 heavy), hence denoted as H2L2. IgA IgM, IgE, etc. are examples of some of the antibodies. They generate humoral immune response (found in blood). T-lymphocytes

− They help B-cells to produce antibodies. They generate cell - mediated immune response. This response helps the body to differentiate between ‘self’ and ‘non-self’ as occurs in case of graft rejection.

Difference between active immunity and passive ­ immunity { Acquired

immunity is of two types:

Active

Immunity - This is the naturally acquired immunity produced in the host body in response to an antigen. Examples: Immunization and body naturally getting immune to a microbe that had caused infection previously are examples of active immunity.

Passive

immunity- When ready-made antibodies are provided to an individual to protect against foreign agents. Examples: Colostrums present in mother’s milk contain IgA. Also, the foetus gets antibodies from mother through placenta.

{ Structure

BIOLOGY

Structure of Antibody {  An

antibody is made up of a variable region and a constant region, and the region that changes to various structures depending on differences in antigens is called the variable region, and the region that has a constant structure is called the constant region.

{  Each

heavy and light chain in an immunoglobulin molecule contains an amino-terminal variable

{ (V)region

that consists of 100 to 110 amino acids and differ from one antibody to another.

{ The

remainder of each chain in the molecule – the constant (C) region exhibits limited variation that defines the two light chain subtypes and the five heavy chains subclasses.

{ Some

heavy chains (α, δ, γ) also contain a proline-rich hinge region.

{ The

amino terminal portions, corresponding to the V regions, bind to antigen; effector functions are mediated by the carboxy-terminal domains.

{ The

ε and μ heavy chains, which lack a hinge region, contain an additional domain in the middle of the molecule. CHO denotes a carbohydrate group linked to the heavy chain.

How does vaccination help? { Vaccines

are nothing but inactivated pathogens.

{ These

inactivated pathogens when introduced in the body produce a primary immune response and antibodies are produced against the pathogen.

{ Memory

B and T-cells are produced.

{ Now

When the pathogen again attacks the person, memory B and T-cells generate a massive immune response and the pathogen is killed.

Problems of immune system

of Antibody

Antibodies are heavy globular plasma proteins. The

basic structure of all antibodies are same.

{ Allergies  Exaggerated

immune response to certain antigens present in environment

Fig 2: Structure of an antibody molecule

89

HUMAN HEALTH AND DISEASES Allergens − Substances in response to which allergy

is produced E.g., dust, pollen, etc. Antibodies

involved − IgE type allergic reactions, chemicals such as histamines and serotonins are released. Symptoms − Sneezing, watery eyes, difficulty in breathing, etc. Allergy test − Patient is injected with small doses of allergens to monitor his response.  Antihistamines, adrenalins, and steroids may be given so that the symptoms of allergy subside. { Autoimmunity In autoimmunity, body generates immune response against its own cells. Reasons − Genetic and other unknown reasons Example − Rheumatoid arthritis is an autoimmune disease. { The B lymphocyte (B cell): It is one of the most important cells of the body. They form part of the adaptive immune response by producing antibodies and presenting antigens to T cells. Once activated, they can mature into plasma cells or memory B cells. The type of B cells are: During

Types of B Cell { Plasma

Cell  Once activated, B cells can differentiate into plasma cells. Plasma cells are large lymphocytes with abundant endoplasmic reticulum, which allows them to produce large quantities of antibodies against specific antigens. They respond to signals from T cells during infection and continue to produce antibodies until the infection is controlled. Plasma cells are often found with chronic inflammation. { Memory B Cell Some B cells will differentiate into memory B cells when activated. These are long-lived cells which remain within the body and allow a more rapid response to future infections. If the host is re-exposed to the same antigen, these cells rapidly proliferate with assistance from T cells. This produces more B cells capable of secreting specific antibodies to the pathogen. This often means that the pathogen can be dealt with before the infection takes hold and becomes symptomatic. T cells (also called T lymphocytes): These are one of the major components of the adaptive immune system. Their roles include directly killing infected host cells, activating other immune cells, producing cytokines and regulating the immune response. The types of T-Cells are: { Cytotoxic T Cells (CD8 T Cells) Cytotoxic T cells kill their target cells, primarily by releasing cytotoxic granules into the cell to be killed. These cells recognise their specific antigen (such as fragments of viruses) when presented by MHC Class I molecules that are present on the surface of all nucleated cells.

MHC

Class I molecules interact with a protein called CD8 on the cytotoxic T cells, which helps to identify this cell type. Cytotoxic T cells require several signals from other cells to be activated, such as from dendritic cells and T helper cells. Their main function is to kill virally infected cells, but they also kill cells with intracellular bacteria or tumorous cells. { T-Helper Cells (Th) (CD4 T Cells) T helper cells have a wider range of effector functions than CD8 T cells and can differentiate into many different subtypes, such as Th1, Th2, Th17 and regulatory T cells. They become activated when they are presented with peptide antigens by MHC Class II molecules, which are expressed on the surface of APCs. MHC Class II molecules interact with a protein called CD4 on the T helper cells, which helps to identify this cell type. The roles of a CD4 T cell may include activating other immune cells, releasing cytokines, and helping B cells to produce antibodies. They help to shape, activate and regulate the adaptive immune response. { Memory T Cells Following an infection, antigen-specific, longlived memory T cells are formed. Memory T cells are important because they can quickly expand to large numbers of effector T cells upon reexposure to the antigen and have a low threshold for activation. They provide the immune system with memory against previously encountered antigens. Memory T cells may either be CD4+ or CD8+. { Major Histocompatibility Complex (MHC) Major histocompatibility complex (MHC), group of genes that code for proteins found on the surfaces of cells that help the immune system recognize foreign substances. MHC proteins are found in all higher vertebrates. In human beings the complex is also called the human leukocyte antigen (HLA) system. The principal function of the MHC is to present antigen to T cells to discriminate between self (our cells and tissues) and non-self (the invaders or modified self).

Antigen Presenting Cells Antigen-presenting cells (APCs) are a heterogeneous group of immune cells that mediate the cellular immune response by processing and presenting antigens for recognition by certain lymphocytes such as T cells. Classical APCs include dendritic cells, macrophages, Langerhans cells and B cells. { Human immune system consists of lymphoid organs, tissues, cells and soluble molecules like antibodies { Lymphoid organs are of two types − primary lymphoid organs and secondary lymphoid organs. Primary lymphoid organs – They consist of bone marrow and thymus. Here, immature lymphocytes are differentiated to form antigen-sensitive lymphocytes.

90 Oswaal CUET (UG) Chapterwise Question Bank { Bone

marrow − Here, all blood cells including B-lymphocytes are produced. { Thymus − It is responsible for maturation of T-lymphocytes. This lobed organ is situated near the heart and keeps on reducing in size as the age increases. Secondary lymphoid organs − Lymphocytes migrate here after attaining maturity. It includes spleen, lymph nodes tonsils, Peyer’s patches, and appendix. { Spleen − Large bean-shaped organ containing lymphocytes and phagocytes, which acts as a filter for blood { Lymph nodes − Located at different points throughout the immune system, they trap the antigens present in lymph or tissue fluid, and these antigens cause activation of lymphocytes and generation of immune response. MALT (Mucosal-associated lymphoid tissue) − Lines major tracts (respiratory, digestive, urogenital, etc); constitutes 50% of lymphoid tissue in body  Problems of immune system { Allergies  Exaggerated immune response to certain antigens present in environment Allergens − Substances in response to which allergy is produced e.g., dust, pollen, etc. Antibodies involved − IgE type During allergic reactions, chemicals such as histamines and serotonins are released. Symptoms − Sneezing, watery eyes, difficulty in breathing, etc. Allergy test − Patient is injected with small doses of allergens to monitor his response.  Antihistamines, adrenalins, and steroids may be given so that the symptoms of allergy subside. { Autoimmunity In autoimmunity, body generates immune response against its own cells. Reasons − Genetic and other unknown reasons Example − Rheumatoid arthritis is an autoimmune disease.

AIDS & Cancer  AIDS (Acquired Immuno Deficiency Syndrome) { Caused by HIV (Human Immunodeficiency Virus) [HIV is a retrovirus (RNA virus)] { Transmission of HIV occurs through: Sexual contact with infected person Sharing infected needles (as in case of intravenous drug abusers) Scan to know Transfusion of contaminated more about blood this topic Infected mother to child through placenta { Time lag between infection and appearance of symptoms − Few HIV Replication cycle months to many years (5-10 years)

BIOLOGY

{ How does AIDS infection spread? Mode of Infection  Virus

enters the body of a person and enters macrophages. Here, virus replicates (viral RNA reverse transcribes to viral DNA, which gets incorporated into hosts DNA and subsequently new viral particles are produced). { Macrophages become a virtual HIV factory. { Thereafter, HIV enters helper T-lymphocytes, replicates, and produces progenies. { As the progenies are released, they attack other T-lymphocytes. { Therefore, T-lymphocytes start decreasing in number and immune response of the person becomes weak. { Even infections which could be overcome easily start aggravating. { Diagnosis of AIDS − By ELISA (Enzyme Linked Immuno Sorbent Assay) { Treatment − No permanent cure; antiretroviral therapies can prolong the life of patient { Prevention of AIDS Ensuring use of disposable syringes Screening blood from blood banks Advocating safe sex NACO (National AIDS Control Organization) and many NGOs are doing a lot to create awareness among people.  Cancer { The process of development of cancer is called oncogenic transformation. { Normal cells have the property of contact inhibition (stoppage of growth on coming in contact with other cells), but cancer cells lose this property. { As a result, cancer cells divide continuously to give rise to mass of cells (tumours). { Tumours are of 2 types − benign and malignant. { Benign tumours − Remain confined to their original location and do not spread { Malignant tumours− These exhibit metastasis i.e., the cells sloughed from such tumours reach distant sites and wherever they reach, new tumour is formed. Malignant tumours actually represent cancer. The cells actively divide, grow, and starve the normal cells of vital nutrients. { Causes of cancer Carcinogens − Physical, chemical, and biological agents that cause cancer Example - ionizing radiations (X-rays and gamma rays), non-ionizing radiations (UV) Oncogenic (cancer-causing) viruses − They have viral oncogenes (cancer- causing genes). Sometimes normal genes in our body called protooncogenes get converted into cellular oncogenes that cause cancer. { Diagnosing cancer Biopsy and histopathological studies Biopsy − Suspected tissue is cut into thin sections and examined microscopically

91

HUMAN HEALTH AND DISEASES

Retro virus

Viral RNA core Virus infects Viral protein normal cell coat

Plasma membrane

Animal cell Viral DNA is produced by reverse transcriptase

Viral RNA is introduced into cell Cytoplasm Viral DNA is incorporated into host genome New viral RNA is produced by the infected cell New viruses are produced

Nucleus

DNA

New viruses can infect other cells

Fig 3: Replication of Retrovirus

Radiography,

CT scan (computed tomography), and MRI (Magnetic resonance imaging) are techniques of diagnosing cancers.

C

T Scan − 3-D imaging of internals of an organ is generated by X-rays.

MRI

Scan − Pathological and physiological changes in a living tissue are detected by using magnetic fields and non-ionising radiations.

Immunological and molecular biological diagnostic

techniques can all be used to detect cancers. Identifying certain genes, which make an individual

susceptible to cancers, can help to prevent cancers. { Treatment

of cancer

Radiotherapy

− Tumour cells are irradiated to death. Also, proper care is taken for protecting surrounding normal tissues.

Chemotherapy

− Drugs specific for particular tumours are used to kill cancer cells. They have side effects such as hair loss, anaemia, etc.

Immunotherapy− Biological response modifiers such

as α- interferons are used. They activate the immune system of patient and helps in destroying the tumour.

Commonly Abused Drugs Alcohol Abuse.  Types of Drugs { Opioids

(Heroin)

Source: Acetylation of morphine extracted from the

latex of poppy plants (Papaver somniferum)

Consumed

by: Snorting or injection

Properties: White,

bitter and odourless

Mode of action: Binds to opioid receptors present in

the CNS and GI tract

Effect: It is a depressant; slows down body functions { Cannabinoids Source:

Inflorescences of the plant Cannabis sativa

Consumed

by: Inhalation or oral ingestion

Mode

of action: Binds to cannabinoid receptors present in the brain

Effect: Affects

the cardiovascular system

{ Cocaine Source:

Coca plant Erythroxylum coca, found in South America

Consumed

by: Snorting

Mode

of action: Interference with transfer of neurotransmitter, dopamine

Effect:

Stimulates the CNS, producing a sense of euphoria and increased energy; excessive dosages cause hallucination

92 Oswaal CUET (UG) Chapterwise Question Bank Drugs Normally Used as Medicines { Drugs

like barbiturates, amphetamines, benzodiazepines, LSD (Lysergic acid diethyl amides) are used as medicines to help patients with mental illness and insomnia.

{ Morphine:

It is a pain killer which is used for patients who have undergone surgery, but it is also abused.

 Effects of Alcohol/ Drug Abuse { Immediate

effect − Vandalism, violence, and reckless

behaviour { Drop

in academic performance, lack of interest in personal hygiene, rebellious behaviour, and change in eating and sleeping patterns, weight and appetite fluctuations

{ Mental, psychological, and financial loss not only to the

Nicotine { Present in tobacco, which is smoked, chewed or snuffed { Mode

of action: Stimulates the adrenal gland to release adrenaline and nor-adrenaline

{ Effect:

BIOLOGY

Increases blood pressure and heart rate

Ill Effects of Smoking { Increased

risk of diseases like bronchitis, emphysema, coronary heart disease, gastric ulcer and cancer (throat, lung and urinary bladder)

{ Increased

carbon monoxide levels in blood, leading to oxygen deficiency

Alcohol / Drug Abuse  Causes of alcohol/ Drug Abuse {  Alcohol

/ drug abuse normally starts in adolescence (period between 12-18 yrs − transition phase between childhood and adulthood).

{ Many adolescents are motivated towards drugs/ alcohol

due to curiosity and experimentation. { Peer pressure, academic stress, unstable family structure

further incline youth towards alcohol/ drug abuse. {  Perception

of consuming alcohol / drug being cool and progressive and use of alcohol/drug in television, movies, etc. further promote this habit.

 Alcohol/ Drug Addiction

user, but also to his family { Those

who take drugs intravenously have a high risk of acquiring deadly diseases such as AIDS and hepatitis B.

{ Damage

to nervous system and liver (cirrhosis)

{ Use

of anabolic steroids by sports-person have adverse effects:

{ In

females − Increase of masculinity, aggressiveness, depression, abnormal menstrual cycle, facial hair growth, enlargement of clitoris, and deepening of voice

{ In

males − Acne, aggressiveness, depression, reduction in size of testicles, decreased sperm production, enlargement of prostate gland, breast enlargement, premature baldness

{ Ultimately,

prolonged use of alcohol/drugs leads to coma and death.

 Effects of Smoking { Increased

risk of diseases like bronchitis, emphysema, coronary heart disease, gastriculcer and cancer (throat, lung and urinary bladder)

{ Increased

carbon monoxide levels in blood, leading to oxygen deficiency

 Preventing Alcohol/ Drug Abuse { It

is better to prevent the inclination of an individual towards alcohol/ drugs right from adolescence. Some of the ways of prevention are:

{ When

Avoid

{ Addiction

Education

a person uses alcohol/ drug repeatedly, he becomes addicted. refers to psychological attachment to certain effects such as euphoria and temporary feeling of wellbeing associated with use of alcohol or drugs.

{ In

addiction, tolerance level of receptors present in our body increases towards the drug.

peer pressure − Understand the unique personality and capabilities of a child and counselling − A child must be taught to accept success and failure equally. Especially during adolescence, he must be inclined towards constructive activities such as music, yoga, sports, reading based on his interest.

{ This

Help

{  Subsequently,

Identifying danger signals − If any sign of symptom

drives the person to use them even when they are not required or when they tend to harm his health / family life. the user runs into a vicious circle of addiction and subsequent dependence.

{ Dependence

leads to manifestation of withdrawal syndrome on discontinuation of use.

{ Withdrawal

syndrome − Anxiety, nausea, sweating, shakiness, and sometimes may be lethal

from parents and peers − This includes proper guidance, advice, and trust to overcome problems such as stress and guilt. of alcohol / drug abuse is seen in the adolescent by family or friends, then it should not be ignored because prevention is better than cure.

{ Seeking medical help − Psychologists and rehabilitation

programs surely help an addict. Medical help should be sought to prevent further damage.

93

HUMAN HEALTH AND DISEASES

OBJECTIVE TYPE QUESTIONS [A] MULTIPLE CHOICE QUESTIONS: 1. Match List-I with List - II. CUET 2023] List-1 (A) Lymphoid Organ

(I) ELISA

(B) Cancer

(II) Widal Test

(C) HIV

(III) Bone marrow and Thymus

(D) Typhoid

(IV) Carcinogens

(a) (b) (c) (d) 2.

Choose the correct answer from the option given below; (A)-(IV), (B)-(II), (C)-(I), (D)-(III) (A)-(III), (B)-(IV), (C)-(I), (D)-(II) (A)-(II), (B)-(III), (C)-(IV), (D)-(I) (A)-(I), (B)-(III), (C)-(II), (D)-(IV) Match List-I with List - II. [CUET 2023] List-1 (Causal Agent)

(1) (2) (3) (4) 3.

List-II

List-II (Disease)

(A) Salmonella typhi

(I) Typhoid

(B) Streptococcus pneumoniae

(II) pneumonia

(C) Rhino Viruses

(III) Common cold

(D) Plasmodium

(IV) Malaria

Choose the correct answer from the option given below; (A)-(I), (B)-(II), (C)-(III), (D)-(IV) (A)-(II), (B)-(I), (C)-(III), (D)-(IV) (A)-(I), (B)-(II), (C)-(IV), (D)-(III) (A)-(III), (B)-(IV), (C)-(I), (D)-(II) _______ is a CNS stimulant as it interferes with the transport of the neuro-transmitter, dopamine. [CUET 2023] (a) Valium (b) Barbiturate (c) Cocaine (d) Opium 4. The statements are not true for allergy: [CUET 2023] (A) Allergy is due to release of chemicals like histamine and serotonin (B) Allergens are the substances which cause allergy (C) The antibodies produced due to allergens are IgA type (D) Adrenalin and steroids quickly enhance the symptoms of allergy. Choose the correct answer from the option given below; (a) (C) and (D) only (b)  (A) and (C) only (c) (A) and (B) only (d)  (B) and (D) only 5. A person complaining of stomach ache, weakness and sustained high fever was recommended Widal test by the doctor. Identify the pathogen responsible for the disease the person is suffering from: (a) Streptococcus pneumonia (b) Rhino Virus (c) Salmonella typhi (d) Plasmodium vivax

6. (a) (b) (c) (d) 7. (a) (b) (c) (d) 8. A. B. C. D. E. (a) (c) 9.

In the life cycle of Plasmodium, the gametocytes develop in WBC of human host RBC of human host Gut of mosquito Saliva of anopheles mosquito Long term use of alcohol specially causes— Liver cirrhosis Arthritis Pulmonary system damage Premature baldness Rearrange the events of life cycle of a retrovirus. New viral RNA produced by infected cell. Reverse transcription. New virus particles inject into other cells. Viral RNA introduced into the cell. Viral DNA incorporates into host genome. Choose the correct answer from the options given below: A, B, D, E, C (b)   B, E, D, A, C C, A, B, D, E (d)   D, B, E, A, C After an HIV infection, the body of the patient starts becoming immunodeficient mainly due to : (a) The damage caused to secondary lymphoid organs. (b) Rapid multiplication of the virus outside the host cells. (c) Progressive decline in the number of helper T-lymphocytes. (d) Infection by a number of pathogens. 10. A patient suffering from diabetes mellitus will have (a) ketonuria and glycosuria (b) uremia and renal calculi (c) hyperglycemia (d) uremia and ketonuria 11. Interferons are most effective in making non-infected cells resistant against the spread of which of the following diseases in human? (a) Ascariasis (b)   Ringworm (c) Amoebiasis (d)   AIDS 12. Identify the disease which is not a sexually transmitted disease? (a) Gonorrhoea (b)   Syphilis (c) Amoebiasis (d)   Chlamydiasis 13. Apis mellifera are killer bees possessing toxic bee venom. Identify the treatment and the type of immunity developed from the given table to treat a person against the venom of this bee. Remedy Immunity (a) Inactivated proteins Active (b) Proteins of the venom Passive (c) Pre-formed antibodies Active Passive (d) Dead microorganisms Passive 14. Synthetic drugs structurally related to adrenaline are: (a) Hallucinogens (b)   Analgesics (c) Amphetamines (d)   Barbiturates

94 Oswaal CUET (UG) Chapterwise Question Bank 15. Which one is correctly matched? (a) Cocaine – Opiate narcotic (b) Bhang – Analgesic (c) Reserpine – Tranquilizer (d) Morphine – Hallucinogen 16. Marijuana is extracted from (a) Dried leaves and flowers of hemp plant (b) Ergot fungus (c) Roots of hemp plant (d) Cocoa plant. K 17. When an apparently healthy person is diagnosed as unhealthy by a psychiatrist, the reason could be that: (a) The patient was not efficient at his work. (b) The patient was not economically prosperous. (c) The patient shows behavioural and social maladjustment. (d) He does not take interest in sports. 18. Diseases are broadly grouped into infectious and noninfectious diseases. In the list given below, identify the infectious diseases. (i) Cancer  (ii)  Influenza  (iii)  Allergy  (iv)  Small pox (A)  (i) and (ii) (b)   (ii) and (iii) (c)   (iii) and (iv) (d)   (ii) and (iv) U 19. One of the following is not the causal organism for ringworm. (a) Microsporum (b)   Trichophyton (c) Epidermophyton (d)   Macrosporum 20. Following toxic substance is responsible for malaria fever: (a) Haemocyanin (b)  Haemoglobin (c) Haemozoin (d)  Haemoriden 21. Which drug is used as medicine to help patients cope with depression and insomnia ? [NCERT Exemplar Q.10 pg.52] (a) Morphine (b)  Amphetamines (c) Codeine (d)   LSD 22. Tobacco consumption is known to stimulate secretion of adrenaline and nor-adrenaline. The component causing this could be [NCERT Exemplar Q.4 pg.51] (a) nicotine (b)  tannic acid (c) curamin (d) catechin 23. The chemical test that is used for diagnosis of typhoid is [NCERT Exemplar Q.3 pg.51] (a) ELISA-Test. (b)   ESR-Test. (c) PCR-Test. (d)   Widal-Test. 24. Many diseases can be diagnosed by observing the symptoms in the patient. Which group of symptoms are indicative of pneumonia? [NCERT Exemplar Q.7 pg.52] (a) Difficulty in respiration, fever, chills, cough, headache (b) Constipation, abdominal pain, cramps, blood clots (c) Nasal congestion and discharge, cough, sore throat, headache (d) High fever, weakness, stomach pain, loss of appetite and constipation 25. A person with sickle cell anaemia is (a) more prone to malaria. (b) more prone to typhoid.

BIOLOGY

(c) less prone to malaria. (d) less prone to typhoid. 26 Haemozoin is : [NCERT Exemplar Q.21 pg.54] (a) a precursor of haemoglobin. (b) a toxin released from Streptococcus infected cells (c) a toxin from Plasmodium infected species. cells (d) a toxin from Haemophilus species. infected cells 27. One of the following is not the causal organism for ringworm. [NCERT Exemplar Q.21 pg.54] (a) Microsporum (b)   Trichophyton (c) Epidermophyton (d)   Macrosporum 28. The sporozoites that cause infection when a female Anopheles mosquito bites a human being are formed in [NCERT Exemplar Q.5 pg.51] (a) liver of human. (b) RBCs of mosquito. (c) salivary glands of mosquito. (d) intestine of human. gut of mosquito 29. Which of the following are the reason(s) for Rheumatoid arthritis? Choose the correct option. [NCERT Exemplar Q.11 pg.52] (i) Lymphocytes become more active. (ii) Body attacks self-cells. (iii) More antibodies are produced in the body. (iv) The ability to differentiate pathogens or foreign molecules from self-cells is lost. (a)  (i) and (ii) (b)   (ii) and (iv) (c)   (iii) and (iv) (d)   (i) and (iii) 30. The substance produced by a cell in viral infection that can protect other cells from further infection is (a) serotonin. (b)   colostrum. (c) interferon. (d)  histamine. [B] ASSERTION AND REASON QUESTIONS: Directions (1-10): In the following questions, a statement of assertion is followed by a statement of reason. Mark the correct choice as: (a) If both Assertion and Reason are true and Reason is the correct explanation of Assertion. (b) If both Assertion and Reason are true but Reason is not the correct explanation of Assertion. (c) If Assertion is true but Reason is false. (d) If both Assertion and Reason are false. 1. Assertion (A): Acquired immunity is pathogen specific. Reason (R): It is not present since birth. 2. Assertion (A): Sharing of injection needles between two individuals is not recommended. Reason (R): This transmits STDs like AIDS and Hepatitis from the diseased person to a healthy person. 3. Assertion (A): Pathogens are disease causing organisms. Reason (R): Salmonella typhi is a pathogenic virus causing tuberculosis. 4. Assertion(A): The colostrum provides passive immunity to the newborn baby.

95

HUMAN HEALTH AND DISEASES



Reason(R): In this, the ready-made antibodies are directly given to protect the body. 5. Assertion (A): Bone marrow and thymus are primary lymphoid organs. Reason(R): These are the organs, to which matured lymphocytes migrate, interact with antigens and then proliferate to become effector cells. 6. Assertion (A): Opioids, also called opiates used to treat pain. Reason (R): Opioids are the drugs which bind to specific opioid receptors present in our central nervous system. 7. Assertion (A): Heroin, the smack is chemically diacetylmorphine. Reason (R): Heroin is obtained by acetylation of morphine. 8. Assertion (A): Cocaine is a drug of abuse. Reason (R): Cocaine interferes with the working of central nervous system. 9. Assertion (A): Smoking increases the carbon monoxide (CO) content in the blood which has greater affinity to haemoglobin than oxygen. Reason (R): CO forms a stable bond with haemoglobin and does not allow binding of oxygen. It reduces concentration of haem bounded oxygen and causes oxygen deficiency in the body. 10. Assertion (A): Ganja is hallucinogen. Reason (R): It alters perception, causes illusion and damages cardiovascular system. [C] COMPETECY/CASE-BASED QUESTIONS: I. Read the passage carefully and answer the question given after the passage from 1–5. Shivani studies in class I. Her parents come from a backward family and are unaware of immunization programme. So, she did not get vaccinated properly. Once playing with her friends in the park, she fell down due to weakness and later complained of high fever, headache and stiffness in her neck. 1. The microbe responsible for Shivani’s illness may be: (a) Vibrio cholerae (b)   Enterovirus (c) Plasmodium (d)   Mycobacterium 2. Which vaccine if administered earlier, would have saved Shivani from the illness she contracted unfortunately? (a) MMR Vaccine (B)  Varicella Vaccine (c) BCG Vaccine (D)  Salk Vaccine 3. The disease that Shivani has contracted spreads through: (a) Mosquito bite (b) Bite of infected dog (c) Faecal oral route (d) Direct contact of an infected person 4. Which serious ailment does the Poliovirus cause? (a) Mental retarded ness (b) Brain haemorrhage (c) Poliomyelitis (D)  Paralysis 5. If Shivani would have been vaccinated then what would be the vaccine that would have been given to Shivani . Which of the following is the most widely used vaccine for the prevention of the above disease? (a) DPT vaccine (b)   OPV vaccine (c) MMR vaccine (d)   BCG vaccine

II. Read the following passage and answers the questions given below 6–10. The natural history of malaria involves cyclical infection of humans and female Anopheles mosquitoes. In humans, the parasites grow and multiply first in the liver cells and then in the red cells of the blood. In the blood, successive broods of parasites grow inside the red cells and destroy them, releasing daughter parasites (“merozoites”) that continue the cycle by invading other red cells.



Study a part of the life cycle of a malarial parasite given below and answer the questions as follow: 6. Which of these is correctly matched? (a) A- Salivary gland of female mosquito releases sporozoites of Plasmodium. (b) B- Gametocytes of Plasmodium enter the mosquito when it bites a malarial patient. (c) C- Fertilisation occurs in the intestine of mosquito. (d) All of these 7. Malaria parasite completes its life-cycle in : (a) One host (B)  Two host (c) Three host (D)  Reservoir host 8. The infectious stage of Plasmodium that enters the human body is : (a) Female gametocytes (b) Male gametocytes (c) Sporozoites (d) Trophozoites 9. Fever in malaria is due to : (a) Entry of sporozoites into blood capillaries. (b) Entry of merozoites into liver cells. (c) Release of haemozoin from red blood cells. (d) Entry of cryptomerozoites into red blood cells. 10. Malaria spreads by : (a) Male Culex (b) Female Culex (c) Male Anopheles (d) Female Anopheles 10. Which of the following is the most common type of symptom that occur in people infected with malaria? (a) Muscle aches and swollen legs and hands (b) Skin rashes and mouth rashes (c) Mild fever and itchy skin (d) High-grade fever and chills

96 Oswaal CUET (UG) Chapterwise Question Bank

BIOLOGY

ANSWER KEY [A] MULTIPLE CHOICE QUESTIONS 1. (b)

2. (a)

3. (c)

4. (c)

5. (c)

6. (b)

7. (a)

8. (d)

9. (c)

10. (a)

11. (d)

12. (c)

13. (c)

14. (c)

15. (c)

16. (a)

17. (c)

18. (d)

19. (d)

20. (c)

21. (b)

22 (d)

23. (a)

24. (a)

25. (c)

26. (c)

27. (d)

28. (d)

29. (b)

30. (c)

8. (c)

9. (a)

10. (a)

9. (c)

10. (d)

[B] ASSERTION & REASON 1. (a)

2. (a)

3. (c)

4. (a)

5. (c)

6. (a)

7. (a)

[C] COMPETENCY/CASE-BASED QUESTIONS 1. (b)

2. (d)

3. (c)

4. (d)

5. (b)

6. (c)

7. (b)

8. (c)

ANSWERS WITH EXPLANATION [A] MULTIPLE CHOICE QUESTIONS 1. Option (b) is correct. Explanation: Bone marrow and thymus are considered as lymphoid organs whereas ELISA test is used to detect HIV, cancer is caused by agents known as carcinogens and typhoid is detected by Widal test. 2. Option (a) is correct Explanation: Salmonella typhi is causative pathogen of typhoid, streptococcus pneumonia Streptococcus pneumoniae is causative pathogen of pneumonia, Rhino virus is causative pathogen of common cold and Plasmodium is causative pathogen of malaria. 3. Option (c) is correct Explanation: Cocaine attaches to the dopamine transporter and blocks the normal recycling process, resulting in a build-up of dopamine in the synapse, which contributes to the pleasurable effects of cocaine. 4. Option (c) is correct Explanation: The antibodies produced due to allergens are IgE type. Adrenaline and steroids decreases allergy symptoms as they reduce inflammation. 5. Option (c) is correct Explanation: The Widal test, developed in 1896 was named after its inventor, Georges-Fernand Widal. It is an agglutination test which detects the presence of serum agglutinins (H and O) in patient’s serum with typhoid and paratyphoid fever. It is characterized by sustained high fever (39° to 40°C), weakness, stomach pain, constipation, headache and loss of appetite. Salmonella typhi is a pathogenic bacterium which causes typhoid fever. 6. Option (b) is correct Explanation: Male and female gametocytes are the components of the malaria parasite life cycle which are taken up from an infected host bloodstream i.e., RBC of human host by mosquitoes and thus mediate disease transmission. 7. Option (a) is correct Explanation: Long term use of alcohol causes liver cirrhosis, as chronic use of alcohol damages nervous system and liver. Arthritis is associated with the inflammation of joints. Pulmonary system damage is associated with smoking tobacco or other forms of air pollutants. Premature baldness can occur due to Deficiency of biotin or growth hormone can cause premature boldness. 8. Option (d) is correct Explanation: The retroviral life cycle begins in the nucleus of an infected cell. A retrovirus inserts its genome into the host’s genome. D - Viral RNA is introduced into the cell. B - Enzyme reverse transcriptase acts on viral RNA. E - Viral

DNA produced is incorporated into the host genome. A - New viral RNA is produced by infected cell. C - New virus particles inject into other cells. 9. Option (c) is correct Explanation: AIDS is caused by HIV transmitted via sexual or blood-blood contact. After entering the human body, the HIV virus attacks and enters the macrophages. Inside the macrophages, the RNA of the virus replicates with the help of enzyme reverse transcriptase and give rise to viral DNA. Then, this viral DNA incorporates into the host DNA and directs the synthesis of virus particles. At the same time. HIV enters the helper T-lymphocytes. It replicates and produces viral progeny here. These newly formed progeny viruses get released into the blood, attacking healthier helper T-lymphocytes in the body. As a result, the number of T-lymphocytes in the body of an infected person decreases progressively, thereby decreasing the immunity of the body. T-helper cells stimulate B-cells to produce antibodies and induce killer-T-cells to destroy the non-self cells (foreign cells). As a result of the decline in the number of helper T-cells, the individual begins to experience opportunistic illnesses brought on by a decline in immunity. 10. Option (a) is correct Explanation: Diabetes mellitus is a metabolic disorder in which the body does not produce enough insulin or does not respond normally to insulin, causing blood sugar (glucose) levels to be abnormally high. A patient suffering from diabetes mellitus will have ketonuria and glycosuria. Glycosuria is the condition in which glucose is present in urine while ketonuria is the condition in which ketone bodies are present in urine. 11. Option (d) is correct. Explanation: Interferons are group of soluble glycoproteins which are produced and released from cells in response to virus infection (and other stimuli). Their production is triggered by the immune system in response to pathogens. Once triggered, they induce number of molecular changes that affect cellular response including cell growth and inflammation. Endogenous interferons play an important role in viral infection of the nervous system. They are also used in the treatment of other viral, autoimmune disorders. 12. Option (c) is correct. Explanation: Amoebiasis (amoebic dysentery) is caused by a protozoa Entamoeba histolytica in the large intestine of human. Houseflies act as mechanical carrier and serve to transmit the parasite from faeces of infected person to food and food products, thereby contaminating them. Hence, drinking water and food contaminated by the faecal matter are the main source of infection. 13. Option (c) is correct. Explanation: Apis mellifera or western honey bee or European honey bee is the most common of the 7–12

HUMAN HEALTH AND DISEASES

species of honey bees worldwide. Mellifera is the Latin word, which means “honey bearing” or “honey carrying”. It is used both for honey production and pollination activities by beekeepers. Honey bee venom is a bitter, colourless liquid and its active protein contains a mixture of proteins that causes local inflammation and act as anti–coagulants. Preformed antibodies are derived from the blood serum of previously infected people or animal. They are used against fast–acting toxins or microbes as in snake bites in order to provide immediate, passive immunisation. 14. Option (c) is correct. Explanation: Amphetamines (stimulants) mimic the effect of the naturally produced hormone adrenaline. 15. Option (c) is correct. Explanation: Cocaine is a natural stimulant. It stimulates nervous system and delays fatigue. Bhang is a hallucinogen. Morphine is a natural opiate derived from opium. Reserpine is a tranquilizer. It has sleep inducing properties 16. Option (a) is correct. Explanation: Marijuana is extracted from leaves and flowers of Cannabis sativa. 17. Option (c) is correct. Explanation: Health is a state of complete physical, mental, and social well-being. So, when an apparently healthy person is diagnosed as unhealthy by a psychiatrist, the reason could be that the patient shows behavioural and social maladjustment due to mental discomfort. 18. Option (d) is correct. Explanation: In the given list, influenza (ii) and small pox (iv) are infectious diseases. Infectious diseases can be spread from one person to another, through contact with body fluids, by aerosols (through coughing and sneezing), or via a vector. Whereas cancer and allergy are non-communicable diseases and cannot spread from infected person to a healthy person 19. Option (d) is correct. Explanation: Ringworm infections are caused by fungi belonging to the genera Microsporum, Trichophyton, and Epidermophyton. Macrosporum is an ecto-mycorrhizal zoosporic fungus causing diseases of economically important vascular plants. 20. Option (c) is correct Explanation : The rupture of RBCs is associated with the release of toxic substance haemozoin which is responsible for the chills and high fever recurring every 3-4 days. 21. Option (b) is correct Explanation: Amphetamines stimulates the nervous system, increases alertness and pro-duces excitement. 22. Option (d) is correct Explanation : In the given list, influenza (ii) and small pox (iv) are infectious diseases. Infectious diseases can be spread from one person to another, through contact with body fluids, by aerosols (through coughing and sneezing), or via a vector. Whereas cancer and allergy are non-communicable diseases and cannot spread from infected persons to healthy person. 23. Option (a) is correct Explanation: Nicotine is highly addictive and harmful for human health. Nicotiana tabacum, the type of nicotine is found in tobacco plants. Nicotine present in tobacco stimulates adrenal gland to release adrenaline and nor-adrenaline hormone which in turn increases the blood pressure and heart rate. 24. Option (a) is correct Explanation: Pneumonia is an infection of lungs which is caused by bacteria, virus, fungi, or parasites. It is a type of chest infection which affects the tiny air sacs (called alveoli) of lungs. In this condition these air sacs get inflamed and fill with fluid which makes it harder to breathe. It is characterised by

97 difficulty in respiration, fever, chills, cough, headache. Constipation, abdominal pain, cramps, blood clots are the symptoms of amoebiasis, caused by Entamoeba histolytica. Common cold or influenza are characterised by nasal congestion and discharge, cough, sore throat, headache. High fever, weakness, stomach pain, loss of appetite, and constipation are the symptoms of typhoid. 25. Option (c) is correct Explanation : A person with sickle cell anaemia is less prone to malaria because in sickle cell anaemia red blood cells becomes abnormally shaped, if they inherit two faulty copies of the gene for the oxygen-carrying protein haemoglobin. The faulty gene persists because even carrying one copy of it confers some resistance to malaria. Therefore, people with normal haemoglobin A are susceptible to death from malaria but people with sickle cell trait who have one gene for haemoglobin A and other for haemoglobin S have a greater chance of surviving malaria and do not suffer adverse consequences from the haemoglobin S gene. 26. Option (c) is correct Explanation : Haemozoin is a toxin released by Plasmodium species, which is responsible for the chill and high fever recurring every three to four days. To continue their lifecycle, Plasmodium enters the human body as sporozoites and multiply within the liver cells, resulting in the rupture of the RBCs. The ruptured RBCs is associated with release of a toxic substance, haemozoin. Streptococcus produces streptomycin and streptococcal pyrogenic exotoxin which shows haemolytic condition. Haemophilus produces cytolethal distending toxin (HdCDT) which inhibit mammals cell proliferation. 27. Option (d) is correct. Explanation : Ringworm infections are caused by fungi belonging to the genera Microsporum, Trichophyton, and Epidermophyton. Macrosporum is an ectomycorrhizal zoosporic fungus causing diseases of economically important vascular plants. 28. Option (d) is correct Explanation :The sporozoites that cause infection, when a female Anopheles mosquito bites a person being are formed in intestine of mosquito. 29. Option (b) is correct Explanation : Rheumatoid arthritis is an autoimmune disorder. It occurs when immune system mistakenly attacks own body’s tissues like, the synovium (the lining of the membranes that surround your joints). The resulting inflammation thickens the synovium, which can eventually destroy the cartilage and bone within the joint. The tendons and ligaments that hold the joint together weaken and stretch. Gradually, the joint loses its shape and alignment. Autoimmunity is an abnormal immune response in which the immune system of the body starts rejecting its own body cells or ‘self-cells and molecules. Sometimes, body loose its ability to differentiate between pathogen or foreign molecules from self-cell and attacks self-cells. This results in damage to the body. 30. Option (c) is correct Explanation : Interferon, belongs to cytokinin, is a protein which enhances the role of immune system. It is produced by virus-infected cells, that inhibit reproduction of the invading virus and induce resistance to further infection. Serotonin is a neurotransmitter that leads to depression. Colostrum is present in the lactating mother’s breast milk which contains antibodies to protect the new born against disease. Histamine is also a neurotransmitter involved in inflammatory response. [B]. ASSERTION & REASON 1. Option (a) is correct. Explanation: Acquired immunity is pathogen specific immunity. It is learned and is not present at birth.

98 Oswaal CUET (UG) Chapterwise Question Bank 2. Option (a) is correct. Explanation: Sharing of injection needles is not recommended so as to avoid the transmission of STDs like AIDS and Hepatitis, from the blood fluid of the diseased person to a healthy person. 3. Option (c) is correct. Explanation: Salmonella typhi is a pathogenic bacterium that causes typhoid fever in human beings. 4. Option (a) is correct. Explanation: Colostrum (Mother’s first milk) contains good amount of ready-made antibodies like IgA, which provide passive immunity to the new born and protects it from various infections. 5. Option (c) is correct. Explanation: Bone marrow and thymus are primary lymphoid organs as here immature lymphocytes differentiate into antigen-sensitive lymphocytes. 6. Option (a) is correct. Explanation: When opioids bind to specific receptors present in our CNS, they block the pain messages sent from the body through the spinal cord to the brain. 7. Option (a) is correct. Explanation: Heroin is obtained by acetylation of morphine, which is extracted from the latex of the poppy plant. 8. Option (c) is correct. Explanation: Cocaine interferes with the transport of the neurotransmitter dopamine, which acts as chemical messenger in our body. 9. Option (a) is correct. Explanation: Smoking increases the carbon monoxide (CO) content in the blood which has greater affinity to haemoglobin than oxygen. CO forms a stable bond with haemoglobin and does not allow binding of oxygen. It reduces concentration of haem bound oxygen and causes oxygen deficiency in the body. Smoking also damages alveolar walls, which reduces respiratory surface (emphysema). 10. Option (a) is correct. Explanation: Ganja is obtained from Cannabis sativa / hemp plant. It is hallucinogenic, alters perception, causes illusion and damages cardiovascular system. [C] COMPETENCY/CASE-BASED QUESTIONS: 1. Option (b) is correct.

24

BIOLOGY

Explanation: The symptoms shown by Shivani suspects of having polio which is caused by a species of Enterovirus. 2. Option (d) is correct. Explanation: Salk Vaccine or inactivated polio virus vaccine if administered earlier would have saved Shivani from contracting the disease. 1 3. Option (c) is correct. Explanation: Polio virus is contagious it spreads through – stool of an infected person, droplets from sneeze, cough of infected person. 1 4. Option (d) is correct. Explanation: The Poliovirus can infect a person’s spinal cord, causing paralysis. 5. Option (b) is correct. Explanation: OPV is the oral polio vaccine. It is the most widely used vaccine for the prevention of poliomyelitis. It induces intestinal immunity due to the production of secretory IgA. This is important for the inhibition of virus replication in the gut. OPV is administered to children under the age group of 5 years to immunize them from polio viruses. 6. Option (c) is correct. Explanation: A—Gametocytes of Plasmodium enter the mosquito when it bites a malarial patient and feed on blood. C—Fertilisation. It occurs in the intestine of mosquito. B—Salivary gland of the female Anopheles mosquito releases sporozoites of Plasmodium. 7. Option (b) is correct. Explanation: The malaria parasite requires two hosts – human and female Anopheles mosquito, to complete its life cycle. 8. Option (c) is correct. Explanation: Life cycle of plasmodium starts with inoculation of sporozoites (infective stage) through the bite of infected female Anopheles mosquitoes. The parasite initially multiplied within the liver cells and then attack the red blood cells (RBCs) resulting in their rupture. 9. Option (c) is correct. Explanation: A toxic substance called haemozoin from the ruptured RBCs is responsible for the chill and high fever. 10. Option (d) is correct. Explanation: Malaria is a vector borne disease that spreads by biting of the female Anopheles mosquito.

Study Time

CHAPTER

9

Max. Time: 1:5 Hours Max. Questions: 50

IMPROVEMENT IN FOOD PRODUCTION

 Revision Notes:

 Plant Breeding { Plant Breeding is the purposeful manipulation of plant species in order to create desired plant species in order to create desired plant types that are better suited for cultivation, give better yields and are disease resistant. {  Classical plant breeding involves crossing or hybridisation of pure lines, followed by artificial selection to produce plants with desirable traits of higher yield, nutrition and resistance to disease. {  In India, agriculture accounts for approximately 33 percent of India’s GDP and employs 62 percent of the population.  The main steps in plant breeding are{  Collection of variability is the collection and preservation of all the different wild varieties, species and relatives of the cultivated species. The entire collection having all the diverse alleles for all genes in a given crop is called germplasm collection. {  Evaluation and selection of parents is the identification of plants with desirable combination of characters. The selected plants are multiplied and used in the process of hybridisation. { Cross hybridisation among the selected parents to obtained desired crop characters for example high protein quality of one parent may need to be combined with disease resistance from another parent. This is possible by cross hybridising the two parents to produce hybrids that genetically combine the desired characters in one plant {  Selection and testing of superior recombinants -The selection process is crucial to the success of the breeding objective and requires careful scientific evaluation of the progeny. This step yields plants that are superior to both of the parents {  Testing, releasing and commercialisation of new cultivars -The newly selected lines are evaluated for their yield and other agronomic traits of quality, disease resistance, etc.  Some Indian Hybrid Crops { Wheat and Rice: Production of wheat and rice increased tremendously between 1960-2000 due to introduction of semidwarf varieties of rice and wheat. This phase is often referred to as the Green Revolution.

 In

1963, several varieties such as Sonalika and Kalyan Sona, which were high yielding and disease resistant were introduced all over the wheat growing field of India. Semi-dwarf rice varieties were derived from IR-8, and Taichung Native-1 were introduced in 1966. Later better-yielding semi-dwarf varieties Jaya and Ratna were developed in India. { Sugar cane: Saccharum barberi and Saccharum officinarum were crossed to get the desirable qualities of high yield, thick stems, high sugar and ability to grow in the sugar cane areas of north India. { Millets:  Hybrid maize, jowar and bajra are developed in India. These varieties are high yielding and resistant to water stress.  Plant Breeding for Disease Resistance { Several fungal, bacterial and viral pathogens affect the yield and quality of plant products. To minimise this loss disease resistant varieties were developed. Breeding is carried out by conventional method or by mutation breeding. { Steps for breeding disease resistant plants: Selection of genome with disease resistant traits Mating of the selected parents Selection of superior hybrids Testing of the hybrid for superior variety Release of the new variety { Some crop varieties bred by hybridisation and selection for disease resistance to fungi, bacterial and viral disease are releasedCrop

Variety

Resistance to diseases

Wheat

Himgiri

Leaf and stripe rust, hill bunt

Brassica

Pusa swarnim (Karan rai)

White rust

Cauliflower

Pusa Shubhra Pusa Snowball K-1

Black rot and Curl blight black rot

Cowpea

Pusa Komal

Bacterial blight

Chilli

Pusa Sadabahar

Chilly mosaic virus, Tobacco mosaic virus and Leaf curl

100 Oswaal CUET (UG) Chapterwise Question Bank

ment in e v o pr roduction fo P

od

BIOLOGY

Im

101

IMPROVEMENT IN FOOD PRODUCTION { Mutation is the process by which genetic variations are

created through changes in the base sequence within genes resulting in the creation of a new character or trait not found in the parental types. It is done by using mutants like chemicals or radiations. This process is called mutation breeding. Example: Mung bean resistance to yellow mosaic virus and powdery mildew were induced by mutation.  Resistance to yellow mosaic virus in bhindi (Abelmoschus esculentus) was transferred from a wild species and resulted in a new variety of A. esculentus called Parbhani kranti.

  Plant breeding for Developing Resistance to

Insect Pests

plant and crop products are destructed by insects and pests on large scale. To prevent this loss new varieties resistance to them are developed. { Steps for breeding disease resistant plants: Selection of genome with disease resistant traits Mating of the selected parents Selection of superior hybrids Testing of the hybrid for superior variety Release of the new variety { Some released crop varieties bred by hybridisation and selection for insect pest resistance are -

Nucleus Plasmalemma Vacuole Cellwall

Plant cell Parent A protoplast

Cellulase + Pectinase Protoplast A Parent B protoplast PEG

Cell fusion

{ Crop

Crop

Variety

Insect Pests

Brassica (rapeseed mustard)

Pusa Gaurav

Aphids

Flat bean

Pusa Sem 2, Pusa Sem 3,

Jassids, aphids and fruit borer

Okra (Bhindi)

Pusa Sawant Pusa A-4

Shoot and Fruit borer

 Tissue Culture: { The capacity to generate whole Scan to know plants form any cell/explant is more about this topic called totipotency. {  Thousands of plants can be produced from expalnts in short interval of time using suitable nutrient medium, Plant Tissue aseptic condition and use of Culture phytohormones. This method of producing thousands of plant is called micropropagation. {  Each of these plants will be genetically identical to the original plant from which they were grown, i.e., they are somaclones. Many important food plants like tomato, banana, apple.  Meristem Culture:  {  The recovery of healthy plants from diseased plants can be done by mersitem culture .Although the plant is infected with a virus, the meristem (apical and axillary) is free of virus. Hence, one can remove the meristem and grow it in vitro to obtain virus-free plants.  Somatic Hybridisation: {  Isolation of single cells from their plants and after digesting their cell wall fusing the cytoplasms of two

Nuclear fusion

Somatic hybrid cell B

different varieties is called somatic hybridisation and these hybrids are called somatic hybrids  Single Cell Protien (SCP) { Single-cell protein refers to the crude, a refined or edible protein extracted from pure microbial cultures, dead, or dried cell biomass. {  They can be used as a protein supplement for both humans or animals. {  Microorganisms like algae, fungi, yeast, and bacteria have very high protein content in their biomass. These microbes can be grown using inexpensive substrates like agricultural waste viz. wood shavings, sawdust, corn cobs etc. and even human and animal waste. { Microbes like Spirullina can be grown easily on materials like waste water from potato processing plants having starch, molasses, animal manure and even sewage to produce large quantities and can serve as food rich in protein minerals, fats, carbohydrates and vitamins.  Biofortification: { Biofortification is the process of breeding crops with higher levels of vitamins and minerals, or higher protein and healthier fats. {  Breeding for improved nutritional qualities have following objectives of improving Protein content and quality. Oil content and quality Vitamin content Micronutrient and mineral content { Examples of Biofortified crops: In 2000, maize hybrids with twice the amount of the amino acids, lysine and tryptophan than the existing hybrids were developed for improving cultivated Wheat variety, Atlas 66, having a high protein content, has been used as a donor for improving cultivated wheat. { The Indian Agriculture Research Institute (IARI), New Delhi have released many varieties of vegetables crops rich in vitamins and minerals like-

102 Oswaal CUET (UG) Chapterwise Question Bank { vitamin A enriched

carrot, spinach and pumpkin C enriched bitter guard, bathua, mustard { iron and calcium enriched spinach and bathua; { protein enriched beans – broad, lablab, French and garden peas. { vitamin

Animal husbandry { Animal husbandry is the science

Scan to know

of caring for and breeding more about this topic domestic animals and the development of genetic traits such as high yielding, disease resistance, etc. that are valuable to humanity. Animal Husbandry {  Animal husbandry involves breeding and raising livestock like buffaloes, cows, goats, camels, horses, sheep etc. and is even extended to poultry farming and fisheries.  Some of the salient practices of animal husbandry

are as follows:

 Management of Farm and Farm Animals { Dairy Farm Management  Dairying is the management of animals for its milk and its product for human consumption. In dairy farm management, we deal with processes and systems that increase yield and improve quality of milk.  Selection of good breeds having high yielding potential, combined with resistance to diseases is very important.  The feeding of cattle should be carried out in a

The

progeny is evaluated. the superior female and superior male are identified for further mating. Inbreeding increases homozygosity so it is only done if a pureline of animals are to be evolved. Inbreeding depression – continued inbreeding reduces fertility and even productivity.  This is called Inbreeding Depression. Whenever this becomes a problem, selected animals of the breeding population should be mated with unrelated superior animals of the same breed.  This usually helps restore fertility and yield. { Out-breeding- is the breeding of unrelated animals, which may be between individuals of same breed but, having no common ancestors or between different breeds (cross breeding) or different species (interspecific hybridisation).  Again,

BIOLOGY

scientific manner (quality and quantity of fodder). have to be housed well, should have proper water and be maintained disease free. Strict cleanliness and hygiene are importance while milking, storage and transport of the milk and its products. {  Poultry Farm Management- Poultry includes domesticated birds used for food or for their eggs. It mainly includes chicken and ducks and with turkey and geese. Important components of poultry farm management includes Selection of disease free and suitable breeds. Proper and safe farm condition. Hygiene and health care. Proper feed and water.  Animal Breeding – It aims at increasing yields of animals and improving the desirable qualities of the product. A breed is a group of animals related by descent and similar in most of characters like general appearance, features, size, configuration etc. There are two kinds of breeding Inbreeding and Outbreeding { Inbreeding – breeding between animals of same breeds. { Outbreeding- crosses between different breeds. { Inbreeding – It is the breeding between animals within the same breed for 4 to 6 generations. The strategy is as follows – Superior females (e.g.; a cow that produces more milk per lactation) and superior males (e.g; a healthy bull) which of the same breed are identified and mated. Cattle

{ Cross

breeding- superior male of one breed are mated with superior female of another females of another breed. Cross breeding allows the desirable qualities of two breeds to be combined. Example; Hisardale is a breed of sheep developed in Punjab by crossing Marino rams and Bikaneri ewes { Out-crossing- this is the practice of mating of animals within the same breed but having no common ancestors on either side of their pedigree up to 4-6 generation. The offspring are called out-cross. Out-crossing often helps to overcome inbreeding depression. { Interspecific hybridisation- male and female animals of two different species are mated. The progeny may combine desirable features of both and parents. Exmule. {  Controlled breeding experiments -They are carried out using artificial insemination. The semen is collected

103

IMPROVEMENT IN FOOD PRODUCTION

from the male that is chosen as a parent and injected into the reproductive tract of the selected female by the breeder. {  The semen can be used immediately or can be frozen and used later. { The frozen semen can also be transported as per the requirement. { The method helps to overcome several problems of normal matings. {  Multiple Ovulation Embryo Transfer Technology (MOET): This technology is used to increase the success rate of artificial insemination. In this method: a cow is administrated hormones (FSH) to induce follicular maturation and super ovulation, instead of one egg; they produce 6-8 eggs. The fertilised eggs 8-32 cells stages, are recovered non-surgically and transferred to surrogate mothers. The genetic mother is available for another round of super ovulation.  Example- High milk-yielding breeds of females cattle and high quality meat- yielding males cattle have been bred by this method to increase heard size in a short time. Bee-keeping:  Apiculture { Bee-keeping or apiculture is the maintenance of hives of honeybees for the production of honey. { Honey is a food of high nutritive value and also finds use in the indigenous systems of medicine. {  It also produces beeswax, used in preparation of

cosmetics and polishes most common species of honey bee is Apis indica. { Bee-keeping is not labour-intensive. { The following points are important for successful beekeeping{ Knowledge of the nature and habits of bees. { Selection of suitable location for keeping the beehives. { Catching and hiving of swarms { Management of beehives during different seasons. { Handling and collection of honey and of beeswax. { Keeping beehives in crop fields during flowering period increases pollination efficiency and improves the yield. { Fishery is an industry, which includes catching, processing or selling of fish, shellfish and other aquatic animals like prawn, crab, lobster, edible oysfegr, etc… (i) The common freshwater fishes are catla, rohu and common carp. (ii) Some of the marine fishes that are eaten include Hilsa, Sardines,Mackerel and Pomfrets. (iii)  Fisheries provides income and employment to millions of fishermen and farmers, especially in coastal states in Indian economy. (iv) To meet the increasing demand on fisheries, different techniques like aquaculture and pisciculture are applied. (v)  Blue revolution is the movement launched to increase the production of fish and fish products. It is being implemented in the same lines as green revolution. { The

OBJECTIVE TYPE QUESTIONS [A] MULTIPLE CHOICE QUESTION: 1. Match List-I with List - II.

[CUET 2023]

List-1



List-II

(A) Sterilized plant part

(I) Pomato

(B) Genetically similer plants

(II) Virus free culture

(C) Meristem

(III) Somaclones

(D) Somatic hybrids

(IV) Explant

Choose the correct answer from the option given below;

(a) (A)-(I), (B)-(II), (C)-(III), (D)-(IV) (b) (A)-(II), (B)-(I), (C)-(III), (D)-(IV) (c) (A)-(I), (B)-(II), (C)-(IV), (D)-(III) (d) (A)-(IV), (B)-(III), (C)-(II), (D)-(I) 2. Pomato is developed by _____ technique. [CUET 2022] (a) Micropropagation

(b) Mutation breeding

(c) Biofortification

(d) Somatic hybridization

3. In tissue culture, the nutrient medium usually contains ________ as a carbon source. [CUET 2022] (a) Sucrose

(b) Maltose

(c) Carbon dioxide

(d) Calcium carbonate

4. Protoplasts are obtained by digestion of ________ of the cells. [CUET 2022] (a) Nuclei (b) Plasma membranes (c) Cell walls (d) Proteins 5. Totipotency can be defined as— (a) Capacity to generate whole plant from any cell/explant. (b) Capacity to survive in unfavourable environment condition. (c) Ability to conduct water in 100 meter tall tree. (d) The duration of dormancy of seeds and other propagules in angiosperms. [CUET 2022] 6. Biofortification refers to: (a) Sources of proteins for animal and human nutrition. (b) Production of somatic hybrids. (c) Use of mutation breeding. (d) Breeding crops with higher levels of vitamins and minerals, or higher protein and healthier fat. 7. Protoplast is. (a) another name for protoplasm. (b) an animal cell. (c) a plant cell without a cell wall. (d) a plant cell.

104 Oswaal CUET (UG) Chapterwise Question Bank 8. Germplasm storing centre in India is: (a) NEERI

(b) FRI

(c) CDRI

(d) ICRISAT.

9. Sonalika and Kalyan Sona are varieties of: (a) wheat.

(b) rice.

(c) millet.

(d) tobacco.

10. Micro-propagation is: (a) propagation of microbes in vitro. (b) propagation of plants in vitro. (c) propagation of cells in vitro. (d) growing plants on smaller scale. 11. The term ‘totipotency’ refers to the capacity of a: (a) cell to generate whole plant. (b) bud to generate whole plant. (c) seed to germinate. (d) cell to enlarge in size. Directions: In the following 12. Inbreeding is carried out in animal husbandry because it: (a) increases vigour. (b) improves the breed. (c) increases heterozygosity. (d) increases homozygosity. 13. To isolate protoplast, one needs: (a) pectinase. (b) cellulase. (c) both pectinase and cellulase. (d) chitinase. 14. In virus-infected plants the meristematic tissues in both apical and axillary buds are free of virus because: (a) the dividing cells are virus resistant. (b) meristems have anti-viral compounds. (c) the cell division of meristems are faster than the rate of viral multiplication. (d) viruses cannot multiply within meristem cell (s). 15. Lysine and tryptophan are: (a) proteins. (b) non-essential amino acids. (c) essential amino acids. (d) aromatic and no acids. 16. The agriculture sector of India employs: (a) 60 per cent of the population. (b) 70 per cent of the population. (c) 30 per cent of the population. (d) 62 per cent of the population. 17. 33 percent of India’s (Gross Domestic Product) comes from: (a) industry. (b) agriculture. (c) export. (d) small-scale cottage industries.

BIOLOGY

18. An explant is (a) dead plant. (b) part of the plant. (c) part of the plant used in tissue culture. (d) part of the plant that expresses a specific gene. 19. Which of the following step is the main root of any plant breeding programme? (a) Genetic variability (b) Evaluation and selection of parents (c) Cross hybridisation among the selected parents (d) Selection and testing of superior recombinants 20. Which of the following is not included in germplasm collection? (a) Wild relatives (b) Old improved varieties (c) Diseased varieties (d) Pure lines 21. The biggest constraint of plant breeding is: [NCERT Exemplar, Q. 15, Pg. 63] (a) availability of desirable gene in the crop and its wild relatives. (b) infrastructure. (c) trained manpower. (d) transfer of genes from unrelated sources. 22. A collection of all the alleles of all the genes of a crop plant is called: [NCERT Exemplar, Q. 25, Pg. 65] (a) germplasm collection. (b) protoplasm collection. (c) herbarium. (d) somaclonal collection. 23. Animal husbandry and plant breeding programmes are the examples of: [NCERT Exemplar, Q. 4, Pg. 61] (a) reverse evolution. (b) artificial selection. (c) mutation. (d) natural selection. 24. More than 70% of livestock population is in: [NCERT Exemplar, Q. 22, Pg. 64] (a) Denmark. (b) India. (c) China. (d) India and China. 25. The chances of contacting bird flu from a properly ooked (>100°C) chicken and egg are: [NCERT Exemplar, Q. 1, Pg. 61] (a) very high. (b) high. (c) moderate. (d) none. 26. Which one of the following products of apiculture is used in cosmetics and polishes? [NCERT Exemplar, Q. 21, Pg. 64] (a) Honey (b) Oil (c) Wax (d) Royal jelly 27. A group of animals, which are related by descent and share many similarities, are referred to as: [NCERT Exemplar, Q. 2, Pg. 61] (a) breed. (b) race. (c) variety. (d) species. 28. Fungicides and antibiotics are chemicals that: (a) enhance yield and disease resistance. (b) kill pathogenic fungi and bacteria, respectively. (c) kill all pathogenic microbes. (d) kill pathogenic bacteria and fungi, respectively.

IMPROVEMENT IN FOOD PRODUCTION

29. Use of certain chemicals and radiation to change the base sequences of genes of crop plants is termed as: [NCERT Exemplar, Q. 9, Pg. 62] (a) recombinant DNA technology. (b) transgenic mechanism. (c) mutation breeding. (d) gene therapy. 30. Micro-propagation is: [NCERT Exemplar, Q. 17, Pg. 63] (a) propagation of microbes in vitro. (b) propagation of plants in vitro. (c) propagation of cells in vitro. (d) growing plants on smaller scale. 30. Several South Indian states raise 2-3 crops of rice annually. The agronomic feature that makes this possible is because of: (a) shorter rice plant (b) better irrigation facilities (c) early yielding rice variety (d) disease resistant rice variety [B] ASSERTION & REASON: Directions: In the following questions a statement of assertion (A) is followed by a statement of reason (R). Mark the correct choice as: (A) Both assertion (A) and reason (R) are true and reason (R) is the correct explanation of assertion (A). (B) Both assertion (A) and reason (R) are true but reason (R) is not the correct explanation of assertion (A). (C) Assertion (A) is true but reason (R) is false. (D) Assertion (A) is false but reason (R) is true 1. Assertion: The queen honey bee copulates only once in her life time. Reason: The honey bee queen can lay fertilized as well as unfertilized eggs. 2. Assertion [A]: Biofortification is a type of new “Green Revolution”. Reason [R]: Biofortification aims to increase nutrient levels in crops which are most common and easily available. 3. Assertion: Biofortification is used to increase nutrient value of crops. Reason: To obtain virus resistant plants meristem culture id done 4. Assertion (A): To increases the productivity of milk,c ross breeding of dairy animals is done Reason (R): Cross breeding increase the lactation period of dairy animals 5. Assertion: A single outcross between a superior animal of the inbreeding population and another superior animal of an unrelated population of the same breed can overcome inbreeding depression. Reason: Crossing unrelated animals decreases the chances of homozygosity in progeny. 6. Assertion (A): Broiler chicken should be provided with Protein and fat-rich food Reason (R): Broiler chicken is grown to provide eggs of good quality.

105 7. Assertion: The yield of milk is is primarily dependent on the quality of breeds in the farm Reason: Selection of good breeds having high yielding potential combined with resistance to disease is very important. 8. Assertion: Light is essential in poultry farm management. Reason: For optimum production of eggs 14-16 hours of light including day light is required 9. Assertion: The single cell protein (SCP) use can help to reduce environmental pollution. Reason: SCP producing organisms SCP can be easily grown on straw, molasses, waste water from potato processing plants, sewage, etc. 10 Assertion: Single cell proteins can help to meet increasing demands of growing population. Reason: SCP now can be produced in high amount commercially, using low cost substrates. [C] COMPETENCY/CASE-BASED QUESTIONS: I. Read the following passage and answer the questions (1-5) given below: Animal breeding is another part of animal husbandry. Breeding of animals aims to develop high-yielding animals and improve the desirable qualities of the produce. Animal breeding includes inbreeding and outbreeding. Inbreeding refers to the practice of mating closely related superior males to superior females within the same breed for 4-6 generations. The superior males and females among the progenies are further mated for developing Mendelian pure lines which are homozygous in nature. However, continued close inbreeding eventually decreases fertility and productivity; a phenomenon known as inbreeding depression. And Out-breeding is the breeding of unrelated animals that can be carried out in three different fashions. When two unrelated individuals of the same breed who do not have a common ancestor for the past 4-6 generations are mated, it is called out-crossing. Mating between a superior male of one breed and a superior female of another breed to give a progeny with the combined qualities of both breeds is known as crossbreeding. When males and females across different related species are mated, it is known as interspecific hybridization. 1. Which of the following is not an aspect of animal breeding? (a) Improve desirable qualities of breeds (b) Increasing the yield (c) Making diseased organisms (d) Making disease resistance breeds 2. What is a breed? (a) Animals related by descent (b) Animals related by an ascent (c) Animals not related at all (d) Clone of animals 3. What is inbreeding? (a) Mating of closely related species (b) Mating of unrelated species (c) Mating of clones (d) Mating of different breeds 4. The aim of Animal breeding programme is: (a) Develop high-yielding animals (b) Improve the desirable qualities of the produce (c) Develop disease resistance animals (d) Both (a) and (b)

106 Oswaal CUET (UG) Chapterwise Question Bank 5. Which breeding is used to overcome inbreeding depression? (a) Out-crossing (b) Cross-breeding (c) Interspecific hybridisation (d) Inbreeding II. Read the following passage and answer the following questions: Tissue Culture is a method under which fragments of plants and animal Tissues are cultivated and grown in a laboratory. The organs, many times, are used for Tissue Culture. Broth and agar are the media used for the growth of the Culture. This method is also known as micropropagation. a sterile workplace, a greenhouse, trained manpower, and a nursery are all it requires. It is beneficial for the production of disease-free plants and plants in developing countries. In developing countries, oil palm, banana, plantain, eggplant, pineapple, rubber tree, tomato, sweet potato have been produced by Tissue Culture.  Tissue culture include Primary culture and secondary culture The important types of tissue culture techniques are Seed Culture -It involve in-vitro derived formation of plant into a laboratory where they proliferate. Embryo Culture:Embryo Culture involves the in-vitro development of an embryo. Callus Culture:- Formation of a complete plant from undifferentiated mass of cells called callus. Organ Culture:- In it any organ of the plant such as a shoot, the leaf can be used as an explant. Protoplast Culture and

BIOLOGY

Meristem Culture 6 Tissue culture technique was first developed by ((a) White (b) Haberlandt (c) Halperin (d) Skoog 7. The meristem culture involves culturing which part of the plant (a) Stem (b) Root (c) Meristem (d) Leaves 8. Which of the following is not an application of tissue culture? (a) Rapid Clonal Propagation (b) Somaclonal Variations (c) Embryo rescue (d) Transgenic plants 9. Plant cell and cell wall together constitute the (a) Mitochondria and cytoplasm (b) Protoplast (c) Cytoplasm and cell wall (d) Plasma membrane without cytoplasm 10. Which of the following plant’s meristem has not been successfully cultured? (a) Banana (b) Apple (c) Sugarcane (d) Potato

ANSWER KEY [A] MULTIPLE CHOICE QUESTIONS 1. (d)

2. (d)

3. (a)

4. (c)

5. (a)

6. (d)

7. (c)

8. (d)

9. (a)

10. (b)

11. (a)

12. (d)

13. (c)

14. (c)

15. (c)

16. (d)

17. (b)

18. (c)

19. (a)

20. (c)

21. (a)

22 (a)

23. (b)

24. (d)

25. (d)

26. (c)

27. (a)

28. (b)

29. (c)

30. (b)

1. (b)

2. (a)

3. (c)

4. (a)

8. (a)

9. (a)

10. (a)

9. (b)

10. (b)

[B] ASSERTION & REASON 5. (a)

6. (c)

7. (b)

[C] COMPETENCY/CASE-BASED QUESTIONS 1. (c)

2. (b, a)

3. (a)

4. (c, d)

5. (a)

6. (b)

7. (c)

8. (c)

ANSWERS WITH EXPLANATION [A] MULTIPLE CHOICE QUESTIONS: 1. Option (d) is correct. Explanation: The explant is the tissue or plant part which is introduced in the culture media for the regeneration of the plant. It is primarily sterilised well to remove all the dirt, bacteria, or infections stuck on the surface of it to ensure that the resulting plant produced will be free from any diseases. Somaclones are the genetically identical plants produced by using the somatic cells or the vegetative parts of the plants through the tissue culture technique Meristem culture involves utilizing apical and axillary meristems as explants. It can help in obtaining virus-free plants as meristems are generally free from virus even if the plant is affected. Pomato was developed by somatic hybridization (protoplast fusion) of two different genera tomato and potato. It is intergeneric somatic hybrid.

2. Option (d) is correct Explanation: Pomato was developed by somatic hybridization (protoplast fusion) of two different genera tomato and potato. It is intergeneric somatic hybrid. 3. Option (a) is correct Explanation: In tissue culture, the nutrient medium-usually contains sucrose as carbon source, Sol. In tissue culture, sucrose is typically used as a carbon source in the nutritional media. 4. Option (c) is correct. Explanation: The two cells of the target plants are first treated with the pectinase and cellulase enzymes in preparation for protoplast fusion. These enzymes break down the cell wall, resulting in the creation of bare protoplasts. 5. Option (a) is correct Explanation: Totipotency is the capacity to generate whole plant from any cell/explants. It is the genetic potential of a plant cell to produce the entire plant. In other words, totipotency is

IMPROVEMENT IN FOOD PRODUCTION

the cell characteristic in which the potential for forming all the cell types in the adult organism is retained. 6. Option (d) is correct Explanation: Biofortification is the process of breeding crops with higher levels of vitamins, minerals, proteins and fat content. This method is employed to improve public health. Single Cell Protein (SCP) is one of the sources of proteins for animal and human nutrition. Somatic hybrids are produced by somatic hybridization. Mutation breeding is one of the methods for making crops disease resistant. 7. Option (3 c) is correct. Explanation: A protoplast is a plant, bacterial or fungal cell whose cell wall is completely or partially removed using either mechanical or enzymatic me 8. Option (d) is correct. Explanation: Germplasm storing centre in India is ICRISAT (International Crops research institute for Semi-Arid tropics located at Hyderabad (Patancheru)). 9. Option (a) is correct. Explanation: Sonalika and Kalyan Sona are varieties of wheat. Sonalika is developed from high yielding, semi-dwarf, fertiliser-responsive wheat variety. They were developed in 1963 at Indian Agriculture Research Institute (IARI), New Delhi, as a part of systematic programme for breeding semidwarf wheat varieties. 10. Option (b) is correct. Explanation: Micro-propagation is propagation of plants invitro (tissue culture) to achieve a large number of plants in very short durations. This results in genetically identical plants which are widely used in forestry and floriculture. 11. Option (a) is correct. Explanation: The capacity of a cell or an explant (any part of plant taken out and grown in test tube) to grow into a whole plant is called totipotency. 12. Option (d) is correct. Explanation: Inbreeding increases homozygosity, that is, state of possessing two identical alleles, one inherited from each parent. It is necessary if we want to evolve a pure line in any animal. Inbreeding exposes harmful recessive genes that are eliminated by selection. It also helps in accumulation of superior genes and elimination of less desirable genes. 13. Option (c) is correct Explanation: Pectin and cellulose are the main components of the cell wall of plant cells. These can be removed by digestion with a combination of pectinase and cellulase enzymes to isolate the protoplast. 14. Option (c) is correct Explanation: In virus-infected plants, the meristematic tissues in both apical and axillary buds are free of virus because the multiplication of meristematic cells is faster than replication of viruses. The main reason behind it is the gene silencing. Neither dividing cells are virus resistant nor do the meristems have antiviral compounds. 15. Option (c) is correct Explanation: Essential amino acids are required for normal health and growth. These are usually supplied with dietary protein because these are not synthesised by the body. Lysine and tryptophan are types of essential amino acids. 16. Option (d) is correct Explanation: India is mainly an agricultural country. The

107 agriculture sector of India employs nearly 62% of the population. 17. Option (b) is correct Explanation: Agriculture accounts for approximately 33% of India’s GDP (gross domestic products). 18. Option (c) is correct. Explanation: Any part of a plant taken out and grown in a test tube under sterile conditions in a special nutrient media is called an explant. 19. Option (a) is correct Explanation: Genetic variability is the root of any breeding programme. In many crops, pre-existing genetic variability is available from wild relatives of the crop. Collection and preservation of all the different wild varieties, species and relatives of the cultivated species is a pre-requisite for effective exploitation of natural 20. Option (c) is correct Explanation: The entire collection of plants or seeds having all the diverse alleles for all genes in a given crop is called as germplasm collection. It consists of all present-day improved varieties, old improved varieties no longer in cultivation, local varieties of all areas, pure lines maintained by plant breeders, wild relatives and related species. 21. Option (a) is correct. Explanation: The biggest constraint of the plant breeding is the availability of limited number of disease resistance genes that are present and identified in various crop varieties or wild relatives. 22. Option (a) is correct. Explanation: The entire collection of plants/seeds which have all the diverse alleles for all genes in a given crop is called germplasm collection. Protoplasm collection refers to collection of protoplast (i.e., plant cell without cell wall). Herbarium is a museum of preserved plants that are used for botanical research, mainly in identification and classification of plants. Soma-clonal collection consists of those plants that are produced from a single cell which are genetically variable from their parents. 23. Option (b) is correct Explanation: Animal husbandry and plant breeding programmes are the examples of artificial selection. Artificial selection is a process in which human choose only those traits or characters which are transferred into their offspring. Mutation is a sudden change in DNA sequence due to mutagenic agents like chemicals and radiations. Natural selection is a slow process by which biological traits become either more or less common in a population as a function of the effect of the changing environment. Reverse evolution or devolution is a concept in which species can change into more primitive forms over time. 24. Option (d) is correct Explanation: It is estimated that more than 70% of the world’s livestock population is in India and China. 25. Option (d) is correct. Explanation: The chances of contacting bird flu from a properly cooked (>100°C) chicken and egg are negligible. Highly pathogenic strains of avian influenza can be found inside and on the surface of eggs. In an infected bird, it may spread to all the parts including the meat. However, proper cooking at or above 70°C, prior to eating, will inactivate the virus in the egg as well as in the meat.

108 Oswaal CUET (UG) Chapterwise Question Bank 26. Option (c) is correct. Explanation: Apiculture (also called bee keeping) is a process of rearing and management of honey bees for commercial production of the following: (i) Bees wax: It is used in industry for the preparation of cosmetics and polishes of various kinds. (ii) Honey: It is a food of high nutritive value and also finds use in the indigenous systems of medicine. (iii) Royal jelly: It is the extra-ordinary source of food for queen bees. It is rich in nutrition values and is believed to be a potent anti-oxidant. Oil is not being considered as a product of apiculture. 27. Option (a) is correct. Explanation: Breed is a group of animals that are related by descent and are similar in most characters like general appearance, features, size, configuration, etc. The other three options are incorrect. Race is used in classification system to categorise humans into large and distinct populations or groups based on anatomical, cultural, ethnic or geographical differences. Variety is a genetically and morphologically distinct subset of a species that is geographically isolated from other populations within that species. A species is defined as a group of individuals that potentially interbreed in nature. 28. Option (b) is correct Explanation: A wide range of pathogen (e.g., fungi, bacteria and virus) affects the yield of cultivated crop species. Fungicides are generally used to control parasitic fungi that either cause economic damage to crop or ornamental plants or endanger the health of domestic animals or hum Whereas antibiotics are medicines that help to stop infections caused by bacteria. Therefore, fungicides and antibiotics are chemicals that kill pathogenic fungi and bacteria, respectively. 29. Option (c) is correct. Explanation: Mutation breeding (also called variation breeding) is a process of induced breeding in crop improvement. It is possible to induce mutations artificially in crop plants through the use of chemical, or radiations (i.e., gamma radiations), and then selecting and using those plants that have the desirable character as a source in breeding. Recombinant DNA technology involves transferring of a desired gene (trans-gene) from an organism and incorporating it to host organism by genetic engineering to produce a recombinant. Gene therapy is performed mainly in hum It involves replacing a defective gene with a normal one. 30. Correct option: (b) Explanation: Micro-propagation is propagation of plants in vitro (tissue culture) to achieve a large number of plants in very short durations. This results in genetically identical plants which are widely used in forestry and floriculture. [B]. ASSERTION AND REASON QUESTIONS: 1. Option (b) is correct Explantation: Queen honey bee only copulates once in her life time with drone (male honeybee). And honeybee can lay both fertilized and unfertilized eggs as well but it is not correct explanation of assertion as after copulation queen honeybee store millions of sperms and they can lay eggs through their life. 2. Option (a) is correct Explantation: Biofortification aims to increase nutrient levels in crops during plant growth rather than through manual means during processing of the crops. It can be thought of as a new Green Revolution, where the aim is to improve public health,

BIOLOGY

or in other words, ensure that everyone can obtain the nutrients required for their growth and development, by producing most commonly available crops with higher nutritional content. 3. Option (c) is correct Biofortification is the process by which the nutritional quality of food crops is improved through agronomic practices, conventional plant breeding, or modern biotechnology. Hence  Biofortification is used to increase the nutrient value of crops is a correct statement. Virus-free plants can be formed by meristem culture because of the viruses spread systemically in the whole plant except the meristems when they colonize a new plant host, via phloem vascular element. Hence, Meristem culture is used to obtain virus-resistant plants is an incorrect statement because meristem culture is used to obtain virus-free plants but not virus-resistant plants. 4. Option (a) is correct Explantation: Cross breeding increases the lactation period of a dairy animal and thus the productivity of milk increases. 5. Option (a) is correct Explantation: Inbreeding involves mating closely related superior animals of a breed, continuously for 4-6 generations. Continuous inbreeding in a small population between related animals of the same breed increases the occurrence of identical (homozygous) alleles in a genotype. It also leads to gradual loss of fertility, productivity and reduced weight in the progeny. This is known as inbreeding depression. This is because closely related parents have a higher chance of transferring similar alleles to the offspring. Thus, crossing unrelated animals decreases the chances of homozygosity in progeny. Thus it is concluded that both assertion and reason are true statements and reason is the correct explanation for assertion. 6. Option (c) is correct Explantation: Broiler chicken requires protein and fat rich food for their fast growth rate • They are mostly raised for meat and not for laying eggs. 7. Option (b) is correct Explantation: Milk yield is primarily dependent on the quality of breeds in the farm, as high milk-yielding breeds of females produce more milk per lactation. Selection of good breeds having high yielding potential combined with resistance to disease is very important because it helps in improving the desirable qualities of the produce as well as gives rise to superior progeny. 8. Option (a) is correct Explantation: A Light is essential for high egg production in poultry birds. For optimum production 14 to 16 hours of light including day light is required, reducing photoperiod can seriously effect egg production as the egg laying patterns can be changed due to one hour of light 9. Option (a) is correct. Explantation: Single Cell Protein or SCP refers to the edible protein extracted from pure microbial cultures, dead, or dried cell biomass. It can be used as an alternative protein source or supplement for both humans or animals . Microorganisms such as unicellular algae, fungi, yeast, bacteria can be used as SCP. Algae such as Spirulina are widely used for the production of SCP as it is rich in protein content. It can be easily grown using inexpensive substrates like agricultural wastes, wood shavings, sawdust, corn cobs, waste water from the potato processing industry, straw, molasses, manure, sewage, human and animal wastes, etc.

IMPROVEMENT IN FOOD PRODUCTION

10. Option (a) is correct Single-cell protein (SCP) is protein-rich biomass that is used as food or feed. It is rich in good quality protein and poor in fats. Yes, single-cell proteins so, it can help to meet the increasing demands of the growing population.  There are several reasons for this claim some of these are listed below:- Some of the reasons are: (i) SCP can be produced using low-cost substrates like dust, paddy, straw, molasses. (ii)  Microorganisms used for SCP have a high rate of multiplication. (iii) Microbial biomass production as SCP is independent of seasonal as well as climatic variations. [C] COMPETENCY/CASE BASED QUESTIONS: 1. Option (c) is correct Explanation: Breeding of animals is an important aspect of animal husbandry. Animal breeding aims at increasing the yield of animals and improving the desirable qualities of their produce and making them disease resistant. 2. Option (b) (a) is correct Explanation: A group of animals related by descent and similar in most characters like general appearance, features, size, configuration, etc., are said to belong to a breed. 3. Option (a) is correct Explanation: Inbreeding refers to the mating of more closely related individuals within the same breed for 4-6 generations. The breeding strategy is as follows-superior males and superior females of the same breed are identified and mated in pairs. 4. Option (c) (d) is correct Explantation: Breeding of animals aims to develop highyielding animals and improve the desirable qualities of the Animals. 5. Option (a) is correct Explanation: Continued inbreeding, especially close inbreeding, usually reduces fertility and even productivity. This is called inbreeding depression. Whenever this becomes

24

109 a problem, selected animals of the breeding population should be mated with unrelated animals of the same breed. This is called as outcrossing and it often helps to overcome inbreeding depression. 6. Option ( b) is correct Explanation: Tissue culture technique was first practised by Haberlandt and Hanning in 1902. Harrison developed this technique. The first successful attempt was made by White in case of Tomato root.Gottlieb Haberlandt was an German botanist who first developed tissue culture. He pointed out for the first time that plants cells are totipotent, which led to establishment of plant tissue culture. He is known as the father of tissue culture. 7. Option ( c) is correct Explanation: When the plants are infected with a virus, the meristem (apical and axillary) is free of the virus because the cell division of meristem is faster than the rate of virus replication and multiplication. 8. Option ( c) is correct Explanation: The applications of tissue culture include Rapid clonal propagation, Somaclonal variations, Transgenic plants, Induction and Selection of mutations, resistance to weedicides and tolerance to stress like pollutants, toxins, salts, drought, floods etc. Embryo rescue is the procedure that involves excising weak, immature plant embryos and culture them in-vitro on specially devised culture medium. 9. Option (b) is correct Explanation: Protoplast is a plant cell without a cell wall. Scientists have been able to isolate single cells from plants and after digesting their cell walls have been able to isolate naked protoplasts (surrounded by plasma membranes). 10. Option (b) is correct Explanation: Culturing of meristems is an important method of the recovery of healthy plants from diseased plants. Hence, one can remove the meristem and grow it in vitro to obtain virusfree plants. Scientists have succeeded in culturing meristems of banana, sugarcane and potato.

Study Time

CHAPTER

10

Max. Time: 1:5 Hours Max. Questions: 50

MICROBES IN HUMAN WELFARE

 Revision Notes:

 Microbial Diversity Microbes are diverse-protozoa, bacteria, fungi and microscopic plants viruses, viroid and also prions that are proteinacious infectious agents. They are found everywhere on earth ranging from soil, air water and some inhabitable places. Bacteria and fungi can be grown on nutritive media to form colonies, which can be seen by necked eyes and very useful in study of microorganisms. Microbes cause many diseases in human beings, plants and animals. Several microorganisms are useful to man in diverse ways.  Microbes in household products household food

processing

{ Microorganisms

like Lactobacillus and other commonly called lactic acid bacteria (LAB) grow in milk and convert it to curd. Scan to know The LAB produces acids that more about coagulate and partially digest this topic the milk proteins. It also improves its nutritional quality by increasing vitamin B12.  In our stomach too, the LAB play Microbes in very beneficial role in checking Household disease-causing microbes. Products {  The dough is used for making foods such as dosa and idli is fermented by bacteria. The puffed-up appearance of dough is due to the production of CO2 gas. The dough used for making bread is fermented using baker’s yeast (Saccharomyces cervisiae). {  Cheese, is one of the oldest food items in which microbes were used. The large holes in ‘Swiss cheese’ are due to production of a large amount of CO2 by a bacterium named Propionibacterium sharmanii. The ‘Roquefort cheese’ is ripened by growing a specific fungus on them for a particular flavour.  Microbes in industrial production A number of products like beverages and antibiotics involve uses of microbes. Production on large scale requires growing microbes in very large vessels called fermenters. { Fermented Beverages- Saccharomyces cerevisiae used for bread-making and commonly called brewer’s yeast, is used for fermenting malted cereals and fruit juices, to produce beverages like wine, bear, whisky and rum. Wine and bear are produced without distillation whereas whisky, brandy and rum are produced by

distillation of the fermented broth. They are chemical substances produced by some microbes and can kill or retard the growth of other microbes. Penicillin was first antibiotic to be discovered. Antibiotics have great improved our capacity to treat deadly diseases such as plague, whooping cough, diphtheria and leprosy. { Chemical, Organic acids , Enzymes and other Bioactive Molecules are commercially produced by microbes. Chemicals : ƒ Aspergillus niger (fungus) – Citric acid ƒ Acetobacter aceti (bacterium) – Acetic acid ƒ Clostridium butylicum (bacterium) – Butyric acid ƒ Lactobacillus (bacterium) – Lactic acid ƒ Saccharomyces cerevisiae (yeast)  – Ethanol Enzymes: ƒ Lipase – used in laundry detergents for removing oily stains ƒ Pectinase and protease – used in for clarifying suspended material from fruit juices. ƒ Streptokinase (Streptococcus bacterium) – used as clot buster (to remove clots) Bioactive molecules: ƒ Cyclosporin A (Trichoderma polysporum fungi) – used as immunosuppressive agent (for organ transplant patients). ƒ Statins (Monascus purpureus yeast) – used as blood cholesterol lowering agents. { Antibiotics-

 Microbes in Sewage Treatment { Municipal waste water (sewage) contains large amount of organic matter and microbes which are pathogenic and cannot be discharged into natural water bodies like rivers and streams. { Sewage is treated in sewage treatment plant to make it less polluting by using heterotrophic microbes naturally present in sewage. { Sewage treatment is done in two stages In Primary treatment, floating debris is removed by sequential filtration. Grit (soil and small pebbles) are removed by sedimentation. Secondary treatment or biological treatment involves passing of primary effluents in large aeration tank to help the growth of aerobic microbes into flocs (masses of bacteria associated with fungal

MICROBES IN HUMAN WELFARE

111

112 Oswaal CUET (UG) Chapterwise Question Bank filaments to form mesh like structures).  These microbes increase the consumption of organic wastes and decrease the BOD (biological oxygen demand)of the effluents. { BOD is the amount of oxygen that would be consumed if all the organic matter in one litre of water were oxidised by bacteria. It measures the amount of organic matter present in the water. Greater the BOD of water more it is polluted. { Once the BOD of sewage or waste water is reduced, the effluent is then passed into a settling tank where the bacterial ‘flocs’ are allowed to sediment. This sediment is called activated sludge. { Sludge is passed into large tanks called anaerobic sludge digesters in which anaerobic bacteria digest the bacteria and fungi in the sludge and produce mixture of gas called biogas, which is a mixture of methane, hydrogen sulphide and carbon dioxide. { The effluents from the secondary treatment plant are released into water bodies.   Microbes in Production of Biogas Energy

generation { Biogas

is a mixture of gases produced by the microbial activity that can be used as fuel. { Certain bacteria that grows anaerobically on cellulosic material produce large amount of methane along with CO2 and H2. These bacteria are collectively called methanogens (Methanobacterium).

BIOLOGY

is removed and supplied through an outlet pipe for consumption. { The spent slurry is removed through another outlet and used as fertilisers. Biogas plant is more often build in rural areas as large amount of cattle dug is available easily.  Microbes as Biocontrol agent { Biocontrol

means use of biochemical method for controlling plant disease and pests. The chemical used as pesticides and insecticides are harmful to human beings and animals.

{ Biological

Scan to know more about this topic

Microbes as

control of pests biocontrol agent and disease is a method of controlling pest on natural prediction rather than chemicals.

{ The organic farmer creates a system where the pests are

not eradicated but kept at manageable level by complex system of check and balance within the living and vibrant ecosystem. For example The

Ladybird and Dragonflies are used to get rid of aphids and mosquitoes respectively.

On

brassicas and fruit tree, to control butterfly caterpillars bacteria Bacillus thuringiensis is used.

{ Biological control developed for use in the treatment of

plant disease. For example The

fungus are free-living fungi that are very common in the root systems that control several plant pathogens.

{ Baculoviruses

are pathogens that attack insects and other arthropods. The majority of baculoviruses used as biological control agents For example The

genus Nucleopolyhedrovirus are major baculoviruses. These viruses are excellent candidates for species-specific, narrow spectrum insecticidal applications.

 Microbes as Biofertilisers { Bio

fertilisers are organisms that enrich the nutrient quality of the soil. The main sources includes bacteria, fungi and cyanobacteria.

Fig. 1: Biogas Plant

Biogas Plant { The

technology of biogas production was developed in India mainly due to the efforts of Indian Agricultural Research Institute (IARI) and Khadi and Village Industries Commission (KVIC). { The excreta of cattle (gobar) is rich in methanogens bacteria and is used for generation of biogas also called as gobar gas. { Biogas plant consists of a concrete tank in which biowastes are collected and slurry of dung is fed. { A floating cover is placed over digester that moves upward when gas is produced. The gas produced

The

root nodule formed by Rhizobium bacteria on root of leguminous plants increase the nitrogen level of soil, necessary for various metabolic processes. and Azospirillum are free living bacteria that live in soil and fix atmospheric nitrogen into organic forms.

Azotobacter

Symbiotic

association of fungi with angiosperm plants (mycorrhiza) also increase the fertility of soil. Glomus form mycorrhiza that absorbs phosphorus from the soil and passes it to the plant. These microbes also provide benefits like resistance to root-borne pathogens, tolerance to salinity and drought.

113

MICROBES IN HUMAN WELFARE Cyanobacteria

(Nostoc, Anabaena, etc), an autotrophic microbes found in aquatic and terrestrial environment fix atmospheric nitrogen. In

paddy field this acts as important biofertiliser. Blue green algae also add organic matter to the soil and increase its fertility.

OBJECTIVE TYPE QUESTIONS [A] MULTIPLE CHOICE QUESTIONS: 1. The inactive protoxin gets converted into an active form due to the ................ .  (CUET 2022) (a) Alkaline pH of insect’s gut (b) Temperature and acidic pH of gut (c) Exposure to light (d) Exposure to light and acidic pH of gut 2. Bacillus thuringiensis is a biocontrol agent against ________ .  (CUET 2022) (a) Nematode (b) Fungal pathogen (c) Insect pests (d) Bacterial pathogen 3. The bacterium responsible for breakdown of cellulose in a biogas plant is:  (CUET 2022) (a) Acetobacter aceti (b) Lactobacillus (c) Clostridium (d) Methanobacterium 4. Which of the following microbe is used in production of antibiotics?  (CUET 2022) (a) Penicillium notatum (b) Streptomyces griseus (c) Aspergillus niger (d) Clostridium butylicium (e) Saccharomyces cerevisiae Choose the correct answer from the options given below: (A) A and B only (B) C and D only (C) B and E only (D) C and E only 5. Statement 1: Baculoviruses belonging to genus Nucleopolyhedrovirus are used as biological control agent. Statement 2: These viruses are used as they are species-specific and have narrow spectrum insecticidal applications, so can be a good insecticide.  (CUET 2022) In the light of above statements, choose the most appropriate answer from the options given below (a) Both statement 1 and statement 2 are correct. (b) Both statement 1 and statement 2 are incorrect. (c) Statement 1 is correct and statement 2 is incorrect. (d) Statement 1 is incorrect and statement 2 is correct. 6. Mr. X had purchased a fruit juice bottle from the market and is puzzled to see that is completely clear. It is due to (a) Lipase and Pectinase  (CUET 2022) (b) Amylase and Polymerase (c) Pectinase and Protease (d) Protease and Ligase 7. Bioactive molecules like statins are produced by: (a) Trichoderma polysporum  (CUET 2022) (b) Monascus purpureus (c) Acetobacter aceti (d) Streptococcus

8. Which one of the following is the correct match of the product and the producer? (a) Cyclosporin A: Trichoderma polysporum (b) Blood cholesterol-increasing statins: Monascus purpureus (c) Antibiotics : Acetobacter aceti (d) Red dye: Methanobacterium 9. Big holes in Swiss cheese are made by: (a) a machine (b) a bacterium that produces methane gas (c) a bacterium producing a large amount of carbon dioxide (d) a fungus that releases a lot of gases during its metabolic activities 10. Methanogenic bacteria are not found in: (a) rumen of cattle (b) gobar gas plant (c) bottom of water-logged paddy fields (d) activated sludge 11. The approach to biological control includes : (a) Import and release of an insect pest to a new area to provide hosts for natural enemies. (b) Import and release of natural enemies from the native home of an alien insect pest that has invaded a new area. (c) Preservation of natural enemies (predators & parasites) that are already established in an area. (d) Use of insecticides to reduce alien insect pests to establish new equilibrium position. 12. Dragonflies indicate positive ecological impact as : (a) The presence of dragonflies indicates polluted water. (b) Dragonfly nymphs selectively eat mosquito larvae. (c) They help to decrease the probability of diseases spread by vectors. (d) Dragonfly do not cause any harm to beneficial species. 13. The most effective stages in the life cycle of dragonfly that eradicates mosquitoes are : (a) Larvae and adult (b) Caterpillar and adult (c) Nymph and adult (d) Pupa and adult 14, What would happen if oxygen availability to activated sludge flocs is reduced? (a) It will slow down the rate of degradation of organic matter. (b) The centre of flocs will become anoxic, which would cause death of bacteria and eventually breakage of flocs. (c) Flocs would increase in size as anaerobic bacteria would grow around flocs. (d) Protozoa would grow in large numbers. 15. The free-living fungus Trichoderma can be used for: (a) killing insects (b) biological control of plant diseases (c) controlling butterfly caterpillars (d) producing antibiotics

114 Oswaal CUET (UG) Chapterwise Question Bank 16. Wastewater treatment generates a large quantity of sludge, which can be treated by: (1) digesters (2) activated sludge (3) chemicals (4) oxidation pond 17. BOD of waste water is estimated by measuring the amount of: (a) total organic matter. (b) biodegradable organic matter. (c) oxygen evolution. (d) oxygen consumption. 18. The free-living fungus Trichoderma can be used for: (a) killing insects. (b) biological control of plant diseases. (c) controlling butterfly caterpillars. (d) producing antibiotics. 19. The large holes in Swiss cheese are due to (a) Production of large amount of O2 (CUET 2023) (b) Citric acid (c) Production of large amount of CO2 (d) Ethyl Alcohol 20. Which one of the following microbes forms symbiotic association with plants and help them in their nutrition ? (a) Azotobacter (b) Aspergillus (c) Glomus (d) Trichoderma 21. Bacillus thuringiensis is widely used as : (a) Insecticide (b) Weedicides (c) Rodenticide (d) All of the above 22. Which of the following is not used as a biopesticide? (a) Xanthomonas campestris (b) Bacillus thuringiensis (c) Trichoderma harzianum (d) Nucleopolyhedrovirus 23. The vitamin whose content increases following the conversion of milk into curd by lactic acid bacteria is:  [NCERT Exemplar, Q. 1, Pg. 69] (a) vitamin C. (b) vitamin D. (c) vitamin B12. (d) vitamin E. 24 Wastewater treatment generates a large quantity of sludge, which can be treated by: [NCERT Exemplar, Q. 2, Pg. 69] (a) digesters. (b) activated sludge. (c) chemicals. (d) oxidation pond. 25. Match the following list of bacteria and their commercially important products. Column A (Bacterium)

Column B (Product)

a) Aspergillus niger

I. Lactic acid

b) Acetobacter aceti

II. Butyric acid

c) Clostridium butylicum

III. Acetic acid

d) Lactobaillus

IV. Citric acid [NCERT Exemplar, Q. 4, Pg. 69]

BIOLOGY

Choose the correct match. (a) A – (ii), B – (iii), C – (iv), D – (i) (b) A – (ii), B – (iv), C – (iii), D – (i) (c) A – (iv), B – (iii), C – (ii), D – (i) (d) A – (iv), B – (i), C – (iii), D – (ii) 26. The primary treatment of waste water involves the removal of: (a) dissolved impurities. (b) stable particles. (c) toxic substances. (d) harmful bacteria. 27. BOD of waste water is estimated by measuring the amount of: (a) total organic matter. (b) biodegradable organic matter. (c) oxygen evolution. (d) oxygen consumption. 28. Match the following list of bioactive substances and their roles. Bioactive substance

Role

a) Statin

I. Removal of oil stains

b) Cyclosporine A

II. Removal of clots from blood vessels

c) Streptokinase

III. Lowering of blood cholesterol

d) Lipase

IV. Immuno-sup-pressive agent

Choose the correct match. (a) A – (ii), B – (iii), C – (i), D – (iv) (b) A – (iv), B – (ii), C – (i), D – (iii) (c) A – (iv), B – (iii), C – (ii), D – (i) (d) A – (iii), B – (iv), C – (ii), D – (i) 29. Which one of the following alcoholic drinks is produced without distillation? [NCERT Exemplar, Q. 8, Pg. 70] (a) Wine (b) Whisky (c) Rum (d) Brandy 30. The technology of biogas production from cow dung was developed in India largely due to the efforts of [NCERT Exemplar, Q. 9, Pg. 70] (a) Gas Authority of India. (b) Oil and Natural Gas Commission. (c) Indian Agricultural Research Institute and Khadi and Village Industries Commission. (d) Indian Oil Corporation. [B] ASSERTION & REASON: Directions: In the following questions, a statement of assertion (A) is followed by a statement of reason (R). Mark the correct choice as: (A) Both assertion (A) and reason (R) are true and reason (R) is the correct explanation of assertion (A). (B) Both assertion (A) and reason (R) are true but reason (R) is not the correct explanation of assertion (A). (C) Assertion (A) is true but reason (R) is false. (D) Assertion (A) is false but reason (R) is true.

115

MICROBES IN HUMAN WELFARE

[C] COMPETENCY/CASE-BASED QUESTIONS: I. Read the passage carefully and answer the question given after the passage from 1–5. Growth of a population with time shows specific and predictable patterns. Two types of growth pattern of population are exponential and logistic growth. When resources in the habitat are unlimited each species has the ability to realise fully its innate potential to grow in



number. Then the population grows in exponential fashion. When the resources are limited, growth curve shows an initial slow rate and then it accelerates and finally slows giving the growth curve which is sigmoid. 1. Identify the incorrect statement : (a) Exponential growth occurs in organisms such as lemmings. (b) Logistic growth is more realistic.

(c) Exponential growth has two phases lag and log. (d) In logistic growth, population passes well beyond the carrying capacity of ecosystem. 2. Identify the correct equation that represents the exponential population growth curve : (a) dN/dt = rN (b) dN/dt = rN (K-N/K) (c) Nt =N0ert (d) Both (a) and (c) 3. The equations correctly represents Verhulst-Pearl logistic growth is : (a) dN/dt = rN(K-N)/K (b) dN/dt = rN/K (c) dN/dt = N(K-N)/K (d) dN/dt = r(K-N)/K 4. The population growth is generally described by the following equation: dN/dt = rN(K – N)/K What does ‘r’ represent in the given equation? (a) Population density at time Y (b) Intrinsic rate of natural increase (c) Carrying capacity (d) The base of natural logarithm 5. Study the population growth curves (A and B) in the given graph and select the incorrect statement

K

A

(N)

Population density

1. Assertion (A): An inoculum of curd is used for curdling of milk. Reason (R): It contains Lactobacillus, which also improves the nutritional quality by increasing vitamin B12. 2. Assertion (A): Statin is produced by a yeast called Trichoderma polysporum. Reason (R): Monascus purpureus lowers the blood cholesterol level in the body. 3. Assertion (A): Activated sludge should have the ability to settle quickly. Reason (R): This is to be done to absorb pathogenic bacteria present in waste water while sinking to the bottom of the settling tank. 4 Assertion (A): Cyanobacteria increases the soil fertility and also add the organic matter to the soil. Reason (R): They can fix free nitrogen in the atmosphere. 5. Assertion (A): Members of the genus Glomus form mycorrhiza. Reason (R): Mycorrhiza is an association of plants and fungi. 6. Assertion (A): Rhizobium leguminosarum is a symbiotic bacterium found in the root nodules of leguminous plants. Reason (R): They have the ability to fix atmospheric nitrogen. 7. Assertion (A): Nucleopolyhedrovirus are useful, as they are good biocontrol agents. Reason (R): They are species-specific, narrow spectrum bioinsecticides. 8. Assertion (A) : Blue green algae are popular as biofertilisers. Reason (R) : Blue green algae cause algal bloom in polluted water bodies. 9. Assertion: Toddy becomes unpalatable after 24 hours. Reason: The fermentation of toddy is continued by naturally occurring yeasts. 10. Assertion : Curd is more nutritious than milk . Reason : LAB present in curd checks the growth of disease- causing microbes.

B

Time (t)

(a) Curve ‘A’ shows exponential growth, represented by equation dN/dt=rN. (b) Curve ‘B’ shows logistic growth, represented by equation dN/dt = r(K-N)/K. (c) Exponential growth curve is considered as more realistic than the logistic growth curve. (d) Curve ‘A’ can also be represented by equation Nt = N0ert. II. Study the following diagram and answer the question given below 6–10. Large quantities of sewage is generated every day in cities and towns, which is treated in Sewage Treatment Plants (STPs) to make it less polluted. Given below is the flow diagram of one of the stages of STP. Observe the given flow diagram and answer the questions accordingly. 6. Primary effluent is passed into large aeration tanks: (a) To allow bacterial flocs to settle down (b) To allow fast growth of useful aerobic microbes into flocs (c) To allow anaerobic sludge digestion (d) None of these 7. Technical term used for sediment formed in sewage treatment is: (a) Flocs (b) Effluents

116 Oswaal CUET (UG) Chapterwise Question Bank (c) (d) 8. (a) (b) (c) (d) 9.

Activated sludge Anaerobic sludge The significance of the above sediment formed is that: It acts as inoculum It serves in formation of flocs It helps in formation of natural water bodies It helps in anerobic sludge digestion Identify the correct set of gases produced during digestion of sludge: (a) methane, hydrogen sulphide and carbondioxide

BIOLOGY

(b) methane, sulphur and carbon dioxide (c) carbon mono-oxide, methane, hydrogen sulphide (d) methane, hydrogen sulphide and nitrogen 10. Secondary treatment of sewage involves: (a) Physical removal of large and small particles from sewage through filtration (b) Anaerobic sludge digestion (c) Sedimentation to remove smaller and larger particles (d) Mechanical agitation coupled with addition of aerobic microbes

ANSWER KEY [A] MULTIPLE CHOICE QUESTIONS 1. (a)

2. (c)

3. (d)

4. (a)

5. (a)

6. (c)

7. (b)

8. (a)

9. (c)

10. (d)

11. (b)

12. (c)

13. (c)

14. (b)

15. (b)

16. (a)

17. (d)

18. (b)

19. (c)

20. (c)

21. (a)

22 (a)

23. (c)

24. (a)

25. (c)

26. (b)

27. (d)

28. (d)

29. (a)

30. (c)

8. (d)

9. (a)

10. (a)

9. (a)

1. (d)

[B] ASSERTION & REASON 1. (a)

2. (d)

3. (c)

4. (a)

5. (a)

6. (a)

7. (a)

[C] COMPETENCY/CASE-BASED QUESTIONS 1. (d)

2. (d)

3. (a)

4. (b)

5. (c)

6. (c)

7. (c)

8. (a)

ANSWERS WITH EXPLANATION [A] MULTIPLE CHOICE QUESTIONS: 1. Option (a) is correct. Explanation: The Bt toxin is produced by the bacterium Bacillus thuringiensis through the process of sporulation. The Bt toxin protein exist as inactive protoxins but once an insect ingests the inactive toxin, it is converted into an active form of toxin due to alkaline pH of the gut which solubilize the crystals. 2. Option (c) is correct Explanation: Bacillus thuringiensis is a bio control agent against insect pests. 3. Option (d) is correct Explanation: The bacteria that are collectively called methanogens and one such bacterium responsible for breakdown cellulose in a biogas plant is Methanobacterium. These bacteria grow anaerobically on cellulosic material produce large amount of methane along with CO2 and H2 after breakdown. 4.. Option (a) is correct Explanation: Alexander Fleming discovered penicillin, the first natural antibiotic in 1928. Penicillium notatum (fungus) produces penicillin, the first antibiotic. Streptomyces griseus is a species of bacteria, they are well known producers of antibiotic, streptomycin. It is the first antibiotic ever reported from a bacterium. 5. Option (a) is correct Explanation: Baculoviruses are biological insecticides used to control pests insects in agriculture and horticulture. The majority of baculoviruses which are used as biological control agents belong to the genus Nucleopolyhedrovirus. They are species-specific, narrow spectrum insecticidal applications.

They do not show negative effects on mammals, birds, plants, fish or even non-target insects. 6. Option (c) is correct Explanation: Clarification, is the removal process of suspended material, is an important step in the production of fruit juice due to its significant effect on the appearance, flavour and commercialization of juice. The enzymes, pectinases and proteases are helpful in clarifying fruit juices. Pectinase, breaks down the pectin polymer structure and reduces the undesirable turbidity. 7. Option (b) is correct Explanation: Statins are produced by Monascus purpureus yeast. They are used as blood-cholesterol lowering agents. These bioactive molecules. It inhibits the enzymes responsible for synthesis of cholesterol. Trichoderma polysporum a fungus produces an immunosuppressive agent cyclosporin A, which is used as an immunosuppressive agent in for organ transplant patients. Acetobacter aceti a bacterium produces acetic acid. Streptococcus produces the clot buster enzyme streptokinase. It is used to remove clots from the blood vessels of patients who have myocardial infarction. 8. Option (a) is correct. Explanation: Cyclosporine A is produced by Trichoderma polysporum (fungus) and is used as an immuno-suppressive agent in organ transplant patients. Statins are produced by Monascus purpureus (yeast). It is used as blood-cholesterol lowering agents. It inhibits the enzymes responsible for synthesis of cholesterol. Acetobacter aceti is a bacterium that produces acetic acid. Methanobacterium is a common methanogen.

MICROBES IN HUMAN WELFARE

9. Option (c) is correct Explanation: Big holes in ‘Swiss cheese’ are made due to production of a large amount of CO2 by bacterium Propionibacterium sharmanii. It releases carbon dioxide when it consumes the lactic acid and forms bubbles. These bubbles form little air pockets, and is responsible for the production of large-holes in the cheese. 10. Option (d) is correct. Explanation: Methanogenic bacteria are anaerobic microorganisms that grow in the presence of carbon dioxide and produce methane gas. These bacteria are not found in activated sludge. The microbes present in the activated sludge are aerobic bacteria that grow rapidly and form flocs. 11. Option (b) is correct. Explanation : The approach to biological control includes Preservation of natural enemies(predators & parasites) that are already established in an area. The release of natural enemies (predators, parasites and pathogens) to control pests is a type of biological control called augmentation. This approach uses commercially available species that are applied in a timely manner to prevent population increases, or to suppress a pest population. Biological control using parasites is generally pestspecific. When multiple pests occur (e.g. aphids, thrips) natural enemies are needed for each pest. In cases where natural enemies are unavailable for augmentation, use of a selected pesticide that spares other natural enemies may be necessary. 12. Option (c) is correct. Explanation : Adult dragonflies are beneficial because they eat pest, flying insects, particularly midges and mosquitoes and help to decrease the probability of diseases spread by vectors. 13. Option (c) is correct. Explanation : Dragonflies are large and heavy-bodied insects. The larvae (or Nymphs), which live in water, eat almost anything living that is smaller than themselves. The adult dragonfly larvae are even known to catch and eat small fish. Usually, they eat bloodworms or other aquatic insect larvae. Dragonfly nymphs are such good hunters because they move really fast - by what looks like jet propulsion. One of the benefits of having dragonflies live nearby is that, they eat many times their weight in mosquitoes every day. Dragonflies also eat many different kinds of flies. Using a basket-like arrangement of their legs, adult dragonflies can actually catch and eat other bugs while they are flying. 14. Option (b) is correct. Explanation : Activated sludge micro-organisms need oxygen as they oxidise wastes to obtain energy for growth. Insufficient oxygen will slow down or kill off aerobic micro-organisms, make facultative organisms work less efficiently and ultimately lead to the breakage of flocs. It will also result in the production of the foul-smelling byproducts of anaerobic decomposition. So, sufficient oxygen must always be sustained in the aeration tank to ensure complete waste stabilisation. 15. Option (b) is correct. Explanation : Trichoderma is a free-living fungus which is common in soil and root eco-systems. It is an effective biocontrol agent and used extensively for soil-borne diseases. It has been also used successfully against many pathogenic fungi which belong to various genera, viz. Fusarium, Phytophthora and Scelerotia. It may suppress the growth of the disease-

117 causing organisms in the rhizosphere through competition and thus reduce disease development. It produces antibiotics and toxins, such as trichothecin and a sesquiterpine, and trichodermin, which have a direct effect on other organisms. 16. Option (a) is correct. Explanation : Sludge is a thick, soft and muddy deposits which can be treated by anaerobic digesters. Anaerobic digesters are large heated tanks in which micro-organisms break down biodegradable material in the absence of oxygen. It is then sent to oxidation ponds where heterotrophic bacteria continue the breakdown of the organic substances and solar ultraviolet (UV) light destroys the harmful bacteria. Chemicals are not used in this treatment. 17. Option (d) is correct. Explanation : Biochemical oxygen demand (BOD) is estimated by measuring the amount of oxygen consumption or is a measure of the organic pollution of water. It refers to the amount of dissolved oxygen required to decompose the organic matter in waste water. A high BOD indicates heavy pollution with little oxygen remaining for fish. 18. Option (b) is correct. Explanation : Trichoderma is a free-living fungus which is common in soil and root eco-systems. It is an effective biocontrol agent and used extensively for soil-borne diseases. It has been also used successfully against many pathogenic fungi which belong to various genera, viz. Fusarium, Phytophthora and Scelerotia. It may suppress the growth of the diseasecausing organisms in the rhizosphere through competition and thus reduce disease development. It produces anti-biotics and toxins, such as trichothecin and a sesquiterpine, and trichodermin, which have a direct effect on other organisms. 19. Option (c) is correct Explanation: Cheeses from Switzerland are renowned for its quality, its purity and its good flavour. Due to production of large amount of Co2 it has big holes . 20. Option (c) is correct. Explanation : Glomus is a genus of arbuscular mycorrhizal fungi and, all species form symbiotic relationships with plant roots. They help the plant in the absorption of nutrients, especially phosphorus from soil. 21. Option (a) is correct. Explanation : Bacillus thuringiensis is to control butterfly caterpillar. These are available in sachets as dried spores which are mixed with water and sprayed on to vulnerable plants such as brassica and fruit trees, where these are eaten by the insect larvae. In the gut of the larvae, the toxin is released and the larvae get killed. 22. Option (a) is correct. Explanation : Xanthomonas campestris is a bacterial species that causes a variety of plant diseases. 23. Option (c) is correct Explanation : Lactic acid bacteria or Lactobacillus converts milk into curd. A small amount of curd, which contains millions of LAB, is added to the fresh milk as inoculums or starter. Then these bacteria multiply at suitable temperature and microbes as Biocontrol Agents convert milk into curd, and thereby improve its nutritional quality by increasing the content of vitamin B12.

118 Oswaal CUET (UG) Chapterwise Question Bank 24. Option (a) is correct. Explanation : Sludge is a thick, soft and muddy deposits which can be treated by anaerobic digesters. Anaerobic digesters are large heated tanks in which micro-organisms break down biodegradable material in the absence of oxygen. It is then sent to oxidation ponds where heterotrophic bacteria continue the breakdown of the organics and solar ultraviolet (UV) light destroys the harmful bacteria. Chemicals are not used in this treatment. 25. Option (c) is correct. Explanation: Column A

Column B

Explanation

A

(iv)

Aspergillus niger produces citric acid.

B

(iii)

Acetobacter aceti produces acetic acid.

C

(ii)

Clostridium butylicum produces butyric acid.

D

(i)

Lactobaillus produces lactic acid.

26. Option (b) is correct Explanation : The primary treatment of waste water involves the physical removal of both larger and smaller particles (stable particles) from the sewage with the help of filtration and sedimentation. These removals occur in the following stages: (i) Initially, floating debris is removed by sequential filtration. (ii) After that grit (made of soil and small pebbles) are removed by sedimentation. All these solids particles that settled form the primary sludge and the supernatant forms the effluent. The effluent is taken for the secondary treatment. The primary treatment does not remove the dissolved impurities, toxic substances and harmful bacteria. 27. Option (d) is correct Explanation : Biochemical oxygen demand (BOD) is estimated by measuring the amount of oxygen consumption or is a measure of the organic pollution of water. It refers to the amount of dissolved oxygen required to decompose the organic matter in waste water. A high BOD indicates heavy pollution with little oxygen remaining for fish. 28. Option (d) ids correct. Explanation: Column A

Column B

Explanation

A

(iv)

Stantins are produced by the Monascus purpureus (a yeast). It helps to lower the cholesterol level in blood.

B

(iii)

Cyclosporine A, produced by the fungus Trichoderma polysporurn, is used as an immune suppressive agent in organ transplant patients.

C

(ii)

Streptokinase is produced by the bacterium Streptococcus. It is a thrombolytic medicine and used as a clot buster' for removing clots from the blood vessels of patients who have undergone myocardial infarction leading to heart attack.

D

(i)

BIOLOGY

Lipases are used in detergent formulation and are helpful in removing oily stains from the laundry.

29. Option (a) is correct. Explanation : Wine and beer are produced without distillation whereas whisky, brandy and rum are produced by distillation of the fermented broth. 30. Option (c) is correct Explanation : Biogas (commonly called gobar gas) is produced by using cattle dung. The technology of biogas production from cow dung was developed in India mainly due to the efforts of Indian Agricultural Research Institute (IARI) and Khadi and Village Industries Commission (KVIC). [B] ASSERTION & REASON: 1. Option (a) is correct. Explanation: The Lactobacillus, produce acids which coagulate and partially digest the milk proteins. An inoculum of curd contains millions of LAB, which convert milk to curd. 2. Option (d) is correct. Explanation: Statin is produced by a yeast called Monascus purpureus. It is a blood cholesterol lowering agent. 3. Option (c) is correct. Explanation: Activated sludge should have the ability to settle quickly so that it can be rapidly pumped back from sedimentation tank to aeration tank. 4. Option (a) is correct. Explanation: Cyanobacteria are autotrophic microbes that can fix atmospheric nitrogen. They are also important biofertilizers that add organic matter to the soil. 5. Option (a) is correct. Explanation: Glomus is a genus of arbuscular mycorrhizal and form a symbiotic association with plants. 6. Option (a) is correct Explanation: Rhizobium leguminosarum is symbiotic bacterium that serves as a biofertilizers. These bacteria fix atmospheric nitrogen into organic forms, which is used by the plant as nutrient. 7. Option (a) is correct. Explanation: Nucleopolyhedrovirus is from the genus of baculoviruses. They are useful, as they are good biocontrol agents. 8. Option (d) is correct. Explanation: Blue-green algae can proliferate in water and can carry out nitrogen fixation. But paddy is the only crop that grows in stagnant water. Most of the other crops do not need stagnant water to grow. This is the main reason of low popularity of blue-green algae as biofertilizer. 9. Option (a) is correct Explanation: Fermented toddy contains about 6 % alcohol. After 24 hours the toddy contains 4 to 5% acetic acid and it is unpalatable as a beverage. The fermentation of toddy is continued by naturally occurring yeasts.

119

MICROBES IN HUMAN WELFARE

10. Option (a) is correct. Explanation: Microorganisms such as Lactobacillus and others commonly called Lactic Acid Bacteria (LAB) grow it milk and convert into curd. During growth, the LAB produces acids that coagulate and partially digest the milk proteins and also check the growth of harmful microorganisms. [C] COMPETENCY/CASE-BASED QUESTIONS: 1. Option (d) is correct. Explanation : In logistic growth population seldom grows beyond the carrying capacity of ecosystem. 2. Option (d) is correct. Explanation : If any species is flourishing under unlimited resources, it would reach exponential growth which can be depicted by equation : dN/dt = rN Where, N= population density at time t; r = intrinsic rate of natural increase. If we derive the integral form of the exponential growth equation, it can be written as : Nt =N0ert Where, Nt, = population density after time t; No is population density at time zero; r = intrinsic rate of natural increase; e is the base of natural logarithm (2.71828). 3. Option (a) is correct. Explanation : The equation that correctly represents VerhulstPearl logistic growth is dN/dt = rN(K-N)/K. 4. Option (b) is correct. Explanation: ‘r’ represents Intrinsic rate of natural increase. 5. Option (c) is correct. Explanation: Since resources of growth for most animal populations are finite and become limiting sooner or later, so the logistic growth model is considered as more realistic.

24

6. Option (c) is correct. Explanation: The primary effluent is transferred into large aeration tanks where it is constantly agitated mechanically and the air having oxygen is pumped into it which allows fast growth of useful aerobic microbes into flocs (masses of bacteria associated with fungal filaments to form mesh-like structures). 7. Option (c) is correct. Explanation: The effluent after primary treatment is passed into the settling tank where the bacterial ‘flocs’ are allowed to settle down and this sediment is called Activated sludge. A small part of the activated sludge is pumped back into the aeration tank to serve as the inoculum(starter). 8. Option (a) is correct. Explanation : The activated sludge is pumped back into the aeration tank to serve as the inoculum(starter). 9. Option (a) is correct. Explanation : During this digestion, bacteria produce a mixture of gases such as methane, hydrogen sulphide, and carbon dioxide. These gases form biogas and can be used as a source of energy. The treated effluent from the secondary treatment plant is generally released into natural water bodies like rivers and streams. 10. Option (d) is correct Explanation: Sewage includes two stages: (i) Primary treatment or plıysical treatment: It involves physical removal of particles from the sewage through filtration and sedimentation. (ii) Secondary treatment or Biological treatment: The primary effluent is passed into large aeration tanks and constantly agitated to allow vigorous growth of useful aerobic microbes into flocs. Once the BOD of sewage water is reduced significantly, the effluent is then passed into a settling tank where the bacterial flocs are allowed to sediment. This sediment is called Activated sludge.

UNIT IV: Biotechnology and its Applications Study Time

CHAPTER

Max. Time: 1:5 Hours Max. Questions: 50

11

BIOTECHNOLOGY: PRINCIPLES AND PROCESSES The function of transferred DNA:

 Revision Notes

 Biotechnology deals with the techniques of using

live organisms or their enzymes for products and processes useful to humans. {  The

term biotechnology was given by Karl Ereky (1919). {  The European Federation of Biotechnology (EFB) defines Biotechnology as ‘the integration of natural science and organisms, cells, parts thereof, and molecular analogues for products and services’. Biotechnology deals with techniques of using live microorganisms, plant or animal cells or their components or enzymes from organisms to produce products and processes (services) useful to human beings.

Principles of Biotechnology



Scan to know

more about two core techniques of this topic modern biotechnology are: (a) Genetic engineering: The technique in which the genetic material (DNA and RNA) Biotechnologyis chemically altered and Principles & introduced into host organisms Processes to change the phenotype is known as genetic engineering. (b) Chemical engineering: It is necessary for chemical engineering processes to grow only the desired microbe / eukaryotic cell in large quantities for the manufacture of antibiotics, vaccines, enzymes, etc. {  Recombinant DNA technology involves combining DNA from two different organisms to generate a recombinant DNA (rDNA). { Recombinant DNA technology involves two basic steps and those are cutting or isolating and transferring or joining. { Stanley Cohen and Herbert Boyer (1972) constructed the first recombinant DNA. They isolated the antibiotic resistance gene by cutting out a piece of DNA from a plasmid. { The organism that contains an artificially inserted gene is known as a transgenic organism or genetically modified organism (GMO). {  The

If a piece of desired DNA is simply somehow transferred to another organism, it may just lie dormant in the cell of another organism. If this piece of desired DNA is integrated into the genome of the recipient organism, it would multiply and be inherited with the host DNA.

Steps in Genetically Modifying an Organism { There

are three basic steps in genetically modifying an organism: (a) Identification of DNA with desirable genes. (b) Introduction of the identified DNA into the host. (c) Maintenance of introduced DNA in the host and transfer of the DNA to its progeny. Tools of Recombinant DNA Technology Scan to know include: more about { Restriction Enzymes this topic { Polymerase enzymes { Ligases { Vectors { Host organisms Tools for Restriction Enzymes (Molecular Scis­ Recombinant DNA Technology sors): { Restriction enzymes belong to a larger class of enzymes called Nucleases. { There are two kinds; Exonucleases and Endonucleases. { Exonucleases remove nucleotides from the ends of the DNA whereas, endonucleases make cuts at specific positions within the DNA. For example, the first restriction endonuclease – Hind II, always cut DNA molecules at a particular point by recognizing a specific sequence of six base pairs. This specific base sequence is known as the Recognition Sequence for Hind II. { Each restriction endonuclease recognises a specific palindromic nucleotide sequence in the DNA. Palindromes are groups of letters that form the same words when read both forward and backwards for example “MALYALAM”. 5′ —— GAATTC —— 3′ 3′ —— CTTAAG —— 5′ The palindrome in DNA is a sequence of base pairs that reads the same on two stands when the orientation of the reading is kept the same.

BIOTECHNOLOGY : PRINCIPLES AND PROCESSES

BIOTECHNOLOGY: PRINCIPLES AND PROCESSES

121

199

122 Oswaal CUET (UG) Chapterwise Question Bank

BIOLOGY

Fig 1: Steps in formation of recombinant DNA by action of restriction endonuclease enzyme- EcoRI.

Separation and isolation of DNA fragments The fragment of DNA obtained by cutting DNA using a restriction enzyme is separated by a technique called gel electrophoresis. Negatively charged DNA fragments can be separated by forcing them to move towards the anode under an electric field through the medium. DNA fragments separate according to their size through the sieving effect provided by agarose gel. {  The separated DNA fragment can be visualized after staining the DNA with ethidium bromide followed by exposure to UV light. Separated bands of DNA are separated from agarose gel and extracted from the gel, called elution. The DNA fragment purified this way is used for recombination. Fig 2: Cloning vector pBR322 { Restriction enzymes cut the strand of DNA a little away

from the centre of the palindrome site between the same two bases on the opposite strands having sticky strand. The stickiness of the strands facilities the action of the enzyme DNA ligase. {  Restriction endonucleases are used in genetic engineering to form recombinant molecules of DNA which are composed of DNA from different sources or genomes. {  When cut the same restriction enzyme the resultant DNA fragments have the same kind of Sticky-ends and can be joined together using DNA ligases. Ligase Enzymes: These enzymes can catalyze the joining (ligation) of two molecules by forming a new chemical bond. This is typically via hydrolysis of a small pendant chemical group on one of the molecules, typically resulting in the formation of new C-O, C-S, or C-N bonds. Polymerase Enzymes: DNA polymerase is a specific class of enzymes found in all living organisms. Its main purpose is to replicate DNA and to help in the repair and maintenance of DNA. The enzyme is critical to the transmission of genetic information from generation to generation.

Fig 3 : A typhical Agrose gel electrophoresis showing migration of undigested

Cloning Vector: { Plasmids

and Bacteriophages is commonly used vector for cloning. {  They have ability to replicate within bacterial cells independent of the control of chromosomal DNA. { Bacteriophages because of their high number per cell, have very high copy numbers of their genome within the bacterial cells.

123

BIOTECHNOLOGY : PRINCIPLES AND PROCESSES

Following features are required to facilitate cloning into a vectora.  Origin of replication (ori) – the Scan to know sequence from where replication more about this topic starts and any piece of DNA when linked to this sequence can be made to replicate within the host cells. This sequence is responsible for controlling the copy number of the Vectors in linked DNA. Recombinant DNA Technology b.  Selectable marker helps in the identifying and eliminating non transformants and selectively permitting the growth of the transformants. Transformation is a procedure through which a piece of DNA is introduced in a host bacterium. Generally, the genes encoding resistance to antibiotics such as ampicillin, chloramphenicol, tetracycline or kanamycin, etc., are considered useful selectable markers for E. coli. c. Cloning sites – to link the foreign DNA, the vector need to have single recognition sites for the commonly used restriction enzymes as presence of more than one recognition sites within the vector will generate several fragments, which will complicate the gene cloning. The ligation of foreign DNA is carried out at a restriction site present in one of the two antibiotic resistance genes. The most efficient method of screening for the presence of recombinant plasmids is based on the principle that the cloned DNA fragment disrupts the coding sequence of a gene. This is termed as Insertional Inactivation. Vectors for cloning genes in plants and animals – Agrobacterium tumefacions (pathogen of dicot plant) is able to deliver a piece of DNA known as ‘T-DNA” to transform normal plant cells into a tumour and direct these tumour cells to produce the chemicals required by the pathogen. The tumour inducing (Ti) plasmid of Agrobacterium tumefaciens has been modified into cloning vector having no more pathogenic to plant.

Processes of Recombinant DNA Technology Recombinant DNA technology involves several steps in specific sequencea. Isolation of DNA. b. Fragmentation of DNA by restriction endonucleases. c. Isolation of a desired DNA fragment. d. Ligation of the DNA fragment into vector. e. Transforming the recombinant DNA into the host. f. Culturing the host cells in a medium at large scale. g. Extraction of the desired product. { Isolation of Genetic material: Genetic material is isolated from other macromolecules by using enzymes such as lysozyme (bacteria), cellulase (plant cells), and chitinase (fungus). DNA that separate out can be removed by spooling. The RNA can be removed by treatment with ribonuclease whereas proteins can be removed by treatment with protease. { Cutting of DNA at specific location is performed by using restriction enzyme and Agarose gel electrophoresis to check the progression of a restriction enzyme digestion. After cutting sources of DNA as well as vector DNA with a specific restriction enzyme to cut out ‘gene of interest’ from the source DNA.

Competent host (For Transformation with Recombinant DNA) {  The

chemical treatment with divalent calcium ions increases the efficiency of host cells (through cell wall pores) to take up the rDNA plasmids. {  rDNA can also be transformed into host cell by incubating both on ice, followed by placing them briefly at 42oC (Heat Shock), and then putting them back on ice. This enables the bacteria to take up the recombinant DNA. { In Micro-injection method, rDNA is directly injected into the nucleus of cells by using a glass micropipette. { Biolistics / Gene gun method, it has been developed to introduce rDNA into mainly plant cells by using a Gene / Particle gun. In this method, microscopic particles of gold / tungsten are coated with the DNA of interest and bombarded onto cells. The last method uses “Disarmed Pathogen” Vectors (Agrobacterium tumefaciens), which when allowed to infect the cell, transfer the recombinant DNA into the host.

Fig 4: Diagrammatic representation of recombinant-DNA technology { Amplification

of Gene of Interest using PCR (Polymerase Chain Reaction) to get multiple copies of the DNA or gene of interest in vitro by using set of primers and enzyme DNA polymerase.

Polymerase chain reaction (PCR) : Each cycle has three steps: (A) Denaturation; (B) Prim­ er annealing; and (C) Extension of primers

124 Oswaal CUET (UG) Chapterwise Question Bank This repeated amplification is done by the use of a thermostable DNA polymerase (isolated from a bacterium, Thermus aquaticus), which remain active during the high temperature induced denaturation of double stranded DNA. {  Insertion of Recombinant DNA into the Host Cell/ Organism includes making the recipient cells

BIOLOGY

competent to receive, take up DNA present in its surrounding etc. The recombinant DNA bearing gene for resistance to an antibiotic is transferred into E.coli cells, the host cell become transformed into ampicillinresistance cells.

Scan to know more about this topic

PCR - Polymerase Chain Reaction

Fig 5: Polymerase Chain reaction

Recovery of foreign gene product – { The

foreign DNA multiplies in plant or animal cell to produce desirable protein. Expression of foreign genes in host cells involve, optimized condition to obtain recombinant protein. {  The recombinant cell is multiplied in a continuous culture system in which used medium is drained out from one side while fresh medium is added from the other to maintain the cells in their physiological active phase.

{  A

bioreactor provides the optimal conditions for achieving the desired product by providing optimum growth conditions (temperature, pH, substrate, salts, vitamins, oxygen). { Downstream Processing involves processes that make the product obtain ready for marketing. This process includes separation and purification called as downstream processing. Suitable preservatives are added to it and send for clinical trial in case of drugs before releasing to market for public use

OBJECTIVE TYPE QUESTIONS [A] MULTIPLE CHOICE QUESTIONS: 1. The transfer of genetic material from one bacterium to another through the mediation of a vector like virus is termed as  [NCERT Exemplar, Q. 3, Pg. 75] (a) transduction. (b) conjugation. (c) transformation. (d) translation. 2. Which of the given statements is correct in the context of observing DNA separated by agarose gel electrophoresis? [NCERT Exemplar, Q. 4, Pg. 75]

(a) DNA can be seen in visible light. (b) DNA can be seen without staining in visible light. (c) Ethidium bromide stained DNA can be seen in visible light. (d) Ethidium bromide stained DNA can be seen under exposure to UV light. 3. ‘Restriction’ in Restriction enzyme refers to (a) cleaving of phosphodiester bond in DNA by the enzyme. [NCERT Exemplar, Q. 5, Pg. 76]

125

BIOTECHNOLOGY : PRINCIPLES AND PROCESSES

(b) cutting of DNA at specific position only. (c) prevention of the multiplication of bacteriophage in bacteria. (d) All of the above 4. In agarose gel electrophoresis, DNA molecules are separated on the basis of their [NCERT Exemplar, Q. 7, Pg. 76] (a) charge only. (b) size only. (c) charge to size ratio. (d) All of the above 5. While isolating DNA from bacteria, which of the following enzymes is not used? [NCERT Exemplar, Q. 9, Pg. 76] (a) Lysozyme (b) Ribonuclease (c) Deoxyribonuclease (d) Protease 6. Which of the following has popularised the PCR (polymerase chain reactions)? [NCERT Exemplar, Q. 10, Pg. 76] (a) Easy availability of DNA template (b) Availability of synthetic primers (c) Availability of cheap deoxyribonucleotides (d) Availability of ‘Thermostable’ DNA polymerase 7. An antibiotic resistance gene in a vector usually helps in the selection of [NCERT Exemplar, Q. 11, Pg. 77] (a) competent cells. (b) transformed cells. (c) recombinant cells. (d) none of the above 8. Significance of ‘heat shock’ method in bacterial transformation is to facilitate [NCERT Exemplar, Q. 12, Pg. 77] (a) binding of DNA to the cell wall. (b) uptake of DNA through membrane transport proteins. (c) uptake of DNA through transient pores in the bacterial cell wall. (d) expression of antibiotic resistance gene. 9. Which of the following bacteria is not a source of restriction endonuclease? [NCERT Exemplar, Q. 14, Pg. 77] (a) Haemophilus influenzae (b) Escherichia coli (c) Agrobacterium tumefaciens (d) Bacillius amyloli 10. Which of the following steps are catalysed by Taq polymerase in a PCR reaction?  [NCERT Exemplar, Q. 15, Pg. 77] (a) Denaturation of template DNA (b) Annealing of primers to template DNA (c) Extension of primer end on the template DNA (d) All of the above 11. The first instance of construction of an artificial recombinant DNA molecule was carried out on the plasmid of : (CUET 2023) (a) Bacillus thuringiensis (b) Escherichia coli (c) Agrobacterium tumifaciens (d) Salmonella typhimurium

12. Agrose, a gel used as matrix in gel electrophoresis is a natural polymer which is extracted from:  (CUET 2023) (a) Soyabean (b) Sea Weeds (c) Sea Anemone (d) Sea Corals 13. Match the features that are required to facilitate cloning of alien DNA into a vector (CUET 2022) Choose the correct answer form the options given below: List I Features to Facilitate

List-I I (Cloning Vector)

(a) Origin of replication (ori)

I. Agrobacterium tumefaciens

(b) Selectable Marker

II. Recognition sites commonly used for restriction enzymes

(c) Cloning sites

III. Helps in identifying and eliminating ­ non-transformants

(d) Vectors for cloning genes in plants

IV. S  equence from where replication starts

14. Recombinant Proteins are expressed in: (a) Cloning vector (b) Heterology Host (c) Homologous chromosomes (d) Promotor 15. How many cycles in a PCR will be required for amplifying the genetic material to about one billion times?  (CUET 2022) (a) 10 cycles (b) 15 cycles (c) 30 cycles (d) 35 cycles 16. In case of COVID positive patients, presence of corona virus is suspected only when the pathogen has produced a disease symptom. But when the symptoms of the disease has not appeared, the corona virus in the body can be detected by–  (CUET 2022) (a) Enzyme linked immuno-sorbent Assay (ELISA) only. (b) Recombinant DNA technology only. (c) Reverse transcriptase Polymerase Chain Reaction (RT-PCR). (d) Widal Test 17. Observe the given figure and name the step used in RecombinantDNA Technology (CUET 2022)

(a) Selecting (b) Scrolling (c) Spiraling (d) Spooling 18. Antibiotic resistance gene in a vector helps in the selection of:  (CUET 2023)

126 Oswaal CUET (UG) Chapterwise Question Bank (a) Recombinant cells (b) Transformed cells (c) Competent Cells (d) Totipotent cells 19. Restriction endonucleases are also called as molecular or biological scissors because:  (a) they cleave base pairs of DNA only at their terminal ends. (b) they cleave one or both the strands of DNA. (c) they act only on single stranded DNA. (d) none of these. 20. The role of DNA ligase in the construction of a recombinant DNA molecule is (a) formation of phosphodiester bond between two DNA fragments. (b) formation of hydrogen bonds between sticky ends of DNA fragments. (c) ligation of all purine and pyrimidine bases. (d) None of the above 21. A recombinant DNA molecule can be produced in the absence of the following (a) Restriction endonuclease (b) DNA ligase (c) DNA fragments (d) Escherichia coli 22. Which of the following steps are catalysed by Taq polymerase in a PCR reaction? (a) Denaturation of template DNA (b) Annealing of primers to template DNA (c) Extension of primer end on the template DNA (d) All of the above 23. PCR technique is best for : (a) DNA synthesis (b) Protein amplification (c) DNA amplification (d) DNA ligation. 24. The cutting of DNA at specific locations became possible with the discovery of : (a) Ligases (b) Restriction enzymes (c) Probes (d) Selectable markers. 25. Which of the following should be chosen for best yield if one were to produce a recombinant protein in large amounts? (a) Laboratory flask of largest capacity (b) A stirred-tank bioreactor without in-lets and outlets (c) A continuous culture system (d) Any of the above 26. Which of the following bacteria is not a source of restriction endonuclease? (a) Haemophilus influenzae (b) Escherichia coli (c) Agrobacterium tumefaciens (d) Bacillius amyloliquefaciens

BIOLOGY

27. Restriction’ in Restriction enzyme refers to  (a) cleaving of phosphodiester bond in DNA by the enzyme. (b) cutting of DNA at specific position only. (c) prevention of the multiplication of bacteriophage in bacteria. (d) all of the above 28. Which of the following statements does not hold true for restriction enzyme? (a) It recognises a palindromic nucleotide sequence. (b) It is an endonuclease. (c) It is isolated from viruses. (d) It produces the same kind of sticky ends in different DNA molecules. 29. The correct order of step in polymerase chain reaction (PCR) is : (a) Extension, Denaturation, Annealing (b) Denaturation, Annealing, Extension (c) Denaturation, Extension, Annealing (d) Annealing, Extension, Denaturation 30. Which among the following is not an application of PCR ?  (a) Gene amplification (b) Diagnosis of pathogens (c) DNA fingerprinting (d) In palaeontology. [B] ASSERTION & REASON: Directions: In the following questions, a statement of assertion (A) is followed by a statement of reason (R). Mark the correct choice as: (a) Both assertion (A) and reason (R) are true and reason (R) is the correct explanation of assertion (A). (b) Both assertion (A) and reason (R) are true but reason (R) is not the correct explanation of assertion (A). (c) Assertion (A) is true but reason (R) is false. (d) Assertion (A) is false but reason (R) is true. 1. Assertion (A): Any fragment of DNA, when linked to the ori region, can be initiated to replicate. Reason (R): Ori is a genetic sequence that acts as the initiation site for replication of DNA. 2. Assertion (A): EcoRI is restriction endonuclease enzyme. Reason (R): Exonuclease removes nucleotides from the ends of DNA. 3. Assertion (A): E. coli having pBR322 with DNA insert at BamHI site cannot grow in medium containing tetracycline.  Reason (R): Recognition site for BamHI is present in tetR region of pBR322. 4. Assertion (A): Thermus aquaticus, is used in PCR technique. Reason (R): It is a heat-stable DNA polymerase. 5. Assertion (A): Agarose gel electrophoresis is used to check the progression of a restriction enzyme digestion.

BIOTECHNOLOGY : PRINCIPLES AND PROCESSES

Reason (R): Restriction enzyme digestions are performed by incubating purified DNA molecules with restriction enzyme. 6. Assertion (A): A primer is a small segment of DNA that binds to a complementary strand of DNA.  Reason (R): Primers are necessary to stop the functioning of DNA polymerase enzyme and, therefore, are necessary in polymerase chain reaction. 7. Assertion (A): β-galactosidase coding sequence act as a selectable marker. Reason (R): Enzyme galactosidase converts the galactose into lactose. 8. Assertion (A): Foreign DNA and vector DNA cut with the help of ligase. Reason (R): Ligase act on sugar phosphate backbone of DNA. 9. Assertion (A): Restriction Endonuclease recognize the Palindromic nucleotide sequence Reason (R): Restriction Endonuclease are specific in their Action 10. Assertion: Genetic engineering requires both nucleases and ligases. Reason: Ligases produce the nick in the recombinant DNA molecule. [C] COMPETENCY/CASE-BASED QUESTIONS: I. Read the passage carefully and answer the question given after the passage from 1–5. Recombinant DNA technology is a technique that alters the phenotype of an entity (host) when a genetically modified vector is introduced and incorporated into the genome of the host. Thus, the process entails introducing a foreign fragment of DNA into the genome containing the desired gene. Unless the vector and source DNA are cut, fragments separated and joined, the desired recombinant vector molecule cannot be created. 1. The enzyme used to cut the DNA of interest and vector DNA during recombinant DNA technology is (a) Restriction Endonuclease (b) Ligase (c) Helicase (d) Cellulase 2. The technique used to separate the cut fragments. (a) DNA Replication (b) Splicing (c) Gel Electrophoresis (d) Elution 3. The process of separation of DNA fragments from Gel Electrophorsis is known as: (a) Downstream processes (b) Elution (c) Expression (d) None of the above 4. The transfer of genetic material from one bacterium to another through the mediation of a vector like virus is termed as (a) transduction. (b) conjugation. (c) transformation. (d) translation.

127 5. The Recombinant DNA technology also known as (a) Genetic Engineering: (b) Molecular Biology (c) Chemical Engineering (d) None of the above II. Study the following diagram and answer the question given below 6–10. Gel electrophoresis is a laboratory method used to separate mixtures of DNA, RNA, or proteins according to molecular size. In gel electrophoresis, the molecules to be separated are pushed by an electrical field through a gel that contains small pores. There are several basic steps to performing gel electrophoresis that will be described below: Pouring the gel, Preparing your samples, Loading the gel, Running the gel (exposing it to an electric field) and Staining the gel. Given below is the diagram of Gel Electrophoresis and answer the Q . no. 1 to 5 on the basis of it

Fig: A typhical Agrose gel electrophoresis showing migration of undigested ( lane 1and digested set of DNA fragments ( lane 2 to 4)

6. In agarose gel electrophoresis, DNA molecules are separated on the basis of their: (a) charge only. (b) size only (c) charge to size ratio. (d) all of the above 7. Agrose Gel is obtained from which class of organisms: (a) Bacteria (b) Fungi (c) Algae (d) Plant 8. Agarose is used in the case of DNA molecules because it has: (a) A greater range of separation (b) low resolving power (c) Both (a) and (b) (d) None of the above 9. Electrophoresis was developed by: (a) Tswett (b) Tsvedberg (c) Tiselius (d) Sanger 10. Which of the following factors does not influence electrophoretic mobility: (a) Molecular weight (b) Shape of molecule (c) Size of molecule (d) Stereochemistry of molecule

128 Oswaal CUET (UG) Chapterwise Question Bank

BIOLOGY

ANSWER KEY [A] MULTIPLE CHOICE QUESTIONS 1. (a)

2. (d)

3. (b)

4. (b)

5. (c)

6. (d)

7. (b)

8. (c)

9. (c)

10. (c)

11. (d)

12. (b)

13. (a)

14. (b)

15. (c)

16. (c)

17. (d)

18. (b)

19. (b)

20. (a)

21. (d)

22 (c)

23. (c)

24. (b)

25. (c)

26. (c)

27. (b)

28. (c)

29. (b)

30. (d)

8. (d)

9. (a)

10. (c)

8. (c)

9. (c)

[B] ASSERTION & REASON 1. (a)

2. (b)

3. (a)

4. (a)

5. (b)

6. (c)

7. (a)

[C] COMPETENCY/CASE-BASED QUESTIONS 1. (a)

2. (c)

3. (b)

4. (a)

5. (a)

6. (b)

7. (c)

1. (d)

ANSWERS WITH EXPLANATION [A] MULTIPLE CHOICE QUESTIONS: 1. Option (a) is correct Explanation: Transduction is the process by which genetic material (DNA) is transferred from one bacterium to another through the mediation of a vector, like virus. Bacterial conjugation is the process of transfer of genetic material (plasmid) between bacterial cells by direct cellto-cell contact or by a bridge-like connection between two cells. Transformation is the genetic alteration of a cell resulting from the direct uptake and ncorporation of exogenous genetic material (exogenous DNA) from its surroundings and taken up through the cell membranes. Translation is the process in which cellular ribosomes create proteins. It is a part of the process of gene expression. 2. Option (d) is correct Explanation: The separated DNA fragments (by the process of gel electrophoresis) are visualised after staining the DNA with ethidium bromide followed by exposure to ultraviolet (UV)-radiation. These fragments are seen as orange coloured bands. 3. Option (b) is correct Explanation: Restriction enzymes (also called molecular scissors) are responsible for cutting DNA. These enzymes belong to a class of enzymes called nucleases and are of two types : (i) Exonuclease which cut DNA at the ends and (ii) endonucleases which make cuts at specific positions within the DNA. The term ‘restriction’ refers to the function of these enzymes in restricting the propagation of foreign DNA of bacteriophage in host bacterium, that is, cutting of DNA, at a specific position only. 4. Option (b) is correct Explanation: In agarose gel electrophoresis, the DNA fragments separate out (resolve) according to their size or length because of the sieving property of agarose gel. It means, the smaller the fragment size, the farther it will move. 5. Option (c) is correct Explanation: Deoxyribonuclease enzyme is not used in the process of isolating DNA from bacteria as this enzyme causes the lysis of DNA molecules.

6. Option (d) is correct Explanation: The polymerase chain reaction (PCR) is a reaction in which amplification of specific DNA sequences is carried out in vitro. Such repeated amplification is achieved by the using thermostable DNA polymerase (isolated from a bacterium, Thermus aquaticus) enzyme which remains active and stable during high temperature and induced denaturation of double-standard DNA. 7. Option (b) is correct Explanation: Selectable markers help in identifying and eliminating non-transformants and selectively permitting the growth of the transformants. The normal E. coli cells do not carry resistance against any antibiotic. Competent bacterial cells are made capable to take foreign DNA with chemical treatment (e.g., calcium chloride). 8. Option (c) is correct Explanation : In chemical method, the cell is treated with specific concentration of a divalent cation such as calcium which increase the pore size in cell wall. The cells are incubated with recombinant DNA on ice, followed by placing them briefly at 42°C and then putting it back on ice. This is called heat shock method. The bacteria now take up these recombinant DNA. 9. Option (c) is correct. Explanation : Agrobacterium tumefaciens is a pathogen of several dicot plants. It delivers a piece of DNA known as T-DNA in the Ti plasmid which transforms normal plant cells into tumour cells to produce chemicals required by pathogens. The restriction enzyme Eco Rl, is isolated from E. coli RY13. The first restriction enzymes Hind II was isolated from bacterium Haemophilus influenzas. The restriction enzyme Bam HI is isolated from Bacillus amyloli. 10. Option (c) is correct Explanation: In polymerase chain reaction, polymerisation or extension step is catalysed by Taq polymerase enzyme. PCR is carried out in the following three steps (i) Denaturation : The double-stranded DNA denatured by applying high temperature of 95°C for 15 seconds. Each separated single strand now acts as template for DNA synthesis.

129

BIOTECHNOLOGY : PRINCIPLES AND PROCESSES

(ii) Annealing : Two sets of primers are added which anneal to the three ends of each separated strand. Primers act as initiators of replication. (iii) Extension : DNA polymerase extends the primers by adding nucleotides complementary to the template provided in the reaction. A thermostable DNA polymerase (Taq DNA polymerase) is used in the reaction which can tolerate the high temperature of the reaction. All these steps are repeated many times to obtain several copies of desired DNA. 11. Option (d) is correct Explantation: Stanley and Herbert Boyer first isolate the plasmid from the bacteria Salmonella typhimurium and construction of an artificial recombinant DNA molecule. 12. Option (b) is correct Explantation: Seaweeds are a good source of nutrients such as proteins, vitamins, minerals, and dietary fiber. Polyphenols, polysaccharides, and sterols, as well as other bioactive molecules, are mainly responsible for the healthy properties associated with seaweed. 13. Option (a) is correct Explanation: Origin of the replication (ori) is the sequence of DNA at which replication starts on a chromosome, plasmid or virus. Selectable markers in vector help in identifying and eliminating nontransformants and selectively permitting the growth of the transformants. Cloning sites are the recognition sites commonly used for restriction enzymes. 14. Option (b) is correct . Explanation: Recombinant proteins are expressed in heterologous host. The hosts which have foreign genes are called heterologous host. 15. Option (c) is correct Explanation: If the process of DNA replication is repeated many times, the segment of DNA can be amplified to approximately billion times, i.e., 1 billion copies are made at the end of 30 PCR cycles. It is possible to generate 2n molecules after 'n' number of cycles. 16. Option (c) is correct Explanation: As in RT-PCR, RNA of retrovirus is reverse transcripted with the help of enzyme reverse transcriptase. DNA formed is amplified with the help of PCR. So, a minute quantity of corona virus can also be detected with the help of RT-PCR. 17. Option (d) is correct Explanation: Spooling which is a method of extraction of substance like DNA in the form of a spool over a glass rod. 18. Option (b) is correct Explantation: Antibiotic resistance gene in a vector helps in the selection of transformed cells. 19. Option (b) is correct. Explanation: Restriction endonucleases naturally target double stranded DNA and could cleave one or both the strands of the DNA.

20. Option (a) is correct. Explanation: DNA ligase (joining or sealing enzymes) are also called genetic gum. They join two individual fragments of double-stranded DNA by forming phosphodiester bonds between them. Thus, they help in sealing gaps in DNA fragments. Therefore, they act as a molecular glue. 21. Option (d) is correct Explanation: A recombinant DNA molecule can be produced in the absence of Escherichia coli. Restriction enzymes and DNA ligases can be used to make a stable recombinant DNA molecule, with DNA fragments that have been spliced together from two different organisms. 22. Option (c) is correct. Explanation: In polymerase chain reaction, polymerisation or extension step is catalysed by Taq polymerase enzyme. PCR is carried out in the following three steps (i) Denaturation : The double-stranded DNA is denatured by applying high temperature of 95°C–97°C for 15 seconds. Each separated single strand now acts as template for DNA synthesis. (ii) Annealing : Two sets of primers are added which anneal to the three ends of each separated strand. Primers act as initiators of replication. (iii) Extension : DNA polymerase extends the primers by adding nucleotides complementary to the template provided in the reaction. A thermostable DNA polymerase (Taq DNA polymerase) is used in the reaction which can tolerate the high temperature of the reaction. All these steps are repeated many times to obtain several copies of desired DNA. 23. Option (c) is correct. Explanation: PCR techniques is helpful to detect very minute traces of virus or bacterial DNA and other multiple copies by DNA amplification. 24. Option (b) is correct. Explanation: Restriction enzymes are the molecular scissors that cuts the DNA from specific recognition site. 25. Option (c) is correct. Explanation : If any protein encoding gene is expressed in a heterologous host, it is called recombinant protein. The cells harbouring cloned genes of interest may be grown on a small scale in the laboratory. The cultures may be used for extracting the desired protein and then purifying it by using different separation techniques. The cells can also be multiplied in a continuous culture system where the used medium is drained out from one side while fresh medium is added from the other to maintain the cells in their physiologically most active log/exponential phase. This type of culturing method produces a larger biomass leading to higher yields of desired protein. 26. Option (c) is correct. Explanation: Agrobacterium tumefaciens is a pathogen of several dicot plants. It delivers a piece of DNA known as T-DNA in the Ti plasmid which transforms normal plant cells into tumour cells to produce chemicals re¬quired by pathogens. The restriction enzyme EcoRl, is isolated from E. coli RY13. The first restriction enzymes Hind

130 Oswaal CUET (UG) Chapterwise Question Bank II was isolated from bacterium Haemophilus influenzae. The restriction enzyme Bam HI is isolated from Bacillus amyloliquefaciens. 27. Option (b) is correct. Explanation: Restriction enzymes (also called molecular scissors) are responsible for cutting DNA. These enzymes belong to a class of enzymes called nucleases and are of two types : (i) Exonuclease which cut DNA at the ends and (ii) endonucleases which make cuts at specific positions within the DNA. 28. Option(c) is correct. Explanation: Restriction enzymes are a protein produced by bacteria that cleaves DNA at specific sites. These are not found in viruses. They are present in bacteria to provide a type of defence mechanism (called the restriction modification system) against bacterial viruses. They are of two types endonuclease and exonuclease. They are indispensable tools in recombinant DNA technology and genetic engineering. 29. Option (b) is correct. Explanation : A single PCR amplification cycle involves three basic steps : denaturation, annealing and extension. PCR stands for polymerase chain reaction in which multiple copies of the gene, for DNA of interest is synthesised in vitro. 30. Option (d) is correct. Polymerase Chain Reaction (PCR) is a powerful molecular biology technique used for amplifying DNA fragments. It has a wide range of applications, including gene amplification, diagnosis of pathogens, and DNA fingerprinting. However, it is not commonly used in paleontology. In paleontology, scientists often work with ancient DNA extracted from fossils or preserved remains of extinct organisms. The DNA in such samples is typically highly degraded and fragmented due to the passage of time. PCR is not an ideal technique for working with ancient DNA because it requires relatively long and intact DNA strands for successful amplification. In paleontology, alternative methods such as DNA sequencing techniques designed for short and degraded DNA fragments are typically used to study ancient DNA. [B] ASSERTION & REASON: 1. Option (a) is correct. Explantation: The process of DNA replication begins at the ori sequence. The presence of ori in a DNA fragment makes it a self-replicating molecule. 2. Option (b) is correct. Explanation: Restriction Endonuclease enzyme which cut the DNA at specific base sequences is both endonucl Ease as well as exonuclease . These enzymes are known as molecular Scissors or biological scissors. 3. Option (a) is correct. Explanation: pBR322 carries recognition sites for number of commonly used restriction enzymes. Recognition site for BamHI is present in tetR region i.e., region responsible for tetracyline resistance. When an

BIOLOGY

insert is added at the BamHI recognition site the gene for tetracycline resistance becomes non-functional and the recombinant bacteria with plasmid pBR322 that has DNA insert at BamHI lose tetracycline resistance. 4. Option (a) is correct. Explanation: Thermus aquaticus, is the source of DNA polymerase because it is a heat-stable DNA polymerase which is used in PCR technique. 5. Option (b) is correct. Explanation: In agarose gel electrophoresis, DNAs fragments cut by restriction enzyme can be arranged according to their sizes. 6. Option (c) is correct. Explanation: A primer is a small segment of DNA that binds to a complementary strand of DNA. Primers are necessary to start the functioning of DNA polymerase enzyme and, therefore, are necessary in polymerase chain reaction. 7. Option (a) is correct. Explanation: Galactosidase is an enzyme that converts the galactose into lactose. This property makes this enzyme to be used as a selectable marker or reporter gene in molecular biology experiments. This property is exploited during the selection of recombinants from the non-recombinants. 8. Option (d) is correct. Explantation: In formation of rDNA, restriction endonucleases cut both foreign DNA and vector DNA and act on sugar phosphate backbone of DNA. 9. Option (a) is correct. Explantation: Restriction Endonuclease bind to the DNA and cut each of the two strands of DNA helix by recognizing the Palindromic sequence. 10. Option (c) is correct Explantation: Nucleases are the enzymes that remove nucleotides or produce nick in the DNA strand. Exonucleases remove nucleotides from the free ends of DNA while endonucleases produce internal nick in DNA. Now, the desired gene is inserted and the cut ends are sealed with the help of DNA ligase. Ligases are also called molecular glue as they join together two strands by forming phosphodiester bonds between adjacent nucleotides. [C] COMPETENCY/CASE-BASED QUESTIONS: 1. Option (a) is correct Explantation: Restriction enzymes cut the strand of DNA a little away from the centre of the palindrome site between the same two bases on the opposite strands in the DNA of interest and Vector DNA. Restriction enzymes are also known as molecular scissors or biological scissors . 2. Option (c) is correct Explantation: Gel electrophoresis is a process of separating various small molecules based on their size and charge. Gel electrophoresis works on the principle of difference in the electric charge of molecules.

131

BIOTECHNOLOGY : PRINCIPLES AND PROCESSES

3. Option (b) is correct.

6. Option (b) is correct

Explantation: The orange coloured bands of DNA are separated From Agrose gel and extracted from the gel piece . 4. Option (a) is correct Explanation: Transduction is the process by which genetic material (DNA) is transferred from one bacterium to another through the mediation of a vector, like virus. Bacterial conjugation is the process of transfer of genetic material (plasmid) between bacterial cells by direct cellto-cell contact or by a bridge-like connection between two cells. Transformation is the genetic alteration of a cell resulting from the direct uptake and in corporation of exogenous genetic material (exogenous DNA) from its surroundings and taken up through the cell membranes. Translation is the process in which cellular ribosomes create proteins. It is a part of the process of gene expression. 5. Option (a) is correct

Explanation: In agarose gel electrophoresis, the DNA fragments separate out (resolve) according to their size or length because of the sieving property of agarose gel. It means, the smaller the fragment size, the farther it will move. 7. Option (c) is correct. Explantation: Agrose gel is obtained from sea weeds which belong to Rhodophyceae class of Alga . 8. Option (c) is correct. Explantation: Agrose permit the formation of bigger pores that can be to dissolve bigger molecules. Agrose gel exhibit greater range of separation and low resolving power. 9. Option ( c) is correct. Explanation: Tiselius developed the electrophoresis technique to separate the DNA fragments. 10. Option (d) is correct

Explantation: Genetic engineering: The technique in which the genetic material (DNA and RNA) is chemically altered and introduced into host organisms to change the phenotype is known as genetic engineering.

24

Explanation: The stereochemistry of molecule won’t have any effect on electrophoretic mobility since it is dependent on velocity and intensity and not alignment.

Study Time

CHAPTER

12

Max. Time: 1:50 Hours Max. Questions: 50

BIOTECHNOLOGY AND ITS APPLICATIONS

 Revision Notes:

Biotechnology deals with industrial-scale production of biopharmaceuticals and biological using genetically modified microbes, fungi, plants and animals. Its application includes therapeutics, diagnostics, genetically modified crops for agriculture, processed food, bioremediation, waste treatment and energy production. The main three critical research areas of biotechnology include –

{ Providing

the best catalyst in the form of an improved organism usually a microbe or pure enzyme. { Creating optimal conditions through engineering for a catalyst to act. {  Downstream processing technologies to purify the protein or organic compounds. Scan to know more about this topic

Applications of Biotechnology

{ Biotechnological

Applications in Agriculture- Food production can be increased by { Agro-chemical based agriculture { Organic agriculture { Genetically engineered crop-based agriculture.  Green Revolution successfully increased food production many folds by using better management practices and the use of agrochemicals, fertilizers and pesticides. Further increase in production is not possible by using these methods. To overcome this genetically modified crop is used.  Plants, bacteria, fungi and animals whose genes have been altered by manipulation are called Genetically Modified Organisms (GMOs). GM plants have many applications Made crops more tolerant of abiotic stresses Reduced reliance on chemical pesticides Helped to reduce post-harvest losses Increased efficiency of mineral usage by plants  Enhanced nutritional value of food, e.g., Vitamin ‘A’ enriched rice.

Application of Biotechnology in the production of pest-resistant plants– Pest-resistant plants decrease the amount of pesticides used. Bt toxin is produced by a bacterium called Bacillus thuringiensis. Bt toxin gene has been cloned from the bacteria and been expressed in plants to provide resistance to insects without the need for insecticides; in effect created a bio-pesticide. Examples are Bt cotton, Bt corn, rice, tomato, potato and soybean etc Bt cotton– Bacterium Bacillus thuringiensis produces proteins that kill certain insects like lepidopterans, coleopterans (beetles) and dipterans (flies, mosquitoes). B. thuringiensis produce crystals that contain a toxic insecticidal protein. This toxic protein is present in the bacterium as inactive protoxins but as soon as an insect ingests the inactive form due to the alkaline pH of the gut, it converts into an active form of toxin and binds to the surface of midgut epithelial cells and creates pores that cause cell swelling and lysis and eventually death of the insect.  The gene from B. thuringiensis has been incorporated into several crop plants like cotton,

BIOTECHNOLOGY AND ITS APPLICATIONS BIOTECHNOLOGY AND ITS APPLICATIONS

133

221

134 Oswaal CUET (UG) Chapterwise Question Bank maize, rice etc. The toxin is coded by a gene named cry. The protein coded by the genes crylAb and cryIIAb control the cotton bollworms, and cryIAb controls corn borer.

Pest Resistant Plants Nematodes

like Meloidegyne incognitia infects the roots of tobacco plants and causes a reduction in yield. The infestation of these nematodes can be prevented by the process of RNA interference (RNAi).  RNAi is present in all eukaryotic organisms as cellular defence by silencing of specific mRNA due to complementary dsRNA molecules that bind to and prevent translation of the mRNA.  The source of complementary dsRNA may be from infection by viruses having RNA genomes or mobile genetic elements that replicate through RNA intermediate. Nematode-specific genes were introduced into host plants using Agrobacterium vectors. The parasite could not survive in a transgenic host expressing specific interfering RNA.

Biotechnological Applications in Medicine :

{ The rDNA technological processes have made immense

impact in the area of healthcare by enabling mass production of safe and more effective therapeutic drugs. {  At present, about 30 recombinant therapeutics have been approved for human use the world over. In India, 12 of these are presently being marketed.

Genetically Engineered Insulin:

Adult –onset diabetes can be controlled by taking insulin at regular intervals. The main source of this insulin was isolation of insulin from animals. Now a day’s insulin can be obtained from bacteria using techniques of biotechnology. {  Insulin was earlier extracted from the pancreas of slaughtered cattle and pigs but insulin from these sources develops allergy or other types of reactions to the foreign protein. { Insulin consists of two short polypeptide chains- chain A and chain B, that are linked together by disulphide bridges.

is absent in mature insulin. The main challenge for the production of insulin using the rDNA technique was getting insulin assembled into a mature form. { An American company, Eli Lilly in 1983 prepared two DNA sequences corresponding to the A and B chain of human insulin and introduced them in plasmids of E.coli to produce insulin chain. Chain A and Chain B were produced separately, extracted and combined by creating disulphide bonds to form human insulin.

Gene Therapy It is a collection of methods that allows the correction of a gene defect that has been diagnosed in a child or embryo. This method is applied in a person with a hereditary disease. In this method, genes are inserted into a person’s cells and tissues to treat a disease. { The correction of gene defect involves the delivery of a normal gene into the individual or embryo to take over the function of and compensate for non-functional genes. { The first clinical gene therapy was done in 1990 on a 4 year old girl with adenosine deaminase (ADA) deficiency. This disorder is caused due to the deletion of the gene for adenosine deaminase which is essential for the immune system to function. This defect can be treated by enzyme replacement therapy in which functional ADA is given to the patient by injection or bone marrow transplant. {  In gene therapy method lymphocytes from the blood of the patient are grown in a culture medium outside the body. A functional ADA cDNA is then introduced into these lymphocytes and returned to the patient. In this method, periodic infusion of such genetically engineered lymphocytes is needed. If a gene isolated from bone marrow cells producing ADA is introduced into cells at early embryonic stages, it could be a permanent cure.

{

Fig. 1: Genetically Engineered Insulin

Maturation Of Proinsulin into insulin: {  In

humans, insulin is synthesised as a prohormone, which contains an extra stretch called C peptide, which

BIOLOGY

Fig 2: Flow diagram on Gene therapy

135

BIOTECHNOLOGY AND ITS APPLICATIONS

 Molecular Diagnosis Conventional method of diagnosis such as serum or urine analysis is not able to early detection of disease-causing pathogens or virus. The following methods can be used to diagnose earlier I. Recombinant DNA technology II. Polymerase Chain Reaction (PCR) III. Enzyme Linked Immuno-sorbent Assay (ELISA). {  Symptoms of the disease appear only when the concentration of the pathogen increased significantly. Low concentrations of bacteria and viruses can be detected by amplification of nucleic acid by PCR. It detects the mutation in the gene in cancer patients. PCR is routinely used to detect HIV in suspected AIDS patients. Genetic disorders can be also detected by using the PCR technique. {  A single-stranded DNA or RNA having radioactive molecules is allowed to hybridise with its complementary DNA in a clone of cells followed by detection using autoradiography. The clone having the mutated gene will not appear on the photographic film. {  ELISA is based on the principle of antigen-antibody interaction. Infection by pathogen can be detected by the presence of antigens like proteins, glycoproteins etc. or by detecting the antibodies synthesised against the pathogen.

Transgenic Animals

Animals that have had their DNA manipulated to possess and express a foreign gene are known as transgenic animals. Transgenic mice, rats, rabbits, pigs, sheep, cows and fish have been produced. Common reasons for the development of transgenic animals{ Normal physiology and development– they are designed to allow the study of gene regulation and their effect on the normal function of the body. By introducing genes from other species that alter the formation of this factor and studying eh biological affects that result. { Study of disease– a number of transgenic animals are designed to increase our understanding of how genes contribute to the development of disease. A transgenic model has been developed for diseases like cancer, cystic fibrosis, Alzheimer’s disease etc. { Biological products–Transgenic animals that produce useful biological products can be created by

the introduction of the portion of DNA (gene) which codes for a particular product such as human protein (alpha – 1-antitrypsin) used to treat emphysema. The first transgenic cow, Rosie, produced human proteinenriched milk (alpha-lactalbumin – 2.4 gm/litre). { Chemical safety testing– transgenic animals are made that carry genes which make them more sensitive to toxic substances than non-transgenic animals. It gives us the results in less time. { Vaccine safety– transgenic mice are developed for used in testing the safety of vaccines before they are used on humans. The Polio vaccine was tested on transgenic mice and then on monkeys.

Ethical Issues: The Indian Government has set up organizations such as GEAC (Genetic Engineering Approval Committee), which will make decisions regarding the validity of GM research and the safety of introducing GM-organisms for public services. Biopatent: A patent is a right granted by a government to an inventor to prevent others from making commercial use of his invention. Now, patents are granted for biological entities and for products derived from biological resources. Biopiracy: It is the term used to refer to the use Scan to know of bio-resources by multinational more about this topic companies and other organizations without proper authorization from the countries and people concerned without compensatory payment. Biopiracy in { An American company in 1977 Biotechnology got patent rights on Basmati rice through the US Patent and Trademark Office. This allowed the company to sell a ‘new variety of Basmati, in the US and abroad. This ‘new’ variety of Basmati had actually been derived from Indian farmer’s varieties. Indian Basmati was crossed with semi-dwarf varieties and claimed as an invention or a novelty. {  Many attempts have also been made to patent uses, products and processes based on Indian traditional herbal medicines, e.g., Neem and turmeric.

OBJECTIVE TYPE QUESTIONS [A] MULTIPLE CHOICE QUESTIONS: 1. Bt cotton is not: [NCERT Exemplar, Q. 1, Pg. 82] (a) a genetically modified (GM) plant. (b) insect resistant. (c) a bacterial gene expressing system. (d) resistant to all pesticides. 2. C-peptide of human insulin is: [NCERT Exemplar, Q. 2, Pg. 82] (a) a part of mature insulin molecule. (b) responsible for formation of disulphide bridges. (c) removed during maturation of pro-insulin to

insulin. (d) responsible for its biological activity. 3. α -1 antitrypsin is: [NCERT Exemplar, Q. 4, Pg. 82] (a) an antacid. (b) an enzyme (c) used to treat arthritis. (d) used to treat emphysema. 4. A probe which is a molecule used to locate specific sequences in a mixture of DNA or RNA molecules could be: [NCERT Exemplar, Q. 5, Pg. 83] (a) a single-stranded RNA. (b) a single stranded DNA.

136 Oswaal CUET (UG) Chapterwise Question Bank (c) either RNA or DNA. (d) can be ss-DNA but not ss RNA. 5. The site of production of ADA in the body is: [NCERT Exemplar, Q. 7, Pg. 83] (a) bone marrow. (b) lymphocytes. (c) blood plasma. (d) monocytes. 6. Pathophysiology is the: [NCERT Exemplar, Q. 9, Pg. 83] (a) study of physiology of a pathogen. (b) study of normal physiology of the host. (c) study of altered physiology of the host. (d) none of the above. 7. The trigger for activation of toxin of Bacillus thuringiensis is: [NCERT Exemplar, Q. 1, Pg. 83] (a) acidic pH of stomach. (b) high temperature. (c) alkaline pH of gut. (d) mechanical action in the insect gut. 8. Golden rice is: [NCERT Exemplar, Q. 11, Pg. 84] (a) a variety of rice grown along the yellow river in china. (b) long stored rice having yellow colour tint. (c) a transgenic rice having gene for β-carotene . (d) wild variety of rice with yellow coloured grains. 9. In RNAi, genes are silenced using: [NCERT Exemplar, Q. 13, Pg. 84] (a) ss DNA. (b) ds DNA. (c) ds RNA. (d) ss RNA. 10. The first clinical gene therapy was done for the treatment of: (CUET 2022) (a) AIDS. (b) cancer. (c) cystic fibrosis. (d) SCID (Severe Combined Immuno Deficiency resulting from deficiency of ADA). 11. The protein formed by the encoded gene expression in a heterologous host is called:  (CUET 2022) (a) Structural protein (b) Recombinant protein (c) Transposons (d) Prohormone 12. The inactive protoxin gets converted into an active form due to the:  (CUET 2022) (a) Alkaline pH of insect's gut (b) Temperature and acidic pH of gut (c) Exposure to light (d) Exposure to light and acidic pH of gut 13. Common term used to refer the use of bioresources by multinational companies and other organisations without proper authorisation and compensatory payment to the countries and people concerned is:  (CUET 2023) (a) Bioinfomatics (b) Biopiracy (c) Biopatenting (d) Biological theft

BIOLOGY

14. Stirred-tank bioreactors have been designed for: (CUET 2022) (a) ensuring anaerobic conditions in culture vessel (b) purification of product (c) addition of preservatives to product (d) availability of oxygen throughout the process 15. During gene therapy, which vector is used to introduce functional ADA cDNA into lymphocyte: (CUET 2022) (a) Plasmid (b) Bacteriophage (c) pBR322 (d) Retrovirus 16. Which of the following genes control corn borer disease?  (CUET 2022) (a) ampR (b) cry I Ac (c) cry I Ab (d) cry II Ab 17. Select the statement that are correct for GM plants with genetic modifications.  (CUET 2022) (a) Made crops tolerant to abiotic stresses. (b) Increased the reliance on chemical pesticides. (c) Helped to reduce post-harvest losses. (d) Decrease efficiency of mineral usage by plants. (e) Enhanced nutritional value of food. Choose the correct answer from the options given below: (a) c and e only (b) b and c only (c) a, c and e only (d) c, d and e only 23 17. The Bt toxin acts on ______ of insect pest: (a) Hind gut (b) Nervous system (c) Reproductive system (d) Midgut 18. The process of RNA interference (RNAi) was used in plant to develop resistance against: (a) Viruses (b) Fungi (c) Nematodes (d) Insects 19. C-peptide of human insulin is:  (a) a part of mature insulin molecule. (b) responsible for formation of disulphide bridges. (c) removed during maturation of pro-insulin to insulin. (d) responsible for its biological activity. 20. In 1990, the first clinical gene therapy was administered to a 4 years old girl with enzyme deficiency of? (a) Adenosine deaminase (ADA) (b) Tyrosine oxidase (c) Monoamine oxidase (d) Glutamate dehydrogenase 21. Silencing of a gene could be achieved through the use of: (a) short interfering RNA (RNAi). (b) antisense RNA. (c) by both (1) and (2). (d) none of the above. 22. A transgenic crop of the future, golden rice has which of the following improved traits? (a) High lysine (essential amino acid) content (b) Insect resistance (c) High protein content (d) High vitamin-A content

BIOTECHNOLOGY AND ITS APPLICATIONS

23. ADA is an enzyme which is deficient in a genetic disorder SCID. What is the full form of ADA: (a) Adenosine deoxyaminase (b) Adenosine deaminase (c) Aspartate deaminase (d) Arginine deaminase 24. What does ‘Bt’ in Bt toxin represent ? (a) Bioterrorism (b) Blue tooth toxin (c) Bleeding toxin (d) Bacillus thuringiensis 25. Transgenic animals are (a) Animals whose DNA is manipulated to possess and express an extra (foreign) gene (b) Animals whose RNA is manipulated to possess and express an extra (foreign) gene (c) Animals whose both DNA and RNA are manip¬ulated to possess and express an extra (foreign) gene. (d) None of the above 26. What are the characteristics of Bt crops (a) Insect Resistance (b) Fungal resistance (c) Drought Resistance (d) All the above 27. A biotechnologist wanted to create a colony of E.coli possessing the plasmid pBR322, sensitive to tetracycline. Which one of the following restriction sites would he use to ligate a foreign DNA? (a) Sal I (b) Pvu I (c) EcoRI (d) Hind III 28. Some strains of Bacillus thuringiensis can kill certain insects such as _______ (a) lepidopterans (b) scorpion (c) fruit fly (d) dragonfly 29. Which of the following is commonly used as a vector for introducing a DNA fragment in human lymphocytes? (a) Retrovirus (b) pBR 322 (c) Lambda phage (d) Ti plasmid 30. Identify which of the following is not an advantage of GM crops? (a) GM plants enhance nutritional value of food. (b) GM plants are more tolerant to abiotic stresses. (c) GM plants have helped to reduce post-harvest losses. (d) GM plants can cause gene transfer to non-target plant species. [B] ASSERTION & REASON: Direction: reason (R). Mark the correct choice as: (A) Both assertion (A) and reason (R) are true and reason (R) is the correct explanation of assertion (A). (B) Both Assertion (A) and Reason (R) are true but Reason (R) is not the correct explanation of Assertion (A). (C) Assertion (A) is true but reason (R) is false. (D) Assertion (A) is false but reason (R) is true. 1. Assertion (A): Norman E. Borlaug was the father of “Green revolution”. Reason (R): Borlaug recommended improved methods

137 of cultivation, and developed a robust strain of wheat dwarf wheat - that was adapted to Mexican conditions. 2. Assertion (A): ‘Bt’ in Bt toxin represent Bacillus thuringiensis. Reason (R) : ‘Bt’ toxin is produced by a virus. 3. Assertion (A): Genetical modification make crops more tolerant to abiotic stress (cold, drought etc.) Reason (R) : Genetically Modified (GMO) genes have been altered by manipulation. 4. Assertion (A): The inactive form of toxic proteins are advantageous to the bacteria producing it. Reason (R) : Low concentration of a bacteria or virus, even at a time when the symptoms of the disease are not visible, can be detected. 5. Assertion (A): Plants with exact desired traits can be created by GE method. Reason (R) : Any gene from an organism can be inserted into another plant by GE method. 6. Assertion (A): Transgenic animals had their DNA manipulated. Reason (R) : 95% of all existing transgenic animals are mice. 7. Assertion (A): GEAC will decide the safety of introducing GM organism for public use. Reason (R): Genetic modifications of organisms may have opposite results when introduced into the ecosystem. 8. Assertion (A): Transgenic mice are being used to test the safety of the polio vaccine. Reason (R) : It could replace the use of monkeys to test the safety of batches of the vaccine. 9. Assertion (A): Rosie was the first transgenic cow. Reason (R) : It contained human alpha lactalbumin. 10. Assertion (A): An American company was allowed to sell a ‘new’ variety of Basmati rice. Reason (R) : Indian Basmati was crossed with semidwarf varieties and claimed as a novelty. [C] COMPETENCY/CASE-BASED QUESTIONS: I. Read the passage carefully and answer the question given after the passage from 1–5. Gene therapy is an attempt to cure a hereditary disease which is present from birth. Gene therapy is a collection of methods that allows correction of a gene defect that has been diagnosed in a child/embryo. In gene therpay genes are inserted into a person’s cells and tissues to treat a disease. Correction of a genetic defect involves delivery of a normal gene into the individual or embryo to take over the function of and compensate for the non-functional gene. The first clinical gene therapy was given in 1990 to a 4-year old girl with adenosine deaminase (ADA) deficiency. This enzyme is crucial for the immune system to function. The disorder is caused due to the deletion of the gene for adenosine deaminase. In some children ADA deficiency can be cured by bone marrow transplantation; in others it can be treated by enzyme replacement therapy, in which functional ADA is given to the patient by injection. But the problem with both of these approaches that they are not completely curative. As a first step towards gene therapy, lymphocytes from the blood of the patient are

138 Oswaal CUET (UG) Chapterwise Question Bank

1.

2.

3.

4.

5.

II.

grown in a culture outside the body. A functional ADA cDNA (using a retroviral vector) is then introduced into these lymphocytes, which are subsequently returned to the patient. The patient requires periodic infusion of such genetically engineered lymphocytes as these cells are not immortal. For permanent cure the gene isolate from marrow cells producing ADA is introduced into cells at early embryonic stages. In which year, first clinical gene therapy was given? (a) In 1902 (b) In 1990 (c) In 1940 (d) In 2010 In the Gene therapy, genes are inserted into _____of the person. (a) Cell (b) Tissue (c) Both (a) and (b) (d) None of these The ADA deficiency causes due to deletion of: (a) Therapy (b) Gene (c) Retrovirus (d) Mitochondria If a person is having hereditary disease since birth then treatment involves: (a) Gene therapy (b) Radiotherapy (c) Chemotherapy (d) Antibiotic intake The ADA deficiency in the children is cured by: (a) Bone marrow Transplantation (b) By taking protein rich diet (c) By taking Antibiotics (d) None of above Study the following diagram and answer the question given below 6–10. Bacillus thuringiensis bacteria produce Bt toxin. Bacterium Bacillus thuringiensis produce proteins that kill certain insects like lepidopterens, colepterans (beetels) and dipterans (flies, mosquitoes). B. thuringiensis produce crystals that contain a toxic insecticidal protein. This toxic protein present in bacterium as inactive protoxins but as soon as insect ingest the inactive form due to alkaline pH of gut, it converted into an active form of toxin and bind to surface of midgut epithelial cells and create pores that cause cell swelling and lysis and eventually death of insect. The gene from B. thuringiensis has been incorporated into several crop plants like cotton, maize, rice etc. The toxin is coded by a gene named cry. The protein coded by the genes crylAb and cryIIAb control the cotton bollworms, cryIAb controls corn borer.

BIOLOGY

(A) Larvae ingest Bt spores and cry proteins

(B) In larval midgut, proteolytic digestion of proteins release cry toxins which binds to epithelial receptors.

(C) Toxin binding causes cell lysis destroying barriers to body cavity. 6. B. thuringiensis forms protein crystals during a particular phase of their growth. These crystals contain a toxic. (a) Proteolytic enzymes (b) Chitin (c) Insecticidal protein (d) Flavoprotein 7. Bt toxin exists as inactive protoxins, it is converted into an active form of toxin due to: (a) Enzymes in insect’s gut. (b) Acidic pH of the insect’s gut. (c) Binding with collagen in insect’s gut (d) Alkaline pH of the insect’s gut. 36 8. The proteins encoded by the genes cry control: (a) Corn borers (b) Cotton bollworms (c) Mosquitoes (d) Nematodes 9. Bacillus thuringiensis is a: (a) air borne bacteria. (b) soil-borne fungus (c) soil borne bacteria (d) food borne bacteria 10. Stem cells can be obtained (a) from embryo (b) from placenta (c) from body parts (d) from an embryo or tissues in the body.

ANSWER KEY [A] MULTIPLE CHOICE QUESTIONS 1. (d)

2. (c)

3. (d)

4. (c)

5. (b)

6. (c)

7. (c)

8. (c)

9. (c)

10. (d)

11. (b)

12. (a)

13. (b)

14. (a)

15. (d)

16. (c)

17. (c)

18. (d)

19. (c)

20. (c)

21. (c)

22 (c)

23. (b)

24. (d)

25. (a)

26. (d)

27. (a)

28. (a)

29. (a)

30. (c)

8. (a)

9. (a)

10. (b)

8. (b)

9. (c)

[B] ASSERTION & REASON 1. (a)

2. (c)

3. (a)

4. (b)

5. (b)

6. (b)

7. (a)

[C] COMPETENCY/CASE-BASED QUESTIONS 1. (b)

2. (b, c)

3. (b)

4. (a)

5. (a)

6. (c)

7. (d)

10. (d)

139

BIOTECHNOLOGY AND ITS APPLICATIONS

ANSWERS WITH EXPLANATION [A] MULTIPLE CHOICE QUESTIONS: 1. Option (d) is correct. Explanation: Bt cotton is a genetically modified (GM) plant which is resistant to insects. These plants are produced by the insertion of one or more genes from a common soil bacterium, Bacillus thuringiensis. These genes encode for the production of insecticidal proteins, and thus, genetically transformed plants produce one or more toxins as they grow 2. Option (c) is correct Explanation: Insulin and C-peptide are linked when first made by the pancreas. C-peptide is removed from the pancreatic beta-cells during cleavage of insulin from pro-insulin. 3. Option (d) is correct Explanation: Transgenic animals that produce useful biological products can be created by the introduction of the portion of DNA (or genes) which codes for a particular product such as human protein (α-1-antitrypsin) used to treat emphysema. 4. Option (c) is correct Explanation: A single-stranded DNA or RNA, tagged with a radioactive molecule (probe) is allowed to hybridise with its complementary DNA in a clone of cells followed by detection using autoradiography. 5. Option (b) is correct Explanation: The first clinical gene therapy was given in 1990 to a 4-year-old girl with adenosine deaminase (ADA) deficiency. This enzyme is crucial for the immune system to function. The disorder is caused due to the deletion of the gene for adenosine deaminase. In some children, ADA deficiency can be cured by bone marrow transplantation. 6. Option (c) is correct Explanation: Study of altered physiology of host is called pathophysiology. 7. Option (c) is correct Explanation: Bt toxin is produced by a bacterium called Bacillus thuringiensis (Bt). The Bt toxin protein exist as inactive pro-toxins, but once an insect ingests the inactive toxin, it is converted into an active form of toxin due to the alkaline pH of the gut which solubilise the crystals. 8. Option (c) is correct Explanation: Golden Rice is a new type of rice that contains beta-carotene (provitamin A), which is converted into vitamin A as needed by the body and gives the grain its golden colour. It is developed through genetic engineering and produces two new enzymes that complete the beta-carotene expression in the rice grain.

the year 1990 to a 4-year old girl with ADA (Adenosine deaminase) deficiency. ADA deficiency causes SCID (Severe Combined Immuno Deficiency) in which B-lymphocytes and T-lymphocytes are not formed. The ADA gene provides instructions for producing the enzyme adenosine aminase. This enzyme is produced in all cells, but the highest levels of adenosine deaminase occur in immune system cells called lymphocytes. The absence of ADA results in a dysfunctional immune system due to build-up of toxic metabolites. 11. Option (b) is correct Explanation: Structural proteins are the most abundant class of proteins in nature and form structural elements. Transposons are repetitive DNA sequences that have the capacity to move from one location to another in genome and prohormones are the precursors of hormone s. Therefore, Option (B) is the correct answer as any protein coding gene is expressed in a heterologous host is called recombinant protein. 12. Option (a) is correct. Explanation: The Bt toxin is produced by the bacterium Bacillus thuringiensis through the process of sporulation. The Bt toxin protein exist as inactive protoxins but once an insect ingests the inactive toxin, it is converted into an active form of toxin due to alkaline pH of the gut which solubilize the crystals. 13. Option (b) is correct Explanation: Biopiracy is the practice of commercial exploitation of biochemicals or genetic materials which occur naturally. Typically, indigenous people have traditional cognition primarily consisting of biological features and genetic diversity of the natural environment from one generation to another. 14. Option (d) is correct. Explanation: The stirred tank bioreactor is well suited for large scale production of micro or­ganism under aseptic condition for a number of days. It can be used easily in research labora­ tory and main advantage is an oxygen delivery system which provides oxygen without any interruption. 15. Option (d) is correct Explanation: The retrovirus is used as a vector to introduce functional ADA cDNA into the lymphocytes, which are subsequently returned to the patient. 16. Option (c) is correct Explanation: Cry gene codes for the Bt toxin. The proteins encoded by the genes cry I Ab control corn borer. 17. Option (c) is correct

Explanation: RNAi takes place in all eukaryotic organisms as a method of cellular defence. This method involves the silencing of a specific mRNA due to a complementary dsRNA molecule that binds to and prevents translation of the mRNA (silencing).

Explanation: GM is a technology that involves inserting DNA into the genome of an organism. To produce a GM plant new DNA is transferred into plant cells. The aim is to introduce a new trait to the plant which does not occur naturally in the species. E.g., in food crops, it include resistance to certain pests, made crops tolerant to abiotic stresses, reduction of spoilage or improving the nutrients content of the crop.

10. Option (d) is correct

18. Option (d) is correct

Explanation: First clinical gene therapy was given in

Explanation: The activated Bt toxin binds to the surface of

9. Option (c) is correct

140 Oswaal CUET (UG) Chapterwise Question Bank midgut epithelial cells and creates pores that cause cell swelling and lysis and eventually cause death of the insect. 19. Option (c) is correct. Explanation: RNA interference (RNAi) is a key biological process in plants to inhibit gene expression both transcriptionally and post-transcriptionally, using three different groups of proteins to resist the virulence of pathogens. 20. Option (d) is correct. Explanation: Cryl. endotoxin obtained from Bacillus thuringiensis is effective against bollworms. A bollworm is a common term for any larva of moth that attacks the fruiting bodies of certain crops, especially cotton. 21. Option (c) is correct.

BIOLOGY

29. Option (a) is correct. Explanation: Retrovirus is commonly used as a vector for introducing a DNA fragment in human lymphocytes. 30. Option (d) is correct Explanation: The protein coded by genes cryIAc and cryIIAb control the cotton bollworms. The protein coded by gene cryIAb controls corn borer. [B] ASSERTION & REASONS: 1. Option (a) is correct. Explanation: Green revolution is the period refers to a process that increases the production of food grains. Prof. Norman E. Borlang developed high - yielding varieties of wheat resistant to disease like rust.

Explanation: RNAi takes place in all eukaryotic organisms as a method of cellular defense. This method involves silencing of a specific mRNA due to a complementary dsRNA molecule that binds to and prevents translation of the mRNA (silencing). The source of this complementary RNA could be from an infection by viruses having RNA genomes or mobile genetic elements (transposons) that replicate via an RNA intermediate.

2. Option (c) is correct

22. Option (c) is correct.

Explanation: Genetically Modified Organisms (GMO) or transgenic organisms are the plants, bacteria, fungi and animals whose genes are altered by manipulation. GMO in plants makes crops more tolerant to abiotic stresses (cold, drought, salt, heat, etc). It helps to reduce post-harvest losses and increases the efficiency of mineral usage by plants. It also enhances the nutritional value of food.

Explanation: Golden rice is a genetically improved variety of rice. It contains more amount of beta-carotene than normal rice varieties. It increases the nutritive value of rice since betacarotene is converted into vitamin A, which is essential for good vision. The rice grains here, are yellow in color due to the presence of beta-carotene. Hence, it is known as golden rice. 23. Option (b) is correct. Explanation: The ADA gene provides instructions for producing the enzyme adenosine deaminase. This enzyme is produced in all cells, but the highest levels of adenosine deaminase occur in immune system cells called lymphocytes, which develop in lymphoid tissues. 24. Option (d) is correct Explanation: Bt toxin is a substance which can be harmful to other organisms and is synthesized a plant species.Bt represents Bacillus thuringiensis that is a bacteria that helps some insects to make toxic proteins. 25. Option (a) is correct. Explanation: Transgenic animals are animals whose DNA is manipulated to possess and express an extra (foreign) gene e.g., Rosie - transgenic cow. 26. Option (d) is correct. Explanation: Bt cotton crops are fungal resistant, insect resistant and drought resistant. 27. Option (a) is correct. Explanation: Sal I is a restriction endonuclease used for molecular biology methods to cleave DNA at the recognition sequence 5′-G/TCGAC-3′, generating fragments with 5′-cohesive ends. 28. Option (a) is correct. Explanation: Bt toxin kills certain insects such as lepidopterans (tobacco budworm, armyworm), coleopterans (beetles) and dipterans (flies, mosquitoes).

Explanation: Bt toxin is produced by a bacterium called Bacillus thuringiensis (Bt). The Bt toxin protein exist as inactive pro-toxins, but once an insect ingests the inactive toxin, it is converted into an active form of toxin due to the alkaline pH of the gut which solubilise the crystals. 3. Option (a) is correct.

4. Option (b) is correct. Explanation: Many proteins including certain toxins are secreted in their inactive form. They get activated, only when exposed to a specific trigger (pH, temperature, etc.). It is advantageous to the bacteria producing it because the bacterium does not get killed due to the action of protein. 5. Option (b) is correct. Explanation: Modern genetic engineering helps to insert any gene from an organism chemically synthesised into another plant. Since only transgenic gene is added so exact desired traits can be created. 6. Option (b) is correct. Explanation: Transgenic animals originates from recombinant DNA technology. 7. Option (a) is correct. Explanation: The Indian Government has set up organisations such as GEAC (Genetic Engineering Approval Committee), which will make decisions regarding the validity of GM research and the safety of introducing GM organisms for public services. 8. Option (a) is correct. Explanation: Transgenic mice are being used to test the safety of the polio vaccine. If successful and found to be reliable, they could replace the use of monkeys to test the safety of batches of the vaccine. 9. Option (a) is correct. Explanation: The milk of the first transgenic cow Rosie contains human alpha lactalbumin and nutrionally a more balanced product for human beings than natural cow milk.

141

BIOTECHNOLOGY AND ITS APPLICATIONS

10. Option (b) is correct.

5. Option (a) is correct

Explanation: In 1997, an American company got patent rights on Basmati rice through the US Patent and Trademark Office. This allowed the company to sell a ‘new’ variety of Basmati which had actually been derived from Indian farmer’s varieties. Indian Basmati was crossed with semi-dwarf varieties and claimed as a novelty.

Explanation: Bone marrow transplantation is the way to cure ADA deficiency in the children.

[C] COMPETENCY/CASE-BASED QUESTIONS:

7. Option (d) is correct

1. Option (b) is correct Explanation: The first clinical gene therapy was given in 1990 to a 4-year old girl with adenosine deaminase (ADA) deficiency. This enzyme is crucial for the immune system to function. The disorder is caused due to the deletion of the gene for adenosine deaminase. 2. Option (c) Both (a) and (b) are correct Explanation: In gene therapy genes are inserted into a person’s cells and tissues to treat a disease. Correction of a genetic defect involves delivery of a normal gene into the individual or embryo to take over the function of and compensate for the non-functional gene. 3. Option (b) is correct Explanation: Adenosine deaminase (ADA) deficiency due to deletion of the ADA gene promoter and first exon by homologous recombination between two Alu elements. 4. Option (a) is correct Explanation: Gene therapy is used to cure hereditary diseases.

24

6. Option (c) is correct Explanation: Bacillus thuringeinsis forms a toxic protein crystals during a particular phase of their growth. These crystals contain a toxic insecticidal protein. Explanation: Bt toxin proteins are present in inactive protoxin form, but become active toxin in the alkaline pH of insect gut. This activated toxin binds to the surface of midgut epithelial cells and creates pores that cause cell swelling and lysis and eventually cause death of the insect. 8. Option (b) is correct Explanation: The protein coded by genes cryIAc and cryIIAb control the cotton bollworms. 9. Option (c) is correct. Explanation : Bt is a microbe naturally found in soil. It makes proteins that are toxic to immature insects (larvae). There are many types of Bt and each target different insect groups. 1 0. Option (d) is correct Explanation: Stem cells can be obtained from an embryo, or by using a specialized body cells (developed by a technique), which behave like embryonic stem cells. These cells are known as induced pluripotent stem cells (IPS cells).

UNIT-V: Ecology and Environment Study Time Max. Time: 1:5 Hours Max. Questions: 50

CHAPTER

13

 Revision Notes

 Habitat: A habitat is the place where an organism lives.  Niche: A niche defines a specific role played by organisms in an ecosystem, i.e. profession.  Ecological niche term was given by Grinnel.

ORGANISMS & ENVIRONMENT Scan to know more about this topic

Habitat & Niche

Characteristics of a Population { Population: A population is defined as a group of individuals of the same species that live in a particular geographical area at a particular time and function as a unit. { A population has several parameters of which the following are the most essential: (i) The number and kind of individuals of a species. (ii) Given area or space. (iii) Time in terms of day, month or year. { A population has certain attributes that an individual organism does not have. For example, individuals may have births and deaths, but a population has birth rates and death rates. { A population at a given time is composed of different individuals of different ages. If the age distribution is plotted for the population, the resulting structure is called age pyramid. The shape of pyramid reflects the shape of the growth status of a population. { Population size or population density (N) is measured in terms of number. { Population Growth: The size of the population is not static. It keeps changing with time, depending upon food and other factors. The main factors that determine population growth are: (i) Natality (number of births) [B] (ii) Mortality (number of deaths) [D] (iii) Immigration (individuals that come into habitat) [I] (iv) Emigration (individuals that leave the habitat) [E] If ‘N’ is population density at time ‘t’, then its density at t +1 is N(t + 1) = Nt + [(B + I) – (D + E)  Growth model: Growth of population takes place

according to the availability of food, habitat condition and presence of other biotic and abiotic factors. There are two main types of models: (i) Exponential Growth: This kind of growth occurs when food and space are available in a sufficient amount. The

population grows exponentially or geometrically. If the size of a population is N, the birth rate is represented as ‘b’ and death rate as ‘d’, then increase and decrease in N during a unit period time ‘t’ will be dN/dt = (b – d) x N Let, (b – d) = r. Then, dN/dt = rN The r in this equation is called ‘intrinsic rate of natural increase’. (ii) Logistic Growth: There is a competition between the individuals of a population for food and space. The ‘fittest’ organism survives and reproduces. This type of growth initially shows a lag phase followed by phases of acceleration and deceleration. K indicates the carrying capacity of the population. K − N  dN /dt = rN   K  Where, N = Population density at time t r =Intrinsic rate of natural increase k = Carrying capacity { Carrying capacity: It is the maximum number of individuals which an environment can support or sustain. It is represented by a constant ‘K’. The population tends to stabilise around the carrying capacity. {  Exponential growth has J-shaped curve and Logistic growth has S-shaped curve or Sigmoid curve. { Homeostasis: The term homeostasis was coined in 1930 by the physician Walter Cannon. { Biome is a major complex biotic community which is characterised by distinctive plant and animal species and maintained under the climatic conditions of the region. { Homeostasis is applied to the tendency for biological systems to resist change and to remain in a state of equilibrium. Scan to know

more about Ecological adaptations: this topic {  An ecological adaptation is a physiological, behavioural or morphological character of an organism that allows it to Ecological survive and reproduce in its adaptations natural environment. {  On the basis of various modes of adaptations, plants are grouped as xerophytes (grow in dry habitat), hydrophytes (grow in water), heliophytes (grow in bright light), halophytes (grow in saline habitat), and sciophytes (grow in partial shades).

143

Envir

onment

ORGANISMS & ENVIRONMENT

144 Oswaal CUET (UG) Chapterwise Question Bank

Population interaction: Scan to know more about {  When various species live this topic together in a community, number of interactions takes place according to their specific needs for food, shelter and Population habits. These interactions can be Interaction of two main types. 1. Intraspecific interactions. 2. Intraspecific interactions. The interspecific interactions are of two types: (a) Cooperative interactions (b) Competitive interactions. {  All animals, plants and microbes in a biological community interact with each other. These interactions may be beneficial, detrimental or neutral to one species or both. The following types of interactions are seen: (a) Predation: It is the interaction between two species members in which the members of one species capture, kill and eat up the members of other species. (b) Parasitism: It is the relationship between two living organisms of different species in which one organism called a parasite obtains its food directly from another living organism called the host. (c) Proto-cooperation: It is the interaction between two living organisms of different species in which both are mutually benefitted but they can live without each other. (d) Competition: It is the rivalry between two or more organisms for obtaining the same resources. (e) Mutualism: It is the interaction between two organisms of different species where both the partners are benefitted but cannot live separately. (f) Amensalism: It is the interaction between two organisms where one is harmed and the other remains unaffected. Population Attributes: Scan to know Population attributes are the more about parameters that help us to study this topic the population. These attributes are measured for a population, not for an organism. There are five essential attributes of a population, Population namely, Natality Rate, Mortality Attributes rate, Sex ratio, Age distribution and Population Density. { Natality rate: It can be defined as the number of births per capita per unit time. { Mortality rate: It can be defined as the number of deaths per capita per unit of time. { Sex ratio: Individuals can be either male or female but the population has a sex ratio. The sex ratio is defined as the number of males to females in a population.

{ Age

Distribution: It refers to the proportion of individuals in different age groups within a population. It helps in understanding the dependency ratio, workforce distribution and potential social and economic implications. We can divide a population into certain categories by taking age as a criterion. A population consists of different age groups as follows: { Pre-reproductive age group: It contains individuals 0 to 14 years of age. { The reproductive age group: It contains individuals 14 to 50 years of age who are reproductively active.. { Post-reproductive age group: It contains individuals more than 50 years of age. { Expanding population pyramid: This pyramid belongs to poor, less developed countries where health facilities are not good and family planning program is not implemented. In an expanding population, the number of individuals in pre-reproductive stage is more than that in reproductive stage. The number of individuals of reproductive age is more than that of post reproductive age. The shape of the expanding population pyramid does not change with time whereas the size of the pyramid increases with time. { Stabilising population pyramid: This pyramid belongs to developing countries like India. Here, the number of individuals in pre-reproductive age is equal to that in reproductive age group. The number of individuals of reproductive age is more than that of post reproductive age. The pyramid is urn-shaped. The shape and size of the pyramid in stabilising population pyramid do not change with time. { Declining population pyramid: This pyramid belongs to highly developed countries. In a declining population, the number of individuals in pre-reproductive age and post reproductive age is less than that in reproductive age. This pyramid is spindle shaped.

Expanding

K

a

Stable

Declining

dN dt = rN

(N)

and behavioral adaptation to environmental variation and stress conditions in animals. occurs in the form of migration, hibernation, mimicry, echolocation, and prevention of freezing.

Population density

{  Physiological

BIOLOGY

b

dN dt = rN

( ) K–N K

Time (t)

(a) When responses are not limiting the growth, plot is exponential. (b) When responses are limiting the growth, plot is logistic. It produces sigmoid curve (S-shaped). K is carrying capacity.

145

ORGANISMS & ENVIRONMENT

OBJECTIVE TYPE QUESTIONS [A] MULTIPLE CHOICE QUESTIONS: 1. A protozoan reproduces by binary fission. What will be the number of protozoans in its population after six generations?  [NCERT Exemplar, Q. 6, Pg. 164] (a) 128 (b) 24 (c) 64 (d) 32 2. What do South American lakes visiting flamingos and resident fishes compete for? (a) Phytoplankton (b) Small fishes (c) Water (d) Zooplankton 3. Heliophytes are: (CUET 2021) (a) salt-loving plants (b) sun-loving plants (c) shade-loving plants (d) water-loving plants 4. What is the phenomenon called in which two species competing for the same resources can avoid competition by choosing? (a) Resource structuring (b) Competition (c) Predation (d) Resource partitioning 5. A kind of population interaction in which one species benefits and the other is neither harmed nor benefited? (a) Commensalism (b) Ammensalism (c) Mutualism (d) Parasitism 6. A population has more young individuals compared to the older individuals. What would be the status of the population after some years? (CUET 2022) (a) It will decline. (b) It will stabilise. (c) It will increase. (d) It will first decline and then stabilise. 7. Which of the following would necessarily decrease the density of a population in a given habitat? (a) Natality > mortality (b) Immigration > emigration (c) Mortality and emigration (d) Natality and immigration 8. Ecological niche is: (a) the surface area of the ocean. (b) an ecologically adapted zone. (c) the physical position and functional role of a species within the community. (d) formed of all plants and animals living at the bottom of a lake 9. Which of the following is a partial root parasite? (a) Sandal wood (b) Mistletoe (c) Orobanche (d) Ganoderma 10. Calculate the birth rate if there were 60 sparrows in a garden last year and through reproduction 24 new sparrows are added this year:  (CUET 2022) (a) 0.4 offsprings per sparrow per year (b) 0.2 offsprings per sparrow per year (c) 0.6 offsprings per sparrow per year (d) 0.1 offsprings per sparrow per year

11. What parameters are used for tiger census in our country’s national parks and sanctuaries? (a) Pug marks only. (b) Pug marks and faecal pellets. (c) Faecal pellets only. (d) Actual head counts. 12. In which condition the prey might become extinct? (a) When the predator is less (b) When the prey is overexploited (c) When the prey is less (d) When the predator is overexploited 13. Select the statement which explains best parasitism. (a) One organism is benefited (b) Both the organisms are benefited (c) One organism is benefited, other is not affected (d) One organism is benefited, other is affected. 14. Ecotone is: (a) a polluted area. (b) the bottom of a lake. (c) a zone of transition between two communities. (d) a zone of developing community. 15. What does population density mean? (a) The number of animals and plants present in a given area (b) The number of individuals in a unit area in a unit time (c) The concentration of human population (d) The number of animals and plants present in a given area 16. Swathi was growing a bacterial colony in a culture flask under ideal laboratory conditions where the resources are replenished. Which of the following equations will represent the growth in this case (Where population size is N, birth rate is b, death rate is d, unit time period is t, and carrying capacity is K). (a) dN/dt = KN (b) dN/dt = r N(K-N/K (c) DN/DT = RN (d) dN/dt = r N(K+N/K) 17. What is an adaptation for climbing and balancing called? (a) Fossorial (b) Arboreal (c) Cursorial (d) Terrestrial 18. Amensalism is an association between two species where: (a) One species is harmed and other is benefitted. (b) One species is harmed and other is unaffected. (c) One species is benefitted and other is unaffected. (d) Both the species are harmed. 19. Which of the following attribute does a population have? (a) Births (b) Birth rate (c) Deaths (d) Pyramid 20. If 100 deaths take place in a population of 10,00,000 per year, then the death rate will be:  (CUET 2022) (a) 0.001 per person per year (b) 0.0001 per person per year (c) 0.1 per person per year (d) 1 per person per year

146 Oswaal CUET (UG) Chapterwise Question Bank 21. What will happen to a well growing herbaceous plant in the forest if it is transplanted outside the forest in a park? (a) It will grow normally. (b) It will grow well because it is planted in the same locality. (c) It may not survive because of change in its microclimate. (d) It grows very well because the plant gets more sunlight. 22. Density of population tells us about: (CUET 2022) (a) total number of individuals of a species (b) total area occupied by a species (c) umber of individuals present per unit space in a given time (d) population growth in a particular time span. 23. What kind of interaction is mutualism? (a) Negative Interspecific (b) Positive Intraspecific (c) Negative Intraspecific (d) Positive Interspecific 24. A population has more young individuals compared to the older individuals. What would be the status of the population after some years? (a) It will decline (b) It will stabilise (c) It will increase (d) It will first decline and then stabilise 25. (p+q)2 = p2 + 2pq + q2 = 1 represents an equation used in: (a) population genetics. (b) Mendelian genetics. (c) biometrics. (d) molecular genetics. 26. What is a relationship in which one parasite feeds on another parasite which in turn feeds on its host called? (a) Pathogen (b) Vector (c) Vehicle (d) Epi-parasite 27. Which is an example of a parasite, in which the host species is closely related to the parasite and is a member of the same family or genus? (a) Endo-parasite (b) Klepto-parasite (c) Adelpho-parasite (d) Ecto-parasite 28. Which of the following forest plants controls the light conditions at the ground? (a) Lianas and climbers (b) Shrubs (c) Tall trees (d) Herbs 29. What is a mutualistic relationship between a fungus and photosynthesizing algae called? (a) Mycorrhiza (b) Lichens (c) Cyanobacteria (d) Sponges 30. Where do ectoparasites reside? (a) Within the blood (b) In the intercellular spaces of host (c) Within the cells (d) On the surface of the host [B] ASSERTION & REASON:

Directions: In the following questions, a statement of assertion (a) is followed by a statement of reason (R). Mark the correct choice as:

(A) Both assertion (a) and reason (R) are true and reason (R) is the correct explanation of assertion (A).

BIOLOGY

(B) Both assertion (a) and reason (R) are true but reason (R) is not the correct explanation of assertion (A). (C) Assertion (a) is true but reason (R) is false. (D) Assertion (a) is false but reason (R) is true. 1. Assertion: The human population represents the logistic growth curve. Reason: When the resources are limited leading to competition between individuals and survival of fittest, the population tends to grow in a logistic manner. 2. Assertion: Daphnia populations in a water body, at different seasons of a year showed marked variations in their body morphology. Reason: Cyclomorphosis in some organisms is influenced by the variations in temperatures prevailing in their waterbodies at different seasons. 3. Assertion (A): Leaf butterfly and stick insect show mimicry to dodge their enemies. Reason (R): Mimicry is a method to acquire body colour blending with the surroundings. 4. Assertion: Clown fish maintains a commensalistic relation with the sea anemone. Reason: In this interaction, one species benefits and the other is neither harmed nor benefited. 5. Assertion (A): Ecological equivalents possess similar types of adaptations. Reason (R): Ecological niche is the total interaction of a species with environment. 6. Assertion (A): The term interbreeding implies sexual reproduction. Reason (R): Interbreeding result in variation among species. 7. Assertion: Species are groups of potentially interbreeding natural populations which are isolated from other such groups. Reason: Distinctive morphological characters are displayed due to reproductive isolation.  8. Assertion (A): Population has certain attributes which an individual organism does not have. Reason (R): Population is a group of people living in well defined area, at a particular time. 9. Assertion (A): “The Biological Species” concept helps us to ask how species are formed. Reason (R): The concept of biological species focuses our attention on the question of how reproductive isolation comes about. 10. Assertion (A): Natality contributes to the increase in population density. Reason (R): Natality refers to the number of births during a given period in the population which is added to the initial density. [C] COMPETENCY/CASE-BASED QUESTIONS: I. Read the passage carefully and answer the question given after the passage from 1–5. Growth of a population with time shows specific and predictable patterns. Two types of growth pattern of population are exponential and logistic growth. When resources in the habitat are unlimited each species has

147

ORGANISMS & ENVIRONMENT

the ability to realise fully its innate potential to grow in number. Then the population grows in exponential fashion. When the resources are limited growth curve shows an initial slow rate and then it accelerates and finally slows giving the growth curve which is sigmoid. 1. Identify the incorrect statement: (a) Exponential growth occurs in organism such as lemmings. (b) Logistic growth is more realistic. (c) Exponential growth has two phases lag and log. (d) In logistic growth, population passes well beyond the carrying capacity of ecosystem. 2. Identify the correct equation that represents the exponential population growth curve: (a) dN/dt = rN (b) dN/dt = rN (K-N/K) (c) Nt =N0ert (d) Both (1) and (3) 3. The equations correctly represents Verhulst-Pearl logistic growth is: (a) dN/dt = rN(K-N)/K (b) dN/dt = rN/K (c) dN/dt = N(K-N)/K (d) dN/dt = r(K-N)/K 4. The population growth is generally described by the following equation: dN/dt = rN(K – N)/K What does ‘r’ represent in the given equation? (a) Population density at time Y (b) Intrinsic rate of natural increase (c) Carrying capacity (d) The base of natural logarithm 5. Study the population growth curves (A and B) in the given graph and select the incorrect statement: (a) Curve ‘A’ shows exponential growth, represented by equation dN/dt=rN.

(b) Curve ‘B’ shows logistic growth, represented by equation dN/dt = r(K-N)/K. (c) Exponential growth curve is considered as more realistic than the logistic growth curve. (d) Curve ‘A’ can also be represented by equation Nt = N0ert. II. Study the following diagram and answer the question given below 6–10. Interactions are either cooperative or competitive. Cooperative interactions are seen mainly during mating, parental care or communication. Competitive interactions may be interspecific or interspecific. Some common type of interactions are also seen like mutualism. In this type of association two types of organisms are observed, viz., microorganisms and higher plants, higher plants and animals, and so on. In such type of association both partners derive benefit from living together. 6. Competition for space, food and light is most severe in: (a) Individual (b) Population (c) Community (d) None of the above 7. Mycorrhiza helps in: (a) Food, preparation (b) Defense mechanism (c) Nutrition up taking (d) Both (a) and (b) 8. Intraspecific competition helps mainly in: (a) Sexual selection (b) Succession (c) Both (a) and (b) (d) None of these. 9. Which of the following type of interactions occur in predation and parasitism? (a) (+,+) (b) (+,0) (c) (+,-) (d) (-,-) 10. Cooperative interactions are seen mainly during: (a) Mating (b) Parental care. (c) Communication. (d) All of the above.

ANSWER KEY [A] MULTIPLE CHOICE QUESTIONS 1. (c)

2. (d)

3. (b)

4. (d)

5. (a)

6. (c)

7. (c)

8. (c)

9. (a)

10. (c)

11. (b)

12. (b)

13. (d)

14. (c)

15. (b)

16. (b)

17. (b)

18. (b)

19. (b)

20. (b)

21. (b)

22 (c)

23. (d)

24. (c)

25. (a)

26. (d)

27. (c)

28. (c)

29. (b)

30. (d)

8. (b)

9. (a)

10. (a)

9. (c)

1. (d)

[B] ASSERTION & REASON 1. (a)

2. (a)

3. (a)

4. (a)

5. (b)

6. (a)

7. (b)

[C] COMPETENCY/CASE-BASED QUESTIONS 1. (d)

2. (d)

3. (a)

4. (b)

5. (c)

6. (b)

7. (c)

8. (a)

ANSWERS WITH EXPLANATION 1. Option (c) is correct. Explanation: Binary fission is a mode of asexual reproduction in protozoan through which it produces two offsprings from parent individual. So, One generation 1: there are 2 protozoans. In 2 generation: there are 2 * 2 = 4 protozoans

In 3rd generation: there are 4*2 = 8 protozoans In 4th generation 4:there are total 16 protozoans. In 5th generation: there are total 32 protozoans In 6th generation: there are 64 protozoans. Thus, the population of protozoan will be 64, after six generations.

148 Oswaal CUET (UG) Chapterwise Question Bank

BIOLOGY

2. Option (d) is correct.

10. Option (c) is correct.

Explanation: The visiting flamingos and resident fishes of the South American lakes compete for zooplankton. It is an example of competition. In competition interaction, the fitness of one organism overpowers the presence and fitness of another.

Explanation: Explanation: Birth rate = Number of individuals born/ Total number of individuals = 24/60 = 0.4 off springs/ sparrow

3. Option (b) is correct.

Explanation: Foot prints (pug marks) and faecal pallets of conserved animals are some parameters which can be used for tiger census in our country’s National park and sanctuaries.

Explanation: Heliophytes are sun loving plants. They are also called sun plants because they grow in open in full sunlight. Prosopis, Betula,Teak, etc., are examples of heliophytes. 4. Option (d) is correct. Explanation: Resource partitioning in which two species competing for the same resources can avoid competition by choosing. In this, the resources are divided by the species within an ecological niche. It is an adaptation of interspecific competition. 5. Option (a) is correct. Explanation: A kind of population interaction in which one species benefits and the other is neither harmed nor benefited is commensalism. It can also be denoted by (+, 0). 6. Option (c) is correct. Explanation: Variations in number of individuals in a population can be expressed as population density and population size. A population of younger individual than older individuals will show positive growth in future (after some yrs.), that is, it will increase after some time. 7. Option (c) is correct. Explanation: Population growth is determined by addition and loss of individual in a population. Population density is the number of individuals present per unit volume/area at given time. Thus, mortality and emigration would necessarily decrease the density of a population in a given habitat due to loss of individual result from mortality (deaths) and emigration. When natality rate will be more than mortality rate and increased immigration rate would necessarily increase the density of a population. 8. Option (c) is correct. Explanation: Ecological niche is ecologically adapted zone, that is, the particular place of habitat and functional role occupied by individual of a species within its ecosystem. Ecological niche have specific characteristics, such as availability of nutrients, temperature, terrain, sunlight, and predators, which influences how populations affect and are affected by resources and enemies. 9. Option (a) is correct. Explanation: Sandal wood tree (Santalum album) is an example of partial root parasite. It grows on the roots of Dalbergia, Albizzia and Eucalyptus. The sucking roots of the plants attack the roots of the host trees and from them nutrients are absorbed. Mistletoe (Viscum) is considered as hemiparasite which derives a part of nourishment from host plant. Orobanche is an obligate holoparasite which attack strategic food crops, such as legumes and vegetables, Ganoderma mushrooms are known as king of herbs for their wonderful medicinal properties. They are commonly known as reishi mushroom. They are widely distributed, shelf like or knob like fungi that feed either as saprobes on dead wood or as parasites on the live wood of hardwood trees.

11. Option (b) is correct.

12. Option (b) is correct. Explanation: The prey might become extinct when the prey itself is overexploited or killed by the predators. Predation is an interaction in which an organism (predator) is benefited by killing another organism (prey). 13. Option (d) is correct. Explanation: Parasitism is an interaction between two species in which one species (parasite) derives benefit while the other species (host) is harmed. For example, ticks and lice (parasites) present on the human body represent this interaction where in the parasites receive benefit (as they derive nourishment by feeding on the blood of humans). On the other hand, these parasites reduce host fitness and cause harm to the human body. 14. Option (c) is correct. Explanation: Ecotone is a transition zone between two communities. The adjacent biotic (natural) communities generally do not possess a fine demarcation line between them. Therefore, the adjacent of two communities is represented by population of both the communities, and this transition zone between two communities is referred to as ecotone. 15. Option (b) is correct. Explanation: The population density tells about the number of individuals in a unit area at a specific time. It is a geographical term. Countries with high population density are said to be over-populated. 16. Option (b) is correct. Explanation: Unlimited resources in any habitat result in increase in number of species. The population grows in an exponential or geometric manner. dN/dt = rN r is known as “intrinsic rate of natural increase“. r is an important factor which help in assessing the impact of any biotic or abiotic factor or population growth. 17. Option (b) is correct. Explanation: The adaptation for climbing and balancing is known as arboreal adaptation. It is also known as scansorial adaptation. It is used by animals for climbing on trees, rocks, walls, etc. This type of adaptation is shown by monkeys, sloths, chameleons, koalas, etc. 18. Option (b) is correct. Explanation: Amensalism is an association between two organisms in which one species is benefitted or unaffected while the other organism is destroyed or inhibited. 19. Option (b) is correct. Explanation: Among the given above birth rate is a population attribute. It is the number of individuals born in a population at a specific time. For humans, it is described as a number of individuals born per 1000 of the population per year.

149

ORGANISMS & ENVIRONMENT

20. Option (b) is correct. Explanation: The death rate=Number of deaths per year/Total population =100/10, 00, 000= 0.0001 person per year. 21. Option (b) is correct. Explanation: In forest ecosystem tall trees of forest plants controls the light condition, that is, intensity duration and quality of light at the ground. A well growing herbaceous plant in forest receive less intensity duration and quality of light, but when it is transplanted in a park outside its natural habitat, the light will be received uninterrupted. So, due to change in its microclimate, it may not survive. 22. Option (c) is correct. Explanation: Population density is a measurement of the number of people in an area. It is an average number. It is generally measured as number of individuals of species present in unit space in a given time. 23. Option (d) is correct. Explanation: Mutualism is a positively directed interspecific relationship. It is a biological interaction between organisms of different species in which each individual receives the benefit. 24. Option (c) is correct. Explanation: Variations in number of individuals in a population is expressed as population density and population size. A population of more young individual than older individuals will show positive growth in future (after some yrs), i.e., it will increase after some time.

growing underneath. Lianas and climber are woody vines which make commensalism association with tall trees. Herbs and shrubs occupy lower strata of forest. 29. Option (b) is correct. Explanation: The mutualistic relationship between a fungus and photosynthesizing algae is called lichens. It is a composite organism and has many colors, forms, and sizes. They are used in the food industry, making dyes and have medicinal use. 30 Option (d) is correct. Explanation: The ectoparasite is organisms that reside on the surface of the host. Some examples of ectoparasites are fleas, lice, mites, ticks, etc. These are obligatory parasites. [B] ASSERTION & REASON: 1. Option (a) is correct. Explanation: The human population represents the logistic growth curve as the number of human beings are increasing rapidly but available resources are not increasing at the same pace. Such a growth pattern is not sustainable because at one point the human population would reach a place where there would not be enough resources for everyone. 2. Option (a) is correct. Explanation: In Daphnia the cyclomorphosis (cyclic change in morphology) is seen. The morphology depends upon the temperature variation in water. So at different seasons of the year the Daphnia shows different morphology.

25. Option (a) is correct.

3. Option (a) is correct.

Explanation: (p+q)2 = p2 + 2pq + q2 = 1 represents an equation used in population genetics. Population genetics is the study of changes in gene frequencies in population of organisms and the effects of such changes on evolution and adaptation. (p+q)2 = p2 + 2pq + q2 = 1 represents the HardyWeinberg principle. This principle says that allele frequencies in a population are stable and is constant from generation to generation (i.e., the gene pool remains a constant). Here, p2 represent the frequency of a homozygous-dominant allele (AA). q2 represents the frequency of homozygous-recessive allele (aa). 2pq is the frequency of heterozygous individual (Aa).

4. Option (a) is correct.

26. Option (d) is correct. Explanation: A relationship in which one parasite feeds on another parasite which in turn feed on its host is called epiparasite. These are also known as secondary or hyper parasites. An example is a bacterium in an insect’s gut that feeds on cattle. 27. Option (c) is correct. Explanation: Adelpho-parasite is an example of a parasite, in which the host species is closely related to the parasite and is a member of the same family or genus. It is a type of hyper-parasitism which commonly seen is Encarsia perplexa, Trissolcus basalis and members of the family Aphelinidae. 28. Option (c) is correct. Explanation: Tall tree of the forest plants controls the light condition at the ground. In a forest ecosystem, light, an important abiotic component controls a number of life processes in organism. In forest, the intensity, duration, and quality of light at ground level is controlled by tall trees, which have higher productivity as compared to shrubs and herbs

Explanation: Leaf butterfly is green in colour and stick insect also mimics in order to escape from the enemies and also to catch prey. The camouflaging mechanism helps the organism to get adapted to its surroundings. The Leaf butterfly is seen more near the leaves while the stick insect camouflages with its surroundings by living on the branches. Explanation: Clown fish maintain a commensalistic relation with the sea anemone. In this interaction, one species benefits and the other is neither harmed nor benefited. Sea anemone has stinging tentacles that provide protection to Clown fish from predators. The sea anemone does not appear to derive any benefit from the clown fish. 5. Option (b) is correct. Explanation: Organisms occupying similar ecological niches but living in different regions are called ecological equivalents. Ecological equivalents possess similar types of adaptations but belong to different taxonomic groups. For example; succulents of American deserts are cacti while those of African desert are euphorbia’s. 6. Option (a) is correct. Explanation: Interbreeding implies sexual reproduction which involve the fusion of two gametes of different species which bring variation among the species. 7. Option (b) is correct. Explanation: A group of individuals resembling each other in morphological, physiological, biochemical and behavioral characters constitute a species. Such individuals can breed among themselves but cannot breed with members other than their own to produce fertile off springs. New species are formed mainly due to reproductive isolation.

150 Oswaal CUET (UG) Chapterwise Question Bank

BIOLOGY

8. Option (b) is correct.

3. Option (a) is correct.

Explanation: Population is a group of people therefore its attributes are different from an individual organism.

Explanation: The equation that correctly represents VerhulstPearl logistic growth is dN/dt = rN(K-N)/K.

9. Option (a) is correct.

4. Option (b) is correct. Explanation: ‘r’ represents Intrinsic rate of natural increase. 5. Option (c) is correct. Explanation: T Since resources of growth for most animal populations are finite and become limiting sooner or later, so the logistic growth model is considered as more realistic. 6. Option (b) is correct. Explanation: Competition is most severe in between closely related organisms, growing in the same area. 7. Option (c) is correct. Explanation: Mycorrhiza is an association of fungus with roots of higher plants. It helps in absorption of water and nutrients like phosphorus from the soil. 8. Option (a) is correct. Explanation: Intraspecific competition is a competition between two individuals from the same species. 9. Option (c) is correct. Explanation: Predation occurs when members of one species eat the other species, while parasitism occurs when one member of a species parasite stay on the other body. In this case, one member of a species is benefited (+) while the other member of a species is harmed (-). 10. Option (d) is correct. Explanation: Cooperative interactions are seen mainly during mating, parental care or communication.

Explanation: Ernst Mayr proposed the biological species concept, which defines species as groups of actually or potentially inter-breeding natural populations which are reproductively isolated from other such groups. 10. Option (a) is correct. Explanation:: Natality refers to the number of births during a given period in the population which is added to the initial density. Hence, it contributes to the increase in population density [C] COMPETENCY/CASE-BASED QUESTIONS: 1. Option (d) is correct. Explanation: In logistic growth population seldom grows beyond the carrying capacity of ecosystem. 2. Option (d) is correct. Explanation: If any species is flourishing under unlimited resources, it would reach exponential growth which can be depicted by equation: dN/dt = rN Where, N = population density at time t; r = intrinsic rate of natural increase. If we derive the integral form of the exponential growth equation, it can be written as: Nt =N0.ert. Where, Nt, = population density after time t; Noise population density at time zero; r = intrinsic rate of natural increase; e is the base of natural logarithm.

24

Study Time Max. Time: 1:5 Hours Max. Questions: 50

CHAPTER

14

ECOSYSTEMS

 Revision Notes:

 Introduction {  An ecosystem is a functional unit of nature, where living organisms interact among themselves and also with the surrounding physical environment. { It is the self-regulating structural and functional unit of the biosphere.  Types of Ecosystems (a) Terrestrial ecosystem: Forest, grassland, desert, etc. (b) Aquatic ecosystem: Pond, lake, wetland, river, estuary and ocean. (c)  Artificial ecosystem: Crop fields and aquarium.  Ecosystem: Structure and Function { An ecosystem consists of biotic and abiotic components. These components function as a unit. {  The vertical distribution of different species occupying different levels is called stratification. { Biotic Factors: This includes all the living organisms in an ecosystem. { Abiotic factors: This includes all the non-living factors in an ecosystem like Water with all dissolved inorganic and organic nutrients, soil rich in nutrients at rock bottom of the pond, solar input, day-length, temperature, and other climatic factors {  The characteristic physical structure of an ecosystem depends on the interaction of biotic and abiotic factors. { Stratification: It is the vertical and also the horizontal distribution of different varieties that occupy different levels. For example, trees occupy the top vertical strata or layers of a forest, shrubs are the second, and herbs and grasses occupy the bottom layers.   Components of Ecosystem There are four main

functions of the ecosystem:

(i) Productivity (ii) Decomposition (iii) Energy flow (iv) Nutrient cycling  The function of an ecosystem is that the conversion of inorganic into organic material by the subsequent steps: { Use of the energy of the sun by the autotrophs. { Consumption of the autotrophs by various heterotrophs. {  Decomposition and mineralization of all the dead organic interest release the nutrients back to the soil for reuse by the autotrophs. { These events are repeated again and again.

 Productivity (a) A constant input of solar energy is the basic requirement for any ecosystem to function and sustain. (b)  The rate of biomass production is called productivity. (c)  The productivity is expressed in terms of g–2yr–1 or (kcal m–2) yr–1. (d) It can be divided into gross primary productivity (GPP) and net primary productivity (NPP).  Primary Productivity (a) The amount of biomass or organic matter produced per unit area over some time by plants during photosynthesis is called primary production. (b)  The primary production is expressed in terms of weight (g–2) or energy (kcal m–2). (c)  Units of primary productivity can be expressed in terms of dry matter produced per unit of area for a given period or energy stored per unit of area for a given period.  Gross Primary Productivity (a) It is the rate of production of organic matter during photosynthesis. (b) A considerable amount of GPP is utilized by plants in respiration.   Net primary productivity : The available biomass for the consumption of heterotrophs or herbivores and decom­ posers is the net primary productivity. NPP = GPP – R  Primary productivity depends on (i) The plant species inhabiting a particular area. (ii) Environmental factors. (iii) Availability of nutrients. (iv) Photosynthetic capacity of plants. Therefore, it varies in different types of ecosystems. (e)  The annual net primary productivity of the whole biosphere is approximately 170 billion tons (dry weight) of organic matter. (f) Of this, despite occupying about 70% of the surface, the productivity of the oceans is only 55 billion tons. The rest of course is on land.  Secondary Productivity It is the rate of formation of new organic matter by consumers.

Producers

152 Oswaal CUET (UG) Chapterwise Question Bank

BIOLOGY

153

ECOSYSTEMS

(i)  It reflects only the utilisation of food for the production of consumer biomass. (ii)  It is the net rate of increase in the biomass of the heterotrophs. (iii) D  ue to this productivity the food is available for the next tropic level. Scan to know

 Decomposition more about this topic (a) It is the breakdown of complex organic matter by decomposers into inorganic substances like carbon dioxide, water and nutrients. Decompositions (b) It is largely an oxygenrequiring process. (c) Detritus (dead plant remains such as leaves, bark, flowers and dead remains of animals, including faecal matter) is the raw material for decomposition. (d) Detrivores: Organisms that feed on detritus are called detritivores. Earthworms, fungi, etc. are examples of detritivores.  Steps in decomposition The important steps in the process of decomposition are fragmentation, leaching, catabolism, humification and mineralisation. (a) Fragmentation It is the breakdown of detritus into smaller particles by detritivores to increases the surface area of detritus particles and makes further decomposition easier. (b) Leaching In this process, water-soluble inorganic nutrients go down into the soil horizon and get precipitated as unavailable salts. (c) Catabolism (i)  Here, the degradation of detritus into simpler inorganic substances takes place by bacterial and fungal enzymes. (ii)  Fragmentation, leaching and catabolism operate simultaneously on the detritus. (d) Humification (i) I t is the accumulation of humus (dark amorphous substance) in soil. (ii) H  umus is resistant to microbial action and decomposes very slowly. (iii) Being colloidal, it serves as a reservoir of nutrients. (e) Mineralisation It is the release of inorganic nutrients due to the degradation of humus by some microbes.  Factors Influencing Decomposition The rate of decomposition is controlled by the chemical composition of detritus and climatic factors. (a) Chemical composition of detritus: The decomposition rate is slower if detritus is rich in lignin and chitin and quicker if detritus is rich in nitrogen and water-soluble substances like sugars. (b) Climatic factors like temperature and soil moisture: –Temperature and soil moisture are the most important climatic factors that regulate decomposition through their effects on the activities of soil microbes.

A warm and moist environment favors decomposition whereas low temperature and anaerobic conditions inhibit decomposition resulting in the buildup of organic materials. – Nutrient immobilisation: At times, the soil nutrients instead of getting mineralised, get bound with the biomass of microbes, and so by temporarily unavailable to other organisms. This incorporation of nutrients in living microbes is called nutrient immobilisation.  Energy Flow Scan to know more about { The sun is the only source of this topic energy for all ecosystems on the earth. { Of the incident solar radiation less than 50% of it is photosynthetically active Energy Flow radiation (PAR). { Plants, photosynthetic and chemosynthetic bacteria (autotrophs) fix solar radiant energy to make food. { Plants capture only 2-10% of the PAR and this small amount of energy sustains the entire living world. So, it is very important to know how the solar energy captured by plants flows through different organisms of an ecosystem. { Ecosystem obeys the first and second laws of Thermodynamics. { The energy of the ecosystem is constant. { They need a constant supply of energy to synthesize the molecules they require, to counteract the universal tendency toward increasing disorderliness.  Producers { The green plants in the ecosystem that capture solar energy and convert it into chemically bound energy are called producers. { All organisms are dependent for their food on producers (green plants), either directly or indirectly. { In a terrestrial ecosystem, major producers are herbaceous and woody plants.

Grazing Food Chain { A

simple grazing food chain Scan to know (GFC) is depicted below: more about Grass - - - - - - - - - - Goat — this topic - - - - - - - - - Man (Producer) (Primary Consumer) (Secondary consumer) { Primary producers in an aquatic ecosystem are phytoplankton, Food Chains & algae and higher plants. Food Webs { The energy trapped by the producer is either passed on to a consumer or the organism dies. { Death of an organism is the beginning of the detritus food chain/web.

 Consumers (Heterotrophs) { These are all animals that depend on plants (directly or indirectly) for their food. { They include: (a) Primary Consumers These are herbivores that feed on plants. e.g., Insects, birds and mammals in the

154 Oswaal CUET (UG) Chapterwise Question Bank terrestrial ecosystem and molluscs in the aquatic ecosystem. (b) Secondary Consumers These are primary carnivores that feed on  herbivores e.g., Frog, fox, man etc. (c) Tertiary Consumers These are secondary carnivores that feed on  primary carnivores.  Detritus Food Chain (DFC) { It begins with dead organic matter. { It is made up of decomposers (saprotrophs) which are heterotrophic organisms. e.g. fungi and bacteria. { They meet their energy and nutrient requirements by degrading dead organic matter or detritus. { Decomposers secrete digestive enzymes that break down dead and waste materials into simple, inorganic materials, which are subsequently absorbed by them. { In an aquatic ecosystem, GFC is the major conduit for energy flow. { In a terrestrial ecosystem, a much larger fraction of energy flows through the DFC than through the GFC. { DFC may be connected with GFC at some levels: some of the organisms of DFC are prey to the GFC animals. { Some animals (cockroaches, crows etc.) are omnivores. { The standing crop is measured as the mass of living organisms (biomass) or the number in a unit area. The biomass of a species is expressed in terms of fresh or dry weight. { Measurement of biomass in terms of dry weight is more accurate. { The number of trophic levels in the grazing food chain is restricted as the transfer of energy follows Lindemann’s 10 % law, which states that only 10% of the energy is transferred to each trophic level from the lower trophic level. { In nature, it is possible to have so many levels – producer, herbivore, primary carnivore, and secondary carnivore in the grazing food chain.  Ecological Pyramids { The representation of a food Scan to know chain in the form of a pyramid more about this topic is called an ecological pyramid. It is the relationship between the producers and consumers of various orders represented graphically. Types of Ecological { The base of each pyramid Pyramids represents the producers (first trophic level) while the apex represents tertiary or top-level consumers or the last trophic level. { Ecological pyramids are of three types: (a) Pyramid of number (b) Pyramid of biomass (c) Pyramid of energy { Any calculations of energy content, biomass or numbers have to include all organisms at that trophic level. { The trophic level represents a functional level, not a

BIOLOGY

species as such. given species may occupy more than one trophic level in the same ecosystem at the same time. For example, A sparrow is a primary consumer when it eats seeds, fruits, and peas, and a secondary consumer when it eats insects and worms. { In most ecosystems, all the pyramids are upright i.e. producers are more in number and biomass than the herbivores and herbivores are more in number and biomass than the carnivores. { Also, energy at a lower trophic level is always more than at a higher level. { Example of inverted pyramids includes insects feeding on a big tree. { Pyramid of biomass in the sea is generally inverted because the biomass of fishes far exceeds that of phytoplankton. { The pyramid of energy is always upright because when energy flows from a trophic level to the next trophic level, some energy is always lost as heat at each step. { A

 Limitations of Ecological Pyramids (a) It does not take into account the same species belonging to two or more trophic levels. (b) It assumes a simple food chain that seldom exists in nature. It does not accommodate a food web. (c)  Saprophytes are not included in ecological pyramids even though they play a vital role in the ecosystem.

Ecological Succession:  Ecological succession is the process

Scan to know of change in the structure of species more about that belongs to an ecological this topic community over time. After a mass extinction, the time scale can be decades or even millions of years. This change occurs orderly and sequential, parallel Ecological with the changes in the physical Succession environment. { Pioneer Species: The species that invade a bare area are called pioneer species. { Pioneer Community: The community that is capable to invade a bare area is known as the pioneer community. { Climax community: A community that is almost near to equilibrium with the environment is called the climax community. { Sere:A sequence of ecological communities arising in an area from the initial pioneer community to the final climax community.   Primary succession: Primary succession is a type of biological and ecological succession of plant life. It occurs in an environment in which a new substrate is deposited. This new substrate is not having any kind of vegetation and other organisms. It also usually lacks soil and organic matter. It occurs in places such as land after a lava flow or area left from the retreated glacier.   Secondary succession: Secondary succession is a type of biological and ecological succession of plant life which takes place in a habitat that has been previously populated but has since been disturbed or damaged. For instance, regions where existing vegetation has been removed (due to tree-felling in a woodland or destructive events such as fires).

155

ECOSYSTEMS

 Succession of plants: { Hydrarch: A succession that begins in a water body or aquatic environment is called Hydrosere or Hydrarch Succession. { Xerarch: Xerosere is a succession of plants that are limited in the availability of water availability. It includes the different stages in xerarch succession. Xerosere originates in extremely dry situations such as sand deserts, dunes, salt deserts, rock deserts, etc. { In secondary succession, the invasion of species depends on the availability of water, conditions of the soil, and the environment. It also depends on whether any seeds or other propagules are present. Because soil is already existent, the pace of succession is substantially faster, and the climax community is reached much sooner. Succession always proceeds towards the mesic community.  Nutrient cycling: A nutrient cycle is defined as the cyclic pathway by which nutrients pass through, to be recycled and reutilised. The pathway comprises cells, organisms, community and ecosystem.  Importance of nutrient cycles: { It is required for the transformation of nutrients from one form to another so that it can be readily utilised by different organisms, e.g. plants cannot take atmospheric nitrogen and it has to be fixed and converted to ammonium and nitrate for uptake. {  Transfer of nutrients from one place to another for utilisation, e.g. air to soil or water { Nutrient cycles keep the ecosystem in equilibrium and help in storing nutrients for future uptake {  Through nutrient cycling, living organisms interact with the abiotic components of their surroundings {  Nutrient recycling involves both biotic and abiotic components. The main abiotic components are air, water, and soil. { The 4 main nutrient cycles are: 1. Carbon cycle. 2. Nitrogen cycle 3. Oxygen cycle 4. Phosphorous cycle.  Carbon cycle: {  Carbon is present mainly as Scan to know carbon dioxide and methane in more about the atmosphere. this topic { There is a continuous exchange of carbon between biotic and abiotic components by the process of photosynthesis and respiration. Carbon Cycle { Atmospheric carbon dioxide is fixed by plants in the process of photosynthesis. { All the living organisms release carbon dioxide during respiration. {  Carbon is released into the atmosphere by burning fossil fuels and auto emissions. Organic carbon from dead and decaying organisms and waste products is released into the atmosphere after decomposition  Phosphorous cycle: Scan to know { The following are the important more about this topic steps of the phosphorous cycle: a) Weathering b) Absorption by plants c) Absorption by Animals d) Return to the Environment through decomposition. Phosphorus Cycle

{  The

natural reservoir of phosphorous is rock in the form of phosphates. { During weathering of rocks minute quantities of phosphates get dissolved in water. {  The phosphorous cycle is one of the slowest bio­ geochemical cycles. { Phosphates enter the trees through their roots and then into the food chain. { The organic wastes and dead organisms are decomposed by phosphate solubilising bacteria which release phosphorous back to the soil. {  The atmospheric inputs of phosphorous through rainfall or gaseous exchange of phosphorous between organisms and the environment are negligible.  Ecological succession: Ecological succession is the  Scan to know more about process by which the mix of this topic species and habitat in an area changes over time. Gradually, these communities replace one another until a “climax community”—like a mature Biogeochemical forest—is reached, or until a Cycle in disturbance, like a fire, occurs. Ecosystem { Types: a) Primary Succession. b) Secondary Succession.  Primary Succession: {  Primary succession is the succession that starts in lifeless areas such as the regions devoid of soil or the areas where the soil is unable to sustain life.  Secondary Succession: {  Secondary succession occurs when the primary ecosystem gets destroyed. E.g. a climax community gets destroyed by fire. It gets decolonized after the destruction. This is known as secondary ecological succession. { Carbon fixation: (balances the atmosphere) Producers pick up CO2 from the atmosphere and convert it into organic compounds in the process of photosynthesis. This also gives food to others outside the ecosystem. E.g.: Tribals and migratory animals. { Pollination is a significant step in the reproduction of plants. It occurs by bees, buffer flies, birds, humans, wind, water etc. Animal depends on plants for food. The absence of these plants will naturally deprive the animal of their food which leads to death. { O2 release: Producers of the ecosystem release a lot of O2 during Photosynthesis. For example, Amazon rainforests are called as lungs of our planet because they produce nearly 20% O2.

IMPORTANT DIAGRAMS Trophic level

No. of Individuals

TC (Tertiary consumer)

3

SC (Secondary consumer) PC (Primary consumer) PP (Primary producer)

3, 54,000 708,000 5, 842,000

Pyramid of numbers in a grassland ecosystem

156 Oswaal CUET (UG) Chapterwise Question Bank Trophic level TC

Dry Mass 2 (kg/m)

Tropic level

1.5

Dry Mass 2 (kg/m)

PC

21 (Zooplankton)

PP SC PC PP

11

BIOLOGY

4 (Phytoplankton)

14.2 Inverted Pyramid of Biomass in Sea Pyramid level Energy level

37 809

14.1 Pyramid of Biomass in most Ecosystems

TC

10 J

SC PC PP

100 J 1000 J 10,000 J

1,000,000 J of Sunlight

14.3 An ideal pyramid of energy

OBJECTIVE TYPE QUESTIONS [A] MULTIPLE CHOICE QUESTIONS: 1. Decomposers like fungi and bacteria are [NCERT Exemplar, Q. 1, Pg. 98] (i) Autotrophs. (ii) Heterotrophs. (iii) Saprotrophs. (iv) Chemoautotroph. Choose the correct answer. (a) i and iii (b) i and iv (c) ii and iii (d) i and ii 2. The process of mineralisation by microorganisms helps in the release of (a) Inorganic nutrients from humus. (b) Both organic and inorganic nutrients from detritus. (c) Organic nutrients from humus. (d) Inorganic nutrients from detritus and formation of humus.  3. Which of the following is not an example of terrestrial ecosystem? (CUET 2022) (a) Wetland (b) Grassland (c) Forest (d) Desert 4. Which is not a structural aspect of the ecosystem? (a) Stratification (b) Diversity (c) Productivity (d) Species composition. 5. An inverted pyramid of biomass can be found in which ecosystem? (a) Forest (b) Marine (c) Grassland (d) Tundra 6. Which of the following ecosystems is most productive in terms of net primary production? [NCERT Exemplar, Q. 6, Pg. 99] (a) Deserts (b) Tropical rainforests (c) Oceans (d) Estuaries 7 Which of the following is not a producer? [NCERT Exemplar, Q. 5, Pg. 99] (a) Spirogyra (b) Agaricus (c) Volvox (d) Nostoc AI

8. What is the spectral range of solar radiation that photosynthetic organisms are able to use in the process of photosynthesis? (a) Photo synthetically Reactive Radiation (b) Photo synthetically Active Radiation (c) Photo synthetically Deactive Radiation (d) Photo synthetically Trapped Radiation 9. An inverted pyramid of biomass can be found in which ecosystem? [NCERT Exemplar, Q. 4, Pg. 98] (a) Forest (b) Marine (c) Grassland (d) Tundra 10. Given below is a simplified model of the phosphorus cycle in terrestrial ecosystems with four blanks. Identify the blanks A, B, C and D. (CUET 2022) C

Consumers D

A

Soil solution

Uptake

Wenthering Run off B

(a) (b) (c) (d) 11. (a) (b) (c) (d)

A: Producer, B: Litter fall, C: Rock mineral, D: Detritus. A: Rock minerals, B: Detritus, C: Litter fall, D: Producers A: Litter fall, B: Producer, C: Rock minerals, D: Detritus A: Detritus, B: Rock minerals, C: Producers, D: Litter fall Which of the following is an ecosystem service provided by a natural ecosystem? Cycling of nutrients. Prevention of soil erosion. Pollutant absorption and reduction of the threat of global warming. All of the above.

157

ECOSYSTEMS

12. Approximately how much of the solar energy that falls on the leaves of a plant is converted to chemical energy by photosynthesis? (a) Less than 1% (b) 2–10% (c) 30%   (d) 50% 13. Among the following, where do you think the process of decomposition would be the fastest? (a) Tropical rain forest (b) Antarctic (c) Dry arid region (d) Alpine region 14. During the process of ecological succession, the changes that take place in communities are [NCERT Exemplar, Q. 11, Pg. 100] (a) O and sequential. (b) Random. (c) Very quick. (d) Not influenced by the physical environment. 15. The detritus food chain (DFC) begins with (CUET 2022) (a) Green plants (b) Carnivore (c) Dead Organic matter (d) Herbivore 16. The Climax community is in a state of (a) Non-equilibrium. (b) Equilibrium. (c) Disorder. (d) Constant change.  17. What is the total number of trophic levels of a balanced ecosystem? (a) 5 (b) 3 (c) 4 (d) 2 18. The sequence of communities of primary succession in water is [NCERT Exemplar, Q. 14, Pg. 100] (a) Phytoplankton, sedges, free-floating hydrophytes, rooted hydrophytes, grasses and trees. (b) Phytoplankton, free-floating hydrophytes, rooted hydrophytes, sedges, grasses and trees. (c) Free-floating hydrophytes, sedges, phytoplankton, rooted hydrophytes, grasses and trees. (d) Phytoplankton, rooted submerged hydrophytes, floating hydrophytes, reed swamp, sedges, meadow and trees. 19. Which one of the following is the simplified sequence of phosphorus cycling in a terrestrial ecosystem? (a) Soil → Producers → Rock minerals → Consumers → Decomposers (b) Rock minerals → Soil → Producers → Consumers → Decomposers (c) Rock minerals → Decomposers → Producers → Consumers (d) Decomposers → Rock minerals → Consumers → Producers  20. The animals that feed on herbivores like insects, birds and mammals in terrestrial ecosystems, are called _____. (CUET 2022) (a) Saprotrophs (b) Primary consumers (c) Secondary carnivores (d) Primary carnivores 21. The zone at the edge of a lake or ocean which is alternatively exposed to air and immersed in water is called [NCERT Exemplar, Q. 18, Pg. 101] (a) pelagic zone. (b) benthic zone (c) lentic one. (d) littoral zone.

22. Nutrient cycles are of two types—  (CUET 2022) (a) Gaseous and solid (b) Liquid and sedimentary (c) Gaseous and sedimentary (d) Aquatic and Gaseous 23. Founder of “ School of Plant Ecology” in India was: (a) R. Mishra (b) KC Mishra (c) RN Singh (d) G.D Pant 24. Out of the total quantity of global carbon, what per cent of carbon is found dissolved in oceans? (a) 50 % (b) 71 % (c) 90 % (d) 25 % 25. Round worms in human intestine belongs to : (a) Trophic level 1 (b) Trophic level 2 (c) Trophic level 3 (d) Trophic level 4 26. How much of the net primary productivity of a terrestrial ecosystem is eaten and digested by herbivores? (a) 1% (b) 10% (c) 40% (d) 90% 27. Which trophic level typically has the fewest organisms but the highest energy content in a pyramid of energy? [NCERT Exemplar, Q. 10, Pg. 99] (a) Producers (b) Primary consumers (c) Secondary consumers (d) Tertiary consumers 28. Which of the following types of ecosystem is expected in an area where evaporation exceeds precipitation, and mean annual rainfall is below 100 mm. [NCERT Exemplar, Q. 17, Pg. 101] (a) Grassland (b) Shrubby forest (c) Desert  (d) Mangrove 29. Edaphic factor refers to (a) Water.  (b) Soil. (c) Relative humidity. (d) Altitude. 30. Select the correct sequence of development of the following stages in primary succession. (CUET 2022) A. Scrub Stage B. Marsh-meadow stage C. Submerged free-floating plant stage D. Reed-swamp stage E. Submerged plant stage Choose the correct answer from the options given below (a) C, E, B, D, A (b) E, C, D, B, A (c) D, E, B, C, A (d) E, C, D, A, B [B] ASSERTION & REASON:

Directions: In the following questions, a statement of assertion (a) is followed by a statement of reason (R). Mark the correct choice as:

(A) Both assertion (A) and reason (R) are true and reason (R) is the correct explanation of assertion (A). (B) Both assertion (A) and reason (R) are true but reason (R) is not the correct explanation of assertion (A). (C) Assertion (A) is true but reason (R) is false. (D) Assertion (A) is false but reason (R) is true. 1. Assertion: The three steps in the decomposition (fragmentation, leaching and catabolism) operate on the detritus simultaneously

158 Oswaal CUET (UG) Chapterwise Question Bank Reason: Humification and mineralisation occur during decomposition in the soil. 

3. Soil aeration is inversely proportional to

2. Assertion: Humus serves as a reservoir of nutrients.

(b) Diameter of soil particles

Reason: Humus is colloidal. 3. Assertion (A): In ecological pyramids, the successive tiers represent successive tropic levels towards the apex.

Reason (R): The base of the pyramid is of producers, the next one above is the herbivores and the top tiers are of carnivores.

4. Assertion: The tropic level represents a functional level, not a species as such. Reason: A given species may occupy more than one tropic level in the same ecosystem at the same time 5. Assertion (A): Ecosystem may be natural or man-made.

Reason (R): The ecosystem is a functional unit of nature. Crop-fields and aquariums may be one of the examples of man-made ecosystems.

6. Assertion (A): The rate of biomass production is called productivity. It is of two types: GPP and NPP.

Reason (R): Gross primary productivity (GPP) is the rate of production of organic matter during photosynthesis. AI

7. Assertion: Low temperature and anaerobiosis inhibit decomposition resulting in a buildup of organic materials Reason: Decomposition is largely an oxygen-requiring process. 8. Assertion (A): The vertical distribution of different species, which occupy different levels is called stratification.

Reason (R): In a forest ecosystem, trees occupy the top strata, followed by shrubs, herbs and grasses.

9. Assertion (A): Plants capture only 2-10 per cent of the photosynthetically active radiation (PAR).

Reason (R): The Sun is the only source of energy for all. 

BIOLOGY

(a) Soil pore spaces (c) Water holding capacity of soil (d) None of the above. 4. Edaphology is concerned with the study of: (a) Soil  

(b) Climate

(c) Biota  

(d) Topography

5. Directions: In the following questions a statement of Assertion (A) is followed by a statement of reason (R). Mark the correct choice as: (a) Both Assertion (A) and reason (R) are true and reason (R) is the correct explanation of Assertion (A). (b) Both Assertion (A) and reason (R) are true but reason (R) is not the correct explanation of Assertion (A). (c) Assertion (A) is true but reason (R) is false. (d) Assertion (A) is false but reason (R) is true.

Assertion (A): Some organisms like Salmon are known to be Euryhaline.



Reason (R): Euryhalines can tolerate a wide range of salinities.

II. Study the following diagram and answer the question given below 6–10.

There are different varieties of living organisms on the earth’s surface. All living organisms interact among themselves and also with the physical environment surrounding them. This is a functional unit of the living world called an ecosystem. The physical environmental factors in an ecosystem constitute the abiotic components. They are mainly of three types i.e., climatic, edaphic, and topographic factors.

6. Who has coined the term ecosystem?

10. Assertion (A): Producers capture only about 1-5% of the total solar radiation.

(a) A.G Tansley

(b) C. Mobius

(c) Forbes 

(d) Sukachev



7. Which among the following is the largest man-made ecosystem?

Reason (R): Most of the solar radiation gets dissipated as heat.

[C] COMPETENCY/CASE-BASED QUESTIONS:

(a) Aquarium

(b) Zoo

I. Read the passage carefully and answer the question given after the passage from 1–5.

(c) Garden 

(d) Agro ecosystem.



(a) Stratification

(c) Diversity

(c) Productivity

(d) Species composition.

All biomes of the earth constitute the earth’s ecosystem. All flora and fauna occur in the lithosphere, atmosphere and hydrosphere along with their environments. Air, water, soil, biota, temperature and light are the major environmental factors that directly affect the flora and fauna in different geological areas of the earth.

1. Climate of an area depends upon: (a) Altitude of the area (b) Latitude of the area (c) Topography (d) Both (1) and (2). 2. Hydrological cycle is concerned with: (a) Lithosphere (b) Hydrosphere (c) Atmosphere (d) All of these.

8. Which is not a structural aspect of the ecosystem?

9. Which of the following is the most stable ecosystem? (a) Ocean 

(b) Desert

(c) Forest 

(d) Mountain

10. Decomposers such as fungi and bacteria are vital to the ecosystem because they: (a) are the primary organic recyclers. (b) are the terminal energy consumers. (c) keep populations in check. (d) Both (a) and (b)

159

ECOSYSTEMS

ANSWER KEY [A] MULTIPLE CHOICE QUESTIONS 1. (c)

2. (a)

3. (a)

4. (c)

5. (b)

6. (b)

7. (b)

8. (b)

9. (b)

10. (d)

11. (d)

12. (b)

13. (a)

14. (a)

15. (c)

16. (b)

17. (c)

18. (b)

19. (b)

20. (d)

21. (d)

22 (c)

23. (a)

24. (b)

25. (c)

26. (b)

27. (d)

28. (c)

29. (b)

30. (b)

8. (b)

9. (a)

10. (a)

9. (a)

1. (d)

[B] ASSERTION & REASON 1. (b)

2. (a)

3. (a)

4. (a)

5. (b)

6. (b)

7. (a)

[C] COMPETENCY/CASE-BASED QUESTIONS 1. (d)

2. (d)

3. (c)

4. (a)

5. (b)

6. (a)

7. (d)

8. (c)

ANSWERS WITH EXPLANATION [A] MULTIPLE CHOICE QUESTIONS: 1. Option (c) is correct. Explanation: Fungi and bacteria have been categorised as heterotrophs and saprotrophs which derive nourishment from the decaying organic matter and thereby decompose complex substances into simpler substances.

8. Option (b) is correct. Explanation: The spectral range of solar radiation that photosynthetic organisms are able to use in the process of photosynthesis is called as Photo synthetically Active Radiation. It is designated as PAR and ranges from 400nm to 700nm.

2. Option (a) is correct.

9. Option (b) is correct.

Explanation: Humus is an organic material formed by the decomposition of leaves. Microorganisms release inorganic nutrients from humus by the process of mineralization.

Explanation: In a terrestrial ecosystem, the biomass of producers is high while biomass decreases at the next trophic levels and it is lowest at the top level. Therefore, pyramid of biomass is upright. Contrary to this, the pyramid of biomass in a sea or a lake is generally inverted because the biomass of fishes (consumers) exceeds that of phytoplankton (producers).

3. Option (a) is correct. Explanation: Wetland is not a terrestrial ecosystem, it is an aquatic ecosystem. Rest all are examples of terrestrial ecosystems. 4. Option (c) is correct. Explanation: Stratification, Diversity and species composition are all structural aspects of an ecosystem. On the other hand, productivity is the rate of biomass accumulation at a specific period by the ecosystem. It is a functional aspect.

10. Option (d) is correct. Explanation: In the given simplified model of phosphorus cycling in a terrestrial ecosystem, the marked “A” is Detritus, label ‘B’ is Rock minerals, label ‘C’ is Producers and label D represents Litterfall.

5. Option (b) is correct. Explanation: In a terrestrial ecosystem biomass of producers is high while biomass decreases at the next trophic levels and it is lowest at the top level. Therefore, pyramid of biomass is upright. Contrary to this, the pyramid of biomass in a sea or a lake is generally inverted because the biomass of fishes (consumers) exceeds that of phytoplankton (producers). 6. Option (b) is correct. Explanation: Primary production is the synthesis of organic compounds from atmospheric or aqueous carbon dioxide. Net primary production is the rate at which all the plants in an ecosystem produce net useful chemical energy; it is equal to the difference between the rate at which the plants in an ecosystem produce useful chemical energy and the rate at which they use some of that energy during respiration. Tropical rainforest ecosystems are most productive in terms of net primary production.

Producers

Consumers Liter fall Detrims Decomposition Soil solution Wenthering

Uptake Run off

Rock minerals

11. Option (d) is correct.

7. Option (b) is correct.

Explanation: Functioning of our ecosystem provides us services like cycling of nutrients, prevention of soil erosion, pollutant absorption and reduction of the threat of global warming, etc.

Explanation: Organisms at options (a), (c) and (d)that is Spirogyra, Volvox and Nostoc are producers. Therefore, Agaricus given under option (b) is a heterotroph which is not a producer.

Explanation: Approximately 2–10% of the solar energy that falls on the leaves of a plant is converted to chemical energy by the process of photosynthesis.

12. Option (b) is correct.

160 Oswaal CUET (UG) Chapterwise Question Bank 13. Option (a) is correct. Explanation: The process of decomposition occurs at a faster rate in warm and moist environments. Tropical rainforests favor these conditions and hence the process of decomposition would be the fastest in tropical rainforests. 14. Option (a) is correct. Explanation: Slow and conspicuously visible change in the species composition of a given area is called ecological succession. A barren area is first inhabited by mosses or lichens which can survive harsh environment and modify the environment for the growth of next species. Followed by changed condition of environment small vegetation and bushes grows and finally trees or climatic species establishes in the area. Therefore, these changes are orderly and sequential parallel with the changes in the physical environment. 15. Option (c) is correct. Explanation: The detritus food chain begins with detritus such as dead bodies of animals or fallen leaves, which are then eaten by decomposers or detrivores. These detrivores are in turn consumed by their predators. The grazing food chain begins with producers, present at the first trophic level. The plant’s biomass is then eaten by herbivores which in turn are consumed by a variety of consumers. Carnivores feed on consumers. Herbivores feed on green plants. 16. Option (b) is correct. Explanation: The climax community is top most stable species suited for the physical environment of ecological succession. Therefore, the climax community is in a state of equilibrium. 17. Option (c) is correct. Explanation: Since the amount of available energy decreases as we move to higher tropic levels very little usable energy remains after four trophic levels. That’s why the food chain has only 3 to 4 tropic levels. 18. Option (b) is correct. Explanation: As the changes in an ecological Successions are orderly, sequential parallel with the changes in the physical environment. Therefore, the sequence of communities of primary succession in water is phytoplankton, free-floating hydrophytes, rooted hydrophytes, sedges, grasses, and trees. 19. Option (b) is correct. Explanation: The phosphorous cycle starts with rock materials and ends with the decomposers. The phosphorous enters the ecosystem through rock minerals and, there is a negligible exchange of gaseous phosphorus between organisms and the environment. 20. Option (d) is correct. Explanation: The animals that feed on herbivores like insects, birds and mammals in terrestrial ecosystems, are called primary carnivores. These are also called secondary consumers. Herbivores are primary consumers. 21. Option (d) is correct. Explanation: The littoral zone is the zone at the edge of a lake or ocean or in an aquatic habitat that is alternatively exposed to air. Therefore, light is also available and immersed in water. This coastal zone is rich in producers from surface to bottom, with a large number of brown and red algae attached to the zone. There are different zones in an aquatic ecosystem. The pelagic zone of water in a sea or lake that is neither close to the bottom nor the shore is a pelagic zone. Benthic zone: It is the bottom part of the water body (aphotic zone), light is not available here and it is dark. Microorganisms and benthic organisms are found in this zone. Lentic zone: It refers to standing or relatively still water in an aquatic ecosystem.

BIOLOGY

22. Option (c) is correct. Explanation: Nutrient cycles are of two types: Gaseous cycle - The exchange of nutrients occurs in gaseous or vapour form. A reservoir pool is an atmosphere or hydrosphere. e.g. Nitrogen; Carbon, Oxygen and Hydrogen cycle. Sedimentary cycle - Sedimentary cycles are a type of biogeochemical cycle, in which the reservoir is Earth’s crust.e.g. Sulphur and Phosphorus cycle. 23. Option (a) is correct. Explanation: Ramdeo Misra is known as the Father of ecology in India. 24. Option (b) is correct. Explanation: Almost 71 per cent of carbon is found dissolved in oceans out of the total quantity of global carbon. Out of this 71 per cent, almost 90 per cent is found in ocean. The oceans consist of 38,000- 40,000 gigatons (Gt) of carbon. 25. Option (c) is correct. Explanation: Rounds worms are parasite of human intestine which are carnivores, thus, they belong to secondary consumer level i.e. they belong to trophic level 3. 26. Option (b) is correct. Explanation: About 10% of the net primary productivity of a terrestrial ecosystem is eaten and digested by herbivores. 27. Ans: Option (d) is correct. Explanation: Tertiary consumers are typically at the top of the food chain and feed on secondary consumers or other tertiary consumers. While they have the highest energy content in their bodies due to consuming organisms lower in the food chain, their population is usually the smallest because there is less energy available as you move up the trophic levels. This phenomenon is known as the “pyramid of energy.” 28. Option (c) is correct. Explanation: When evaporation exceeds precipitation, the soil moisture supporting vegetation will deplete and therefore growth of vegetation will cease. Hence, a desert ecosystem, where vegetation is almost nil, is expected in an area where evaporation exceeds precipitation, and mean annual rainfall is below 100 mm. 29. Option (b) is correct. Explanation: Locality factors or the environment of soil is described by the term Edaphic factor. 30. Option (b) is correct. Explanation: Ecological succession is the gradual change in the species composition of a given area. In primary succession in water, the pioneers are replaced with time by rooted submerged plants (submerged plant stage)—Submerged freefloating plant stage—Reed swamps stage—Marsh meadow stage—scrub stage—Forest. [B] ASSERTION & REASON: I. 1. Option (b) is correct. Explanation: The three steps in the decomposition (fragmentation, leaching and catabolism) operate on the detritus simultaneously. Humification & mineralisation occur during decomposition in the soil. 2. Option (a) is correct. Explanation: Being colloidal (organic colloid) humus serves as a reservoir of nutrients. 3. Option (a) is correct. Explanation: Each successive level of nourishment as represented by links of the food chain is known as the trophic level. Organisms whose food is obtained from plants by the

161

ECOSYSTEMS

same number of steps belong to the same trophic level. Thus, green plants (producers) occupy the first trophic level; plant eaters (primary consumers) the secondary level, and carnivores which eat herbivores, the third level. 4. Option (a) is correct. Explanation: The trophic level represents a functional level, not a species as such. A given species may occupy more than one trophic level in the same ecosystem at the same time. 5. Option (b) is correct. Explanation: Ecosystem may be natural or man-made. It is a functional unit of nature. Crop fields and aquariums may be some examples of man-made ecosystems. 6. Option (b) is correct. Explanation: Gross primary productivity (GPP) of an ecosystem is the rate of production of organic matter during photosynthesis. Net primary productivity (NPP) is the amount of organic matter stored by producers per unit area per unit time. Gross primary productivity minus respiration losses is the net primary productivity i.e., NPP=GPP−R. 7. Option (a) is correct. Explanation: Low temperature and aerobiosis inhibit decomposition resulting in the building of organic materials because decomposition is largely an oxygen-requiring process. 8. Option (b) is correct. Explanation: The vertical distribution of different species occupying different levels is called stratification. For example, trees occupy the top vertical strata or layer of a forest, shrubs the second and herbs and grasses occupy the bottom layers. 9. Option (a) is correct. Explanation: Explanation The sun is the ultimate source of energy for all living organisms. About 1−5% of incident solar energy or 2−10% of PAR is captured by the photosynthetic organisms for the synthesis of organic matter (Gross primary productivity). 10. Option (a) is correct. Explanation: Green plants capture about 1% of the energy of sunlight that falls on their leaves for the process of photosynthesis. This energy is converted into chemical energy as food.

24

[C] COMPETENCY/CASE-BASED QUESTIONS: 1. Option (d) is correct. Explanation: The altitude of an area can impact like climate that is mostly rainy and have low temperature while the altitude of an area impact the ultimate with respect to the equator and poles. 2. Option (d) is correct. Explanation: The lithosphere is concerned with land, the hydrosphere with water bodies and the atmosphere is concerned with space. The hydrological cycle circulates in all these three subsystems. 3. Option (c) is correct. Explanation: More will be the water holding capacity of the soil, less will be the air spaces less soil aeration. 4. Option (a) is correct. Explanation: Edaphology is the study of soil useful for plant growth. 5. Option (b) is correct. Explanation: Euryhaline animals like salmon can adopt a wide range of salinity as they are found in areas where rivers and sea meet. 6. Option (a) is correct. Explanation: A.G Tansley in 1935 coined the term Ecosystem. 7. Option (d) is correct. Explanation: Manmade ecosystems are called Agro ecosystems. It is an ecosystem on agricultural land. 8. Option (c) is correct. Explanation: Stratification, Diversity and species composition are all structural aspects of an ecosystem. On the other hand, productivity is the rate of biomass accumulation at a specific period by the ecosystem. It is a functional aspect. 9. Option (a) is correct. Explanation: Oceans are the most stable ecosystems. Other terrestrial ecosystems undergo changes and succession of biotic components. But oceans remain stable for a long duration of time. 10. Option (d) is correct. Explanation: Decomposers are vital to the health of the ecosystem because they prevent nutrients from being tied up in one place for extended periods.

Study Time Max. Time: 1:5 Hours Max. Questions: 50

CHAPTER

15

BIODIVERSITY AND ITS CONSERVATION

 Revision Notes:

  Biodiversity: It is the diversity (or heterogeneity) of biological organisation ranging from cellular macromolecules to biomes.   Edward Wilson: Popularized the term ‘biodiversity’ to describe the combined diversity at all levels of biological organization.  Levels of Biodiversity {  Biodiversity has been divided into three hierarchical levels of biological organization. 1. Genetic diversity (a)  Diversity shown by a single species at the genetic level. e.g., Rauwolfia vomitoria in Himalaya shows genetic variation in the potency and concentration of the chemical, reserpine. (b) India has more than 50,000 different strains of rice and 1,000 varieties of mango. 2. Species diversity Diversity at the species level. e.g., Western Ghats have greater amphibian species than the Eastern Ghats. 3. Ecological diversity Diversity at ecosystem level. For e.g., In India, deserts, rain forests, mangroves, coral reefs, wetlands, estuaries & alpine meadows, all can be seen, whereas the Scandinavian countries (like, Norway, Sweden) have less ecological diversity. {  Number of Species on Earth (Global Species Diversity): {  According to IUCN or International Union for Conservation of Nature & Natural Resources (2004) more than 1.5 million species have been described so far. { According to Robert May, the global estimate is about 7 million (considering the species are to be discovered in the tropics. i.e., only 22% of the total species have been recorded so far). {  Animals are more diverse (above 70%) than Plants including plantae and fungi (22%). { Most species-rich taxonomic group among animals are: Insects (70%, i.e., out of every 10 animals, 7 are insects). { Number of fungal species is more than the combined

total of the species of fishes, amphibians, reptiles and mammals. { Biologists are not sure about total number of prokaryotic species because: (a) Conventional taxonomic methods are not suitable for identifying microbial species. (b) Many species are not culturable under laboratory conditions.

Patterns of Biodiversity {  Biodiversity

is not uniform throughout the world. It varies with the change in latitude and altitude, it is affected by latitudinal gradients and species-area relationship. Following are the main patterns of biodiversity. (a) Latitudinal Gradients Species diversity decreases from the equator to the poles. Tropics (latitudinal range of 23.5° N to 23.5° S) have more species than temperate or polar areas. Tropical forest region like Ecuador has up to 10 times species of vascular plants as compared to a forest of equal area in a temperate region like the Midwest of USA. Tropical Amazonian rain forest (South America) has the greatest biodiversity on earth. Biodiversity (species richness) is highest in tropics because (i)  Tropics had more evolutionary time. (ii)  Relatively constant environment (less seasonal). (iii)  They receive more solar energy which contributes to greater productivity. (b) Species - Area Relationship According to the study of Alexander von Humboldt (German naturalist & geographer) in South American jungles, within a region, species richness increases with an increase in explored area, but only up to a limit. Relation between species richness and area for a wide variety of taxa (like, angiosperm plants, birds, freshwater fishes) gives a rectangular hyperbola.

163

BIODIVERSITY AND ITS CONSERVATION BIODIVERSITY AND ITS CONSERVATION

271

164 Oswaal CUET (UG) Chapterwise Question Bank S = CA

Z

Species richness

Log S = log C + Z log A

le

ca

s log

-

log

log

Area On a logarithmic scale, the relationship is a straight line or linear, described by the equation: log S = log C + Z log A where, S = Species richness, A = Area, C = Y-intercept, Z = slope of the line (regression co-efficient) The value of Z lies in the range of 0.1 to 0.2. In the species-area relationship among the large areas like entire continents, the slope of the line is steeper (Z value: 0.6 to 1.2). e.g., for frugivorous birds and mammals in the tropical forests of different continents, the slope is 1.15.

Importance of Species Diversity to the Ecosystem { For many decades, ecologists believed that communities

with more species, generally, tend to be more stable than those with fewer species. { A stable community should not show too much variation in productivity from year to year; it must be either resistant or resilient to occasional disturbances (natural or man-made), and it must also be resistant to invasions by alien species. { David Tilman found that plots with more species showed less year-to-year variation in total biomass. { He also showed that in his experiments, increased diversity contributed to higher productivity. { A rich biodiversity is not only essential for ecosystem health but imperative for the survival of the human race on this planet. { Stanford ecologist Paul Ehrlich explained the effect of loss of species through his ‘rivet popper hypothesis’.

Rivet popper hypothesis: This hypothesis was used by Stanford ecologist Paul Ehrlich. In an airplane, all parts are joined together using thousands of rivets (species). If every passenger travelling in air plane starts popping a rivet to take home (causing a species to become extinct), it may not affect flight safety (proper functioning of ecosystem) initially, but as more and more rivets are removed, the planes become dangerously weak over a period of time. Loss of rivets on the wings (key species that drives major ecosystem functions) is more serious threat to flight safety than loss of a few rivets on the seats or windows inside the plane.

Loss of Biodiversity

BIOLOGY

Scan to know

more about Red List (2004) says that this topic 784 species (338 vertebrates, 359 invertebrates and 87 plants) became extinct in the last 500 years. e.g., Dodo (Mauritius), Quagga (Africa), Thylacine Loss of Biodiversity (Australia), Stellar ’s sea cow (Russia) and 3 subspecies (Bali, Java, Caspian) of the tiger. { 27 species have disappeared in the last 20 years. { The extinctions across taxa are not random. Some groups (like amphibians) appear to be more vulnerable to extinction. { More than 15,500 species are facing the threat of extinction. { 12% birds, 23% mammals, 32% amphibians, 31% gymnosperm species face the threat of extinction. { On earth, there have been five mass extinctions of species and at present ‘Sixth Extinction’ is in progress. { The current extinction rate is 100 - 1000 times faster than in pre-human times. If this trend continues, nearly 50% species might be extinct within the next 100 years. { IUCN

Impacts of Loss of Biodiversity (a) Decline in plant production. (b)  Lowered resistance to some environmental perturbations such as drought. (c)  Increased variability in ecosystem processes such as plant productivity, water use and pest and disease cycles.

Causes of Biodiversity Losses (‘The Evil Quartet’) { “The

Evil Quartet” is the phrase coined by Jared Diamond to describe the four human induced causes of extinction. (a) Habitat Loss and Fragmentation It is the most important cause. For e.g., Tropical rain forests (loss from 14% to 6%). Thousands of hectares of rain forests are being lost within hours. The Amazon rain forest (‘lungs of the planet’) is being cut for cultivating soya beans or for the conversion of grasslands for cattle. When large habitats are broken up into small fragments due to various human activities, mammals and birds requiring large territories and certain animals with migratory habits are badly affected, leading to population decline. (b) Over-exploitation The dependence of humans on nature for food and shelter led to the over-exploitation of natural resources. Example: Many species like Stellar ’s sea cow, Passenger pigeon, etc, became extinct due to overexploitation. Many marine fish populations around the world are over-harvested, endangering the continued existence of some commercially important species.

165

BIODIVERSITY AND ITS CONSERVATION

(c) Alien Species Invasions When alien species are introduced unintentionally or deliberately, some of them turn invasive, and cause the decline or extinction of indigenous species. These alien species cause decline or extinction of indigenous species. Example: (a) The Nile Perch introduced in Lake Victoria (East Africa) caused extinction of more than 200 species of cichlid fish. (b)  Invasive weed species like carrot grass (Parthenium), Lantana and water hyacinth (Eichhornia) caused damage to our native species. (c) The illegal introduction of the African Catfish (Clarias gariepinus) for aquaculture is posing a threat to the indigenous catfish (Clarias batrachus) in our rivers. (d) Co-extinction When a species becomes extinct, the plant and animal species associated with it also become extinct. Example: (a) Extinction of the parasites takes place when the host is extinct. (b)  In co-evolved plant-pollinator mutualism extinction of one leads to the extinction of the other.

Conservation Processes There are three main reasons for conserving the biodiversity which are categorized as follows: (a) Narrowly Utilitarian Arguments Humans derive economic benefits from nature such as food, firewood, fibre, construction material, industrial products (tannins, lubricants, dyes, resins, perfumes) and medicines. More than 25% of the drugs are derived from plants. 25,000 species of plants have medicinal value. Exploring molecular, genetic and species-level diversity for i.e., ‘bioprospecting’ products of economic importance may enormously benefit nations with rich biodiversity. (b) Broadly Utilitarian Arguments Biodiversity has many ecosystem services. Amazon forest produces 20% of total O2 in the earth’s atmosphere by the process of photosynthesis. Pollination service takes place through bees, bumblebees, birds and bats. Aesthetic pleasures such as walking through thick woods, watching spring flowers in full bloom or waking by hearing a bulbul’s song in the morning. Other indirect benefits are pest control, climate moderation and flood control. (c) Ethical Arguments Every species has an intrinsic value. We have a moral duty to take care for their wellbeing.

Conservation of Biodiversity Types of Conservation

Scan to know

more about (a) In situ conservation (on site) this topic  It is the conservation of genetic resources within natural or human-made ecosystems in which they occur. hotspots of biodiversity Examples: Protected areas such as National Parks, Sanctuaries, Biosphere reserves, cultural landscapes, national monuments. (i) National Park ƒ Strictly reserved for the welfare of the wildlife where private ownership, cultivation, grazing, etc., are prohibited. ƒ There are 90 national parks in India. (ii) Sanctuary ƒ Here, protection is given only to the animals. ƒ Collection of timbers, minor forest products and private ownership are allowed so long as they do not harm the animals. ƒ There are 553 wildlife sanctuaries in India. (iii) Biosphere Reserves ƒ Areas of land or coastal environments ƒ to conserve the ecosystem and genetic ƒ resources contained therein. ƒ There are 18 biosphere reserves in India. (iv) Sacred Forests (Sacred Groves) ƒ Sacred groves are highly Scan to know protected forests because more about of religious and cultural this topic traditions. ƒ Sacred groves in Khasi and Jaintia Hills in Meghalaya. Sacred Groves ƒ  Aravalli Hills of Rajasthan. ƒ Western Ghat regions of Karnataka & Maharashtra. ƒ Sarguja, Chanda and Bastar areas of ƒ Madhya Pradesh. ƒ In Meghalaya, the sacred groves are the last refuges for a large number of rare and threatened plants. (v) Hot spots: It is a Scan to know biogeographic region which more about serve as significant reservoir this topic of bio-diversity but they are threatened due to degradation, illegal logging, etc. Biodiversity and ƒ  These are the richest Conservation and the most threatened reservoirs of plant and animal life on earth. ƒ There are 36 hotspots in the world.

166 Oswaal CUET (UG) Chapterwise Question Bank total, all the biodiversity hotspots cover less than 2% of the earth’s land area but could reduce the ongoing extinctions by almost 30%. ƒ  Three main hotspots (Western Ghats and Sri Lanka, Indo-Burma and Himalaya) cover India’s biodiversity regions. (b) Ex situ conservation (off site)  It is the conservation of organisms outside their habitats.  In this approach, threatened animals and plants are taken out from their natural habitat and placed in a special setting where they can be protected and given special care. For e.g., genetic resource centres, zoological parks, botanical gardens, gene banks etc. In recent years, ex-situ conservation has advanced by preserving the gametes of threatened species in viable and fertile condition for long periods using

BIOLOGY

cryopreservation techniques, eggs can be fertilised in-vitro, and plants can be propagated using tissue culture methods. Seeds of different genetic strains of commercially important plants can be kept for long periods in seed banks.

ƒ In

International Efforts for Conserving Biodiversity { The

Earth Summit (Rio de Janeiro, 1992) -Three objectives: (a) Conservation of biodiversity (b) Sustainable use of biodiversity (c) Sharing of benefits in the utilization of genetic resources. { The World Summit on Sustainable Development (Johannesburg, South Africa, 2002): 190 countries pledged to reduce the current rate of biodiversity loss.

OBJECTIVE TYPE QUESTIONS [A] MULTIPLE CHOICE QUESTIONS: 1. Biodiversity is affected by: (a) latitudinal gradients (b) species-area relationship (c) Both (a) and (b) (d) None of the above  2. The historic convention on Biological diversity held in Rio de Jenerio in 1992 is also called: [NCERT Exemplar, Q. 19, Pg. 19] (a) The world summit. (b) MAB programme. (c) The earth summit. (d) G-16 summit. 3. Diversity of habitat over the total landscape is called: (a) β –diversity (b) ɣ-diversity. (c) Landscape diversity (d) Ecosystem diversity  4. Which of the following area is India’s one of the biodiversity hotspots? (CUET 2023) (a) Eastern ghats. (b) Indo-Burma plains. (c) Aravali hills. (d) Indo-Gangetic plains. 5. Introduction of which one of the following organism’s species did cause decline or extinction of indigenous species?  (CUET 2022) (a) Eicchornia Crassipes (b) Nile Perch (c) Clarias gariepinus (d) Steller’s Sea cow 6. What is common to Lantana, Eichhornia and African catfish? (a) All are endangered species of India. (b) All are key stone species. (c) All are mammals found in India. (d) All the species are neither threatened, nor indigenous species of India. 7. Which one of the following is an endangered plant species of India?

(a) (b) (c) (d) 8.

Rauwolfia serpentina Santalum album (Sandal wood) Cycas beddonei All of the above  Which of the following statements is correct? [NCERT Exemplar, Q. 13, Pg. 108] (a) Parthenium is an endemic species of our coumtry. (b) African catfish is not a threat to indigenous catfishes. (c) Steller’s sea cow is an extinct animal. (d) Lantana is popularly known as carrot grass. 9. Following are the animals that recently became extinct, except: (CUET 2022) (a) Dodo (b) Quagga (c) Thylacine (d) Clarias garipinus 10. Modern ex-situ conservation includes: (1) Cryopreservation techniques. (2) in vitro-fertilisation. (3) propagation of plants by using tissue culture methods. (a) Only 1. (b) Only 2. (c) 2 and 3. (d) All of these. 11. The extinction of passenger pigeon was due to: (a) Increased number of predatory birds. (b) Over-exploitation by hum (c) Non-availability of the food. (d) Bird flu virus infection.  12. Amongst the animal groups given below, which one has the highest percentage of endangered species? [NCERT Exemplar, Q. 9, Pg. 107] (a) Insects (b) Mammals (c) Amphibians (d) Reptiles 13. On a logarithmic scale, how is the equation S = CAz described? (a) log C / Z log A = log S] (b) log C + Z log A = log S (c) log C x log ZA = log S (d) -Z log A – log C = log S

BIODIVERSITY AND ITS CONSERVATION

14. In which range the value of Z lies in Q-13. (a) 0.1 to 0.2 (b) 1 to 2 (c) 0.001 to 0.002 (d) 10 to 20 15. Which one of the following is not a major characteristic feature of biodiversity hot-spots? [NCERT Exemplar, Q. 5, Pg. 106] (a) Large number of species (b) Abundance of endemic species (c) Large number of exotic species (d) Mostly located in polar regions 16. The species that invades a bare area is called .................... . (CUET 2022) (a) Seral stage (b) Alien species (c) Endemic species (d) Pioneer species 17. Which of the following countries has the highest biodiversity? [NCERT Exemplar, Q. 1, Pg. 106] (a) Brazil (b) South Africa (c) Russia (d) India 18. Where among the following will you find pitcher plant? [NCERT Exemplar, Q. 4, Pg. 106] (a) Rain forest of North-East India (b) Sundarbans (c) Thar desert (d) Western ghats 19. How much area of the Earth’s surface do the biodiversity hotspots cover? (a) 8 % (b) 5 % (c) 10 % (d) 2 %  20. Sacred groves are specially useful in: (a) Preventing soil erosion (b) Year-round flow of water in rivers (c) Conserving rare and threatened species (d) Generating environmental awareness 21. By which of the following technique the gametes of threatened species are preserved in viable and fertile conditions for long periods? (a) Botanical gardens (b) Cryopreservation techniques (c) Zoological parks (d) Wildlife safari parks  22. What is common to the techniques (i) in-vitro fertilisation, (ii) Cryopreservation and (iii) tissue culture? (a) All are in-situ conservation methods. (b) All are ex-situ conservation methods. (c) All require ultra-modern equipment and large space. (d) All are methods of conservation of extinct organisms. 23. Which of the following is not a cause for loss of biodiversity? [NCERT Exemplar, Q. 2, Pg. 106] (a) Destruction of habitat (b) Invasion by alien species (c) Keeping animals in zoological parks (d) Over­exploitation of natural resources 24. Which one of the following shows maximum genetic diversity in India ? (a) Rice (b) Maize (c) Mango (d) Groundnut 

167 25. Which of the following represent maximum number of species among global biodiversity? (a) Mosses and ferns (b) Algae (c) Lichens (d) Fungi 26. Which region had a long evolutionary time for species diversification? (a) Temperate environments (b) Polar environments (c) Tropical environments (d) Arctic environments 27. Cryopreservation of gametes of threatened species in viable and fertile condition can be referred to as: (a) In-situ conservation of biodiversity (b) advanced ex-situ conservation of biodiversity. (c) In-situ conservation by sacred groves. (d) In-situ cryo-conservation of biodiversity 28. The one-horned rhinoceros is specific to which of the following sanctuary? [NCERT Exemplar, Q. 8, Pg. 107] (a) Bhitar Kanika (b) Bandipur (c) Kaziranga (d) Corbett park 29. The active chemical drug reserpine is obtained from [NCERT Exemplar, Q. 16, Pg. 108] (a) Datura. (b) Rauwolfia. (c) Atropa. (d) Papaver. 30. What is common to the following plants: Nepenthes, Psilotum, Rauwolfia & Aconitum? [NCERT Exemplar, Q. 7, Pg. 107] (a) All are ornamental plants. (b) All are phylogenetic link species. (c) All are prone to over-exploitation. (d) All are exclusively present in the eastern Himalayas. [B] ASSERTION & REASON: Directions: In the following questions, a statement of assertion (a) is followed by a statement of reason (R). Mark the correct choice as: (A) Both assertion (A) and reason (R) are true and reason (R) is the correct explanation of assertion (A). (B) Both assertion (A) and reason (R) are true but reason (R) is not the correct explanation of assertion (A). (C) Assertion (A) is true but reason (R) is false. (D) Assertion (A) is false but reason (R) is true. 1. Assertion (A): Sacred groves are highly protected. Reason (R): They are of religious importance to the communities. 2. Assertion: Process of extinction is random and fast. Reason: Any species which can not adapt itself according to the environment cannot survive. 3. Assertion (A): Western ghats of India are one of the hotspots of biodiversity. Reason (R): Western ghats have greater amphibian diversity than Eastern ghats.  4. Assertion (A): Loss of habitat is the main cause of loss of biodiversity. Reason (R): This causes the increase in edge area and reduction in core area.

168 Oswaal CUET (UG) Chapterwise Question Bank 5. Assertion (A): Percentage of animals is more than the percentage of plants. Reason (R): 70% of all the species are animals, while plants contribute nearly 22% of the total. 6. Assertion (A): Biologists are not sure about total number of prokaryotic species. Reason (R): Nearly 45,000 species of plants and twice as many of animals have been recorded from India. 7. Assertion (A): Biosphere reserves are also included under the ex-situ conservation strategies. Reason (R): Cropping and grazing are allowed in the transition zone of biosphere reserve. 8. Assertion (A): There is more species biodiversity in tropical latitudes than in temperate ones. Reason (R): Tropical environments, unlike temperate ones, are less seasonal, relatively more constant and predictable  9. Assertion (A): Many species like Stellar ’s sea cow, passenger pigeon, etc., became extinct due to over exploitation. Reason (R): Over exploitation is one of the major causes of biodiversity loss. 10. Assertion (A): Biodiversity means diversity at species level. Reason (R): Biodiversity is a diversity not only at species level but at all levels of biological organization.  [C] COMPETENCY/CASE-BASED QUESTIONS: I. Read the passage carefully and answer the question given after the passage from 1–5. he biological wealth of our planet has been declining rapidly and the accusing finger is clearly pointing to human activities. The colonisation of tropical pacific islands by humans is said to have led to extinction of more than 2000 species of native birds. The IUCN red list documents the extinction of 784 species. Since the origin and diversification of life on earth, there were five episodes of mass extinction of species. Current species extinction rate as estimated to be 100-1000 times faster. There are four major causes called “The Evil quartet” Habitat loss, over exploitation, Alien species invasion and co-extinction. Conservation efforts are necessary to preserve biodiversity and protect endangered species and their habitats. 1. Tissue culture and cryopreservation are the techniques used: (a) In in-situ conservation. (b) To control conservation of medicinal plants only. (c) In ex-situ conservation method. (d) Used in conventional breeding programme. 2. Mr. X wants to understand the ‘Evil Quartet’ to know the reasons of Biodiversity loss. It is: (a) Habitat loss and fragmentation only. (b) Over exploitation and Alien species invasions only. (c) Co-extinctions and Alien-species invasion.

BIOLOGY

(d) Habitat loss, over exploitation, Alien species invasions and co-extinctions 3. Which one of the following is NOT the reason for decline in biodiversity? (a) Pollution (b) Agriculture (c) Sacred groves (d) Fishing 4. Select the one which is NOT an ex-situ conservation of biodiversity. (a) Zoological parks (b) Botanical gardens (c) National parks (d) Seed banks 5. Identify the in-situ ways to conserve biodiversity from the following: (1) Zoological parks (2) Eggs fertilised in-vitro (3) Sacred groves (4) Biosphere reserves Choose the correct answer from the options: (a) 1, 2 only (b) 2, 3 only (c) 3, 4 only (d) 1, 4 only 2. Read the following text and answer the following questions based on the same: Excessive exploitation of species, whether a plant or animal reduces the size of its population, so it becomes vulnerable to extinction. Dodo and passenger pigeon have become extinct due to over exploitation by hum Thus, the world is facing accelerated rates of species extinctions, largely due to human interference. 6. Identify a cause of biodiversity loss that is not included in evil quartet? (a) Co-extinction (b) Pollution (c) Alien species invasion (d) Habitat loss and fragmentation 7. The species that have become extinct due to over exploitation is/are: (a) Stellar’s sea cow (b) Yucca moth (c) Blatta orientalis (d) Nile Perch 8. Factors which make species susceptible to extinction are: (a) Small population size (b) Higher status of tropic level. (c) Inability to switch over to alternate food. (d) All of the above 9. Which term is used to refer the species whose populations have been seriously depleted and whose ultimate security is not assured? (a) Threatened species (b) Endangered species (c) Vulnerable species (d) Rare species 10. Blue whale is placed under: (a) Endangered (b) Critically endangered (c) Rare (d) Extinct

169

BIODIVERSITY AND ITS CONSERVATION

ANSWER KEY [A] MULTIPLE CHOICE QUESTIONS 1. (c)

2. (c)

3. (b)

4. (b)

5. (b)

6. (d)

7. (d)

8. (c)

9. (d)

10. (d)

11. (b)

12. (c)

13. (b)

14. (a)

15. (d)

16. (d)

17. (a)

18. (a)

19. (d)

20. (c)

21. (b)

22 (b)

23. (c)

24. (a)

25. (d)

26. (c)

27. (b)

28. (c)

29. (b)

30. (c)

8. (a)

9. (b)

10. (d)

9. (b)

1. (a)

[B] ASSERTION & REASON 1. (a)

2. (d)

3. (b)

4. (a)

5. (a)

6. (c)

7. (d)

[C] COMPETENCY/CASE-BASED QUESTIONS 1. (c)

2. (d)

3. (c)

4. (a)

5. (c)

6. (b)

7. (a)

8. (d)

ANSWERS WITH EXPLANATION [A] MULTIPLE CHOICE QUESTIONS: 1. Option (c) is correct. Explanation: Biodiversity is not uniform throughout the world. It varies with the change in latitude and altitude, it is affected by latitudinal gradients and species-area relationship. 2. Option (c) is correct. Explanation: The United Nations Conference on Environment and Development (UNCED), also known as the Rio de Janeiro Earth Summit, the Rio Summit, the Rio Conference, and the Earth Summit was a major United Nations conference held in Rio de Janeiro in 1992. The Earth Summit called upon all nations to take appropriate measures for conservation of biodiversity and sustainable utilisation of its benefits. 3. Option (b) is correct. Explanation: Gamma diversity is the diversity of habitat over a total landscape. Gamma diversity (γ - diversity) is the total species diversity in a landscape 4. Option (b) is correct. Explanation: The correct answer is the Indo-Burma plains. There are 4 biodiversity hotspots: the Himalayas, the Western Ghats, the Indo-Burma region, and the Sundaland. 5. Option (b) is correct. Explanation: The Nile perch introduction into Lake Victoria led eventually to the extinction of ecologically unique assemblage of more than 200 species of cichlid fish in the lake. 6. Option (d) is correct. Explanation: Lantana, Eichhornia (water hyacinth) and African catfish (Clarias gariepinus) are neither threatened, nor indigenous species of India. They all are alien (or exotic) species which are invasive and have a harmful impact on the indigenous species leading to their extinction. 7. Option (d) is correct. Explanation: All of the above-mentioned plants are endangered plant species of India. Rauwolfia serpentina (sarpgandha), Santalum album (sandal wood) and Cycas beddonei are facing the threat of extinction due their medicinal and commercial importance. 8. Option (c) is correct. Explanation: Stellar’s sea cow (Russia) is a recently extinct animal. Parthenium (carrot grass) is an exotic weed, which grows rapidly and adversely affects the native species. African catfish is also an alien species which adversely affects the

growth of indigenous Clarias batrachus - our local species. Lantana camara, another alien species, is also strongly competing with native species. 9. Option (d) is correct. Explanation: Clarias garipinus is indigenous to the inland waters of much of Africa and they are also endemic in Asia Minor in countries such as Israel. It is commonly known as African Catfish. It is a hardy fish that can be densely stocked in low oxygen waters making it ideal for culture in areas with a limited water supply. It’s high fecundity, fast growth rate resistance to disease, etc., lead to its illegal introduction for agriculture purposes is posing a threat to it. 10. Option (d) is correct. Explanation: Ex situ conservation involves conservation of threatened animals and plants away from their natural habitat in a special protected environment. Examples of ex situ conservation methods include in vitro fertilisation, tissue culture, cryopreservation, gene banks, zoological parks, botanical gardens etc 11. Option (b) is correct. Explanation: Extinction of passenger pigeon (Ectopistes migratorius) was due to the combined effects of their natural dramatic population fluctuations and human over-exploitation. Factors like increase in number of predatory birds, nonavailability of food and bird flu virus infection did not contribute to the extinction of passenger pigeon. 12. Option (c) is correct. Explanation: Amphibians have the highest percentage presently, 32% of all the amphibian species in the world face the threat of extinction. Other than these, 23% of all mammal species and 12% of all bird species also facing the risk. 13. Option (b) is correct. Explanation: Explanation: On a logarithmic scale, S = CAz is described as log S = log C + Z log A. Here S = Species richness, A = Area explored, Z = slope of the line (regression coefficient), and C = Y-intercept. 14. Option (a) is correct. Explanation: According to ecologists, the value of Z ranges from 0.1 to 0.2. It is regardless of the taxonomic region or group and the slopes of the regression line are mostly similar. 15. Option (d) is correct. Explanation: Biodiversity hot-spots specifically refer to 25 biologically rich areas around the world that have lost at least 70% of their original habitat. They are characterised by the

170 Oswaal CUET (UG) Chapterwise Question Bank large number of flora and fauna, abundance of endemic species and also large number of alien or exotic species. They are mostly found in tropical and temperate regions. There are no biodiversity hot-spots in polar regions. 16. Option (d) is correct. Explanation: The species that invade a bare area is called pioneer species. 17. Option (a) is correct. Explanation: Brazil is the country with the greatest biodiversity of flora and fauna on the planet. It has the highest number of species of known mammals and fresh-water fish, and more than 50,000 species of trees and bushes. It takes first place in plant diversity. The climate of Brazil remains relatively undisturbed, constant and predictable giving tune for diversification, which favours rich biodiversity. 18. Option (a) is correct. Explanation: Pitcher plant (Nepenthes) is an insectivorous plant, found in rain forest of North-East India. They generally grow in nitrogen-deficient soil. Sundarbans are rich in fauna and flora. Mangrove plants are the characteristic of Sundarban Western Ghats are rich in biodiversity. Xerophytes are more common in Thar deserts. 19. Option (d) is correct. Explanation: A biogeographic region with significant levels of biodiversity having a large number of endemic species that are threatened by human habitation is known as biodiversity hotspots. In all, there are 34 biodiversity hotspots present throughout the world covering up to 2 % of the Earth’s surface. 20. Option (c) is correct. Explanation: Sacred grooves are method for in situ conservation of biodiversity and conserving rare threatened species. 21. Option (b) is correct. Explanation: The technique by which the gametes, organelles, cells, tissues, or any other biological construct is preserved by cooling the samples to very low temperatures such that they remain in viable and fertile conditions for long periods is known as cryopreservation. It is used for the conservation of the threatened species. 22. Option (b) is correct. Explanation: In-vitro fertilisation, cryopreservation and tissue culture are all ex-situ conservation methods. Ex-situ (off site) conservation is a set of conservation techniques which involves the transfer of a target species away from its native habitat. At present, gametes of threatened species can be preserved in viable conditions (at very low temperature −196°C) for longer duration by cryopreservation method. Fertilisation can be achieved in laboratory through the process of in-vitro fertilization, and species can be propagated by tissue-culture method which is a plant breeding principle. 23. Option (c) is correct. Explanation: Keeping animals in zoological parks is not a cause for loss of biodiversity. The important factors causing loss of biodiversity are as follows: (i) Destruction of natural habitat (primary cause). (ii) Introduction of exotic (alien species) and indigenous species. (iii) Over-exploitation of natural resources. (iv) Co-extinction of species. 24. Option (a) is correct. Explanation: Rice has more than 50,000 genetically different strains while Mango has 1000 varieties in India

BIOLOGY

25. Option (d) is correct. Explanation: The number of fungi species in the world is more than the combined total of the species of fish, amphibians, reptiles & mammals. 26. Option (c) is correct. Explanation: Tropical environments have remained undisturbed for millions of years that are a long evolutionary time. Due to such environmental consistency, they promote specialization in the niche and lead to a greater diversification of species. 27. Option (b) is correct. Explanation: In recent years, ex-situ conservation has advanced by preserving the gametes of threatened species in viable and fertile condition for long periods using cryopreservation techniques. 28. Option (c) is correct. Explanation: One-horned rhinoceros is specific to Kaziranga National Park, which is situated in Golaghat and Nagaon districts of (Assam). Bhitar Kanika National Park is located in Odisha and is specific for salt water crocodile. Bandipur National Park (Mysore) too is specific for tigers. Corbett National Park is situated in district Nainital (Uttarakhand) and is specific for tiger. It is the first National Park of India, which is famous for tigers. 29. Option (b) is correct. Explanation: Reserpine is an alkaloid found in the roots of Rauwolfia serpentina and R. vomitoria. It is used with or without other medications to treat high blood pressure (hypertension). It lowers blood pressure by slowing down the nervous system. This allows blood vessels to relax and dilate (widen), which helps heart beats more slowly and improves blood flow. 30. Option (c) is correct. Explanation: All of the above plants are prone to overexploitation due to their respective properties. Nepenthes (pitcher plant) are an insectivorous plant. Psilotum is a pteridophyte Rauwolfia and Aconitum are medicinal plants. [B] ASSERTION REASON QUESTIONS: 1. Option (a) is correct. Explanation: Sacred groves are highly protected by certain communities because they are of religious importance to the communities. They have a significant role in in-situ conservation. 2. Option (d) is correct. Explanation: The process of extinction is slow and unevenly distributed. For example certain species of amphibians are slowly more susceptible to extinct than the others. 3. Option (b) is correct. Explanation: Western ghats is one of the eight hotspots of biological diversity in the world due to its high level of biological diversity and endemism. 4. Option (a) is correct. Explanation: There are many reasons behind the loss of biodiversity among them habitat loss is the major one; others are over-exploitation, alien-species invasions, co-extinctions and deforestation. It the habitat loss writinues, total core area and cohesion of habitat are retuned thereby increasing the edge density. 5. Option (a) is correct. Explanation: Animals constitute more than `70%` of all the species recorded whereas plants constitute around `22%` of total number of species.

BIODIVERSITY AND ITS CONSERVATION

6. Option (c) is correct. Explanation: Biologists are not sure about total number of prokaryotic species because: (a) Conventional taxonomic methods are not suitable for identifying microbial species. (b) Many species are not culturable under laboratory conditions. 7. Option (d) is correct. Explanation: Biosphere reserves are included under the in situ conservation strategies. 8. Option (a) is correct. Explanation: Biodiversity (species richness) is highest in tropics because (i) Tropics had more evolutionary time. (ii) Relatively constant environment (less seasonal). (iii) They receive more solar energy which contributes to greater productivity. 9. Option (b) is correct. Explanation: Many species like Stellar’s sea cow, passenger pigeon, etc., became extinct due to over exploitation. The dependence of humans on nature for food and shelter led to over exploitation of natural resources. It is one of the major causes of biodiversity loss. 10. Option (d) is correct. Explanation: The term biodiversity is popularized by Edward Wilson. It is a diversity at all levels of biological organization from macromolecules within cells to the biomes. [C] COMPETENCY/CASE-BASED QUESTIONS: 1. Option (c) is correct. Explanation: Tissue culture and cryopreservation are the techniques used in ex-situ conservation method. It is the conservation of organisms outside their habitats. 2. Option (d) is correct. Explanation: Habitat loss & fragmentation, exploitation, alien species invasions & co-extinctions are four major causes of biodiversity losses, which together are called “The Evil Quarter”. 3. Option (c) is correct. Explanation: Sacred groves are forests which are regenerated around places of worship. Sacred groves help in protection of many rare, threatened and endemic species of plants and

24

171 animals found in an area. Pollution, agriculture and fishing are the reasons for decline in biodiversity. 4. Option (a) is correct. Explanation: National parks are in-situ conservation of biodiversity. Zoological parks, botanical gardens and seed banks are ex-situ conservation methods for the protection of species. 5. Option (c) is correct. Explanation: Sacred groves and biosphere reserve are in-situ conservation strategies for protection of biodiversity. In situ conservation (on site) is the conservation of genetic resources within natural or human-made ecosystems in which they occur. Zoological parks are ex-situ conservation strategies for biodiversity protection. Ex situ conservation (off site) is the conservation of organisms outside their habitats. When the fertilization of egg is done outside the body, it is called in-vitro fertilization. 6. Option (b) is correct. Explanation: Evil quartet, i.e., four major causes of biodiversity loss is habitat loss and fragmentation, over exploitation, alien species invasion and co-extinction. Pollution is also one of the causes of biodiversity loss but included in evil quartet. 7. Option (a) is correct. Explanation: Steller’s sea cow has become extinct in the last 500 years due to over exploitation by hum 8. Option (d) is correct. Explanation: Population traits which make species susceptible to extinction are: small population, higher status of trophic level and inability to switch over to alternate foods. 9. Option (b) is correct. Explanation: Those species whose populations have been seriously depleted and whose ultimate security is not assured are known as endangered species. 10. Option (a) is correct. Explanation: Today, blue whales are listed as endangered under the Endangered Species Act and protected under the Marine Mammal Protection Act as they were significantly getting depleted by commercial whaling activities worldwide.

Study Time Max. Time: 1:5 Hours Max. Questions: 50

CHAPTER

16

ENVIRONMENTAL ISSUES

 Revision Notes:

 Introduction { Human population explosion increases the demand for food, water, home, electricity, roads, automobiles, etc. { It leads to pollution of air, water and soil and depletion of valuable natural resources. { Pollution is any undesirable change in physical, chemical or biological characteristics of air, land, water or soil. { Agents that cause pollution are called pollutants. { The Government of India has passed the Environment (Protection) Act, 1986 to control environmental pollution and protect and improve the quality of our environment.

Air Pollution and Its Control

 Causes of Air Pollution { Particulate and gaseous air pollutants from smokestacks of thermal power plants, Scan to know smelters and other industries more about this topic release particulate and gaseous air pollutants. {  According to Central Pollution Control Board (CPCB), par­ ticulate size of less than 2.5 mm Air Pollution in diameter (PM 2.5) causes the greatest harm to human health. { Pollutants from automobiles. { Use of leaded petrol.  Harmful Effects of Air Pollution { Air pollutants cause injury to all living organisms. { They reduce the growth and yield of crops and cause premature death of plants. { Air pollutants affect the animals and humans respiratory system. { It causes respiratory problems, irritation, inflammations and damage to lungs and premature deaths.  Control of Air Pollution (a) Particulate matters must be separated/filtered out before releasing the harmless gases into the atmosphere. (b) Use of catalytic converters (having Platinum-Palladium & Rhodium as the catalysts). It reduces the emission of poisonous gases.  This converts unburnt hydrocarbons to CO2 and water, and carbon monoxide and nitric oxide to CO2 and nitrogen gas, respectively.

 Motor

vehicles having catalytic converter should use unleaded petrol because lead in the petrol inactivates the catalyst. (c) Proper maintenance of automobiles along with use of lead-free petrol or diesel can reduce the pollutants they emit. (d) Catalytic converters, having expensive metals namely platinum-palladium and rhodium as the catalysts, are fitted into automobiles for reducing emission of poisonous gases. (e) Phasing out of old vehicles (f) Use of low-sulphur and aromatic content in petrol and diesel (g) Application of pollution-level norms for vehicles, etc.   Controlling Vehicular Air Pollution: A Case

Study of Delhi {  In

Delhi, compressed natural gas (CNG) in public transport (buses) is used. { CNG is better than petrol and diesel because— (a) CNG burns most efficiently and very little of it is left unburnt. (b) CNG is cheaper than petrol or diesel, cannot be siphoned off by thieves and adulterated like petrol or diesel. { The main problem with CNG is(a) The difficulty of laying down pipelines to deliver. (b)  Distribution points/pumps and ensuring uninterrupted supply.  Electrostatic Precipitator {  It is the electrical device widely used to remove particulate matter present in the exhaust of thermal power plants and vehicular exhaust. { It can remove over 99% of particulate matter present in the exhaust from a thermal power plant. { The electrons released from electrode wires (maintained at several thousand volts) attach to dust particles and give a negative charge. { The collecting plates are earthed so that they attract the charged dust particles. {  The velocity of air between the plates must be low enough to allow the dust to fall. { A scrubber removes gases like SO2 { In a scrubber, the exhaust is passed through a spray of water or lime. {  Very small particulates are not removed by this precipitator.

ENVIRONMENTAL ISSUES

173

174 Oswaal CUET (UG) Chapterwise Question Bank   Laws & policies in India to control vehicular

pollution

(a) Auto fuel policy It has laid out a roadmap to cut down vehicular pollution in Indian cities. It has steadily reduced the sulphur and aromatic content in petrol and diesel fuels. (b) Euro II norms It stipulates that sulphur should be at 350 parts-permillion (ppm) in diesel and 150 ppm in petrol.  Aromatic hydrocarbons are to be contained at 42%of the concerned fuel. The goal is to reduce sulphur to 50 ppm in petrol and diesel and bring down the level to 35%. Vehicle engines will also need to be upgraded. (c) The Bharat Stage II All automobiles and fuel have to meet the Euro III emission specifications in 11 cities from 1 April 2005 and had to meet the Euro-IV norms by 1 April 2010. The rest of the country have Euro-III emission norm compliant automobiles and fuels by 2010. Water Pollution and Its Control { Water bodies are the lifeline of all living organisms. {  Due to human activities, the ponds, lakes, stream, rivers, estuaries and oceans are becoming polluted. {  The Government of India has passed the Water (Prevention and Control of Pollution) Act, 1974 to safeguard our water resources.

Domestic Sewage and Industrial Effluents { A

mere 0.1 % impurities make domestic sewage unfit for human use. {  The composition of waste water contains suspended solids (sand, silt, clay etc), colloidal materials (faecal matter, bacteria, cloth, paper, fibres etc.) and dissolved materials (nutrients like nitrate, NH3, phosphate, Na,Ca etc). { Solids are easy to remove. {  Removal of dissolved materials, organic compounds and toxic metal ions are most difficult. {  Domestic sewage mainly contains biodegradable organic matter. {  It is decomposed by microorganisms, which can multiply using these organic substances as substrates and hence utilize some of the components of sewage. {  Sewage from homes and hospitals may contain undesirable pathogens and its disposal into water causes serious diseases like dysentery, typhoid, jaundice, cholera, etc. {  Industrial (petroleum, metal, paper manufacturing, chemical manufacturing, etc.) wastewater contains toxic substances like heavy metals such as mercury, cadmium, copper, lead, etc. and organic compounds. { Some toxic substances (mercury, DDT, etc) present in industrial wastewater, cause biological magnification (biomagnifications) in the aquatic food chain.

BIOLOGY

Biochemical Oxygen Demand - BOD { The amount of biodegradable organic matter in sewage water is estimated by measuring biochemical oxygen demand (BOD). { During biodegradation, microorganisms consume a lot of O2. { It results in a sharp decline in dissolved O2. This causes the death of aquatic organisms. Algal Bloom {  Presence of large amounts of nutrients in water also causes excessive growth of planktonic algae (algal bloom). {  It imparts a distinct colour to the water bodies and deteriorates the water quality resulting in death of fishes. {  Some bloom-forming algae are extremely toxic to human beings and animals. { The water hyacinth (Eichhornia crassipes) is the most problematic aquatic weed (Terror of Bengal’). { They grow faster than our ability to remove them. { They grow abundantly in eutrophic water bodies. { It leads to an imbalance in the ecosystem dynamics of the water body.

Biomagnification { Biomagnification

is the accumulation of the toxicant at successive trophic levels. { The organism in each trophic level cannot metabolize or excrete the toxicant (like mercury, calcium and DDT), and is thus passed on to the next trophic level. { Biomagnification of DDT dichlorodiphenyl trichloroethane) in an Aquatic Food Chain { Water (DDT: 0.003 ppm) → zooplankton (0.04 ppm) → small fish (0.5 ppm) → large fish (2 ppm) → birds (25 ppm). {  DDT disturbs calcium metabolism in birds, which causes thinning of eggshell and their premature breaking. { It causes a decline in bird populations.

Eutrophication { It

is the natural ageing of a lake by nutrient enrichment of water. { In a young lake the water is cold and clear. {  With time, streams draining into the lake introduce nutrients like N2, P, etc, which encourage the growth of aquatic organisms. {  As the lake’s fertility increases, plants and animals grow rapidly, and organic remains are deposited on the lake bottom. {  Thus, the lake grows shallower and warmer, with warm-water organisms. { Marsh plants take root in the shallows and fill in the original lake basin. { Eventually, the lake becomes a bog, finally converting into the land. { Depending on the climate, size of the lake and other factors, the eutrophication may span thousands of years.

175

ENVIRONMENTAL ISSUES { However,

pollutants like effluents from industries and homes accelerate the ageing process. This phenomenon is called Cultural or Accelerated Eutrophication. {  The prime contaminants are nitrates and phosphates, which act as plant nutrients. {  They over stimulate the growth of algae, causing unsightly scum and unpleasant odours, and robbing the water of dissolved oxygen vital to other aquatic life. { At the same time, other pollutants flowing into a lake may poison whole populations of fish; whose decomposed remains further deplete the waters's dissolved oxygen content.

Ecological Sanitation { It

is a sustainable system for handling human excreta, using dry composting toilets. { This is a practical, hygienic, efficient and cost-effective solution to human waste disposal. {  Most importantly with this composting method, human excreta can be recycled into a resource (as natural fertiliser), which reduces the need for chemical fertilisers. { There are ‘EcoSan’ toilets in many areas of Kerala and Sri Lanka.

Solid Wastes { Solid

wastes refer to everything Scan to know that goes out in the trash. more about {  These are discarded solid this topic materials which are produced due to various human activities. {  Municipal solid wastes are wastes from homes, offices, stores, schools, hospitals, etc., Solid Waste Mgt that are collected and disposed of by the municipality. { The municipal solid wastes include paper, food wastes, plastics, glass, metals, rubber, leather, textile, etc. { Burning reduces the volume of the wastes, although it is generally not burnt to completion and open dumps often serve as the breeding ground for rats and flies.

Sanitary Landfills { These

were adopted as the substitute for open-burning dumps. { In a sanitary landfill, wastes are dumped in a depression or trench after compaction, and covered with dirt every day. { Landfills are also not much of a solution since the amount of garbage generation especially in the metros has increased so much that these sites are getting filled too. {  Also there is a danger of seepage of chemicals, etc., from these landfills polluting the underground water

resources.

Types of Solid Wastes { All

wastes can be categorized into three types namely: (a) Bio-degradable (b) Recyclable (c) Non-biodegradable

Plastic Wastes { It

is important that all garbage generated is sorted to recycle or reuse. { Kabadiwallas and rag-pickers help to separate materials for recycling. { The biodegradable materials can be put into deep pits in the ground and be left for natural breakdown. { This leaves only the non-biodegradable to be disposed off. {  We are increasing the use of non-biodegradable products. e.g., plastic packets of eatables such as biscuit packet, milk and water in polybags, packed fruits and vegetables (in polystyrene and plastic packaging) etc. { State Governments are trying to push for a reduction in the use of plastics and encouraging the use of ecofriendly packaging. { We can use carrying a cloth or other natural fibre carrybags instead of polythene bags for shopping.

Hospital Wastes { Hospital wastes contain disinfectants and other harmful

chemicals, and also pathogenic micro-organisms. incinerators are used to dispose a hospital wastes.

{ The

E-wastes {  Irreparable

computers and other electronic goods are known as electronic wastes (e-wastes). { They are buried in landfills or incinerated. { Over half of the e-wastes generated in the developed world are exported to developing countries, mainly to China, India and Pakistan, where metals like copper, iron, silicon, nickel and gold are recovered during the recycling process. { Developed countries have specifically built facilities for recycling of e-wastes. {  Recycling in developing countries often involves manual participation thus exposing workers to toxic substances present in e-wastes. {  Recycling is the only solution for the treatment of e-waste, provided it is carried out in an environment friendly manner.

Agro-Chemicals and Their Effects { In the wake of the green revolution, the use of inorganic

fertilisers, pesticides, herbicides, fungicides, etc. has increased manifold for enhancing crop production. {  These are toxic to non-target organisms that are important components of the soil ecosystem. { These can be biomagnified in terrestrial ecosystems. { Chemical fertilisers cause eutrophication. Greenhouse Effect & Global ­Warming {  Greenhouse is a small glass­house used for growing plants during winter under controlled conditions. { The glass panel lets the light in, but does not allow heat to escape.

176 Oswaal CUET (UG) Chapterwise Question Bank { Therefore,

the greenhouse warms up. effect is a natural phenomenon responsible for heating of Earth’s surface and atmosphere. { It maintains the present average temperature (15°C). { Without the greenhouse effect, the average temperature at Earth surface would have been very cold. (–18°C). { Clouds and gases reflect about 1/4th of the incoming solar radiation, and absorb some of it. {  But almost half of incoming solar radiation falls on Earth’s surface heating it, while a small proportion is reflected { Earth’s surface re-emits heat as infrared radiation. { But a part of infrared is absorbed by atmospheric gases (CO2, CH4, etc.) and so cannot escape into space. { These greenhouse gases (commonly - carbon dioxide and methane) radiate heat energy, and a major part of which again comes to Earth’s surface, thus heating it. { These gases cause the greenhouse effect. {  Increase in the level of greenhouse gases has led to global warming (overheating of Earth land). { During the past century, the temperature of Earth has increased by 0.6°C, most of it during the last 3 decades. { Greenhouse

Global Warming { The gradual rise in temperature of the earth surface due

to the accumulation of greenhouse gases is called as global warming. It has led to deleterious changes in the environment resulting in odd climatic changes (e.g., El Nino effect). { Impacts of Climate Change: (a) It has been observed that in the past three decades, the average temperature of the Earth has increased up to 0.6ºC. As a result, the natural water cycle has been disturbed, which has resulted in abrupt changes in the pattern of rainfall. It also changed the amount of rainwater. (b) Melting of Polar ice caps and mountain glaciers. This has caused a rise in the sea level,leading to the inundation of coastal regions. (c) The upper parts of the atmosphere have become cooler due to reduced passage of longwave radiations. This in turn had led to shrinkage of the atmosphere. { Climate Change mitigation: (a) Climate change is a real and serious issue. We need to act promptly to mitigate (reducing the severity, seriousness or painfulness of something) its effects. (b) Climate change mitigation consists of actions to limit the magnitude or rate of long-term climate change. It generally involves reductions in human (anthropogenic) emissions of greenhouse gases (GHGs). Mitigation may also be achieved by increasing the capacity of carbon sinks, e.g., through reforestation. (c) Mitigation policies can substantially reduce the risks associated with human-induced global warming. { Ozone Depletion in the Stratosphere There are two types of ozone- Bad ozone and Good ozone.

‘Bad’ ozone

BIOLOGY

is an air pollutant and is formed in the lower atmosphere (troposphere). It harms plants and animals. The ‘good’ ozone is found in the stratosphere.  It acts as a shield absorbing ultraviolet radiation from the sun.  UV rays are highly injurious since they cause mutation.  The thickness of the ozone (O3) in a column of air from the ground to the top of the atmosphere is measured in terms of Dobson units (DU). Ozone is continuously formed by the action of UV rays on molecular oxygen, and also degraded into molecular oxygen in the stratosphere.  Production and degradation of ozone in the stratosphere should be balanced.  But the balance is disrupted due to ozone degradation by chlorofluorocarbons (CFCs) gas. CFCs (used as refrigerants) move upward and reach the stratosphere. UV rays act on them releasing Cl atoms.  In the presence of Cl (catalyst), ozone degrades releasing molecular oxygen (O2) causing ozone depletion.  It has formed on ozone hole over the Antarctic region.  UV radiations of wavelengths shorter than UVB, are almost completely absorbed by Earth’s atmosphere. { Effects of UV-B (a) UV-B causes mutation of DNA. (b) It causes ageing of the skin, damage to skin cells and skin cancers. (c) A high dose of UV-B causes inflammation of the cornea (snow-blindness), cataract, etc. (d) It permanently damages the cornea. { Montreal Protocol  The Montreal Protocol (an international treaty in Canada, 1987) was signed to control the emission of ozone depleting substances. Many more efforts have been made and protocols have laid down definite road maps, separately for developed and developing countries, for reducing the emission of CFCs and other ozone depleting chemicals. { Deforestation It is the conversion of forested areas to non-forested ones. Almost 40% of forests have been lost in the tropics, compared to only 1% in the temperate region.  National Forest Policy (1988) of India has recommended 33% forest cover for the plains and 67% for the hills. We have only 19.4% of forest cover (it was about 30% at the beginning of the 20th century). { Reasons of Deforestation (a) Conversion of forest to agricultural land.

177

ENVIRONMENTAL ISSUES

(b) For timber, firewood, cattle ranching, etc. (c) Slash & burn agriculture (Jhum cultivation) in the north-eastern states of India. In this, the farmers cut down the trees of the forest and burn the plant remains. The ash is used as a fertiliser and the land is then used for farming or cattle grazing. After cultivation, the area is left for several years to allow its recovery. In earlier days, enough time-gap was given for recovery. With increasing population and repeated cultivation, this recovery phase is decreased, resulting in deforestation. { Consequences of Deforestation (a) CO2 concentration in the atmosphere is enhanced because trees that could hold a lot of carbon in their biomass are lost with deforestation (b) Loss of biodiversity due to habitat destruction. (c) Disturbs hydrological cycle (d) Soil erosion and desertification { Reforestation (a) The process of restoring a forest that once existed in the past is known as reforestation. (b) It may occur naturally in a deforested area. (c) We can speed it up by planting trees.

arrange wood for constructing a new palace. minister and workers went to a forest near a village, inhabited by Bishnois. The Bishnois thwarted them from cutting down the trees. A Bishnoi woman, Amrita Devi hugged a tree. Sadly, the king’s men cut down the tree along with Amrita Devi. Her three daughters and hundreds of other Bishnois followed her , and thus lost their lives saving trees. Government of India has instituted the Amrita Devi Bishnoi Wildlife Protection Award for individuals or communities from rural areas for extraordinary courage and dedication in protecting wildlife. (b) Chipko Movement of Garhwal Himalayas  In 1974, local women of Garhwal Himalayas participated to protect trees from the axe of contractors by hugging them. Realizing the significance of participation by local communities, the Government of India in the 1980s has introduced the concept of Joint Forest Management (JFM) to work closely with the local communities for protecting and managing forests.  In return for their services to the forest, the communities get the benefit of various forest products (e.g., fruits, gum, rubber, medicine, etc.), and thus the forest can be conserved in a sustainable manner. The

Two Case Studies of People’s Participation in ­Conservation of Forests (a) Bishnoi Movement In 1731, the king of Jodhpur in Rajasthan asked to

OBJECTIVE TYPE QUESTIONS [A] MULTIPLE CHOICE QUESTIONS: 1. The green scum seen in the fresh water bodies is (a) blue green algae. (b) red algae. (c) green algae. (d) both (a) and (c) 2. Algal blooms do not cause— (a) Imbalance in ecosystem dynamics. (b) Deterioration of the water quality and fish mortality. (c) Reduction in BOD. (d) Increase in organic matters in water body. 3. How is the amount of biodegradable organic matter in sewage water estimated? (a) Chemical Oxygen Demand (b) Physical Oxygen Demand (c) Biological Oxygen Demand (d) Mathematical Oxygen Demand 4. According to the Central Pollution Control Board, particles that are responsible for causing great harm to human health are of diameter (a) 2.50 micrometers. (b) 5.00 micrometers. (c) 10.00 micrometers. (d) 7.5 micrometers. 5. The material generally used for sound proofing of rooms like a recording studio and auditorium, etc. is (a) cotton. (b) coir.

(c) wood. (d) styrofoam. 6. Match List I with List II (CUET 2022) List-I

List-II

A. Catalytic converter

I. Particulate matter

B. Incinerators

II. Organic Waste

C. Electrostatic precipitator

III. Hospital Waste

D. Sewage treatment plant

IV. Carbon monoxide and nitrogen oxides

(a) (b) (c) (d) 7. (a) (c) 8. A. B. C. D.

Choose the correct answer form the options given below: A - III, B - II, C - IV, D - I A - IV, B - III, C - I, D - II A - II, B - III, C - I, D - IV A - I, B - II, C - IV, D - III Compressed Natural Gas (CNG) is Propane. (b) Methane. Ethane. (d) Butane. Arrange the following options in a sequential manner to demonstrate degradation caused by improper resource utilisation and maintenance. (CUET 2022) Barren patches of land that become large and lead to desertification. Fertile top soil can be removed easily by over cultivation. Deposition of thin crust of salt on land surface. Unrestricted grazing of animals.

178 Oswaal CUET (UG) Chapterwise Question Bank E. Irrigation without proper drainage of water leading to water logging in the soil. Choose the correct answer from the options given below: (a) A, B, C, D, E (b) C, D, A, B, E (c) B, D, E, C, A (d) E, D, B, C, A 9. Among the following which one causes more indoor chemical pollution? (a) Burning coal (b) Burning cooking gas (c) Burning mosquito coil (d) Room spray 10. How is the value of COD compared to BOD? (a) Much higher (b) Zero (c) Much lower (d) Low 11. What phenomenon occurs due to the accumulation of certain pollutants that increase in concentration along the food chain? (a) Eutrophication (b) Pollution (c) Deforestation (d) Bio-magnification 12. Why is it necessary to remove sulphur from petroleum products? (a) To reduce the emission of sulphur dioxide in exhaust fumes. (b) To increase efficiency of automobiles engines. (c) To use sulphur removed from petroleum for commercial purposes. (d) To increase the life span of engine silencers. 13. Prime contaminant of lake for eutrophication is: (a) Dissolved oxygen (b) Algal bloom (c) Nitrate and phosphate (d) Fungi and Bacteria 14. What of the following was adopted as the substitute for open-burning dumping grounds? (a) Lakes (b) Sanitary landfills (c) Land (d) Forests 15. The loudness of a sound that a person can withstand without discomfort is about (a) 150 dB. (b) 215 dB. (c) 30 dB. (d) 80 dB. 16. Which of the following element is most hazardous to humans? (a) Phosphorous (b) Sulfur (c) Carbon (d) Uranium 17. The major source of noise pollution, worldwide is due to (a) office equipment. (b) transport system. (c) sugar, textile and paper industries. (d) oil refineries and thermal power plants 18. Match List-I with List-II (CUET 2022) List-I

List-II

A. Catalytic converter

I. Solid wastes

B. Electrostatic precipitator

II. High noise level

C. Earmuffs

III. Particulate matter

D. Lands fills

IV. Carban monoxide and nitrogen oxide

Choose the correct answer from the options given below: (a) A-II, B-III, C-I, D-IV (b) A-I, B-IV, C-III, D-II (c) A-IV, B-III, C-II, D-I (d) A-III, B-IV, C-II, D-I

BIOLOGY

19. Which of the following is a cyclic and zero waste procedure? (a) Radioactive waste management (b) Oil-spill cleaning (c) Integrated organic farming (d) Reduction in thermal heat 20. Non-biodegradable pollutants are created by (a) nature (b) excessive use of resources (c) humans (d) natural disasters 21. Match the items in column I and column II and choose the correct option. Column-I

Column-II

A. UV

(i) Bio-magnification

B. Biodegradable organic matter

(ii) Eutrophication

C. DDT

(iii) Snow blindness

D. Phosphates (iv) BOD The correct matches are (a) A – (ii), B – (i), C – (iv), D – (iii) (b) A – (iii), B – (ii), C – (iv), D – (i) (c) A – (iii), B – (iv), C – (i), D – (ii) (d) A – (iii), B – (i), C – (iv), D – (ii) 22. Catalytic converters are fitted into automobiles to reduce emission of harmful gases. Catalytic converters change unburnt hydrocarbons into (a) carbon dioxide and water. (b) carbon monoxide. (c) methane. (d) carbon dioxide and methane 23. Chipko Movement of Garhwal Himalayas (a) Government of India has instituted the Amrita Devi Bishnoi Wildlife Protection Award for individuals or communities from rural areas for extraordinary courage and dedication in protecting wildlife. (b) In 1974, local women of Garhwal Himalayas participated to protect trees from the axe of contractors by hugging them. (c) Realizing the significance of participation by local communities, the Government of India in the 1980s has introduced the concept of Joint Forest Management (JFM) to work closely with the local communities for protecting and managing forests. (d) In return for their services to the forest, the communities get the benefit of various forest products (e.g., fruits, gum, rubber, medicine, etc.), and thus the forest can be conserved in a sustainable manner. 24. What does CPCB stand for? (a) Central Particulate Control Board (b) Central Panama Channel Board (c) Central Pollution Control Board (d) Central Pollution Channel Board 25. Which of the following causes bio-magnification? (a) SO2 (b) Mercury (c) DDT (d) Both (b) & (c) 26. Which of the following is used as an atmospheric pollution indicator? (a) Lepidoptera (b) Lichens (c) Lycopersicon (d) Lycopodium

179

ENVIRONMENTAL ISSUES

27. How is the thickness of the ozone in a column of air from the ground to the top of the atmosphere measured? (a) Dobson units (b) Hertz (c) Centimeter (d) Ampere 28. Match correctly the following and choose the correct option. Column-I A. Environment Protection Act

Column-II (i) 1974

B. Air Prevention and Control of (ii) 1987 Pollution C. Water Act

(iii) 1986

D.  Amendment of Air Act to (iv) 1981 include noise The correct matches are (a) A – (iii), B – (iv), C – (i), D – (ii) (b) A – (i), B – (iii), C – (ii), D – (iv) (c) A – (iv), B – (i), C – (ii), D – (iii) (d) A – (iii), B – (iv), C – (ii), D – (i) 29. Which act was passed to protect and improve the quality of our environment? (a) The Mental Healthcare Act (b) The Coal Mines Act (c) The National Sports University Act (d) Environment Protection Act 30. In Global Warming, the temperature of which part of the Atmosphere increases? (a) Troposphere (b) Mesosphere (c) Stratosphere (d) Ionosphere. [B] ASSERTION & REASON: Directions: In the following questions, a statement of assertion (A) is followed by a statement of reason (R). Mark the correct choice as: (A) Both assertion (A) and reason (R) are true and reason (R) is the correct explanation of assertion (A). (B) Both assertion (A) and reason (R) are true but reason (R) is not the correct explanation of assertion (A). (C) Assertion (A) is true but reason (R) is false. (D) Assertion (A) is false but reason (R) is true. 1. Assertion: The temperature of the Earth has increased by 0.6°C. Reason: Intensity of sun rays reaching the surface has increased. 2. Assertion: Carbon dioxide and methane are commonly known as green grass. Reason: Greenhouse gases absorb longwave infra-red radiation from the earth and emit it again towards the earth, which results in heating of the earth’s surface. 3. Assertion (A): Montreal protocol suggested to control the emission of ozone depleting substances. Reason (R): Montreal protocol, in an international treaty signed at Geneva in 1987. 4. Assertion: Chipko movement was started in Gharwal region of Himalayas in 1974. Reason: It is a movement, lead by people to protect trees by hugging them. 5. Assertion (A): Noise is an air pollutant. Reason (R): Noise causes psychological and physiological disorders in humans.

6. Assertion (A): Electrostatic precipitator is used to remove particulate matter. Reason (R): It can remove only 80 percent of particulate matter present in the exhaust from a thermal power plant. 7. Assertion: Suspended particulate matter (SPM) is an important pollutant released by diesel vehicles. Reason: Catalytic converters greatly reduce pollution caused by automobiles. 8. Assertion (A): Bharat stage IV emission norms have been in place since April 2010, for 4 wheelers in 13 mega cities of India. Reason (R): Green muffler scheme refers to the plantation of trees and shrubs along road sides and is effective to control noise pollution only. 9. Assertion (A): Water pollutants are measured by BOD.  Reason (R): If BOD is more, the water is polluted. 10. Assertion (A): Excess of nitrates in drinking water are harmful for infants. Reason (R): Nitrates are responsible for blue baby syndrome. [C] COMPETENCY/CASE-BASED QUESTIONS: I. Read the passage carefully and answer the question given after the passage from 1–5. The greenhouse effect is a natural phenomenon that is responsible for heating of Earth’s surface and atmosphere. Due to the greenhouse effect, their is an increase in the atmospheric concentration of CO2. increase in global temperature called Global warming. The rise in temperature of the Earth results in odd climatic changes, melting of polar ice caps, rising of sea level, changes in rainfall pattern. 1. Which of the following pair does not represent greenhouse gases ? (a) CO2 and CH4 (b) CH4 and N2O (c) O3 and SO2 (d) N2O and CFCs. 2. What does the increase in the level of greenhouse gases lead to? (a) No change in ecosystem cycles (b) Proper plant growth (c) Proper rainfall (d) Global warming 3. Maximum greenhouse gases are released by which of the following country ? (a) India (b) France (c) USA (d) Britain. 4. Which one of the following ways can be used to control global warming? (a) Increasing deforestation (b) Reducing efficiency of energy usage (c) Reducing the use of fossil fuel (d) Increasing the growth of human population. 5. Directions: In the following questions a statement of Assertion (A) is followed by a statement of reason (R). Mark the correct choice as: (a) Both Assertion (A) and reason (R) are true and reason (R) is the correct explanation of Assertion (A). (b) Both Assertion (A) and reason (R) are true but reason (R) is not the correct explanation of Assertion (A).

180 Oswaal CUET (UG) Chapterwise Question Bank (c) Assertion (A) is true but reason (R) is false. (d) Assertion (A) is false but reason (R) is true. Assertion (A): The rise in temperature of the Earth results in La Nina. Reason (R): La Nina trade winds are stronger than usual, pushing more warm water toward Asia. II. Study the following diagram and answer the question given below 6–10. Increasing human population, has led to an increase in demand for food, water, shelter etc. These requirements are being fulfilled at the cost of our natural resources. With increasing population, the rate of pollution has also increased. Pollution is any undesirable change in physical, chemical or biological characteristics of air, land, water or soil. Agents which bring undesirable change are called pollutants. To improve the quality of our environment, the Government of India has passed Environment Protection Act.

BIOLOGY

6. The are substances that are chemical, physical, or biological materials that contaminate the environment are called: (a) Waste (b) Pollutant (c) Hazards material (d) All of the above. 7. Air pollutants that is not coming directly from the pollution source: (a) NOx (b) Ozone (c) Hydrocarbons (d) All of the above. 8. Acid rain is rainfall with a pH of: (a) 7 (b) 6.56 (c) < 4.5 (d) 11. 9. Most hazardous pollutant of automobile exhaust is: (a) Mercury (b) Copper (c) Arsenic (d) Lead. 10. Environment protection act passed in: (a) 1986 (b) 1977. (c) 1980. (d) 1991.

ANSWER KEY MULTIPLE CHOICE QUESTIONS 1. (d)

2. (b)

3. (c)

4. (a)

5. (d)

6. (b)

7. (b)

8. (c)

9. (a)

10. (a)

11. (d)

12. (a)

13. (c)

14. (b)

15. (d)

16. (d)

17. (b)

18. (d)

19. (c)

20. (d)

21. (b)

22 (c)

23. (d)

24. (c)

25. (d)

26. (b)

27. (a)

28. (a)

29. (d)

30. (d)

1. (c)

2. (a)

3. (c)

4. (a)

8. (c)

9. (a)

10. (a)

1. (c)

2. ( )

3. (c)

8. (c)

9. (d)

10. (a)

ASSERTION & REASON 5. (b)

6. (c)

7. (b)

COMPETENCY/CASE-BASED QUESTIONS 4. (c)

5. (d)

6. (b)

7. (b)

ANSWERS WITH EXPLANATION MULTIPLE CHOICE

5. Option (d) is correct.

1. Option (d) is correct.

Explanation: The material generally used for sound proofing of rooms like recording studio, cinema hall, auditorium, etc. which absorb sound and facilitate in proofing is Styrofoam. Styrofoam is a polystyrene foam material which is always made in a sheet form and is generally in blue colour. Ear plugs (device used by working person of commercial and industrial zone) are made up of very fine glass wood or cotton wood impregnated with wax while earmuffs contain fluid seals or plastic foam for absorbing sound.

Explanation: The green scum seen in the fresh water bodies mainly consists of green algae and blue-green algae, while the red algae is mostly marine. 2. Option (b) is correct. Explanation: Algal blooms are formed as a result of overproduction of algae in a water body that is nutrient-rich or eutrophicated water-bodies. They can cause harmful effects on aquatic species by increasing organic matter and thus BOD of water body increases too. 3. Option (c) is correct. Explanation: The amount of biodegradable organic matter in sewage water estimated with the help of biological oxygen demand. It is used for indicating the organic strength of wastewater. It is designated as BOD. 4. Option (a) is correct. Explanation: According to the Central Pollution Control Board (CPCB), particles that are responsible for causing great harm to human health are of diameter 2.50 micrometers. According to CPCB, particulate matter of 2.5 micrometer or less in diameter are responsible for causing great harm to human health. The particular matter of 2.5 micrometers or less in size are inhaled deep into the respiratory tract (up to lungs) and causes respiratory problems and may even lead to death.

6. Option (b) is correct. Explanation: When exhaust passes through catalytic converter then carbon monoxide and nitrogen oxide are converted to carbon dioxide and nitrogen gas. The use of incinerator is crucial for disposal of hospital waste. Incineration is the high temperature burning of a waste. Electrostatic precipitator can remove over 99% of particulate matter. They worked on the principle of the attraction of a charged particle for an oppositely charged collector. In sewage treatment plant biodegradation of organic matter occurs by microorganisms. 7. Option (b) is correct. Explanation: Compressed natural gas (CNG) is the best and cheap fuel which is used to power vehicles, mostly buses and trucks. It has replaced petrol and diesel fuel as it is considered as clean fuel. It consists of around 90% methane, by compressing it to less than 1% of volume it occupies at standard atmospheric

ENVIRONMENTAL ISSUES

pressure. Propane and butane together form LPG or liquefied petroleum gas while ethane is used in chemical industry to produce ethene. 8. Option (c) is correct. Explanation: The loss of top soil due to wind, rain and other forces is a natural process but some other activities like overcultivation, unrestricted grazing of animals, poor irrigation practices result in barren patches of land. Barren lands’ refers to area of land where plant growth is sparse, stunted and/or contain limited biodiversity. When large barren patches extend over time, desert is created. 9. Option (a) is correct. Explanation: Burning coal is the cause of major indoor chemical pollution. It releases carbon content in the atmosphere which combines with oxygen to form carbon dioxide, a threat to environment or may result in the formation of carbon monoxide which is a threat to human health. Burning of coal also releases inhale able particle nitrogen oxides, sulphur dioxides, metal and silicates. Although of mosquito coil and room spray causes indoor chemical pollution but their effects are long term. 10. Option (a) is correct. Explanation: Chemical Oxygen Demand is designated by COD. It is the measure of the amount of oxygen necessary to oxidize all the pollutants present in water. The value of the biological oxygen demand is less than the chemical oxygen demand. 11. Option (d) is correct. Explanation: Biomagnification, also known as bioamplification is the accumulation of certain pollutants (toxins) in the tissues of organisms whose concentration increases along the food chain. These toxins accumulate in the body because they can’t be digested by organisms. 12. Option (a) is correct. Explanation: SO2 is a very harmful pollutant. It can damage the vegetation by causing chlorosis and also causes acid rain damaging the building and plants. Therefore, it is necessary to remove sulphur from petroleum product (diesel) as it reduces the emission of oxides of sulphur like SO2 and SO3 in the exhaust fumes 13. Option (c) is correct. Explanation: Eutrophication is a natural process in which an entire body of water, or its port get enriched with minerals and nutrients. The prime contaminants that caused such eutrophication are nitrates and phosphates. It leads to an increased algal growth, as the level of nutrients increases. 14. Option (b) is correct. Explanation: The sanitary landfill is an engineered method of controlled disposal where waste is isolated from the environment until it is safe. It is a substitute for open-burning dumps containing degradable, semi-degradable, and nonbiodegradable waste. 15. Option (d) is correct. Explanation: Relative loudness of sound ranges from 30 dB– 60 dB and is generally not hazardous. The loudness above 60– 80 dB, is not much uncomfortable while prolonged exposure to noise level above 80 dB is painful and gradually leads to permanent loss of hearing ability or deafness. The permissible sound level in different areas is shown in the given table. 16. Option (d) is correct.

181 Explanation: The most hazardous element among the following is uranium. It is a silvery-grey metal and its isotopes are highly unstable and radioactive. This is extremely harmful to humans and creates various disorders and can be fatal. 17. Option (b) is correct. Explanation: The major source of noise pollution, worldwide is due to transport system, that is, transport vehicles (both public and private). The other sources of noise pollution are various industries like textile, printing, sugar, engineering, and agricultural implements. 18. Option (d) is correct. Explanation: Catalytic converters are devices fitted in automobiles to reduce vehicular pollution. As the vehicular discharge passes through catalytic converter, carbon monoxide and nitric oxide released by catalytic converters are converted into carbon dioxide and nitrogen dioxide respectively. Electrostatic precipitator is the device widely used to remove particulate matter such as dust,smoke etc., from air using force of an electrostatic charge. It can remove over 99% particulate matter present in the exhaust from the thermal power plant. High noise level in an area is eliminated by using earmuffs. Solid wastes arc trashed in a depression or trench considered as landfills. 19. Option (c) is correct. Explanation: A cyclical and zero waste procedure that primarily aims at cultivating the land and raising crops in such a way that the waste products of one process is used as a nutrient for other process is known as integrated organic farming. It maximizes income in relation to area, time, and efforts. 20. Option (d) is correct. Explanation: Non-biodegradable pollutants are those pollutants which are not bro-ken down into simpler substances by natural biological processes. These pollutants include plastics, tin container, heavy metals, radioactive substances, etc. These are created by human activities, like industrialisation. Whereas biodegradable pollutant created by humans (for example paper, household waste like peel of vegetables fruits, etc.) are degraded or disposed quickly by biological processes. 21. Option (b) is correct. Explanation: A-(iii): Snow blindness is the inflammation and photophobia caused by exposure of the eyes to ultraviolet rays) reflected from snow or ice. B-(iv): Biological oxygen demand (BOD) is related to biodegradable organic matter. It is the amount of dissolved oxygen that must be present in water in order to decompose the organic matter in the water, used as a measure of the degree of pollution. C-(i): DDT is a colorless odorless substance which is used as an insecticide. It is toxic to animals and is known to accumulate in the tissues in a process called bio-magnification. D-(ii): Eutrophication is a process by which a body of water becomes enriched in dissolved nutrients (such as phosphates) that stimulate the growth of aquatic plant life usually resulting in the depletion of dissolved oxygen. 22. Option (c) is correct. Explanation: Catalytic converters contain costly metals like rhodium and platinum-palladium as catalysts, and when exhaust gas passes to fitted catalytic converter, the unburnt hydrocarbons (cause of cancer) are oxidised into water and carbon dioxide. Catalytic converters are mainly designed to reduce immediate, local air pollution.

182 Oswaal CUET (UG) Chapterwise Question Bank

BIOLOGY

23. Option (b) is correct.

2. Option (a) is correct.

Explanation: In 1974, local women of Garhwal Himalayas participated to protect trees from the axe of contractors by hugging them. 24. Option (c) is correct.

Explanation: The greenhouse effect is  a natural process that warms the Earth’s surface. When the Sun’s energy reaches the Earth’s atmosphere, some of it is reflected back to space and some is absorbed and re-radiated by greenhouse gases. 3. Option (c) is correct.

Explanation: Central Pollution Control Board is designated as CPCB. It was formed in the year 1974 under the Ministry of Environment, Forest and Climate Change. It promotes the prevention, control, and abatement of pollution. 25. Option (d) is correct. Explanation: Bio-magnifications (also called as bioaccumulation) is the accumulation of non-biodegradable toxic material like mercury (Hg), DDT, etc., in different trophic levels. In the process of bio-magnification, the concentration of non-biodegradable pollutants increases at each successive trophic level of the food chain, thus, harming the environment at an alarming rate, whereas SO2(sulphur dioxide) is an air pollutant and acts as precursor of acid rain. 26. Option (b) is correct. Explanation: Lichens can be used as an atmospheric pollution indicator because they do not grow in areas that are polluted. They are sensitive (especially phycobiont) to oxides of nitrogen and sulphur, hence unable to synthesise organic food and do not grow well. Therefore, lichens grow in areas where sulphur dioxide pollution is very less (e.g.,in Manali and Darjeeling). Lepidoptera is an insect order. Lycopersicon is scientific name of tomato. Lycopodium is an example of pteridophyte. 27. Option (a) is correct. Explanation: Dobson unit (DU) is used to measure the thickness of the ozone in a column of air from the ground to the top of the atmosphere. A layer of pure ozone that is 0.01 mm thick at standard temperature and pressure is equivalent to one Dobson unit. 28. Option (a) is correct. Explanation: A-(iii): Environment Protection Act came into force on November 1986 at the birth anniversary of late Prime Minister of India Smt. Indira Gandhi. B-(iv): Air prevention and control of pollution Act came into force in 1981 which is meant for the preservation of quality of air. C-(i): Water Act 1974 is meant for restoration of quality of all type of surface and groundwater. D-(ii): In 1987 Amendment of Air Act to include noise into pollution came into existence 29. Option (d) is correct. Explanation: Explanation: The Environment Protection Act was passed by the Government of India in order to control environmental (air, water, and soil) pollution. It was passed in the year 1986. 30. Option (d) is correct. Explanation: Global Warming is the process of heating up the Earth’s atmosphere. It has existed since the beginning of industrialisation, but with the increase in emission of Greenhouse gases, the rate of global warming has also increased. Human activities like burning of fossil fuels and discharge of industrial gases has led to the accumulation of greenhouse gases in the Earth’s atmosphere.  [B] ASSERTION & REASON: 1. Option (c) is correct. Explanation: Assertion is true but reason is false, because the rise in temperature is due to the greenhouse effect.

Explanation: Montreal protocol was signed at Montreal (Canada) in 1987. 4. Option (a) is correct. Explanation: Chipko movement was started in the Garwal region of the Himalayas in 1984 to protect trees from indiscriminate cutting due to human activities. 5. Option (b) is correct. Explanation: Both assertion and reason are true. According to the Air Act which came into force in 1981 and later amended in 1987 has included noise as an air pollutant. 6. Option (c) is correct. Explanation: Assertion is true, but reason is false as an electrostatic precipitator can remove over 99 percent particulate matter present in the exhaust from a thermal power plant. 7. Option (b) is correct. Explanation: SPM (Suspended Particulate Matter) is defined as particles floating in the air with a diameter below 10 μm. Studies have shown that high SPM concentrations in the air can have a detrimental impact on respiratory organs. SPM generation from natural sources (e.g., volcanoes or dust storms) and human activities (vehicles, incinerators and industrial plants). Catalytic converters is a devices designed to reduce the amount of emissions from automobiles. The current (socalled three way) systems use a heated metal catalyst to reduce the emissions of carbon monoxide (CO), hydrocarbons, and nitric oxide (NO), all of which contribute to the formation of photochemical smog. 8. Option (c) is correct. Explanation: Bharat stage emission standards are emission standards issued by the Government of India to regulate the emission of air pollutants from internal combustion of engine equipments of motor vehicles. Bharat Stage IV norms have been in place for 4-wheelers in 13 mega cities of India since April 2010. Green muffler or green belt vegetation is rows of trees and shrubs grown and maintained to serve as noise absorbers. It also reduces air pollution because the trees and shrubs absorb pollution gases and cause settling of suspended particulate matter. 9. Option (a) is correct. Explanation: Water pollutants are commonly measured by their main common denominator, called BOD (Biochemical Oxygen Demand), i.e., the amount of free oxygen absorbed by extraneous substances from water. If water is polluted, it will consume more oxygen, thereby enhancing the BOD of water. 10. Option (a) is correct. Explanation: Excess of nitrates in drinking water are harmful for human health and may be fatal for infants. Excessive use of fertilizers often leads to accumulations of nitrates in water. In infants, excess nitrate reacts with hemoglobin to form nonfunctional methaemoglobin that impairs oxygen transport. This condition is termed as methaemnoglobinemia or blue baby syndrome. This disease can damage respiratory and vascular systems and even cause suffocation.

183

ENVIRONMENTAL ISSUES

[C] COMPETENCY/CASE-BASED QUESTIONS: 1. Option (c) is correct.

pushing more warm water toward Asia.

Explanation: CO2,CH4,N2O and CFCs are the green house gases because this gases in the earth’s atmosphere trap heat. 2. Option (d) is correct

Explanation: Pollutants are substances that are chemical, physical, or biological materials that contaminate the environment and cause pollution. These may cause soil, water, or air pollution. Pollutants cause both environmental as well as health problems.

Explanation: Global warming is the phenomenon occurring due to an increase in the level of greenhouse gases. This leads to considerable heating and an increase in temperature of Earth leading to deleterious changes in the environment. This causes odd climatic changes. 3. Option (c) is correct. Explanation: The largest source of greenhouse gas emissions from human activities in the United States is from burning fossil fuels for electricity, heat, and transportation. 4. Option (c) is correct. Explanation: The phenomenon occurring due to an increase in the level of greenhouse gases is known as global warming. This leads to an increase in Earth’s temperature and leading to deleterious changes in the environment. This can be controlled by reducing the use of fossil fuels. 5. Option (d) is correct. Explanation: The rise in temperature of the Earth results in El Nino effect. El Niño refers to the above-average sea-surface temperatures that periodically develop across the east-central equatorial Pacific. La NIna trade winds are stronger than usual,

24

6. Option (b) is correct.

7. Option (b) is correct. Explanation: Examples of a secondary air pollutant include ozone, which is formed when hydrocarbons (HC) and nitrogen oxides (NOx) combine in the presence of sunlight. 8. Option (c) is correct. Explanation: The pH of acid-impacted rain is generally below 4.5 on the pH scale. 9. Option (d) is correct. Explanation: Lead is the most hazardous metal pollutant of automobile exhaust. It has various adverse effects on the body, including neurological and gastrointestinal effects. Its use in petrol to improve its quality. 10. Option (a) is correct. Explanation: Air (Prevention And Control Of Pollution) Act, 1977. Forest Conservation Act, 1980. Environmental Protection Act, 1986. Public Liability Insurance Act 1991.

Writing Your Notes